Local Board Exams Reviewers

October 22, 2017 | Author: Vivien Marie Noel | Category: Cutaneous Conditions, Epidemics, Epidemiology, Burn, Outbreak
Share Embed Donate


Short Description

Download Local Board Exams Reviewers...

Description

50 item Pharmacology Exam 50 item Pharmacology Exam 1. A client with myasthenia gravis reports the occurrence of difficulty chewing. The physician prescribes pyridostigmine bromide (Mestinon) to increase muscle strength for this activity. The nurse instructs the client to take the medication at what time, in relation to meals? a. after dinner daily when most fatigued b. before breakfast daily c. as soon as arising in the morning d. thirty minutes before each meal 2. A client is advised to take senna (Senokot) for the treatment of constipation asks the nurse how this medication works. The nurse responds knowing that it: a. accumulates water in the stool and increases peristalsis b. stimulates the vagus nerve c. coats the bowel wall d. adds fiber and bulk to the stool 3. A client is receiving heparin sodium by continuous intravenous infusion. The nurse monitors the client for which adverse effect of this therapy? a. decreased blood pressure b. increased pulse rate c. ecchymoses d. tinnitus 4. A client is being treated for acute congestive heart failure (CHF) and the client’s vital signs are as follows: BP 85/50 mm Hg; pulse, 96 bpm; respirations, 26 cpm. The physician prescribes digoxin (Lanoxin). To evaluate a therapeutic effectiveness of this medication, the nurse would expect which of the following changes in the client’s vital signs? a. BP 85/50 mm Hg, pulse 60 bpm, respirations 26 cpm b. BP 98/60 mm Hg, pulse 80 bpm, respirations 24 cpm c. BP 130/70 mm Hg, pulse 104 bpm, respirations 20 cpm d. BP 110/40 mm Hg, 110 bpm, respirations 20 cpm 5. Diazepam (Valium) is prescribed for a client with anxiety. The nurse instructs the client to expect which side effect? a. incoordination b. cough

c. tinnitus d. hypertension 6. A client receives oxytocin (Pitocin) to induce labor. During the administration of the oxytocin, it is most important for the nurse to monitor: a. urinary output b. fetal heart rate c. central venous pressure d. maternal blood glucose 7. A clinic nurse is performing assessment on a client who is being seen in the clinic for the first time. When asking about the client’s medication history, the client tells the nurse that he takes nateglinide (Starlix). The nurse then questions the client about the presence of which disorder that is treated with this medication? a. hypothyroidism b. insomnia c. type 2 diabetes mellitus d. renal failure 8. A client who is taking rifampin (Rifadin) as part of the medication regimen for the treatment of tuberculosis calls the clinic nurse and reports that her urine is a red-orange color. The nurse tells the client to: a. come to the clinic to provide a urine sample b. stop the medication until further instructions are given by the physician c. take the medication dose with an antacid to prevent this adverse effect d. expect a red-orange color in urine, feces, sweat, sputum, and tears as a harmless side effect 9. A nurse is caring for a client with a tracheostomy that has been diagnosed with a respiratory infection. The client is receiving vancomycin hydrochloride (Vancocin) 500 mg intravenously every 12 hours. Which of the following would indicate to the nurse that the client is experiencing an adverse effect of the medication? a. decreased hearing acuity b. photophobia c. hypotension d. bradycardia 10. A nurse is caring for a client with a diagnosis of metastatic breast carcinoma who is receiving tamoxifen citrate (Nolvadex) 10 mg orally twice daily. Which of the following would indicate to the nurse that the client is experiencing a side effect related to the medication?

a. hypetension b. diarrhea c. nose bleeds d. vaginal bleeding 11. A client has just been given a prescription for diphenoxylate with atropine (Lomotil). The nurse teaches the client which of the following about the use of this medication? a. drooling may occur while taking this medication b. irritability may occur while taking this medication c. this medication contains a habit-forming ingredient d. take the medication with a laxative of choice 12. A nurse is gathering data from client about the client’s medication history and notes that the client is taking tolterodine tartrate (Detrol LA). The nurse determines that the client is taking the medication to treat which disorder? a. glaucoma b. renal insufficiency c. pyloric stenosis d. urinary frequency and urgency 13. A client has an order to receive psyllium (Metamucil) daily. The nurse administers this medication with: a. a multivitamin and mineral supplement b. a dose of an antacid c. applesauce d. eight ounces of liquid 14. A nurse is teaching a client taking cyclosporine (Sandimmune) after renal transplant about medication information. The nurse tells the client to be especially alert for: a. signs of infection b. hypotension c. weight loss d. hair loss 15. A nurse reinforces dietary instruction for the client receiving spironolactone (Aldactone). Which food would the nurse instruct the client to avoid while taking this medication? a. crackers b. shrimp c. apricots d. popcorn

16. Oral lactulose (Chronulac) is prescribed for the client with a hepatic disorder and the nurse provides instructions to the client regarding this medication. Which statement by the client indicates a need for further instructions? a. “I need to take the medication with water’” b. “ I need to increase fluid intake while taking the medication” c. “ I need to increase fiber in the diet” d. “I need to notify the physician of nausea occurs” 17. A home care nurse provides instructions to a client taking digoxin (Lanoxin) 0.25 mg daily. Which statement by the client indcates a need for further instructions? a. “I will take my prescribed antacid if I become nauseated” b. “It is important to have my blood drawn when prescribed” c. “I will check my pulse before I take my medication” d. “I will carry a medication identification card with me” 18. A client with anxiety disorder is taking buspirone (BuSpar) and tells the nurse that it is difficult to swallow the tablets. The nurse tells the client to: a. dissolve the tablet in a cup of coffee b. crush the tablet before taking it c. call the physician for a change in medication d. mix the tablet uncrushed in custard 19. A nurse is caring for a child with CHF provides instructions to the parents regarding the administration of digoxin (Lanoxin). Which statement by the mother indicates a need for further instructions? a. “If my child vomits after I give the medication, I will not repeat the dose” b. “I will check my child’s pulse before giving the medication” c. “I will check the dose of the medication with my husband before I give the medication” d. “I will mix the medication with food” 20. A nurse provides instructions to a client who will begin an oral contraceptives. Which statement by the client indicates the need for further instructions? a. “I will take one pill daily at the same time every day” b. “I will not need to use an additional birth control method once I start these pills” c. “If I miss a pill I need to take it as soon as I remember” d. “If I miss two pills I will take them both as soon as I remember and I will take two pills the next day also”

21. A nurse provides instructions to a client taking clorazepate (Tranxene) for management of an anxiety disorder. The nurse tells the client that: a. drowsiness is a side effect that usually disappears with continued therapy b. if dizziness occurs, call the physician c. smoking increases the effectiveness of the medication d. if gastrointestinal disturbances occur, discontinue the medication 22. A client with Parkinson’s disease has begun therapy with levodopa (L-dopa). The nurse determines that the client understands the action of the medication if the client verbalizes that results may not be apparent for: a. 24 hours b. Two to three days c. One week d. Two to three weeks 23. A nurse in a physician’s office is reviewing the results of a client’s phenytoin (Dilantin) level drawn that morning. The nurse determines that the client has a therapeutic drug level if the client’s result was: a. 3 mcg/ml b. 8 mcg/ml c. 15 mcg/ml d. 24mcg/ml 24. A nurse is caring for a client with a genitourinary tract infection receiving amoxicillin (Augmentin) 500 mg every 8 hours. Which of the following would indicate to the nurse that the client is experiencing an adverse effect related to the medication? a. hypertension b. nausea c. headache d. watery diarrhea 25. A nurse is caring for a client with glaucoma who receives a daily dose of acetazolamide (Diamox). Which of the following would indicate to the nurse that the client is experiencing an adverse effect of the medication? a. constipation b. difficulty swallowing c. dark-colored urine and stools d. irritability

26. A nurse is caring for a client with a diagnosis of meningitis who is receiving amphotericin B (Fungizone) intravenously. Which of the following would indicate to the nurse that the client is experiencing an adverse effect related to the medication? a. nausea b. decreased urinary output c. muscle weakness d. confusion 27. A nurse has formulated a nursing diagnosis of Disturbed Body Image for a client who is taking spironolactone (Aldactone). The nurse based this diagnosis on assessment of which side effect of the medication? a. edema b. weight gain c. excitability d. decreased libido 28. A nurse is caring for the client with a history of mild heart failure who is receiving diltiazem hydrochloride (Cardizem) for hypertension. The nurse would assess the client for: a. bradycardia b. wheezing c. peripheral edema and weight gain d. apical pulse rate lower than baseline 29. The wound of a client with an extensive burn injury is being treated with the application of silver sulfadiazine (Silvadene). Which symptom would indicate to the nurse that the client is experiencing a side effect related to systemic absorption? a. pain at the wound site b. burning and itching at the wound site c. a localized rash d. photosensitivity 30. A nurse is caring for a client with a diagnosis of rheumatoid arthritis who is receiving sulindac (Clinoril) 150 mg po twice daily. Which finding would indicate to the nurse that the client is experiencing a side effect related to the medication? a. diarrhea b. photophobia c. fever d. tingling in the extremities

31. The nurse notes that the client is receiving filgrastim (Neupogen). The nurse checks which of the following to determine medication effectiveness? a. neutrophil count b. platelet count c. blood urea nitrogen d. creatinine level 32. A nurse is monitoring a client who is taking fluphenazine decanoate (Prolixin) for signs of leucopenia. Which finding indicates a sign of this blood dyscrasia? a. blurred vision b. constipation c. sore throat d. dry mouth 33. A nurse is administering amphotericin B (Fungizone) to a client intravenously to treat a fungal infection. The nurse monitors the result of which electrolyte study during therapy with this medication? a. sodium b. potassium c. calcium d. chloride 34. A clinic nurse asks a client with diabetes mellitus being seen in the clinic for the first time to list the medications that she is taking. Which combination of medications taken by the client should the nurse report to the physician? a. Acetohexamide (Dymelor) and trimethoprim-sulfamethoxazole (Bactrim) b. Chlorpropamide (Diabenase) and amitriptyline (Elavil) c. Glyburide (DiaBeta) and Lanoxin (Digoxin) d. Tolbutamide (Orinase) and amoxicillin (Amoxil) 35. A nurse is caring for a client receiving streptogramin (Synercid) by intravenous intermittent infusion for the treatment of a bone infection develops diarrhea. Which nursing action would the nurse implement? a. administer an antidiarrheal agent b. notify the physician c. discontinue the medication d. monitor the client’s temperature 36. A client has been taking fosinopril (Monopril) for 2 months. The nurse determines that the client is having the intended effects of therapy if the nurse notes which of the following?

a. lowered BP b. lowered pulse rate c. increased WBC d. increased monocyte count 37. A client is taking labetalol (Normodyne). The nurse monitors the client for which frequent side effect of the medication? a. tachycardia b. impotence c. increased energy level d. night blindness 38. An older client has been using cascara sagrada on a long-term basis. The nurse determines that which laboratory result is a result of the side effects of this medication? a. sodium 135 mEq/L b. sodium 145 mEq/L c. potassium 3.1 mEq/L d. potassium 5.0 mEq/L 39. A client has an order to begin short-term therapy with enoxaparin (Lovenox). The nurse explains to the client that this medication is being ordered to: a. dissolve urinary calculi b. reduce the risk of deep vein thrombosis c. relieve migraine headaches d. stop progression of multiple sclerosis 40. Quinidine gluconate (Dura Quin) is prescribed for a client. The nurse reviews the client’s medical record, knowing that which of the following is a contraindication in the use of this medication? a. complete atrioventricular (AV) block b. muscle weakness c. asthma d. infection 41. A client has been taking benzonatate (Tessalon) as ordered. The nurse tells the client that this medication should do which of the following? a. take away nausea and vomiting b. calm the persistent cough c. decrease anxiety level d. increase comfort level

42. Auranofin (Ridaura) is prescribed for a client with rheumatoid arthritis, and the nurse monitors the client for signs of an adverse effect related to the medication. Which of the following indicates an adverse effect? a. nausea b. diarrhea c. anorexia d. proteinuria 43. A nurse is providing instructions to a client regarding quinapril hydrochloride (Accupril). The nurse tells the client: a. to take the medication with food only b. to rise slowly from a lying to a sitting position c. to discontinue the medication if nausea occurs d. that a therapeutic effect will be noted immediatedly 44. A female client tells the clinic nurse that her skin is very dry and irritated. Which product would the nurse suggest that the client apply to the dry skin? a. glycerin emollient b. aspercreame c. myoflex d. acetic acid solution 45. A client with advanced cirrhosis of the liver is not tolerating protein well, as eveidenced by abnormal laboratory values. The nurse anticipates that which of the following medications will be prescribed for the client? a. lactulose (Chronulac) b. ethacrynic acid (Edecrin) c. folic acid (Folvite) d. thiamine (Vitamin B1) 46. A nurse is planning dietary counseling for the client taking triamterene (Dyrenium). The nurse plans to include which of the following in a list of foods that are acceptable? a. baked potato b. bananas c. oranges d. pears canned in water 47. A client is taking famotidine (Pepcid) asks the home care nurse what would be the best medication to take for a headache. The nurse tells the client that it would be best to take:

a. aspirin (acetylsalicylic acid, ASA) b. ibuprofen (Motrin) c. acetaminophen (Tylenol) d. naproxen (Naprosyn) 48. A nurse has taught a client taking a xanthine bronchodilator about beverages to avoid. The nurse determines that the client understands the information if the client chooses which of the following beverages from the dietary menu? a. chocolate milk b. cranberry juice c. coffee d. cola 49. A client with histoplasmosis has an order for ketoconazole (Nizoral). The nurse teaches the client to do which of the following while taking this medication? a. take the medication on an empty stomach b. take the medication with an antacid c. avoid exposure to sunlight d. limit alcohol to 2 ounces per day 50. A nurse is preparing the client’s morning NPH insulin dose and notices a clumpy precipitate inside the insulin vial. The nurse should: a. draw up and administer the dose b. shake the vial in an attempt to disperse the clumps c. draw the dose from a new vial d. warm the bottle under running water to dissolve the clump Answer Key: http://www.file2upload.com/file/31124/50-item-pharmacology-exam-with-answers-andrationale-pdf.html password iamhere

50 item Integumentary Exam 1. A nurse is caring for a burn client who has sustained thoracic burns and smoke inhalation and is risk for impaired gas exchange. The nurse avoids which action in caring for this client? a. repositioning the client from side to side every 2 hours b. maintaining the client in a supine position with the head of the bed elevated c. suctioning the airway as needed d. providing humidified oxygen as prescribed 2. A client sustains a burn injury to the entire right arm, entire right leg, and anterior thorax. According to the rule of nine’s the nurse determines that what body percent was injured? Answer: ______________________________________ 3. A nurse assesses a burn injury and determines that the client sustained a full-thickness fourth-degree burn if which of the following is noted at the site of injury? a. a wet shiny weeping wound surface b. a dry wound surface c. charring at the wound site d. blisters 4. A client is brought to the emergency room following a burn injury. In assessment the nurse notes that the client’s eyebrow and nasal hairs are singed. The nurse would identify this type of burn as: a. thermal b. electrical c. radiation d. chemical 5. A nurse assesses the carbon monoxide level of a client following a burn injury and notes that the level is 8%. Based on this level, which finding would the nurse expect to note during the assessment of the client? a. tachycardia b. tachypnea c. coma d. impaired visual acuity

6. A nurse assesses the client’s burn injury and determines that the client sustained a partial-thickness superficial burn. Based on this determination, which finding did the nurse note? a. a wet, shiny, weeping wound b. a dry wound surface c. charring at the wound site d. absence of wound sensation 7. A nurse assesses the client’s burn injury and determines that the client sustained a partial-thickness deep burn. Based on this determination, which finding did the nurse note? a. a wet, shiny, weeping wound surface b. a dry wound surface c. charring at the wound site d. total absence of wound sensation 8. On assessment of a child, the nurse notes the presence of white patches on the child’s tongue and determines that they may be indicative of candidiasis (thrush). The nurse understands that the white patches of candidiasis (thrush): a. adhere to the tongue even when scraped with tongue blade b. cause the tongue to bleed continuously around the patch c. produce a red circle in the center of the white lesion d. will occur only in the tongue 9. On assessment, a nurse notes a flat brown circular nevi on the skin of a client that measures less than one centimeter. The client asks, “Is this cancer?” The nurse makes which response to the client? a. “These are likely to be benign moles.” b. “These require immediate attention because they are probably cancer.” c. “These indicate malignancy.” d. “These are probably verrucae.” 10. A nurse is performing a skin assessment on a client. The nurse understands that moles with variegated color, irregular borders, and/or an irregular surface should be considered: a. suspicious b. normal c. common d. benign 11. A client is diagnosed with herpes zoster (shingles). Which pharmacological therapy would the nurse expect to be prescribed to treat this disorder?

a. tetracycline hydrochloride (achromycin) b. erythromycin base (e-mycin) c. acyclovir (zovirax) d. indomethacin (indocin) 12. A nurse reviews the record of a client diagnosed with pemphigus and notes that the physician has documented the presence of Nikolsky’s sign. Based on this documentation, which of the following would the nurse expect to note? a. client complains of discomfort behind the knee on forced dorsiflexion of the foot b. a spasm of the facial muscles elicited by tapping the facial nerve in the region of the parotid gland c. carpal spasm elicited by compressing the upper arm d. the epidermis of the client’s skin can be rubbed off by slight friction or injury 13. A hospitalized client is diagnosed with scabies. Which of the following would a nurse expect to note on inspection of the client’s skin? a. the appearance of vesicles or pustules b. the presence of white patches scattered about the trunk c. multiple straight or wavy threadlike lines beneath the skin d. patchy hair loss and round, red macules with scales 14. A client is seen in the health care clinic and the physician suspects herpes zoster. The nurse prepares the items needed to perform the diagnostic test to confirm this diagnosis. Which item will the nurse obtain? a. a biopsy kit b. a wood’s light c. a culture swab and tube d. a patch test kit 15. A nurse reviews the health care record of a client diagnosed with herpes zoster. Which finding would the nurse expect to note as characteristic of this disorder? a. a generalized red body rash that causes pruritus b. small blue-white spots with a red base noted on the extremities c. a fiery red edematous rash on the cheeks and neck d. clustered and grouped skin vesicles 16. A client returns to the clinic for a follow-up treatment following a skin biopsy of a suspicious lesion performed 1 week ago. The biopsy report indicated that the lesion is a squamous cell carcinoma. The nurse plans care knowing that which of the following describes the characteristic of this type of a lesion?

a. it is highly metastatic b. it does not metastasize c. it is characterized by local invasion d. it is encapsulated 17. A nurse reviews the record of a client scheduled for removal of a skin lesion. The record indicates that the lesion is an irregularly shaped, pigmented papule with a bluetoned color. The nurse determines that this description of the lesion is characteristic of: a. melanoma b. basal cell carcinoma c. squamous cell carcinoma d. actinic keratosis 18. A nurse is reviewing the nursing care plan for a client for whom a stage 4 decubiti ulcer has been documented. Which of the following would the nurse expect to note on assessment of the client? a. a reddened area that returns to a normal skin color after 15 to 20 minutes of pressure relief b. intact skin c. an area in which the top layer of skin is missing d. a deep ulcer that extends into muscle and bone. 19. A nurse notes documentation of a stage 3 pressure ulcer in a client’s record. Which of the following would the nurse expect to note on assessment of the client? a. a deep ulcer that extends into muscle and bone b. a deep ulcer that extends into the dermis and the subcutaneous tissue c. an area in which the top layer of skin is missing d. a reddened area that returns to normal skin color after 15 to 20 minutes of pressure relief 20. A client is in the health care clinic for complaints of pruritus. Following diagnostic studies, it has been determined that there is not a pathophysiological process causing the pruritus. The nurse prepares instructions for the client to assist in reducing the problem and tells the client to: a. use a dehumidifier in the home b. ensure that the temperature in the home is high, especially during the winter months c. use a cool-mist vaporizer, especially during the winter months d. avoid use of skin moisturizers following a bath 21. A client is seen in the health care clinic because of complaints of lesions on the elbows and the knees. The lesions are red raised papules, and large plaques covered by

silvery scales are also noticed on the elbows and the knees. Psoriasis is diagnosed and the nurse provides information about treatment to the client. The nurse determines that the client needs additional information if the client states that which of the following is a component of the treatment plan? a. tar baths b. ultraviolet light treatments c. topical lubricants d. systemic corticosteroids 22. A client is seen in the health care clinic and a biopsy is performed on a skin lesion that the physician suspects malignant melanoma. The nurse prepares a plan of care for the client based on which characteristics of this type of skin cancer? a. it is an aggressive cancer that requires aggressive therapy to control its rapid spread b. it is a slow-growing cancer and seldom metastasizes c. it can grow so large that an entire area, such as the nose, the lip, or the ear must be removed and reconstructed if it occurs on the face d. it is the most common form of skin cancer 23. A nurse is caring for a client brought to the emergency room following a burn injury that occurred in the basement of the home. Which initial finding would indicate the presence of inhalation injury? a. expectoration of sputum tinged with blood b. the presence of singed nasal hair c. absent breath sounds in the lower lobes bilaterally d. tachycardia 24. A nurse is caring for a client who arrives at the emergency room with the emergency medical services team following a severe burn injury from an explosion. Once the initial assessment has been performed by the physician and life-threatening dysfunctions have been addressed, the nurse reviews the physician’s orders anticipating that which pain medication will be prescribed? a. intravenous (IV) morphine sulfate b. aspirin with oxycodone (percodan) via nasogastric tube c. acetaminophen (tylenol) with codeine sulfate d. morphine sulfate by the subcutaneous route 25. A nurse is assessing the operative site in a client who underwent a breast reconstruction. The nurse is inspecting the flap and the areola of the nipple and notes that the areola is a deep red color around the edge. The nurse takes which action first? a. document the findings

b. elevate the breast c. encourage nipple massage d. notify the physician 26. A nurse performs a skin assessment on an assigned client and notes the presence of lesions that are red-tan scaly plaques. The nurse documents this findings as: a. seborrhea b. xerosis c. pruritus d. actinic keratoses 27. A community health nurse has provided fire safety instructions to a group of individuals who are part of a disaster response team. Which statement by a group member indicates a need for further instructions? a. “the victim may be rolled on the ground to extinguish the flames” b. “a blanket or another cover can be used to smother the flames” c. “flames should be doused with water” d. “keep the victim in standing position so flames won’t spread to other parts of the body” 28. A community health nurse is providing a teaching session to firefighters in a small community regarding care to a victim at the scene of a burn injury. The community health nurse instructs the firefighters that in the event of a tar burn the immediate action would be to: a. cool the injury with water b. remove all clothing immediately c. remove the tar from the burn injury d. leave any clothing that is saturated with tar in place 29. The client who sustained an inhalation injury arrives in the emergency department. On assessment of the client, the nurse notes that the client is very confused and combative. The nurse determines that the client is experiencing: a. anxiety b. fear c. hypoxia d. pain 30. The client is diagnosed with stage 1 of Lyme disease. The nurse assesses the client for the hallmark characteristic of this stage. Which assessment finding would the nurse expect to note?

a. dizziness and headaches b. enlarged and inflamed joints c. arthralgias d. skin rash 31. The emergency department nurse is performing an assessment on a client who has sustained circumferential burns of both legs. Which assessment would be the priority in caring for this client? a. assessing peripheral pulses b. assessing neurological status c. assessing urine output d. assessing blood pressure 32. The nurse is reviewing the discharge instructions for a client who had skin biopsy. Which statement by the client indicates a need for further instructions? a. “I will watch for any drainage from the wound” b. “I will return tomorrow to have the sutures removed” c. “I will use antibiotic ointment as prescribed” d. “I will keep the dressing dry” 33. The nurse preparing to assist the physician to examine the client’s skin with a Wood’s light would do which of the following? a. obtain an informed consent b. tell the client that the procedure is painless c. shave the skin site d. prepare a local anesthetic 34. The nurse provides discharge instructions to a client following patch testing. Which instruction would the nurse provide to the client? a. return to the clinic in 2 weeks for the initial reading b. reapply the patch if it comes off c. continue all current activities d. keep the test sites dry 35. A nurse is preparing a client for skin grafting and notes that the physician has documented that the client is scheduled for heterograft. The nurse understands that the heterograft used for the burn client is skin from: a. another species b. a cadaver c. the burned client d. a skin bank

36. Following assessment and diagnostic evaluation, it has been determined that the client has Stage II of Lyme disease. The nurse expects to note which assessment finding that is most indicative of this stage? a. erythematous rash b. cardiac conduction defects c. arthralgias d. enlargement of joints 37. The clinic nurse reads the chart of a client that was seen by the physician and notes that the physician has documented that the client has Stage III of Lyme disease. Which clinical manifestation would the nurse expect to note in the client? a. a generalized skin rash b. a cardiac dysrhythmia c. complaints of joint pain d. paralysis in the extremity where the tick bite occurred 38. A female client arrives at the health care clinic and tells the nurse that she was bitten by a tick and would like to be tested for Lyme disease. The client tells the nurse that she removed the tick and flushed it down the toilet. Which nursing action is appropriate? a. refer the client for a blood test immediately b. inform the client that the tick is needed to perform the test c. inform the client that she will need to return in 6 weeks to be tested because testing before this time is not reliable d. ask the client about the size and color of the tick 39. The client suspected of having Stage I of Lyme disease is seen in the health care clinic and is told that the Lyme disease test is positive. The client asks the nurse about the treatment for the disease. The nurse responds to the client, anticipating which of the following to be part of the treatment plan? a. no treatment unless symptoms develop b. a 3-week course of oral antibiotic therapy c. treatment with intravenous penicillin G d. ultraviolet light therapy 40. The client with acquired immunodeficiency syndrome (AIDS) is suspected of having cutaneous Kaposi’s sarcoma. The nurse prepares the client for which test that will confirm the presence of this type of sarcoma? a. sputum culture b. liver biopsy c. punch biopsy of the lesion

d. white blood cell count 41. The client who is newly admitted to the hospital for treatment of acute cellulitis of the lower left leg asks the nurse about the nature of the disorder. The nurse would respond that cellulitis is actually: a. a skin infection into the deep dermis and subcutaneous fat b. an acute superficial infection c. an inflammation of the epidermis d. an epidermal infection caused by Staphylococcus 42. A nurse is preparing a plan of care for a client with a diagnosis of acute cellulitis of the lower leg. The nurse anticipates which measure will be prescribed to treat this condition? a. warm moist compresses to the affected area b. cold compresses to the affected area c. heat lamp treatments 4 times daily d. alternating hot to cold compresses every 2 hours 43. A clinic nurse provides instructions to a client who will be taking isotretinoin (Accutane) for severe cystic acne. Which statement by the client indicates the need for further instructions? a. “I need to return to the clinic for a blood test to check my triglyceride level” b. “The medication may cause my lips to burn” c. “The medication may cause dryness and burning in my eyes” d. “I need to take vitamin A supplements to improve the effectiveness of this treatment” 44. A client sustained full-thickness burns to both hands from scalding water. A sheet graft was surgically applied to the wounds. The nurse tells the client that this type of graft is indicated for which of the following primary purposes? a. better adherence to the wound bed b. better cosmetic result c. better donor site availability d. easier to care for initially 45. A client sustained a major burn is beginning to take an oral diet again. The nurse plans to encourage the client to eat variety of which of the following types of foods to best help in continued wound healing and tissue repair? a. high carbohydrate and low protein b. high fat and low carbohydrate c. high protein and high fat d. high protein and high carbohydrate

46. A client with a major burn is admitted to the emergency department. The nurse anticipates that which of the following routes will be ordered for analgesics for this client? a. intramuscular b. intravenous c. oral d. subcutaneous 47. A nurse is performing a skin assessment of a client who is immobile and notes the presence of partial thickness skin loss of the upper layer of the skin in the sacral area. The nurse documents these findings as a: a. stage 1 pressure ulcer b. stage 2 pressure ulcer c. stage 3 pressure ulcer d. stage 4 pressure ulcer 48. A student nurse is instructed by the registered nurse to monitor a client who has dark skin for cyanosis. The registered nurse determines that the student needs instructions regarding physical assessment techniques for the dark-skinned client if the student states that the best area to assess for cyanosis was in the: a. nail beds b. lips c. sclera of the eye d. tongue 49. A client with severe psoriasis has a nursing diagnosis of Chronic Low Self-Esteem. The nurse uses which therapeutic strategy when working with this client? a. listening attentively b. pretending not to notice affected skin areas c. keeping communications brief d. approaching the client in a formal manner 50. A nurse caring for a client who sustained a high-voltage electrical injury analyzes the client’s test results. Which finding would the nurse interpret as increasing the client’s risk of developing acute tubular necrosis? a. myoglobin in the urine b. carbonaceous sputum c. hyperkalemia d. cloudy cerebrospinal fluid

Answer Key http://www.file2upload.com/file/31122/50-item-integumentary-exam-with-answers-andrationale-pdf.html password iamhere COMMUNITY HEALTH NURSING EXAMINATION PART I COMMUNITY HEALTH NURSING EXAMINATION PART I By : Budek http://pinoybsn.blogspot.com Outline : I. Epidemiology II. Vital Statistics III. FHSIS IV. COPAR V. Health Education SITUATION : Epidemiology and Vital statistics is a very important tool that a nurse could use in controlling the spread of disease in the community and at the same time, surveying the impact of the disease on the population and prevent it’s future occurrence. 1. It is concerned with the study of factors that influence the occurrence and distribution of diseases, defects, disability or death which occurs in groups or aggregation of individuals. A. Epidemiology B. Demographics C. Vital Statistics D. Health Statistics 2. Which of the following is the backbone in disease prevention? A. Epidemiology B. Demographics C. Vital Statistics D. Health Statistics 3. Which of the following type of research could show how community expectations can result in the actual provision of services? A. Basic Research B. Operational Research

C. Action Research D. Applied Research 4. An outbreak of measles has been reported in Community A. As a nurse, which of the following is your first action for an Epidemiological investigation? A. Classify if the outbreak of measles is epidemic or just sporadic B. Report the incidence into the RHU C. Determine the first day when the outbreak occurred D. Identify if it is the disease which it is reported to be 5. After the epidemiological investigation produced final conclusions, which of the following is your initial step in your operational procedure during disease outbreak? A. Coordinate personnel from Municipal to the National level B. Collect pertinent laboratory specimen to confirm disease causation C. Immunize nearby communities with Measles D. Educate the community in future prevention of similar outbreaks 6. The main concern of a public health nurse is the prevention of disease, prolonging of life and promoting physical health and efficiency through which of the following? A. Use of epidemiological tools and vital health statistics B. Determine the spread and occurrence of the disease C. Political empowerment and Socio Economic Assistance D. Organized Community Efforts 7. In order to control a disease effectively, which of the following must first be known? 1. The conditions surrounding its occurrence 2. Factors that do not favor its development 3. The condition that do not surround its occurrence 4. Factors that favors its development A. 1 and 3 B. 1 and 4 C. 2 and 3 D. 2 and 4 8. All of the following are uses of epidemiology except: A. To study the history of health population and the rise and fall of disease B. To diagnose the health of the community and the condition of the people C. To provide summary data on health service delivery D. To identify groups needing special attention

9. Before reporting the fact of presence of an epidemic, which of the following is of most importance to determine? A. Are the facts complete? B. Is the disease real? C. Is the disease tangible? D. Is it epidemic or endemic? 10. An unknown epidemic has just been reported in Barangay Dekbudekbu. People said that affected person demonstrates hemorrhagic type of fever. You are designated now to plan for epidemiological investigation. Arrange the sequence of events in accordance with the correct outline plan for epidemiological investigation. 1. Report the presence of dengue 2. Summarize data and conclude the final picture of epidemic 3. Relate the occurrence to the population group, facilities, food supply and carriers 4. Determine if the disease is factual or real 5. Determine any unusual prevalence of the disease and its nature; is it epidemic, sporadic, endemic or pandemic? 6. Determine onset and the geographical limitation of the disease. A. 4,1,3,5,2,6 B. 4,1,5,6,3,2 C. 5,4,6,2,1,3 D. 5,4,6,1,2,3 E. 1,2,3,4,5,6 11. In the occurrence of SARS and other pandemics, which of the following is the most vital role of a nurse in epidemiology? A. Health promotion B. Disease prevention C. Surveillance D. Casefinding 12. Measles outbreak has been reported in Barangay Bahay Toro, After conducting an epidemiological investigation you have confirmed that the outbreak is factual. You are tasked to lead a team of medical workers for operational procedure in disease outbreak. Arrange the correct sequence of events that you must do to effectively contain the disease 1. Create a final report and recommendation 2. Perform nasopharyngeal swabbing to infected individuals 3. Perform mass measles immunization to vulnerable groups 4. Perform an environmental sanitation survey on the immediate environment 5. Organize your team and Coordinate the personnels 6. Educate the community on disease transmission

A. 1,2,3,4,5,6 B. 6,5,4,3,2,1 C. 5,6,4,2,3,1 D. 5,2,3,4,6,1 13. All of the following are function of Nurse Budek in epidemiology except A. Laboratory Diagnosis B. Surveillance of disease occurrence C. Follow up cases and contacts D. Refer cases to hospitals if necessary E. Isolate cases of communicable disease 14. All of the following are performed in team organization except A. Orientation and demonstration of methodology to be employed B. Area assignments of team members C. Check team’s equipments and paraphernalia D. Active case finding and Surveillance 15. Which of the following is the final output of data reporting in epidemiological operational procedure? A. Recommendation B. Evaluation C. Final Report D. Preliminary report 16. The office in charge with registering vital facts in the Philippines is none other than the A. PCSO B PAGCOR C. DOH D. NSO 17. The following are possible sources of Data except: A. Experience B. Census C. Surveys D. Research 18. This refers to systematic study of vital events such as births, illnesses, marriages, divorces and deaths

A. Epidemiology B. Demographics C. Vital Statistics D. Health Statistics 19. In case of clerical errors in your birth certificate, Where should you go to have it corrected? A. NSO B. Court of Appeals C. Municipal Trial Court D. Local Civil Registrar 20. Acasia just gave birth to Lestat, A healthy baby boy. Who are going to report the birth of Baby Lestat? A. Nurse B. Midwife C. OB Gyne D. Birth Attendant 21. In reporting the birth of Baby Lestat, where will he be registered? A. At the Local Civil Registrar B. In the National Statistics Office C. In the City Health Department D. In the Field Health Services and Information System Main Office 22. Deejay, The birth attendant noticed that Lestat has low set of ears, Micrognathia, Microcephaly and a typical cat like cry. What should Deejay do? A. Bring Lestat immediately to the nearest hospital B. Ask his assistant to call the nearby pediatrician C. Bring Lestat to the nearest pediatric clinic D. Call a Taxi and together with Acasia, Bring Lestat to the nearest hospital 23. Deejay would suspect which disorder? A. Trisomy 21 B. Turners Syndrome C. Cri Du Chat D. Klinefelters Syndrome 24. Deejay could expect which of the following congenital anomaly that would accompany this disorder?

A. AVSD B. PDA C. TOF D. TOGV 26. Which presidential decree orders reporting of births within 30 days after its occurrence? A. 651 B. 541 C. 996 D. 825 25. These rates are referred to the total living population, It must be presumed that the total population was exposed to the risk of occurrence of the event. A. Rate B. Ratio C. Crude/General Rates D. Specific Rate 26. These are used to describe the relationship between two numerical quantities or measures of events without taking particular considerations to the time or place. A. Rate B. Ratios C. Crude/General Rate D. Specific Rate 27. This is the most sensitive index in determining the general health condition of a community since it reflects the changes in the environment and medical conditions of a community A. Crude death rate B. Infant mortality rate C. Maternal mortality rate D. Fetal death rate 28. According to the WHO, which of the following is the most frequent cause of death in children underfive worldwide in the 2003 WHO Survey? A. Neonatal B. Pneumonia C. Diarrhea D. HIV/AIDS

29. In the Philippines, what is the most common cause of death of infants according to the latest survey? A. Pneumonia B. Diarrhea C. Other perinatal condition D. Respiratory condition of fetus and newborn 30. The major cause of mortality from 1999 up to 2002 in the Philippines are A. Diseases of the heart B. Diseases of the vascular system C. Pneumonias D. Tuberculosis 31. Alicia, a 9 year old child asked you “ What is the common cause of death in my age group here in the Philippines? “ The nurse is correct if he will answer A. Pneumonia is the top leading cause of death in children age 5 to 9 B. Malignant neoplasm if common in your age group C. Probability wise, You might die due to accidents D. Diseases of the respiratory system is the most common cause of death in children 32. In children 1 to 4 years old, which is the most common cause of death? A. Diarrhea B. Accidents C. Pneumonia D. Diseases of the heart 33. Working in the community as a PHN for almost 10 years, Aida knew the fluctuation in vital statistics. She knew that the most common cause of morbidity among the Filipinos is A. Diseases of the heart B. Diarrhea C. Pneumonia D. Vascular system diseases 34. Nurse Aida also knew that most maternal deaths are caused by A. Hemorrhage B. Other Complications related to pregnancy occurring in the course of labor, delivery and puerperium C. Hypertension complicating pregnancy, childbirth and puerperium

D. Abortion SITUATION : Barangay PinoyBSN has the following data in year 2006 1. July 1 population : 254,316 2. Livebirths : 2,289 3. Deaths from maternal cause : 15 4. Death from CVD : 3,029 5. Deaths under 1 year of age : 23 6. Fetal deaths : 8 7. Deaths under 28 days : 8 8. Death due to rabies : 45 9. Registered cases of rabies : 45 10. People with pneumonia : 79 11. People exposed with pneumonia : 2,593 12. Total number of deaths from all causes : 10,998 The following questions refer to these data 35. What is the crude birth rate of Barangay PinoyBSN? A. 90/100,000 B. 9/100 C. 90/1000 D. 9/1000 36. What is the cause specific death rate from cardiovascular diseases? A. 27/100 B. 1191/100,000 C. 27/100,000 D. 1.1/1000 37. What is the Maternal Mortality rate of this barangay? A. 6.55/1000 B. 5.89/1000 C. 1.36/1000 D. 3.67/1000 38. What is the fetal death rate? A. 3.49/1000 B. 10.04/1000 C. 3.14/1000 D. 3.14/100,000

39. What is the attack rate of pneumonia? A. 3.04/1000 B. 7.18/1000 C. 32.82/100 D. 3.04/100 40. Determine the Case fatality ratio of rabies in this Barangay A. 1/100 B. 100% C. 1% D. 100/1000 41. The following are all functions of the nurse in vital statistics, which of the following is not? A. Consolidate Data B. Collects Data C. Analyze Data D. Tabulate Data 42. The following are Notifiable diseases that needs to have a tally sheet in data reporting, Which one is not? A. Hypertension B. Bronchiolitis C. Chemical Poisoning D. Accidents 43. Which of the following requires reporting within 24 hours? A. Neonatal tetanus B. Measles C. Hypertension D. Tetanus 44. Which Act declared that all communicable disease be reported to the nearest health station? A. 1082 B. 1891 C. 3573 D. 6675

45. In the RHU Team, Which professional is directly responsible in caring a sick person who is homebound? A. Midwife B. Nurse C. BHW D. Physician 46. During epidemics, which of the following epidemiological function will you have to perform first? A. Teaching the community on disease prevention B. Assessment on suspected cases C. Monitor the condition of people affected D. Determining the source and nature of the epidemic 47. Which of the following is a POINT SOURCE epidemic? A. Dengue H.F B. Malaria C. Contaminated Water Source D. Tuberculosis 48. All but one is a characteristic of a point source epidemic, which one is not? A. The spread of the disease is caused by a common vehicle B. The disease is usually caused by contaminated food C. There is a gradual increase of cases D. Epidemic is usually sudden 49. The only Microorganism monitored in cases of contaminated water is A. Vibrio Cholera B. Escherichia Coli C. Entamoeba Histolytica D. Coliform Test 50. Dengue increase in number during June, July and August. This pattern is called A. Epidemic B. Endemic C. Cyclical D. Secular SITUATION : Field health services and information system provides summary data on health service delivery and selected program from the barangay level up to the national

level. As a nurse, you should know the process on how these information became processed and consolidated. 51. All of the following are objectives of FHSIS Except A. To complete the clinical picture of chronic disease and describe their natural history B. To provide standardized, facility level data base which can be accessed for more in depth studies C. To minimize recording and reporting burden allowing more time for patient care and promotive activities D. To ensure that data reported are useful and accurate and are disseminated in a timely and easy to use fashion 52. What is the fundamental block or foundation of the field health service information system? A. Family treatment record B. Target Client list C. Reporting forms D. Output record 53. What is the primary advantage of having a target client list? A. Nurses need not to go back to FTR to monitor treatment and services to beneficiaries thus saving time and effort B. Help monitor service rendered to clients in general C. Facilitate monitoring and supervision of services D. Facilitates easier reporting 54. Which of the following is used to monitor particular groups that are qualified as eligible to a certain program of the DOH? A. Family treatment record B. Target Client list C. Reporting forms D. Output record 55. In using the tally sheet, what is the recommended frequency in tallying activities and services? A. Daily B. Weekly C. Monthly D. Quarterly 56. When is the counting of the tally sheet done?

A. At the end of the day B. At the end of the week C. At the end of the month D. At the end of the year 57. Target client list will be transmitted to the next facility in the form of A. Family treatment record B. Target Client list C. Reporting forms D. Output record 58. All but one of the following are eligible target client list A. Leprosy cases B. TB cases C. Prenatal care D. Diarrhea cases 59. This is the only mechanism through which data are routinely transmitted from once facility to another A. Family treatment record B. Target Client list C. Reporting forms D. Output record 60. FHSIS/Q-3 Or the report for environmental health activities is prepared how frequently? A. Daily B. Weekly C. Quarterly D. Yearly 61. Nurse Budek is preparing the reporting form for weekly notifiable diseases. He knew that he will code the report form as A. FHSIS/E-1 B. FHSIS/E-2 C. FHSIS/E-3 D. FHSIS/M-1 62. In preparing the maternal death report, which of the following correctly codes this occurrence?

A. FHSIS/E-1 B. FHSIS/E-2 C. FHSIS/E-3 D. FHSIS/M-1 63. Where should Nurse Budek bring the reporting forms if he is in the BHU Facility? A. Rural health office B. FHSIS Main office C. Provincial health office D. Regional health office 64. After bringing the reporting forms in the right facility for processing, Nurse Budek knew that the output reports are solely produced by what office? A. Rural health office B. FHSIS Main office C. Provincial health office D. Regional health office 65. Mang Raul entered the health center complaining of fatigue and frequent syncope. You assessed Mang Raul and found out that he is severely malnourished and anemic. What record should you get first to document these findings? A. Family treatment record B. Target Client list C. Reporting forms D. Output record 66. The information about Mang Raul’s address, full name, age, symptoms and diagnosis is recorded in A. Family treatment record B. Target Client list C. Reporting forms D. Output record 67. Another entry is to be made for Mang Raul because he is in the target client’s list, In what TCL should Mang Raul’s entry be documented? A. TCL Eligible Population B. TCL Family Planning C. TCL Nutrition D. TCL Pre Natal

68. The nurse uses the FHSIS Record system incorrectly when she found out that A. She go to the individual or FTR for entry confirmation in the Tally/Report Summary B. She refer to other sources for completing monthly and quarterly reports C. She records diarrhea in the Tally sheet/Report form with a code FHSIS/M-1 D. She records a Child who have frequent diarrhea in TCL : Under Five 69. The BHS Is the lowest level of reporting unit in FHSIS. A BHS can be considered a reporting unit if all of the following are met except A. It renders service to 3 barangays B. There is a midwife the regularly renders service to the area C. The BHS Have no mother BHS D. It should be a satellite BHS 70. Data submitted to the PHO is processed using what type of technology? A. Internet B. Microcomputer C. Supercomputer D. Server Interlink Connections SITUATION : Community organizing is a process by which people, health services and agencies of the community are brought together to act and solve their own problems. 71. Mang ambo approaches you for counseling. You are an effective counselor if you A. Give good advice to Mang Ambo B. Identify Mang Ambo’s problems C. Convince Mang Ambo to follow your advice D. Help Mang Ambo identify his problems 72. As a newly appointed PHN instructed to organize Barangay Baritan, Which of the following is your initial step in organizing the community for initial action? A. Study the Barangay Health statistics and records B. Make a courtesy call to the Barangay Captain C. Meet with the Barangay Captain to make plans D. Make a courtesy call to the Municipal Mayor 73. Preparatory phase is the first phase in organizing the community. Which of the following is the initial step in the preparatory phase? A. Area selection B. Community profiling C. Entry in the community

D. Integration with the people 74. the most important factor in determining the proper area for community organizing is that this area should A. Be already adopted by another organization B. Be able to finance the projects C. Have problems and needs assistance D. Have people with expertise to be developed as leaders 75. Which of the following dwelling place should the Nurse choose when integrating with the people? A. A simple house in the border of Barangay Baritan and San Pablo B. A simple house with fencing and gate located in the center of Barangay Baritan C. A modest dwelling place where people will not hesitate to enter D. A modest dwelling place where people will not hesitate to enter located in the center of the community 76. In choosing a leader in the community during the Organizational phase, Which among these people will you choose? A. Miguel Zobel, 50 years old, Rich and Famous B. Rustom, 27 years old, Actor C. Mang Ambo, 70, Willing to work for the desired change D. Ricky, 30 years old, Influential and Willing to work for the desired change 77. Which type of leadership style should the leaders of the community practice? A. Autocratic B. Democratic C. Laissez Faire D. Consultative 78. Setting up Committee on Education and Training is in what phase of COPAR? A. Preparatory B. Organizational C. Education and Training D. Intersectoral Collaboration E. Phase out 79. Community diagnosis is done to come up with a profile of local health situation that will serve as basis of health programs and services. This is done in what phase of COPAR?

A. Preparatory B. Organizational C. Education and Training D. Intersectoral Collaboration E. Phase out 80. The people named the community health workers based on the collective decision in accordance with the set criteria. Before they can be trained by the Nurse, The Nurse must first A. Make a lesson plan B. Set learning goals and objective C. Assess their learning needs D. Review materials needed for training 81. Nurse Budek wrote a letter to PCSO asking them for assistance in their feeding programs for the community’s nutrition and health projects. PCSO then approved the request and gave Budek 50,000 Pesos and a truckload of rice, fruits and vegetables. Which phase of COPAR did Budek utilized? A. Preparatory B. Organizational C. Education and Training D. Intersectoral Collaboration E. Phase out 82. Ideally, How many years should the Nurse stay in the community before he can phase out and be assured of a Self Reliant community? A. 5 years B. 10 years C. 1 year D. 6 months 83. Major discussion in community organization are made by A. The nurse B. The leaders of each committee C. The entire group D. Collaborating Agencies 84. The nurse should know that Organizational plan best succeeds when 1. People sees its values 2. People think its antagonistic professionally 3. It is incompatible with their personal beliefs

4. It is compatible with their personal beliefs A. 1 and 3 B. 2 and 4 C. 1 and 2 D. 1 and 4 85. Nurse Budek made a proposal that people should turn their backyard into small farming lots to plant vegetables and fruits. He specified that the objective is to save money in buying vegetables and fruits that tend to have a fluctuating and cyclical price. Which step in Community organizing process did he utilized? A. Fact finding B. Determination of needs C. Program formation D. Education and Interpretation 86. One of the critical steps in COPAR is becoming one with the people and understanding their culture and lifestyle. Which critical step in COPAR will the Nurse try to immerse himself in the community? A. Integration B. Social Mobilization C. Ground Work D. Mobilization 87. The Actual exercise of people power occurs during when? A. Integration B. Social Mobilization C. Ground Work D. Mobilization 88. Which steps in COPAR trains indigenous and informal leaders? A. Ground Work B. Mobilization C. Core Group formation D. Integration 89. As a PHN, One of your role is to organize the community. Nurse Budek knows that the purposes of community organizing are 1. Move the community to act on their own problems 2. Make people aware of their own problems 3. Enable the nurse to solve the community problems

4. Offer people means of solving their own problems A. 1,2,3 B. 1,2,3,4 C. 1,2 D. 1,2,4 90. This is considered the first act of integrating with the people. This gives an in depth participation in community health problems and needs. A. Residing in the area of assignment B. Listing down the name of person to contact for courtesy call C. Gathering initial information about the community D. Preparing Agenda for the first meeting SITUATION : Health education is the process whereby knowledge, attitude and practice of people are changed to improve individual, family and community health. 91. Which of the following is the correct sequence in health education? 1. Information 2. Communication 3. Education A. 1,2,3 B. 3,2,1 C. 1,3,2 D. 3,1,2 92. The health status of the people is greatly affected and determined by which of the following? A. Behavioral factors B. Socioeconomic factors C. Political factors D. Psychological factors 93. Nurse Budek is conducting a health teaching to Agnesia, 50 year old breast cancer survivor needing rehabilitative measures. He knows that health education is effective when A. Agnesia recites the procedure and instructions perfectly B. Agnesia’s behavior and outlook in life was changed positively C. Agnesia gave feedback to Budek saying that she understood the instruction D. Agnesia requested a written instruction from Budek

94. Which of the following is true about health education? A. It helps people attain their health through the nurse’s sole efforts B. It should not be flexible C. It is a fast and mushroom like process D. It is a slow and continuous process 95. Which of the following factors least influence the learning readiness of an adult learner? A. The individuals stage of development B. Ability to concentrate on information to be learned C. The individual’s psychosocial adaptation to his illness D. The internal impulses that drive the person to take action 96. Which of the following is the most important condition for diabetic patients to learn how to control their diet? A. Use of pamphlets and other materials during instructions B. Motivation to be symptom free C. Ability of the patient to understand teaching instruction D. Language used by the nurse 97. An important skill that a primigravida has to acquire is the ability to bathe her newborn baby and clean her breast if she decides to breastfeed her baby, Which of the following learning domain will you classify the above goals? A. Psychomotor B. Cognitive C. Affective D. Attitudinal 98. When you prepare your teaching plan for a group of hypertensive patients, you first formulate your learning objectives. Which of the following steps in the nursing process corresponds to the writing of the learning objectives? A. Planning B. Implementing C. Evaluation C. Assessment 99. Rose, 50 years old and newly diagnosed diabetic patient must learn how to inject insulin. Which of the following physical attribute is not in anyway related to her ability to administer insulin? A. Strength

B. Coordination C. Dexterity D. Muscle Built 100. Appearance and disposition of clients are best observed initially during which of the following situation? A. Taking V/S B. Interview C. Implementation of the initial care D. Actual Physical examination Answer Key http://pinoybsn.blogspot.com/2006/11/community-health-nursing-examination_06.html 100 item Comprehensive Exam II 1. In a child with suspected coarctation of the aorta, the nurse would expect to find A) Strong pedal pulses B) Diminishing cartoid pulses C) Normal femoral pulses D) Bounding pulses in the arms 2. The nurse is caring for a child receiving chest physiotherapy (CPT). Which of the following actions by the nurse would be appropriate? A) Schedule the therapy thirty minutes after meals B) Teach the child not to cough during the treatment C) Confine the percussion to the rib cage area D) Place the child in a prone position for the therapy 3. A client was admitted to the psychiatric unit with major depression after a suicide attempt. In addition to feeling sad and hopeless, the nurse would assess for A) Anxiety, unconscious anger, and hostility B) Guilt, indecisiveness, poor self-concept C) Psychomotor retardation or agitation D) Meticulous attention to grooming and hygiene 4. A victim of domestic violence states to the nurse, "If only I could change and be how my companion wants me to be, I know things would be different." Which would be the best response by the nurse? A) "The violence is temporarily caused by unusual circumstances; don’t stop hoping for a change." B) "Perhaps, if you understood the need to abuse, you could stop the violence."

C) "No one deserves to be beaten. Are you doing anything to provoke your spouse into beating you?" D) "Batterers lose self-control because of their own internal reasons, not because of what their partner did or did not do." 5. A nurse is to present information about Chinese folk medicine to a group of student nurses. Based on this cultural belief, the nurse would explain that illness is attributed to the A) Yang, the positive force that represents light, warmth, and fullness B) Yin, the negative force that represents darkness, cold, and emptiness C) Use of improper hot foods, herbs and plants D) A failure to keep life in balance with nature and others 6. A polydrug user has been in recovery for 8 months. The client has began skipping breakfast and not eating regular dinners. The client has also started frequenting bars to "see old buddies." The nurse understands that the client’s behavior is a warning sign to indicate that the client may be A) headed for relapse B) feeling hopeless C) approaching recovery D) in need of increased socialization 7. At the day treatment center a client diagnosed with Schizophrenia - Paranoid Type sits alone alertly watching the activities of clients and staff. The client is hostile when approached and asserts that the doctor gives her medication to control her mind. The client's behavior most likely indicates A) Feelings of increasing anxiety related to paranoia B) Social isolation related to altered thought processes C) Sensory perceptual alteration related to withdrawal from environment D) Impaired verbal communication related to impaired judgment 8. A client is admitted with the diagnosis of meningitis. Which finding would the nurse expect in assessing this client? A) Hyperextension of the neck with passive shoulder flexion B) Flexion of the hip and knees with passive flexion of the neck C) Flexion of the legs with rebound tenderness D) Hyperflexion of the neck with rebound flexion of the legs 9. Post-procedure nursing interventions for electroconvulsive therapy include A) Applying hard restraints if seizure occurs B) Expecting client to sleep for 4 to 6 hours C) Remaining with client until oriented D) Expecting long-term memory loss

10. The nurse is talking to parents about nutrition in school aged children. Which of the following is the most common nutritional disorder in this age group? A) Bulimia B) Anorexia C) Obesity D) Malnutrition 11. The nurse assesses a client who has been re-admitted to the psychiatric in-patient unit for schizophrenia. His symptoms have been managed for several months with fluphenazine (Prolixin). Which should be a focus of the first assessment? A) Stressors in the home B) Medication compliance C) Exposure to hot temperatures D) Alcohol use 12. The nurse admits a client newly diagnosed with hypertension. What is the best method for assessing the blood pressure? A) Standing and sitting B) In both arms C) After exercising D) Supine position 13. The nurse is caring for a client who has developed cardiac tamponade. Which finding would the nurse anticipate? A) Widening pulse pressure B) Pleural friction rub C) Distended neck veins D) Bradycardia 14. At the geriatric day care program a client is crying and repeating "I want to go home. Call my daddy to come for me." The nurse should A) Invite the client to join the exercise group B) Tell the client you will call someone to come for her C) Give the client simple information about what she will be doing D) Firmly direct the client to her assigned group activity 15. When teaching adolescents about sexually transmitted diseases, what should the nurse emphasize that is the most common infection? A) Gonorrhea B) Chlamydia C) Herpes D) HIV 16. A 38 year-old female client is admitted to the hospital with an acute exacerbation of asthma. This is her third admission for asthma in 7 months. She describes how she

doesn't really like having to use her medications all the time. Which explanation by the nurse best describes the long-term consequence of uncontrolled airway inflammation? A) Degeneration of the alveoli B) Chronic bronchoconstriction of the large airways C) Lung remodeling and permanent changes in lung function D) Frequent pneumonia 17. The mother of a 15 month-old child asks the nurse to explain her child's lab results and how they show her child has iron deficiency anemia. The nurse's best response is A) "Although the results are here, your doctor will explain them later." B) "Your child has less red blood cells that carry oxygen." C) "The blood cells that carry nutrients to the cells are too large." D) "There are not enough blood cells in your child's circulation." 18. Privacy and confidentiality of all client information is legally protected. In which of these situations would the nurse make an exception to this practice? A) When a family member offers information about their loved one B) When the client threatens self-harm and harm to others C) When the health care provider decides the family has a right to know the client's diagnosis D) When a visitor insists that the visitor has been given permission by the client 19. At a well baby clinic the nurse is assigned to assess an 8 month-old child. Which of these developmental achievements would the nurse anticipate that the child would be able to perform? A) Say 2 words B) Pull up to stand C) Sit without support D) Drink from a cup 20. First-time parents bring their 5 day-old infant to the pediatrician's office because they are extremely concerned about its breathing pattern. The nurse assesses the baby and finds that the breath sounds are clear with equal chest expansion. The respiratory rate is 38-42 breaths per minute with occasional periods of apnea lasting 10 seconds in length. What is the correct analysis of these findings? A) The pediatrician must examine the baby B) Emergency equipment should be available C) This breathing pattern is normal D) A future referral may be indicated 21. A 30 month-old child is admitted to the hospital unit. Which of the following toys would be appropriate for the nurse to select from the toy room for this child? A) Cartoon stickers B) Large wooden puzzle C) Blunt scissors and paper D) Beach ball

22. A 2 year-old child has just been diagnosed with cystic fibrosis. The child's father asks the nurse "What is our major concern now, and what will we have to deal with in the future?" Which of the following is the best response? A) "There is a probability of life-long complications." B) "Cystic fibrosis results in nutritional concerns that can be dealt with." C) "Thin, tenacious secretions from the lungs are a constant struggle in cystic fibrosis." D) "You will work with a team of experts and also have access to a support group that the family can attend." 23. A mother asks the nurse if she should be concerned about the tendency of her child to stutter. What assessment data will be most useful in counseling the parent? A) Age of the child B) Sibling position in family C) Stressful family events D) Parental discipline strategies 24. During an examination of a 2 year-old child with a tentative diagnosis of Wilm's tumor, the nurse would be most concerned about which statement by the mother? A) My child has lost 3 pounds in the last month. B) Urinary output seemed to be less over the past 2 days. C) All the pants have become tight around the waist. D) The child prefers some salty foods more than others. 25. A client is admitted with a pressure ulcer in the sacral area. The partial thickness wound is 4cm by 7cm, the wound base is red and moist with no exudate and the surrounding skin is intact. Which of the following coverings is most appropriate for this wound? A) Transparent dressing B) Dry sterile dressing with antibiotic ointment C) Wet to dry dressing D) Occlusive moist dressing 26. A 65-year-old Hispanic-Latino client with prostate cancer rates his pain as a 6 on a 0to-10 scale. The client refuses all pain medication other than Motrin, which does not relieve his pain. The next action for the nurse to take is to A) Ask the client about the refusal of certain pain medications B) Talk with the client's family about the situation C) Report the situation to the health care provider D) Document the situation in the notes 27. The nurse is caring for a client with an unstable spinal cord injury at the T7 level. Which intervention should take priority in planning care? A) Increase fluid intake to prevent dehydration B) Place client on a pressure reducing support surface C) Use skin care products designed for use with incontinence

D) Increase caloric intake to aid healing 28. A client is experiencing hallucinations that are markedly increased at night. The client is very frightened by the hallucinations. The client’s partner asked to stay a few hours beyond the visiting time, in the client’s private room. What would be the best response by the nurse demonstrating emotional support for the client? A) "No, it would be best if you brought the client some reading material that she could read at night." B) "No, your presence may cause the client to become more anxious." C) "Yes, staying with the client and orienting her to her surroundings may decrease her anxiety." D) "Yes, would you like to spend the night when the client’s behavior indicates that she is frightened?" 29. The nurse is caring for residents in a long term care setting for the elderly. Which of the following activities will be most effective in meeting the growth and development needs for persons in this age group? A) Aerobic exercise classes B) Transportation for shopping trips C) Reminiscence groups D) Regularly scheduled social activities 30. Which type of accidental poisoning would the nurse expect to occur in children under age 6? A) Oral ingestion B) Topical contact C) Inhalation D) Eye splashes 31. A mother wants to switch her 9 month-old infant from an iron-fortified formula to whole milk because of the expense. Upon further assessment, the nurse finds that the baby eats table foods well, but drinks less milk than before. What is the best advice by the nurse? A) Change the baby to whole milk B) Add chocolate syrup to the bottle C) Continue with the present formula D) Offer fruit juice frequently 32. A nurse is conducting a community wide seminar on childhood safety issues. Which of these children is at the highest risk for poisoning? A) 9 month-old who stays with a sitter 5 days a week B) 20 month-old who has just learned to climb stairs C) 10 year-old who occasionally stays at home unattended D) 15 year-old who likes to repair bicycles

33. The nurse assesses delayed gross motor development in a 3 year-old child. The inability of the child to do which action confirms this finding? A) Stand on 1 foot B) Catch a ball C) Skip on alternate feet D) Ride a bicycle 34. The nurse is making a home visit to a client with chronic obstructive pulmonary disease (COPD). The client tells the nurse that he used to be able to walk from the house to the mailbox without difficulty. Now, he has to pause to catch his breath halfway through the trip. Which diagnosis would be most appropriate for this client based on this assessment? A) Activity intolerance caused by fatigue related to chronic tissue hypoxia B) Impaired mobility related to chronic obstructive pulmonary disease C) Self care deficit caused by fatigue related to dyspnea D) Ineffective airway clearance related to increased bronchial secretions 35. A nurse is caring for a client with multiple myeloma. Which of the following should be included in the plan of care? A) Monitor for hyperkalemia B) Place in protective isolation C) Precautions with position changes D) Administer diuretics as ordered 36. A client was admitted to the psychiatric unit with a diagnosis of bipolar disorder. He constantly bothers other clients, tries to help the housekeeping staff, demonstrates pressured speech and demands constant attention from the staff. Which activity would be best for the client? A) Reading B) Checkers C) Cards D) Ping-pong 37. What is the most important aspect to include when developing a home care plan for a client with severe arthritis? A) Maintaining and preserving function B) Anticipating side effects of therapy C) Supporting coping with limitations D) Ensuring compliance with medications 38. A pre-term newborn is to be fed breast milk through nasogastric tube. Why is breast milk preferred over formula for premature infants? A) Contains less lactose B) Is higher in calories/ounce C) Provides antibodies D) Has less fatty acid

39. Which of the following nursing assessments in an infant is most valuable in identifying serious visual defects? A) Red reflex test B) Visual acuity C) Pupil response to light D) Cover test 40. Which nursing action is a priority as the plan of care is developed for a 7 year-old child hospitalized for acute glomerulonephritis? A) Assess for generalized edema B) Monitor for increased urinary output C) Encourage rest during hyperactive periods D) Note patterns of increased blood pressure 41. The nurse should recognize that physical dependence is accompanied by what findings when alcohol consumption is first reduced or ended? A) Seizures B) Withdrawal C) Craving D) Marked tolerance 42. The nurse is preparing a 5 year-old for a scheduled tonsillectomy and adenoidectomy. The parents are anxious and concerned about the child's reaction to impending surgery. Which nursing intervention would be best to prepare the child? A) Introduce the child to all staff the day before surgery B) Explain the surgery 1 week prior to the procedure C) Arrange a tour of the operating and recovery rooms D) Encourage the child to bring a favorite toy to the hospital 43. During the evaluation phase for a client, the nurse should focus on A) All finding of physical and psychosocial stressors of the client and in the family B) The client's status, progress toward goal achievement, and ongoing re-evaluation C) Setting short and long-term goals to insure continuity of care from hospital to home D) Select interventions that are measurable and achievable within selected timeframes 44. The client who is receiving enteral nutrition through a gastrostomy tube has had 4 diarrhea stools in the past 24 hours. The nurse should A) Review the medications the client is receiving B) Increase the formula infusion rate C) Increase the amount of water used to flush the tube D) Attach a rectal bag to protect the skin 45. A client is receiving nitroprusside IV for the treatment of acute heart failure with pulmonary edema. What diagnostic lab value should the nurse monitor in relation to this medication?

A) Potassium B) Arterial blood gasses C) Blood urea nitrogen D) Thiocyanate 46. The nurse is talking with a client. The client abruptly says to the nurse, "The moon is full. Astronauts walk on the moon. Walking is a good health habit." The client’s behavior most likely indicates A) Neologisms B) Dissociation C) Flight of ideas D) Word salad 47. The nurse is assessing a child for clinical manifestations of iron deficiency anemia. Which factor would the nurse recognize as cause for the findings? A) Decreased cardiac output B) Tissue hypoxia C) Cerebral edema D) Reduced oxygen saturation 48. A Hispanic client in the postpartum period refuses the hospital food because it is "cold." The best initial action by the nurse is to A) Have the unlicensed assistive personnel (UAP) reheat the food if the client wishes B) Ask the client what foods are acceptable or bad C) Encourage her to eat for healing and strength D) Schedule the dietitian to meet with the client as soon as possible 49. In planning care for a child diagnosed with minimal change nephrotic syndrome, the nurse should understand the relationship between edema formation and A) Increased retention of albumin in the vascular system B) Decreased colloidal osmotic pressure in the capillaries C) Fluid shift from interstitial spaces into the vascular space D) Reduced tubular reabsorption of sodium and water 50. A client is admitted with a diagnosis of hepatitis B. In reviewing the initial laboratory results, the nurse would expect to find elevation in which of the following values? A) Blood urea nitrogen B) Acid phosphatase C) Bilirubin D) Sedimentation rate 51. The nurse is monitoring the contractions of a woman in labor. A contraction is recorded as beginning at 10:00 A.M. and ending at 10:01 A.M. Another begins at 10:15 A.M. What is the frequency of the contractions? A) 14 minutes B) 10 minutes

C) 15 minutes D) Nine minutes 52. A recovering alcoholic asked the nurse, "Will it be ok for me to just drink at special family gatherings?" Which initial response by the nurse would be best? A) "A recovering person has to be very careful not to lose control, therefore, confine your drinking just at family gatherings." B) "At your next AA meeting discuss the possibility of limited drinking with your sponsor." C) "A recovering person needs to get in touch with their feelings. Do you want a drink?" D) "A recovering person cannot return to drinking without starting the addiction process over." 53. Which of the actions suggested to the RN by the PN during a planning conference for a 10 month-old infant admitted 2 hours ago with bacterial meningitis would be acceptable to add to the plan of care? A) Measure head circumference B) Place in airborne isolation C) Provide passive range of motion D) Provide an over-the-crib protective top 54. A victim of domestic violence tells the batterer she needs a little time away. How would the nurse expect that the batterer might respond? A) With acceptance and views the victim’s comment as an indication that their marriage is in trouble B) With fear of rejection causing increased rage toward the victim C) With a new commitment to seek counseling to assist with their marital problems D) With relief, and welcomes the separation as a means to have some personal time 55. A nurse is assigned to a client who is a new admission for the treatment of a frontal lobe brain tumor. Which history offered by the family members would be anticipated by the nurse as associated with the diagnosis and communicated? A) "My partner's breathing rate is usually below 12." B) "I find the mood swings and the change from a calm person to being angry all the time hard to deal with." C) "It seems our sex life is nonexistant over the past 6 months." D) "In the morning and evening I hear complaints that reading is next to impossible from blurred print." 56. A client who has been drinking for five years states that he drinks when he gets upset about "things" such as being unemployed or feeling like life is not leading anywhere. The nurse understands that the client is using alcohol as a way to deal with A) Recreational and social needs B) Feelings of anger C) Life’s stressors D) Issues of guilt and disappointment

57. The nurse would expect the cystic fibrosis client to receive supplemental pancreatic enzymes along with a diet A) High in carbohydrates and proteins B) Low in carbohydrates and proteins C) High in carbohydrates, low in proteins D) Low in carbohydrates, high in proteins 58. The nurse is discussing nutritional requirements with the parents of an 18 month-old child. Which of these statements about milk consumption is correct? A) May drink as much milk as desired B) Can have milk mixed with other foods C) Will benefit from fat-free cow's milk D) Should be limited to 3-4 cups of milk daily 59. A postpartum mother is unwilling to allow the father to participate in the newborn's care, although he is interested in doing so. She states, "I am afraid the baby will be confused about who the mother is. Baby raising is for mothers, not fathers." The nurse's initial intervention should be what focus? A) Discuss with the mother sharing parenting responsibilities B) Set time aside to get the mother to express her feelings and concerns C) Arrange for the parents to attend infant care classes D) Talk with the father and help him accept the wife's decision 60. A client with emphysema visits the clinic. While teaching about proper nutrition, the nurse should emphasize that the client A) Eat foods high in sodium increases sputum liquefaction B) Use oxygen during meals improves gas exchange C) Perform exercise after respiratory therapy enhances appetite D) Cleanse the mouth of dried secretions reduces risk of infection 61. The nurse is assigned to a client who has heart failure . During the morning rounds the nurse sees the client develop sudden anxiety, diaphoresis and dyspnea. The nurse auscultates, crackles bilaterally. Which nursing intervention should be performed first? A) Take the client's vital signs B) Place the client in a sitting position with legs dangling C) Contact the health care provider D) Administer the PRN antianxiety agent 62. Based on principles of teaching and learning, what is the best initial approach to preop teaching for a client scheduled for coronary artery bypass? A) Touring the coronary intensive unit B) Mailing a video tape to the home C) Assessing the client's learning style D) Administering a written pre-test

63. An eighteen month-old has been brought to the emergency room with irritability, lethargy over 2 days, dry skin and increased pulse. Based upon the evaluation of these initial findings, the nurse would assess the child for additional findings of A) Septicemia B) Dehydration C) Hypokalemia D) Hypercalcemia 64. A nurse is doing preconceptual counseling with a woman who is planning a pregnancy. Which of the following statements suggests that the client understands the connection between alcohol consumption and fetal alcohol syndrome? A) "I understand that a glass of wine with dinner is healthy." B) "Beer is not really hard alcohol, so I guess I can drink some." C) "If I drink, my baby may be harmed before I know I am pregnant." D) "Drinking with meals reduces the effects of alcohol." 65. The nurse is performing an assessment on a child with severe airway obstruction. Which finding would the nurse anticipate finding? A) Retractions in the intercostal tissues of the thorax B) Chest pain aggravated by respiratory movement C) Cyanosis and mottling of the skin D) Rapid, shallow respirations 66. The father of an 8 month-old infant asks the nurse if his infant's vocalizations are normal for his age. Which of the following would the nurse expect at this age? A) Cooing B) Imitation of sounds C) Throaty sounds D) Laughter 67. The nurse is planning to give a 3 year-old child oral digoxin. Which of the following is the best approach by the nurse? A) "Do you want to take this pretty red medicine?" B) "You will feel better if you take your medicine." C) "This is your medicine, and you must take it all right now." D) "Would you like to take your medicine from a spoon or a cup?" 68. The nurse is providing instructions to a new mother on the proper techniques for breast feeding her infant. Which statement by the mother indicates the need for additional instruction? A) "I should position my baby completely facing me with my baby's mouth in front of my nipple." B) "The baby should latch onto the nipple and areola areas." C) "There may be times that I will need to manually express milk." D) "I can switch to a bottle if I need to take a break from breast feeding."

69. Which of these parents’ comment for a newborn would most likely reveal an initial finding of a suspected pyloric stenosis? A) I noticed a little lump a little above the belly button. B) The baby seems hungry all the time. C) Mild vomiting that progressed to vomiting shooting across the room. D) Irritation and spitting up immediately after feedings. 70. The nurse prepares for a Denver Screening test with a 3 year-old child in the clinic. The mother asks the nurse to explain the purpose of the test. What is the nurse’s best response about the purpose of the Denver? A) It measures a child’s intelligence. B) It assesses a child's development. C) It evaluates psychological responses. D) It helps to determine problems. 71. The school nurse suspects that a third grade child might have Attention Deficit Hyperactivity Disorder. Prior to referring the child for further evaluation, the nurse should A) Observe the child's behavior on at least 2 occasions B) Consult with the teacher about how to control impulsivity C) Compile a history of behavior patterns and developmental accomplishments D) Compare the child's behavior with classic signs and symptoms 72. Immediately following an acute battering incident in a violent relationship, the batterer may respond to the partner’s injuries by A) Seeking medical help for the victim's injuries B) Minimizing the episode and underestimating the victim’s injuries C) Contacting a close friend and asking for help D) Being very remorseful and assisting the victim with medical care 73. The nurse, assisting in applying a cast to a client with a broken arm, knows that A) The cast material should be dipped several times into the warm water B) The cast should be covered until it dries C) The wet cast should be handled with the palms of hands D) The casted extremity should be placed on a cloth-covered surface 74. The nurse is caring for a toddler with atopic dermatitis. The nurse should instruct the parents to A) Dress the child warmly to avoid chilling B) Keep the child away from other children for the duration of the rash C) Clean the affected areas with tepid water and detergent D) Wrap the child's hand in mittens or socks to prevent scratching 75. In evaluating the growth of a 12 month-old child, which of these findings would the nurse expect to be present in the infant? A) Increased 10% in height

B) 2 deciduous teeth C) Tripled the birth weight D) Head > chest circumference 76. In taking the history of a pregnant woman, which of the following would the nurse recognize as the primary contraindication for breast feeding? A) Age 40 years B) Lactose intolerance C) Family history of breast cancer D) Uses cocaine on weekends 77. The nurse enters a 2 year-old child's hospital room in order to administer an oral medication. When the child is asked if he is ready to take his medicine, he immediately says, "No!". What would be the most appropriate next action? A) Leave the room and return five minutes later and give the medicine B) Explain to the child that the medicine must be taken now C) Give the medication to the father and ask him to give it D) Mix the medication with ice cream or applesauce 78. A mother asks about expected motor skills for a 3 year-old child. Which of the following would the nurse emphasize as normal at this age? A) Jumping rope B) Tying shoelaces C) Riding a tricycle D) Playing hopscotch 79. A 4 year-old child is recovering from chicken pox (varicella). The parents would like to have the child return to day care as soon as possible. In order to ensure that the illness is no longer communicable, what should the nurse assess for in this child? A) All lesions crusted B) Elevated temperature C) Rhinorrhea and coryza D) Presence of vesicles 80. A home health nurse is caring for a client with a pressure sore that is red, with serous drainage, is 2 inches in diameter with loss of subcutaneous tissue. The appropriate dressing for this wound is A) A transparent film dressing B) Wet dressing with debridement granules C) Wet to dry with hydrogen peroxide D) Moist saline dressing 81. A diabetic client asks the nurse why the health care provider ordered a glycolsylated hemoglobin (HbA) measurement, since a blood glucose reading was just performed. You will explain to the client that the HbA test: A) Provides a more precise blood glucose value than self-monitoring

B) Is performed to detect complications of diabetes C) Measures circulating levels of insulin D) Reflects an average blood sugar for several months 82. The nurse is caring for a client with COPD who becomes dyspneic. The nurse should A) Instruct the client to breathe into a paper bag B) Place the client in a high Fowler's position C) Assist the client with pursed lip breathing D) Administer oxygen at 6L/minute via nasal cannula 83. A 24 year-old male is admitted with a diagnosis of testicular cancer. The nurse would expect the client to have A) Scrotal discoloration B) Sustained painful erection C) Inability to achieve erection D) Heaviness in the affected testicle 84. After successful alcohol detoxification, a client remarked to a friend, "I’ve tried to stop drinking but I just can’t, I can’t even work without having a drink." The client’s belief that he needs alcohol indicates his dependence is primarily A) Psychological B) Physical C) Biological D) Social-cultural 85. The nurse is planning care for a 2 year-old hospitalized child. Which of the following will produces the most stress at this age? A) Separation anxiety B) Fear of pain C) Loss of control D) Bodily injury 86. A 9 year-old is taken to the emergency room with right lower quadrant pain and vomiting. When preparing the child for an emergency appendectomy, what must the nurse expect to be the child's greatest fear? A) Change in body image B) An unfamiliar environment C) Perceived loss of control D) Guilt over being hospitalized 87. In preparing medications for a client with a gastrostomy tube, the nurse should contact the health care provider before administering which of the following drugs through the tube? A) Cardizem SR tablet (diltiazem) B) Lanoxin liquid C) Os-cal tablet (calcium carbonate)

D) Tylenol liquid (acetaminophen) 88. The nurse is assigned to care for a client newly diagnosed with angina. As part of discharge teaching, it is important to remind the client to remove the nitroglycerine patch after 12 hours in order to prevent what condition? A) Skin irritation B) Drug tolerance C) Severe headaches D) Postural hypotension 89. What is the major developmental task that the mother must accomplish during the first trimester of pregnancy? A) Acceptance of the pregnancy B) Acceptance of the termination of the pregnancy C) Acceptance of the fetus as a separate and unique being D) Satisfactory resolution of fears related to giving birth 90. The nurse is caring for a depressed client with a new prescription for an SSRI antidepressant. In reviewing the admission history and physical, which of the following should prompt questions about the safety of this medication? A) History of obesity B) Prescribed use of an MAO inhibitor C) Diagnosis of vascular disease D) Takes antacids frequently 91. The nurse detects blood-tinged fluid leaking from the nose and ears of a head trauma client. What is the appropriate nursing action? A) Pack the nose and ears with sterile gauze B) Apply pressure to the injury site C) Apply bulky, loose dressing to nose and ears D) Apply an ice pack to the back of the neck 92. A nurse aide is taking care of a 2 year-old child with Wilm's tumor. The nurse aide asks the nurse why there is a sign above the bed that says DO NOT PALPATE THE ABDOMEN? The best response by the nurse would be which of these statements? A) "Touching the abdomen could cause cancer cells to spread." B) "Examining the area would cause difficulty to the child." C) "Pushing on the stomach might lead to the spread of infection." D) "Placing any pressure on the abdomen may cause an abnormal experience." 93. The nurse is caring for a client with a deep vein thrombosis. Which finding would require the nurse's immediate attention? A) Temperature of 102 degrees Fahrenheit B) Pulse rate of 98 beats per minute C) Respiratory rate of 32 D) Blood pressure of 90/50

94. A client admits to benzodiazepine dependence for several years. She is now in an outpatient detoxification program. The nurse must understand that a priority during withdrawal is A) Avoid alcohol use during this time B) Observe the client for hypotension C) Abrupt discontinuation of the drug D) Assess for mild physical symptoms 95. The nurse will administer liquid medicine to a 9 month-old child. Which of the following methods is appropriate? A) Allow the infant to drink the liquid from a medicine cup B) Administer the medication with a syringe next to the tongue C) Mix the medication with the infant's formula in the bottle D) Hold the child upright and administer the medicine by spoon 96. A client refuses to take the medication prescribed because the client prefers to take self-prescribed herbal preparations. What is the initial action the nurse should take? A) Report the behavior to the charge nurse B) Talk with the client to find out about the preferred herbal preparation C) Contact the client's health care provider D) Explain the importance of the medication to the client 97. The nurse is teaching diet restrictions for a client with Addison's disease. The client would indicate an understanding of the diet by stating A) "I will increase sodium and fluids and restrict potassium." B) "I will increase potassium and sodium and restrict fluids." C) "I will increase sodium, potassium and fluids." D) "I will increase fluids and restrict sodium and potassium." 98. A nurse arranges for a interpreter to facilitate communication between the health care team and a non-English speaking client. To promote therapeutic communication, the appropriate action for the nurse to remember when working with an interpreter is to A) Promote verbal and nonverbal communication with both the client and the interpreter B) Speak only a few sentences at a time and then pause for a few moments C) Plan that the encounter will take more time than if the client spoke English D) Ask the client to speak slowly and to look at the person spoken to 99. The most common reason for an Apgar score of 8 and 9 in a newborn is an abnormality of what parameter? A) Heart rate B) Muscle tone C) Cry D) Color

100. The nurse is caring for several 70 to 80 year-old clients on bed rest. What is the most important measure to prevent skin breakdown? A) Massage legs frequently B) Frequent turning C) Moisten skin with lotions D) Apply moist heat to reddened areas Answer Key http://pinoybsn.blogspot.com/2006/11/100-item-comprehensive-exam-ii-with.html 100 items Comprehensive Exam Choose the BEST answer 1. The nurse enters the room as a 3 year-old is having a generalized seizure. Which intervention should the nurse do first? A) Clear the area of any hazards B) Place the child on the side C) Restrain the child D) Give the prescribed anticonvulsant 2. A client has just returned to the medical-surgical unit following a segmental lung resection. After assessing the client, the first nursing action would be to A) Administer pain medication B) Suction excessive tracheobronchial secretions C) Assist client to turn, deep breathe and cought D) Monitor oxygen saturation 3. A nurse from the surgical department is reassigned to the pediatric unit. The charge nurse should recognize that the child at highest risk for cardiac arrest and is the least likely to be assiged to this nurse is which child? A) Congenital cardiac defects B) An acute febrile illness C) Prolonged hypoxemia D) Severe multiple trauma 4. Which of the following would be the best strategy for the nurse to use when teaching insulin injection techniques to a newly diagnosed client with diabetes? A) Give written pre and post tests B) Ask questions during practice C) Allow another diabetic to assist D) Observe a return demonstration 5. The nurse is assessing a 2 year-old client with a possible diagnosis of congenital heart disease. Which of these is most likely to be seen with this diagnosis?

A) Several otitis media episodes in the last year B) Weight and height in 10th percentile since birth C) Takes frequent rest periods while playing D) Changing food preferences and dislikes 6. The nurse is reassigned to work at the Poison Control Center telephone hotline. In which of these cases of childhood poisoning would the nurse suggest that parents have the child drink orange juice? A) An 18 month-old who ate an undetermined amount of crystal drain cleaner B) A 14 month-old who chewed 2 leaves of a philodendron plant C) A 20 month-old who is found sitting on the bathroom floor beside an empty bottle of diazepam (Valium) D) A 30 month-old who has swallowed a mouthful of charcoal lighter fluid 7. A 23 year-old single client is in the 33rd week of her first pregnancy. She tells the nurse that she has everything ready for the baby and has made plans for the first weeks together at home. Which normal emotional reaction does the nurse recognize? A) Acceptance of the pregnancy B) Focus on fetal development C) Anticipation of the birth D) Ambivalence about pregnancy 8. Upon examining the mouth of a 3 year-old child, the nurse discovers that the teeth have chalky white-to-yellowish staining with pitting of the enamel. Which of the following conditions would most likely explain these findings? A) Ingestion of tetracycline B) Excessive fluoride intake C) Oral iron therapy D) Poor dental hygiene 9. Which of the following should the nurse teach the client to avoid when taking chlorpromazine HCL (Thorazine)? A) Direct sunlight B) Foods containing tyramine C) Foods fermented with yeast D) Canned citrus fruit drinks 10. The nurse is discussing dietary intake with an adolescent who has acne. The most appropriate statement for the nurse is A) "Eat a balanced diet for your age." B) "Increase your intake of protein and Vitamin A." C) "Decrease fatty foods from your diet." D) "Do not use caffeine in any form, including chocolate." 11. The nurse is caring for a child who has just returned from surgery following a tonsillectomy and adenoidectomy. Which action by the nurse is appropriate?

A) Offer ice cream every 2 hours B) Place the child in a supine position C) Allow the child to drink through a straw D) Observe swallowing patterns 12. The nurse is caring for a client with acute pancreatitis. After pain management, which intervention should be included in the plan of care? A) Cough and deep breathe every 2 hours B) Place the client in contact isolation C) Provide a diet high in protein D) Institute seizure precautions 13. The nurse is caring for a client with trigeminal neuralgia (tic douloureaux). To assist the client with nutrition needs, the nurse should A) Offer small meals of high calorie soft food B) Assist the client to sit in a chair for meals C) Provide additional servings of fruits and raw vegetables D) Encourage the client to eat fish, liver and chicken 14. A client treated for depression tells the nurse at the mental health clinic that he recently purchased a handgun because he is thinking about suicide. The first nursing action should be to A) Notify the health care provider immediately B) Suggest in-patient psychiatric care C) Respect the client's confidential disclosure D) Phone the family to warn them of the risk 15. The initial response by the nurse to a delusional client who refuses to eat because of a belief that the food is poisoned is A) "You think that someone wants to poison you?" B) "Why do you think the food is poisoned?" C) "These feelings are a symptom of your illness." D) "You’re safe here. I won’t let anyone poison you." 16. A client has just been admitted with portal hypertension. Which nursing diagnosis would be a priority in planning care? A) Altered nutrition: less than body requirements B) Potential complication hemorrhage C) Ineffective individual coping D) Fluid volume excess 17. The nurse in a well-child clinic examines many children on a daily basis. Which of the following toddlers requires further follow up? A) A 13 month-old unable to walk B) A 20 month-old only using 2 and 3 word sentences C) A 24 month-old who cries during examination

D) A 30 month-old only drinking from a sippy cup 18. Which of the following conditions assessed by the nurse would contraindicate the use of benztropine (Cogentin)? A) Neuromalignant syndrome B) Acute extrapyramidal syndrome C) Glaucoma, prostatic hypertrophy D) Parkinson's disease, atypical tremors 19. A 15 year-old client with a lengthy confining illness is at risk for altered growth and development of which task? A) Loss of control B) Insecurity C) Dependence D) Lack of trust 20. The nurse is caring for a client with cirrhosis of the liver with ascites. When instructing nursing assistants in the care of the client, the nurse should emphasize that A) The client should remain on bed rest in a semi-Fowler's position B) The client should alternate ambulation with bed rest with legs elevated C) The client may ambulate and sit in chair as tolerated D) The client may ambulate as tolerated and remain in semi-Fowlers position in bed 21. In providing care to a 14 year-old adolescent with scoliosis, which of the following will be most difficult for this client? A) Compliance with treatment regimens B) Looking different from their peers C) Lacking independence in activities D) Reliance on family for their social support 22. The nurse is preparing to perform a physical examination on an 8 month-old who is sitting contentedly on his mother's lap. Which of the following should the nurse do first? A) Elicit reflexes B) Measure height and weight C) Auscultate heart and lungs D) Examine the ears 23. Which of these principles should the nurse apply when performing a nutritional assessment on a 2 year-old client? A) An accurate measurement of intake is not reliable B) The food pyramid is not used in this age group C) A serving size at this age is about 2 tablespoons D) Total intake varies greatly each day 24. The nurse is assessing a client with delayed wound healing. Which of the following risk factors is most important in this situation?

A) Glucose level of 120 B) History of myocardial infarction C) Long term steroid usage D) Diet high in carbohydrates 25. Which of the following nursing assessments indicate immediate discontinuance of an antipsychotic medication? A) Involuntary rhythmic stereotypic movements and tongue protrusion B) Cheek puffing, involuntary movements of extremities and trunk C) Agitation, constant state of motion D) Hyperpyrexia, severe muscle rigidity, malignant hypertension 26. A client with HIV infection has a secondary herpes simplex type 1 (HSV-1) infection. The nurse knows that the most likely cause of the HSV-1 infection in this client is A) Immunosuppression B) Emotional stress C) Unprotected sexual activities D) Contact with saliva 27. The nurse measures the head and chest circumferences of a 20 month-old infant. After comparing the measurements, the nurse finds that they are approximately the same. What action should the nurse take? A) Notify the health care provider B) Palpate the anterior fontanel C) Feel the posterior fontanel D) Record these normal findings 28. At a routine clinic visit, parents express concern that their 4 year-old is wetting the bed several times a month. What is the nurse's best response? A) "This is normal at this time of day." B) "How long has this been occurring?" C) "Do you offer fluids at night?" D) "Have you tried waking her to urinate?" 29. A client was admitted to the psychiatric unit after refusing to get out of bed. In the hospital the client talks to unseen people and voids on the floor. The nurse could best handle the problem of voiding on the floor by A) Requiring the client to mop the floor B) Restricting the client’s fluids throughout the day C) Withholding privileges each time the voiding occurs D) Toileting the client more frequently with supervision 30. The nurse is caring for a client with a sigmoid colostomy who requests assistance in removing the flatus from a 1 piece drainable ostomy pouch. Which is the correct intervention? A) Piercing the plastic of the ostomy pouch with a pin to vent the flatus

B) Opening the bottom of the pouch, allowing the flatus to be expelled C) Pulling the adhesive seal around the ostomy pouch to allow the flatus to escape D) Assisting the client to ambulate to reduce the flatus in the pouch 31. The nurse is teaching parents of an infant about introduction of solid food to their baby. What is the first food they can add to the diet? A) Vegetables B) Cereal C) Fruit D) Meats 32. When counseling parents of a child who has recently been diagnosed with hemophilia, what must the nurse know about the offspring of a normal father and a carrier mother? A) It is likely that all sons are affected B) There is a 50% probability that sons will have the disease C) Every daughter is likely to be a carrier D) There is a 25% chance a daughter will be a carrier 33. When teaching a client with chronic obstructive pulmonary disease about oxygen by cannula, the nurse should also instruct the client's family to A) Avoid smoking near the client B) Turn off oxygen during meals C) Adjust the liter flow to 10 as needed D) Remind the client to keep mouth closed 34. The nurse is caring for a post-op colostomy client. The client begins to cry saying, "I'll never be attractive again with this ugly red thing." What should be the first action by the nurse? A) Arrange a consultation with a sex therapist B) Suggest sexual positions that hide the colostomy C) Invite the partner to participate in colostomy care D) Determine the client's understanding of her colostomy 35. A schizophrenic client talks animatedly but the staff are unable to understand what the client is communicating. The client is observed mumbling to herself and speaking to the radio. A desirable outcome for this client’s care will be A) Expresses feelings appropriately through verbal interactions B) Accurately interprets events and behaviors of others C) Demonstrates improved social relationships D) Engages in meaningful and understandable verbal communication 36. A 7 year-old child is hospitalized following a major burn to the lower extremities. A diet high in protein and carbohydrates is recommended. The nurse informs the child and family that the most important reason for this diet is to A) Promote healing and strengthen the immune system

B) Provide a well balanced nutritional intake C) Stimulate increased peristalsis absorption D) Spare protein catabolism to meet metabolic needs 37. The parents of a 7 year-old tell the nurse their child has started to "tattle" on siblings. In interpreting this new behavior, how should the nurse explain the child's actions to the parents? A) The ethical sense and feelings of justice are developing B) Attempts to control the family use new coping styles C) Insecurity and attention getting are common motives D) Complex thought processes help to resolve conflicts 38. A school nurse is advising a class of unwed pregnant high school students. What is the most important action they can perform to deliver a healthy child? A) Maintain good nutrition B) Stay in school C) Keep in contact with the child's father D) Get adequate sleep 39. A client continually repeats phrases that others have just said. The nurse recognizes this behavior as A) Autistic B) Ecopraxic C) Echolalic D) Catatonic 40. A client is admitted for hemodialysis. Which abnormal lab value would the nurse anticipate not being improved by hemodialysis? A) Low hemoglobin B) Hypernatremia C) High serum creatinine D) Hyperkalemia 41. The nurse is caring for a 7 year-old child who is being discharged following a tonsillectomy. Which of the following instructions is appropriate for the nurse to teach the parents? A) Report a persistent cough to the health care provider B) The child can return to school in 4 days C) Administer chewable aspirin for pain D) The child may gargle with saline as necessary for discomfort 42. The nurse is caring for a 14 month-old just diagnosed with Cystic Fibrosis. The parents state this is the first child in either family with this disease, and ask about the risk to future children. What is the best response by the nurse? A) 1in 4 chance for each child to carry that trait B) 1in 4 risk for each child to have the disease

C) 1in 2 chance of avoiding the trait and disease D) 1in 2 chance that each child will have the disease 43. The nurse is performing an assessment on a client with pneumococcal pneumonia. Which finding would the nurse anticipate? A) Bronchial breath sounds in outer lung fields B) Decreased tactile fremitus C) Hacking, nonproductive cough D) Hyperresonance of areas of consolidation 44. During seizure activity which observation is the priority to enhance further direction of treatment? A) Observe the sequence or types of movement B) Note the time from beginning to end C) Identify the pattern of breathing D) Determine if loss of bowel or bladder control occurs 45. Which of the following statements describes what the nurse must know in order to provide anticipatory guidance to parents of a toddler about readiness for toilet training? A) The child learns voluntary sphincter control through repetition B) Myelination of the spinal cord is completed by this age C) Neuronal impulses are interrupted at the base of the ganglia D) The toddler can understand cause and effect 46. A client complaining of severe shortness of breath is diagnosed with congestive heart failure. The nurse observes a falling pulse oximetry. The client's color changes to gray and she expectorates large amounts of pink frothy sputum. The first action of the nurse would be which of the following? A) Call the health care provider B) Check vital signs C) Position in high Fowler's D) Administer oxygen 47. The nurse is caring for a client with benign prostatic hypertrophy. Which of the following assessments would the nurse anticipate finding? A) Large volume of urinary output with each voiding B) Involuntary voiding with coughing and sneezing C) Frequent urination D) Urine is dark and concentrated 48. An anxious parent of a 4 year-old consults the nurse for guidance in how to answer the child's question, "Where do babies come from?" What is the nurse's best response to the parent? A) "When a child asks a question, give a simple answer." B) "Children ask many questions, but are not looking for answers." C) "This question indicates interest in sex beyond this age."

D) "Full and detailed answers should be given to all questions." 49. A 3 year-old child is treated in the emergency department after ingestion of 1ounce of a liquid narcotic. What action should the nurse do first? A) Provide the ordered humidified oxygen via mask B) Suction the mouth and the nose C) Check the mouth and radial pulse D) Start the ordered intravenous fluids 50. The charge nurse on the eating disorder unit instructs a new staff member to weigh each client in his or her hospital gown only. What is the rationale for this nursing intervention? A) To reduce the risk of the client feeling cold due to decreased fat and subcutaneous tissue B) To cover the bony prominence and areas where there is skin breakdown C) So the client knows what type of clothing to wear when weighed D) To reduce the tendency of the client to hide objects under his or her clothing 51. In teaching parents to associate prevention with the lifestyle of their child with sickle cell disease, the nurse should emphasize that a priority for their child is to A) Avoid overheating during physical activities B) Maintain normal activity with some restrictions C) Be cautious of others with viruses or temperatures D) Maintain routine immunizations 52. The nurse understands that during the "tension building" phase of a violent relationship, when the batterer makes unreasonable demands, the battered victim may experience feelings of A) Anger B) Helplessness C) Calm D) Explosive 53. A parent has numerous questions regarding normal growth and development of a 10 month-old infant. Which of the following parameters is of most concern to the nurse? A) 50% increase in birth weight B) Head circumference greater than chest C) Crying when the parents leave D) Able to stand up briefly in play pen 54. The nurse has been assigned to these clients in the emergency room. Which client would the nurse go check first? A) Viral pneumonia with atelectasis B) Spontaneous pneumothorax with a respiratory rate of 38 C) Tension pneumothorax with slight tracheal deviation to the right D) Acute asthma with episodes of bronchospasm

55. The nurse is assessing a 4 year-old for possible developmental dysplasia of the right hip. Which finding would the nurse expect? A) Pelvic tip downward B) Right leg lengthening C) Ortolani sign D) Characteristic limp 56. A 2 year-old child has recently been diagnosed with cystic fibrosis. The nurse is teaching the parents about home care for the child. Which of the following information is appropriate for the nurse to include? A) Allow the child to continue normal activities B) Schedule frequent rest periods C) Limit exposure to other children D) Restrict activities to inside the house 57. The nurses on a unit are planning for stoma care for clients who have a stoma for fecal diversion. Which stomal diversion poses the highest risk for skin breakdown A) Ileostomy B) Transverse colostomy C) Ileal conduit D) Sigmoid colostomy 58. A client is unconscious following a tonic-clonic seizure. What should the nurse do first? A) Check the pulse B) Administer Valium C) Place the client in a side-lying position D) Place a tongue blade in the mouth 59. The nurse is teaching a client who has a hip prostheses following total hip replacement. Which of the following should be included in the instructions for home care? A) Avoid climbing stairs for 3 months B) Ambulate using crutches only C) Sleep only on your back D) Do not cross legs 60. A nurse who travels with an agency is uncertain about what tasks can be performed when working in a different state. It would be best for the nurse to check which resource? A) The state nurse practice act in which the assignment is made B) With a nurse colleague who has worked in that state 2 years ago D) The Nursing Social Policy Statement within the United States C) The policies and procedures of the assigned agency in that state

61. Parents of a 7 year-old child call the clinic nurse because their daughter was sent home from school because of a rash. The child had been seen the day before by the health care provider and diagnosed with Fifth Disease (erythema infectiosum). What is the most appropriate action by the nurse? A) Tell the parents to bring the child to the clinic for further evaluation B) Refer the school officials to printed materials about this viral illness C) Inform the teacher that the child is receiving antibiotics for the rash D) Explain that this rash is not contagious and does not require isolation 62. What principle of HIV disease should the nurse keep in mind when planning care for a newborn who was infected in utero? A) The disease will incubate longer and progress more slowly in this infant B) The infant is very susceptible to infections C) Growth and development patterns will proceed at a normal rate D) Careful monitoring of renal function is indicated 63. While teaching a client about their medications, the client asks how long it will take before the effects of lithium take place. What is the best response of the nurse? A) Immediately B) Several days C) 2 weeks D) 1 month 64. The nurse is caring for a 12 year-old with an acute illness. Which of the following indicates the nurse understands common sibling reactions to hospitalization? A) Younger siblings adapt very well B) Visitation is helpful for both C) The siblings may enjoy privacy D) Those cared for at home cope better 65. Following a cocaine high, the user commonly experiences an extremely unpleasant feeling called A) Craving B) Crashing C) Outward bound D) Nodding out 66. One reason that domestic violence remains extensively undetected is A) Few battered victims seek medical care B) There is typically a series of minor, vague complaints C) Expenses due to police and court costs are prohibitive D) Very little knowledge is currently known about batterers and battering relationships 67. When making a home visit to a client with chronic pyelonephritis, which nursing action has the highest priority? A) Follow-up on lab values before the visit

B) Observe client findings for the effectiveness of antibiotics C) Ask for a log of urinary output D) As for the log of the oral intake 68. When a client is having a general tonic clonic seizure, the nurse should A) Hold the client's arms at their side B) Place the client on their side C) Insert a padded tongue blade in client's mouth D) Elevate the head of the bed 69. The nurse is teaching a client with dysrhythmia about the electrical pathway of an impulse as it travels through the heart. Which of these demonstrates the normal pathway? A) AV node, SA node, Bundle of His, Purkinje fibers B) Purkinje fibers, SA node, AV node, Bundle of His C) Bundle of His, Purkinje fibers, SA node , AV node D) SA node, AV node, Bundle of His, Purkinje fibers 70. Clients with mitral stenosis would likely manifest findings associated with congestion in the A) Pulmonary circulation B) Descending aorta C) Superior vena cava D) Bundle of His 71. In assessing the healing of a client's wound during a home visit, which of the following is the best indicator of good healing? A) White patches B) Green drainage C) Reddened tissue D) Eschar development 72. The nursing intervention that best describes treatment to deal with the behaviors of clients with personality disorders include A) Pointing out inconsistencies in speech patterns to correct thought disorders B) Accepting client and the client's behavior unconditionally C) Encouraging dependency in order to develop ego controls D) Consistent limit-setting enforced 24 hours per day 73. A client has received her first dose of fluphenazine (Prolixin) 2 hours ago. She suddenly experiences torticollis and involuntary spastic muscle movement. In addition to administering the ordered anticholinergic drug, what other measure should the nurse implement? A) Have respiratory support equipment available B) Immediately place her in the seclusion room C) Assess the client for anxiety and agitation D) Administer prn dose of IM antipsychotic medication

74. The nurse asks a client with a history of alcoholism about the client’s drinking behavior. The client states "I didn’t hurt anyone. I just like to have a good time, and drinking helps me to relax." The client is using which defense mechanism? A) Denial B) Projection C) Intellectualization D) Rationalization 75. The nurse is teaching a smoking cessation class and notices there are 2 pregnant women in the group. Which information is a priority for these women? A) Low tar cigarettes are less harmful during pregnancy B) There is a relationship between smoking and low birth weight C) The placenta serves as a barrier to nicotine D) Moderate smoking is effective in weight control 76. The nurse is caring for a client with end stage renal disease. What action should the nurse take to assess for patency in a fistula used for hemodialysis? A) Observe for edema proximal to the site B) Irrigate with 5 mls of 0.9% Normal Saline C) Palpate for a thrill over the fistula D) Check color and warmth in the extremity 77. Which therapeutic communication skill is most likely to encourage a depressed client to vent feelings? A) Direct confrontation B) Reality orientation C) Projective identification D) Active listening 78. The nurse walks into a client's room and finds the client lying still and silent on the floor. The nurse should first A) Assess the client's airway B) Call for help C) Establish that the client is unresponsive D) See if anyone saw the client fall 79. What is the best way for the nurse to accomplish a health history on a 14 year-old client? A) Have the mother present to verify information B) Allow an opportunity for the teen to express feelings C) Use the same type of language as the adolescent D) Focus the discussion of risk factors in the peer group 80. A new nurse on the unit notes that the nurse manager seems to be highly respected by the nursing staff. The new nurse is surprised when one of the nurses states: "The manager

makes all decisions and rarely asks for our input." The best description of the nurse manager's management style is A) Participative or democratic B) Ultraliberal or communicative C) Autocratic or authoritarian D) Laissez faire or permissive 81. A 2 year-old child is being treated with Amoxicillin suspension, 200 milligrams per dose, for acute otitis media. The child weighs 30 lb. (15 kg) and the daily dose range is 20-40 mg/kg of body weight, in three divided doses every 8 hours. Using principles of safe drug administration, what should the nurse do next? A) Give the medication as ordered B) Call the health care provider to clarify the dose C) Recognize that antibiotics are over-prescribed D) Hold the medication as the dosage is too low 82. The nurse is performing a developmental assessment on an 8 month-old. Which finding should be reported to the health care provider? A) Lifts head from the prone position B) Rolls from abdomen to back C) Responds to parents' voices D) Falls forward when sitting 83. The nurse is participating in a community health fair. As part of the assessments, the nurse should conduct a mental status examination when A) An individual displays restlessness B) There are obvious signs of depression C) Conducting any health assessment D) The resident reports memory lapses 84. The nurse caring for a 14 year-old boy with severe Hemophilia A, who was admitted after a fall while playing basketball. In understanding his behavior and in planning care for this client, what must the nurse understand about adolescents with hemophilia? A) Must have structured activities B) Often take part in active sports C) Explain limitations to peer groups D) Avoid risks after bleeding episodes 85. When assessing a client who has just undergone a cardioversion, the nurse finds the respirations are 12. Which action should the nurse take first? A) Try to vigorously stimulate normal breathing B) Ask the RN to assess the vital signs C) Measure the pulse oximetry D) Continue to monitor respirations

86. In order to enhance a client's response to medication for chest pain from acute angina, the nurse should emphasize A) Learning relaxation techniques B) Limiting alcohol use C) Eating smaller meals D) Avoiding passive smoke 87. The primary nursing diagnosis for a client with congestive heart failure with pulmonary edema is A) Pain B) Impaired gas exchange C) Cardiac output altered: decreased D) Fluid volume excess 88. After talking with her partner, a client voluntarily admitted herself to the substance abuse unit. After the second day on the unit the client states to the nurse, "My husband told me to get treatment or he would divorce me. I don’t believe I really need treatment but I don’t want my husband to leave me." Which response by the nurse would assist the client? A) "In early recovery, it's quite common to have mixed feelings, but unmotivated people can’t get well." B) "In early recovery, it’s quite common to have mixed feelings, but I didn’t know you had been pressured to come." C) "In early recovery it’s quite common to have mixed feelings, perhaps it would be best to seek treatment on an outclient bases." D) "In early recovery, it’s quite common to have mixed feelings. Let’s discuss the benefits of sobriety for you." 89. Clients taking which of the following drugs are at risk for depression? A) Steroids B) Diuretics C) Folic acid D) Aspirin 90. The nurse is assessing a client on admission to a community mental health center. The client discloses that she has been thinking about ending her life. The nurse's best response would be A) "Do you want to discuss this with your pastor?" B) "We will help you deal with those thoughts." C) "Is your life so terrible that you want to end it?" D) "Have you thought about how you would do it?" 91. The nurse is caring for a client 2 hours after a right lower lobectomy. During the evaluation of the water-seal chest drainage system, it is noted that the fluid level bubbles constantly in the water seal chamber. On inspection of the chest dressing and tubing, the nurse does not find any air leaks in the system. The next best action for the nurse is to

A) Check for subcutaneous emphysema in the upper torso B) Reposition the client to a position of comfort C) Call the health care provider as soon as possible D) Check for any increase in the amount of thoracic drainage 92. The nurse is caring for a newborn who has just been diagnosed with hypospadias. After discussing the defect with the parents, the nurse should expect that A) Circumcision can be performed at any time B) Initial repair is delayed until ages 6-8 C) Post-operative appearance will be normal D) Surgery will be performed in stages 93. A client has been receiving lithium (Lithane) for the past two weeks for the treatment of bipolar illness. When planning client teaching, what is most important to emphasize to the client? A) Maintain a low sodium diet B) Take a diuretic with lithium C) Come in for evaluation of serum lithium levels every 1-3 months D) Have blood lithium levels drawn during the summer months 94. When an autistic client begins to eat with her hands, the nurse can best handle the problem by A) Placing the spoon in the client’s hand and stating, "Use the spoon to eat your food." B) Commenting "I believe you know better than to eat with your hand." C) Jokingly stating, "Well I guess fingers sometimes work better than spoons." D) Removing the food and stating "You can’t have anymore food until you use the spoon." 95. A client develops volume overload from an IV that has infused too rapidly. What assessment would the nurse expect to find? A) S3 heart sound B) Thready pulse C) Flattened neck veins D) Hypoventilation 96. A neonate born 12 hours ago to a methadone maintained woman is exhibiting a hyperactive MORO reflex and slight tremors. The newborn passed one loose, watery stool. Which of these is a nursing priority? A) Hold the infant at frequent intervals. B) Assess for neonatal withdrawl syndrome C) Offer fluids to prevent dehydration D) Administer paregoric to stop diarrhea 97. While planning care for a preschool aged child, the nurse understands developmental needs. Which of the following would be of the most concern to the nurse? A) Playing imaginatively

B) Expressing shame C) Identifying with family D) Exploring the playroom 98. A depressed client who has recently been acting suicidal is now more social and energetic than usual. Smilingly he tells the nurse "I’ve made some decisions about my life." What should be the nurse’s initial response? A) "You’ve made some decisions." B) "Are you thinking about killing yourself?" C) "I’m so glad to hear that you’ve made some decisions." D) "You need to discuss your decisions with your therapist." 99. The nurse is caring for 2 children who have had surgical repair of congenital heart defects. For which defect is it a priority to assess for findings of heart conduction disturbance? A) Artrial septal defect B) Patent ductus arteriosus C) Aortic stenosis D) Ventricular septal defect 100. The nurse is caring for a post myocardial infarction client in an intensive care unit. It is noted that urinary output has dropped from 60 -70 ml per hour to 30 ml per hour. This change is most likely due to A) Dehydration B) Diminished blood volume C) Decreased cardiac output D) Renal failure Answer Key http://pinoybsn.blogspot.com/2006/10/100-item-comprehensive-exam-with.html Nursing Practice (Integrated Exam) with Answer Keys Nursing Practice (Integrated Exam) Here's something for all the Filipino Nursing Students. Answer keys are found after the last item of exam. Due to time constraints, I will not be able to provide you with rationales. Kindly post comments if there are any misleading answers. Thank you.. /brewed

Degree of Question’s Difficulty (3) – Difficult question (2) – Average question (1) – Easy question Situation 1: Mr. Santiago has a long history of smoking; he is currently diagnosed with COPD. He is admitted for a pulmonary work up. 1. His arterial blood gas results are PO2 of 85, PCO2 of 40 and HCO3 of 24. Which of the following should be initiated? (3) a. Administer O2 at 2L to prevent him from becoming hypoxic. b. No action is necessary; this is within normal range for a COPD client. c. Anticipate the development of metabolic acidosis and administer Na HCO3. d. Position him in high Fowler’s and anticipate him to need assisted ventilation 2. He finds that after smoking or exercise, he experiences difficulty of breathing, headaches and nausea. These are symptoms of: (2) a. Increased level of carbon dioxide b. Decreased level of arterial oxygen c. Decreased level of carbon dioxide d. Very rapid breathing and metabolic acidosis 3. To encourage proper breathing exercises, which of the following should the nurse teach? (1) a. Encourage pursed lip breathing b. Inhalation should be 2 to 3x that of exhalation c. Encourage high abdominal breathing using the muscles of the diaphragm d. Inhale through the mouth and out through the nose. 4. Which set of blood gases would indicate respiratory acidosis? (2) a. pH 7.0; PCO2 42mmHg; HCO3 21mEq/L b. pH 7.46; PCO2 38mmHg; HCO3 28mEq/L c. pH 7.35; PCO2 44mmHg; HCO3 25mEq/L d. pH 7.32; PCO2 48mmHg; HCO3 22mEq/L 5. He is to receive an IV of Lactated Ringer’s, 1000 cc to run for 8 hours. The drip factor is 10 gtt/cc. How many drops per minute should you regulate the IV? (1) a. 24 gtt/min b. 12 gtt/min c. 21 gtt/min d. 30 gtt/min Situation 2: Nurse Jeddah is the staff nurse assigned in the Medical Ward of a secondary hospital. 6. The physician ordered reverse isolation for Mr. Perez with second degree burns. While performing reverse isolation technique, Nurse Freud should understand that: (2) a. it is not necessary to use sterile linen if the linen has been properly washed.

b. only some persons who come in direct contact with the client need to wear gloves andmask c. sterile gown and gloves must be worn while caring for Mr. Perez d. it is not necessary to wear a mask 7. During the stage of diuresis, there is resorption of fluid into the intravascular compartment and increase urinary output. Which electrolyte imbalance is most frequently associated with this stage? (3) a. hypernatremia, hyperkalemia, carbonic acid deficit b. hyponatremia, hyperkalemia, bicarbonate excess c. hyponatremia, hypokalemia, bicarbonate deficit d. hypernatremia, hypokalemia, carbonic acid excess 8. The priority nursing care for patient suffering from stroke during acute phase is to: (3) a. maintain respiratory and cardiac functions b. prevent contracture and deformities c. maintain optimal nutrition d. provide sensory stimulation 9. The nurse in the clinic would assess a 4-month-old who is in acute respiratory distress when which of the following is observed? (2) a. resting respiratory rate of 35 breaths/min b. flaring of nares c. diaphragmatic respiration d. bilateral breath sounds 10. In assessing patient for signs of impending respiratory failure, an early symptom that the nurse should look for is: (1) a. Kussmaul’s respiration b. cyanosis c. tachypnea d. bradypnea Situation 4: Nurse Kitchie is caring to clients with tuberculosis at San Lazaro Hospital 11. Which of the following symptoms is common in clients with tuberculsosis? (1) a. Mental status changes b. Increased appetite c. Dyspnea on exertion d. Weight loss 12. Nurse Kitchie obtains a sputum specimen from a client for laboratory study. Which of he following laboratory techniques is most commonly used to identify tubercle bacilli in sputum? (1) a. Dark-field illumination b. Sensitivity Testing c. Acid – fast staining d. Agglutination 13. A client has a positive Mantoux test. Nurse Kitchie correctly interprets this reaction to mean that the client has: (2) a. active tuberculosis

b. had contact with Mycobacterium tuberculosis c. developed a resistance to tubercle bacilli d. developed passive immunity to tuberculosis 14. Nurse Kitchie should teach clients that the most common route of transmitting tubercle bacilli from person to person is through contaminated: (1) a. dust particles b. droplet nuclei c. water d. eating utensils 15. The client is to be discharged home with a community health nursing follow – up. Of the following interventions, which would have the highest priority? (2) a. Offering the client emotional support b. Teaching the client about the disease and treatment c. Coordinating various agency services d. Assessing the clients environment for sanitation Situation 4 Mang Tomas with advanced chronic obstructive pulmonary disease (COPD) reports steady weight loss and being “is too tired from just breathing to eat, is admitted in the hospital. 16. Which of the following physical assessment findings would Nurse Pepsi expect to find for Mang Tomas? (2) a. Increased anteroposterior chest diameter b. Underdeveloped neck veins c. Collapsed neck veins d. Increased chest excursions with respiration 17. Which of the following nursing diagnoses would be most appropriate when planning nutritional interventions? (1) a. Altered Nutrition: Less than body requirements r/t fatigue b. Activity Intolerance r/t dyspnea c. Weight loss related to COPD d. Ineffective breathing pattern r/t alveolar hypoventilation 18. Nurse Pepsi’s priority goal for Mang Tomas is: (2) a. Maintaining functional ability b. Minimizing chest pain c. Increasing carbon dioxide levels in the blood d. Treating infectious agents 19. Which of the following diets would be most appropriate for Mang Tomas? (2) a. Low fat, low cholesterol b. Bland, soft diet c. High calorie, high CHON d. Low sodium diet 20. When developing a discharge plan to manage the care for Mang Tomas, Nurse Pepsi should anticipate that the client will do which of the following? (3) a. Develop infections easily b. Maintain current status

c. Require less supplemental oxygen d. Show permanent improvement Situation 5: Tuberculosis (TB) has been declared a global emergency in 1993 by the WHO. 21. TB ranks sixth in the leading causes of morbidity. Nurse Miranda recognizes that the most hazardous periods for development of clinical disease is: (1) a. 1 year b. 2 ½ years c. 3 months d. 6 months 22. Which of the following statements is the primary preventive measure for PTB? (2) a. Provide public health nursing and outreach services b. BCG vaccination of newborn, infants and grade 1 or school entrants c. Make available medical, laboratory and x –ray facilities d. Educate the public in mode of spread and methods of control 23. The source of infection in PTB is through which of the following? (1) a. Contamination of potable water b. Direct connection with injected persons c. Crowded living patterns d. Sexual intercourse 24. In TB control program, DOH has specific objectives, one on prevention is another program focused on children. Which one below is the program? (1) a. Sputum collection and examination b. Tuberculin for skin testing c. EPI for BCG vaccine d. Maternal and child health nursing 25. BCG is given to protect the baby from infection at what age? (1) a. At birth b. At 1 month c. At 2 month d. At 9 month Situation 6: The national objectives for maintaining the health of all Filipinos is a primary responsibility of the Department of Health. 26. The Department of Health Program has a mission which includes all of the following, except: (1) a. Ensure accessibility b. Quality of health care c. Improve quality of life d. Health for all Filipinos in the year 2020 27. Which of the following are primary strategies to achieve health goals? (1) a. Assurance of health care b. Development of national standards and objectives

c. Support to local health system development d. All of the above 28. Which of the following statements best describes the DOH vision? (1) a. Ensure accessibility b. Achievement of quality health care for all c. Health for all Filipinos d. Promotion of health education 29. Which of the following is the mission of the DOH? (1) a. Promote healthy lifestyle b. Ensure accessibility and quality of health care c. Reduce morbidity and mortality d. Improve general heath status of the people 30. Which of the following is not a primary strategy to achieve health goals? (1) a. Support of local health system b. Development of national standards for health c. Assurance of health care for all d. Funding from non – government organizations 3Situation 7: Asthma results in diffuse obstructive and restrictive airway disease of inflammation and bronchoconstriction. With increasing pollution in our environment, both children and adults are now affected with asthma. 31. As a nurse you know that there are many elements that provoke the attack. Which among the elements is a common allergen to both the children and adult patients? (1) a. Cigarette smoke b. Dust – mite c. Perfume d. Flowers 32. If a child has asthma, what nursing diagnosis can you make that will direct your nursing intervention? (2) a. Parental anxiety r/t respiratory distress in child b. Child fear r/t asthma c. Impaired breathing mechanisms r/t bronschospasm d. Fatigue r/t respiratory distress 33. As a nurse, what body organs and sense can you utilize in rural areas that can crudely assess presence of asthma in children? (2) a. heart, eyes and ears b. eyes and mind c. eyes, ears and touch d. ears and heart 34. In case of asthmatic attack, what position can a nurse advise patients to take? (1) a. Semi – fowler’s b. Sitting c. Lying down in bed d. Prone lying

35. Of what use is the Peak Expiratory Low Rate (PEFR) as a monitoring device for nurses? (3) a. Guide to respiratory therapy with medications b. Help in planning of an appropriate therapeutic regimen c. Monitor breathing capacity d. Know adequate transfer of gases across alveolar capillaries Situation 8: Fear and anxiety are conditions that bring about acid-base imbalances. 36. What is the primary nursing responsibility when a patient presents respiratory distress? (2) a. Get ready with complete ECG cart at the side b. Life-threatening measures are readied c. History taking and keen assessment of respiratory problem d. Positioning and oxygen therapy on hand 37. When a patient is fearful and anxious, what is the condition indicative of? (3) a. Respiratory alkalosis b. Metabolic acidosis c. Metabolic alkalosis d. Respiratory acidosis 38. When a patient shows manifestation of an acid-base imbalance, what is the nursing responsibility? (2) a. Explain procedure and protocol of care b. Take arterial blood gases every hour c. Assess respiratory and neurologic status every 2-4 hours d. Administer medication to help ventilation 39. Chow, 2-year-old has difficulty of breathing without any previous cause. What can a nurse do at this very moment? (2) a. Positional nursing care – head part higher b. Administer oxygen and fluids c. Hook to mechanical ventilation d. Monitor intake and output 40. If Chow asks for food and drink, which among the following will you give him? (3) a. Soft drinks b. Ice cream c. Hot milk and crackers d. Warm congee Situation 9: “Acute Respiratory Infection (ARI) especially pneumonia leads as the cause of illness and death among Filipino children”, claims the Department of Health. 41. Which of the following is the main factor that contributes to the problem of getting sick of pneumonia among children below 5 years old? (3) a. Poor follow-up compliance to treatment b. Lack of advocates to gain local government support c. Inability of health worker to refer immediate treatment

d. Failure of mother to recognize early signs and symptoms 42. What is the program mandated to reduced mortality of acute respiratory illness? (1) a. Pneumonia control program b. Stop pneumonia c. Control of ARI (CARI) d. “Ask, Look and Listen” 43. Which classification of pneumonia has the child if 3 or more danger signs are present? (2) a. Severe b. Pneumonia c. No Pneumonia d. Very severe 44. When the child is to receive antibiotic, where should the first dose be given? (1) a. In the school b. In the health center c. In the home d. In the referral system 45. Which of the following is your topic for health teaching to mothers of children with pneumonia? (2) a. Breastfeeding b. Family planning c. Use of Assess Card d. Play therapy Situation 10: The Tochan family is in crisis situation. Mr. Tochan, 60-year-old has emphysema and is in ZMDH. He was admitted last September 5, 2006. His wife Vina, 50-year-old is taking care of her husband in the hospital. He is under close observation with O2 inh/NC @ 2LPM. 46. Upon auscultation, rales are heard in Mr. Tochan’s left lower lung segment. One of the orders in the care and treatment of Mr. Tochan was postural drainage once a day. To perform the procedure it would involve placing Mr. Tochan on: (2) a. back with a pillow under his hips b. left side with a pillow under his hips c. right side with a pillow under his hips d. abdomen with a pillow under his chest 47. Mr. Tochan’s condition resulted in COPD. His blood pH is 7.33 and he is restless. Accordingly, her nursing care plan should include: (3) a. increasing his O2 flow rate b. removing his secretions from his respiratory tract c. limiting his fluid intake d. administering hypnotics as ordered 48. Most of the time Mr. Tochan is allowed to rest. At 12 noon CBC & ABG were done. The results showed PaO2 - 92%, PCO2 - 46, RBC- 4.8 mx10, WBC - 11000, Hgb.12g/dl. The best nursing action would be: (2) a. monitor Mr. Tochan

b. Increase O2 inh c. call the MD d. start an antibiotic 449. He complained of inability to produce sputum. “I feel I have something in my lungs that I need to cough out.” Nurse Pia would be most helpful if she: (2) a. calls the physician b. administer mucolytic c. give health teaching d. limit fluid intake 50. Chest physiotherapy and deep breathing exercises were encouraged for Mr. Tochan. When is the most appropriate time to perform such procedure? (1) a. after lunch, before napping b. before breakfast c. after breakfast, before am care d. time element is not important Situation 11: At around 7:30 am the following day, Nurse Pia endorsed Mr. Tochan to Nurse Vega as having a temperature reading of 38.8ºC/ax. She also endorsed that he’d been restless during the night. 51. The most appropriate nursing action would be: (3) a. check vital signs b. call the physician c. administer cooling measures d. administer antipyretics 52. One of the nurses came at the bedside of Mr. Tochan and states: “You look like you are having difficulty of breathing.” The nurse’s statement is: (3) a. appropriate because difficulty of breathing is expected from COPD b. appropriate because it states what the nurse is observing c. inappropriate because the nurse made a conclusion without validating d. inappropriate because the nurse should wait for the client to speak first 53. Later that day, Mr. Tochan had bouts of productive cough. The most effective infection control is for the nurse to: (1) a. monitor the temperature b. push oral fluids c. have the client cover his mouth when coughing d. do not allow visitors for the client 54. In respiratory infection, the sputum is highly contagious. In the chain of infection the sputum is: (1) a. portal of entry b. infectious agent c. reservoir d. portal of exit 55. The following conversation took place at Mr. Tochan’s bedside while the morning shift nurse was making her rounds: Nurse: “Mr. Tochan, I will be teaching you deep breathing exercises.”

Tochan: “I would prefer that we wait for my wife. She knows what to do.” Nurse: “You should not rely on your wife. I will show you how to do it effectively.” The nurse’s last statement is: (2) a. displaying a value of judgment b. appropriate because it encourage independence c. the client must realize that the wife has other things to do d. inappropriate because patients are always right Situation 12: The Department of Health promotes use of herbal drugs. As a public health nurse, you implement the program on traditional medicine in the community. 56. To promote the use of herbal medicines, which of the following projects would you encourage the people in the community to do? (1) a. Backyard herbal gardening b. Plant a tree today c. Save Mother Earth d. Clean and Green 57. Which of the following herbal plants is used for respiratory problems such as asthma, cough and fever? (1) a. Lagundi b. Sambong c. Niyog-niyogan d. Yerba Buena 58. Which of the following aromatic herbs for body pain, rheumatism and arthritis is used by older persons? (1) a. Sambong b. Yerba Buena c. Carmona-rosa d. Alusimang Bato 59. Which of the following herbal plants is used for mild non-insulin dependent diabetes mellitus? (1) a. Alusimang Bato b. Bawang c. Carmona-rosa d. Ampalaya 60. In the use of herbal medicines, which of the following statement is incorrect? (2) a. Avoid the use of insecticides as these may leave poison on plants b. Use only the part of the plant being advocated c. In preparation, use a clay pot and cover it while boiling at low heat d. Follow accurate dose of suggested preparation Situation 13: Mr. Ang Lee has chronic cough and dyspnea. Her physician made a diagnosis of Acute Pulmonary Emphysema. 61. He has dyspnea with mild exertion. What is the probable cause of this? (2) a. Impaired diffusion between the alveolar air and blood

b. Thrombic obstruction of pulmonary arterioles c. Decrease tone of the diaphragm d. Lowered oxygen carrying capacity of the RBC 62. Which of the following tissue changes is a characteristic of emphysema? (2) a. Overdistention, inelasticity and rupture of alveoli b. Accumulation of pus in the pleural space c. Filling of air passage by inflammatory alveoli d. Accumulation of fluids in the pleural sac 63. While waiting for the resident-on-duty to perform the physical examination, Mr. Lee would be most comfortable in which position? (1) a. Sitting on edge of bed b. Lying flat on bed c. Reclining in his left side d. Supine with head elevated 64. The primary objective of your nursing care management for him is to improve her quality of life. Which of the following would not be included in your therapeutic approach? (3) a. Prevention of infection b. Prompt treatment of infection c. Providing supportive care d. Maintenance of a conducive environment 65. Which of the following hygienic care would be most appropriate for Mr. Lee? (2) a. Proper care of finger and toenails b. Not allowing him to have daily baths c. Checking the VS every four hours d. Providing oral care at least three times a day 5Situation 14: Mrs. Meow, 46-year-old with asthmatic attack is admitted in the medical ward of Rico Hospital. 66. Your finding in your assessment would include the following, except: (1) a. Ability only to speak a few words without taking a deep breath b. Tachycardia, cool and moist skin c. Air hunger and presence of wheezing sounds d. Tachycardia, warm and moist skin 67. With your assessment, which of these symptoms would you expect to develop later? (2) a. Nasal flaring b. Lips pursed in an effort to exhale c. Cyanosis d. Use of accessory muscles for breathing 68. Which has the fewer tendencies to precipitate or trigger asthmatic attack? (1) a. Air pollution b. Cold weather c. Changes in climate d. Mold, house dust

69. The least of nursing care that you would do with Mrs. Meow is to: (2) a. Give Bricanyl tablet to ease breathing b. Keep his back always dry c. Keep the siderails up at all times d. Force fluids to liquefy the secretions 70. The most comfortable position for Mrs. Meow to assume during asthmatic attack is: (1) a. Sitting b. Orthopneic c. Fowler’s d. Supine Situation 15: Mr. Kaldero, age 38, is referred to the local hospital clinic after his tuberculin skin test was found to be positive. He is admitted for further diagnosis and evaluation. Medications ordered for Mr. Cordero are 300 mg Isoniazid (INH) p.o. daily, 300 mg Rifampicin p.o. daily, 100 mg Pyridoxine (Vitamin B6) p.o. daily, regular diet and bed rest. 71. Which would most likely confirm Mr. Kaldero’s diagnosis of tuberculosis? (1) a. Creatinine kinase test b. Chest x-ray c. Sputum smear and culture d. White blood cell count 72. Which clinical manifestations would the nurse expect in a patient with TB? (2) a. Hemoptysis and weight gain b. Productive cough and afternoon elevated temp c. Dry cough and blood streaked sputum d. Night sweats and urticaria 73. Which nursing activity would be most therapeutic while Mr. Kaldero is on bed rest? (2) a. Encouraging family and friends to visit 3x a day b. Assisting him in walking to the lounge c. Assisting him with ROM exercises d. Encouraging him to visit other patients 74. Possible adverse effects of Isoniazid therapy include: (2) a. Peripheral neuritis, tachycardia, and insomnia vertigo b. Fever and GI dysfunction c. Hepatic dysfunction, headache and d. Hepatic dysfunction and kidney damage 75. After a week in the hospital, Mr. Kaldero is ready for discharge. When providing discharge instructions, the nurse should discuss all of the following, except: a. The plan for regular follow-up care b. The possible adverse effects of his medications c. The need to discontinue INH if nausea occurs d. The need to cover his nose and mouth when coughing

Situation 16: Reason Blade, R.N., is the staff nurse on duty at the Medical Ward of Aquinas University Hospital. 76. In Bed No. 1 is Mr. Monterey, a 50-year-old client with asthma. Your nursing management for him is: (2) a. Administer Alevaire inhalation to soften secretions b. Force fluids c. Administer Bricanyl d. Give fruit juice 77. To ease his breathing, which position would make him most comfortable? (1) a. Dorsal recumbent b. Sim’s c. Orthopneic d. Fowler’s 78. Mr. Tatad, who is in Bed No. 3 is suffering from COPD. You informed him that the most effective bronchodilator is: (2) a. Lukewarm lemonade b. Deep breathing d. Steam inhalation d. Mild mucolytic agent 79. What diet is best recommended for him considering his existing Valsalva maneuver problem? (2) a. Full liquid diet b. Bland diet c. High in fiber and bulk d. Soft, high in protein 80.Which of the following will not promote effective clearing of Mr. Tatad’s tracheobronchial secretions? (2) a. Assuming postural drainage b. Administering Alevaire medications c. Doing deep breathing exercises every 2 hours d. Doing coughing technique effectively Situation 17: In a developing country like the Philippines, accumulation of fluid in the pleural cavity commonly results from tuberculosis. 81. Anatomically, the inner part of the thoracic cavity is lined by the parietal membrane, while the membrane that envelops the lung is called: (1) a. visceral membrane b. cell membrane c. plasma membrane d. pulmonary membrane 82. Mang Jose is prepared for CTT. Nurse A would know that CTT stands for: (2) a. Chest Tube Thoracostomy b. Central Thoracic Test c. Critical Terminal Treatment

d. Close Tube Thoracostomy 83. The correct position of Mang Jose to assume during CTT is: (1) a. supine position b. sitting on a chair, leaning on the back rest c. high fowler’s with arm of affected side above the head d. side lying on high fowler’s at the side of the bed 84. During the rounds, Nurse A noticed that the chest tube was accidentally removed by the client. The best appropriate nursing action Nurse A should take is: (3) a. shout for help b. reinsert the chest tube immediately c. apply vaselinized sterile gauze and pressure dressing and notify surgeon immediately d. ignore it, the client can reinsert it by himself 685. Nursing responsibilities in caring for patients with CTT are the following except: (3) a. ensure that the drainage bottle is at the level of the patient’s chest b. monitor water-seal drainage bottle to ensure fluid level is above drain tube c. coil tubing carefully to avoid kinking d. prepare two clamps at bedside and take with patient when brought out of the room Situation 18: Calamares, 25-year-old, employee was brought to the ER because of severe allergic reaction. She complained of difficulty of breathing. Oxygen inhalation 3L was ordered stat. Intravenous solution started. 86. Which of the following physiologic needs has the highest priority for Calamares? (1) a. Fluid b. Nutrition c. Oxygen d. Low Temperature 87. A symptomatic patient like Calamares would consider which of the following as her most important needs? (2) a. Relief from her health problem b. Assistance with family and financial responsibilities c. Understanding of her personal concern d. Solution for the office problem 88. Illness prevention activities are generally designed to help client attain which of the following? (2) a. Promote habits related to good health b. Identify disease symptoms c. Manage stress d. Hospitalization 89. Nurse Lavigña provided Calamares and her family with information regarding the client’s care. This constitute as: (1) a. Patient advocate role b. Surrogate role c. Educator role d. Counselor role

90. Which of the following is the most important precautionary measure in administering oxygen inhalation to be observed that will benefit the client in particular and hospital in general? (2) a. Setting up a bottle of sterile water to humidify the oxygen before it is administered to client b. “Crack” the oxygen tank before it is wheeled/brought to client’s room c. Lubricate the tip of the catheter with mineral oil that is water soluble d. Hang a “No Smoking” sign on the oxygen tank and a similar warning at the door. Situation 19: Mr. Murdocks, a 65-year-old retired steel mill worker, is admitted to the unit with dyspnea-upon-exertion. He has a long history of smoking. Initial assessment findings include barrel chest, ankle edema, persistent cough with copious sputum production and variable wheezing on expiration. 91. Mr. Murdocks’ ankle edema and respiratory problems should make the nurse suspect hypertrophy of which heart chamber? (2) a. Left atrium b. Right atrium c. Left ventricle d. Right ventricle 92. The physician orders an Aminophylline IV drip for Mr. Murdocks. The nurse should be alert for which sign of drug toxicity? (2) a. Depression b. Lethargy c. Tachycardia d. Cyanosis 93. ABG measurements reveal a ph of 7.25, PaCO2 of 52 mmHg and a HCO3 level of 25 mEq/L. The result indicates what acid-base imbalance? (2) a. Respiratory alkalosis, uncompensated b. Respiratory acidosis, uncompensated c. Metabolic alkalosis, compensated d. Metabolic acidosis, compensated 94. Mr. Murdocks is unable to exhale efficiently and becomes short of breath. The best nursing intervention would be, to teach him: (1) a. Pursed lip breathing b. Coughing technique c. Postural drainage d. Relaxation technique 95. The physician orders postural drainage. Which statement about postural drainage is most accurate? (2) a. Postural drainage uses gravity to augment mucociliary clearing mechanisms and drain retained secretions b. All patients with COPD are positioned the same way during postural drainage c. Postural drainage involves rhythmic clapping of the chest wall with cupped hands d. postural drainage is effective only when performed for 1 hour or longer

Situation 20: Marisse, a BSN student was assigned in the medical ward. She is to administer medication under the supervision of her clinical instructor. 96. When administering drugs, the nurse compares the label of the drug container with the medicine card correctly except: (2) a. Before removing the container from the drawer or shelf b. As the amount of drug ordered is removed from it c. Before resuming the container to the storage d. Before directly administering the drug 97. What is the best way of identifying the right client ideally? (1) a. Check the medicine tickets against the client’s identification b. Nurse speaks the name of the client c. Nurse ask the client’s relative to state the full name d. Nurse consults the physician 98. If the doctors orders q8h, what does this mean? (1) a. The medication should be given during the waking hours b. The medication should be given round the clock c. Both a and b d. None of the above 99. The expectorant guiafenesin (Robitussin) 300 mg p.o. has been ordered. The bottle is labeled 100mg/5mL. How many mL should be given? (2) a. 13 mL b. 14 mL c. 15 mL d. 16 mL 100. The physician orders: Administer Ampicillin 50 mg. oral suspension p.o q6 hours for 7 days. Stock dose is 125 mg/5mL in 30 mL bottle. How many bottles of the medication will you request? (3) a. 5 bottles b. 4 bottles c. 3 bottles d. 2 bottles ANSWER KEYS: 1. B 2. A 3. A 4. D 5. C 6. C 7. C 8. A 9. B 10. C 11. D 12. C 13. B

14. B 15. B 16. A 17. A 18. A 19. C 20. A 21. D 22. B 23. C 24. C 25. A 26. D 27. A 28. A 29. B 30. A 31. C 32. B 33. A 34. C 35. C 36. C 37. C 38. C 39. C 40. C 41. D 42. C 43. A 44. A 45. A 46. A 47. C 48. B 49. D 50. D 51. D 52. A 53. C 54. D 55. C 56. A 57. A 58. B 59. C

60. D 61. C 62. A 63. C 64. A 65. A 66. D 67. A 68. B 69. C 70. A 71. D 72. B 73. A 74. A 75. A 76. A 77. C 78. D 79. C 80. B 81. A 82. D 83. D 84. B 85. B 86. C 87. D 88. B 89. C 90. A 91. B 92. C 93. A 94. D 95. A 96. C 97. A 98. D 99. C 100. C /end of nursing practice (integrated examination) This material may not be published, broadcast, rewritten or redistributed. Thank you. BREWED

75 items MS Random questions 1. The nursing care plan for a toddler diagnosed with Kawasaki Disease (mucocutaneous lymph node syndrome) should be based on the high risk for development of which problem? A) Chronic vessel plaque formation B) Pulmonary embolism C) Occlusions at the vessel bifurcations D) Coronary artery aneurysms 2. A nurse has just received a medication order which is not legible. Which statement best reflects assertive communication? A) "I cannot give this medication as it is written. I have no idea of what you mean." B) "Would you please clarify what you have written so I am sure I am reading it correctly?" C) "I am having difficulty reading your handwriting. It would save me time if you would be more careful." D) "Please print in the future so I do not have to spend extra time attempting to read your writing." 3. The nurse is discussing negativism with the parents of a 30 month-old child. How should the nurse tell the parents to best respond to this behavior? A) Reprimand the child and give a 15 minute "time out" B) Maintain a permissive attitude for this behavior C) Use patience and a sense of humor to deal with this behavior D) Assert authority over the child through limit setting 4. An ambulatory client reports edema during the day in his feet and an ankle that disappears while sleeping at night. What is the most appropriate follow-up question for the nurse to ask? A) "Have you had a recent heart attack?" B) "Do you become short of breath during your normal daily activities?" C) "How many pillows do you use at night to sleep comfortably?" D) "Do you smoke?" 5. The nurse is planning care for a client during the acute phase of a sickle cell vasoocclusive crisis. Which of the following actions would be most appropriate? A) Fluid restriction 1000cc per day B) Ambulate in hallway 4 times a day C) Administer analgesic therapy as ordered D) Encourage increased caloric intake 6. While working with an obese adolescent, it is important for the nurse to recognize that obesity in adolescents is most often associated with what other behavior?

A) Sexual promiscuity B) Poor body image C) Dropping out of school D) Drug experimentation 7. A nurse and client are talking about the client’s progress toward understanding his behavior under stress. This is typical of which phase in the therapeutic relationship? A) Pre-interaction B) Orientation C) Working D) Termination 8. A nurse is eating in the hospital cafeteria when a toddler at a nearby table chokes on a piece of food and appears slightly blue. The appropriate initial action should be to A) Begin mouth to mouth resuscitation B) Give the child water to help in swallowing C) Perform 5 abdominal thrusts D) Call for the emergency response team 9. The emergency room nurse admits a child who experienced a seizure at school. The father comments that this is the first occurrence, and denies any family history of epilepsy. What is the best response by the nurse? A) "Do not worry. Epilepsy can be treated with medications." B) "The seizure may or may not mean your child has epilepsy." C) "Since this was the first convulsion, it may not happen again." D) "Long term treatment will prevent future seizures." 10. A nurse admits a 3 week-old infant to the special care nursery with a diagnosis of bronchopulmonary dysplasia. As the nurse reviews the birth history, which data would be most consistent with this diagnosis? A) Gestational age assessment suggested growth retardation B) Meconium was cleared from the airway at delivery C) Phototherapy was used to treat Rh incompatibility D) The infant received mechanical ventilation for 2 weeks 11. Parents of a 6 month-old breast fed baby ask the nurse about increasing the baby's diet. Which of the following should be added first? A) Cereal B) Eggs C) Meat D) Juice 12. A victim of domestic violence states, "If I were better, I would not have been beat." Which feeling best describes what the victim may be experiencing? A) Fear B) Helplessness

C) Self-blame D) Rejection 13. The nurse is assessing the mental status of a client admitted with possible organic brain disorder. Which of these questions will best assess the function of the client's recent memory? A) "Name the year." "What season is this?" (pause for answer after each question) B) "Subtract 7 from 100 and then subtract 7 from that." (pause for answer) "Now continue to subtract 7 from the new number." C) "I am going to say the names of three things and I want you to repeat them after me: blue, ball, pen." D) "What is this on my wrist?" (point to your watch) Then ask, "What is the purpose of it?" 14. Which oxygen delivery system would the nurse apply that would provide the highest concentrations of oxygen to the client? A) Venturi mask B) Partial rebreather mask C) Non-rebreather mask D) Simple face mask 15. A nurse is caring for a client who had a closed reduction of a fractured right wrist followed by the application of a fiberglass cast 12 hours ago. Which finding requires the nurse’s immediate attention? A) Capillary refill of fingers on right hand is 3 seconds B) Skin warm to touch and normally colored C) Client reports prickling sensation in the right hand D) Slight swelling of fingers of right hand 16. Included in teaching the client with tuberculosis taking INH about follow-up home care, the nurse should emphasize that a laboratory appointment for which of the following lab tests is critical? A) Liver function B) Kidney function C) Blood sugar D) Cardiac enzymes 17. Which client is at highest risk for developing a pressure ulcer? A) 23 year-old in traction for fractured femur B) 72 year-old with peripheral vascular disease, who is unable to walk without assistance C) 75 year-old with left sided paresthesia and is incontinent of urine and stool D) 30 year-old who is comatose following a ruptured aneurysm 18. Which contraindication should the nurse assess for prior to giving a child immunization? A) Mild cold symptoms

B) Chronic asthma C) Depressed immune system D) Allergy to eggs 19. The nurse is caring for a 2 year-old who is being treated with chelation therapy, calcium disodium edetate, for lead poisoning. The nurse should be alert for which of the following side effects? A) Neurotoxicity B) Hepatomegaly C) Nephrotoxicity D) Ototoxicity 20. A newborn is having difficulty maintaining a temperature above 98 degrees Fahrenheit and has been placed in a warming isolette. Which action is a nursing priority? A) Protect the eyes of the neonate from the heat lamp B) Monitor the neonate’s temperature C) Warm all medications and liquids before giving D) Avoid touching the neonate with cold hands 21. At a senior citizens meeting a nurse talks with a client who has diabetes mellitus Type 1. Which statement by the client during the conversation is most predictive of a potential for impaired skin integrity? A) "I give my insulin to myself in my thighs." B) "Sometimes when I put my shoes on I don't know where my toes are." C) "Here are my up and down glucose readings that I wrote on my calendar." D) "If I bathe more than once a week my skin feels too dry." 22. A 4 year-old hospitalized child begins to have a seizure while playing with hard plastic toys in the hallway. Of the following nursing actions, which one should the nurse do first? A) Place the child in the nearest bed B) Administer IV medication to slow down the seizure C) Place a padded tongue blade in the child's mouth D) Remove the child's toys from the immediate area 23. The nurse is at the community center speaking with retired people. To which comment by one of the retirees during a discussion about glaucoma would the nurse give a supportive comment to reinforce correct information? A) "I usually avoid driving at night since lights sometimes seem to make things blur." B) "I take half of the usual dose for my sinuses to maintain my blood pressure." C) "I have to sit at the side of the pool with the grandchildren since I can't swim with this eye problem." D) "I take extra fiber and drink lots of water to avoid getting constipated.”

24. The nurse is teaching a parent about side effects of routine immunizations. Which of the following must be reported immediately? A) Irritability B) Slight edema at site C) Local tenderness D) Temperature of 102.5 F 25. A client is admitted with the diagnosis of pulmonary embolism. While taking a history, the client tells the nurse he was admitted for the same thing twice before, the last time just 3 months ago. The nurse would anticipate the health care provider ordering A) Pulmonary embolectomy B) Vena caval interruption C) Increasing the coumadin therapy to an INR of 3-4 D) Thrombolytic therapy 26. A woman in her third trimester complains of severe heartburn. What is appropriate teaching by the nurse to help the woman alleviate these symptoms? A) Drink small amounts of liquids frequently B) Eat the evening meal just before retiring C) Take sodium bicarbonate after each meal D) Sleep with head propped on several pillows 27. The nurse is teaching the mother of a 5 month-old about nutrition for her baby. Which statement by the mother indicates the need for further teaching? A) "I'm going to try feeding my baby some rice cereal." B) "When he wakes at night for a bottle, I feed him." C) "I dip his pacifier in honey so he'll take it." D) "I keep formula in the refrigerator for 24 hours." 28. For a 6 year-old child hospitalized with moderate edema and mild hypertension associated with acute glomerulonephritis (AGN), which one of the following nursing interventions would be appropriate? A) Institute seizure precautions B) Weigh the child twice per shift C) Encourage the child to eat protein-rich foods D) Relieve boredom through physical activity 29. Which statement by the client with chronic obstructive lung disease indicates an understanding of the major reason for the use of occasional pursed-lip breathing? A) "This action of my lips helps to keep my airway open." B) "I can expel more when I pucker up my lips to breathe out." C) "My mouth doesn't get as dry when I breathe with pursed lips." D) "By prolonging breathing out with pursed lips the little areas in my lungs don't collapse."

30. A 57 year-old male client has hemoglobin of 10 mg/dl and a hematocrit of 32%. What would be the most appropriate follow-up by the home care nurse? A) Ask the client if he has noticed any bleeding or dark stools B) Tell the client to call 911 and go to the emergency department immediately C) Schedule a repeat Hemoglobin and Hematocrit in 1 month D) Tell the client to schedule an appointment with a hematologist 31. Which response by the nurse would best assist the chemically impaired client to deal with issues of guilt? A) "Addiction usually causes people to feel guilty. Don’t worry, it is a typical response due to your drinking behavior." B) "What have you done that you feel most guilty about and what steps can you begin to take to help you lessen this guilt?" C) "Don’t focus on your guilty feelings. These feelings will only lead you to drinking and taking drugs." D) "You’ve caused a great deal of pain to your family and close friends, so it will take time to undo all the things you’ve done." 32. An adolescent client comes to the clinic 3 weeks after the birth of her first baby. She tells the nurse she is concerned because she has not returned to her pre-pregnant weight. Which action should the nurse perform first? A) Review the client's weight pattern over the year B) Ask the mother to record her diet for the last 24 hours C) Encourage her to talk about her view of herself D) Give her several pamphlets on postpartum nutrition 33. Which of the following measures would be appropriate for the nurse to teach the parent of a nine month-old infant about diaper dermatitis? A) Use only cloth diapers that are rinsed in bleach B) Do not use occlusive ointments on the rash C) Use commercial baby wipes with each diaper change D) Discontinue a new food that was added to the infant's diet just prior to the rash 34. A 16 year-old client is admitted to a psychiatric unit with a diagnosis of attempted suicide. The nurse is aware that the most frequent cause for suicide in adolescents is A) Progressive failure to adapt B) Feelings of anger or hostility C) Reunion wish or fantasy D) Feelings of alienation or isolation 35. A mother brings her 26 month-old to the well-child clinic. She expresses frustration and anger due to her child's constantly saying "no" and his refusal to follow her directions. The nurse explains this is normal for his age, as negativism is attempting to meet which developmental need? A) Trust

B) Initiative C) Independence D) Self-esteem 36. Following mitral valve replacement surgery a client develops PVC’s. The health care provider orders a bolus of Lidocaine followed by a continuous Lidocaine infusion at a rate of 2 mgm/minute. The IV solution contains 2 grams of Lidocaine in 500 cc’s of D5W. The infusion pump delivers 60 microdrops/cc. What rate would deliver 4 mgm of Lidocaine/minute? A) 60 microdrops/minute B) 20 microdrops/minute C) 30 microdrops/minute D) 40 microdrops/minute 37. A couple asks the nurse about risks of several birth control methods. What is the most appropriate response by the nurse? A) Norplant is safe and may be removed easily B) Oral contraceptives should not be used by smokers C) Depo-Provera is convenient with few side effects D) The IUD gives protection from pregnancy and infection 38. The nurse is caring for a client in the late stages of Amyotrophic Lateral Sclerosis (A.L.S.). Which finding would the nurse expect? A) Confusion B) Loss of half of visual field C) Shallow respirations D) Tonic-clonic seizures 39. A client experiences post partum hemorrhage eight hours after the birth of twins. Following administration of IV fluids and 500 ml of whole blood, her hemoglobin and hematocrit are within normal limits. She asks the nurse whether she should continue to breast feed the infants. Which of the following is based on sound rationale? A) "Nursing will help contract the uterus and reduce your risk of bleeding." B) "Breastfeeding twins will take too much energy after the hemorrhage." C) "The blood transfusion may increase the risks to you and the babies." D) "Lactation should be delayed until the "real milk" is secreted." 40. A client complained of nausea, a metallic taste in her mouth, and fine hand tremors 2 hours after her first dose of lithium carbonate (Lithane). What is the nurse’s best explanation of these findings? A) These side effects are common and should subside in a few days B) The client is probably having an allergic reaction and should discontinue the drug C) Taking the lithium on an empty stomach should decrease these symptoms D) Decreasing dietary intake of sodium and fluids should minimize the side effects

41. The nurse is caring for a post-surgical client at risk for developing deep vein thrombosis. Which intervention is an effective preventive measure? A) Place pillows under the knees B) Use elastic stockings continuously C) Encourage range of motion and ambulation D) Massage the legs twice daily 42. The parents of a newborn male with hypospadias want their child circumcised. The best response by the nurse is to inform them that A) Circumcision is delayed so the foreskin can be used for the surgical repair B) This procedure is contraindicated because of the permanent defect C) There is no medical indication for performing a circumcision on any child D) The procedure should be performed as soon as the infant is stable 43. The nurse is teaching parents about the treatment plan for a 2 weeks-old infant with Tetralogy of Fallot. While awaiting future surgery, the nurse instructs the parents to immediately report A) Loss of consciousness B) Feeding problems C) Poor weight gain D) Fatigue with crying 44. An infant weighed 7 pounds 8 ounces at birth. If growth occurs at a normal rate, what would be the expected weight at 6 months of age? A) Double the birth weight B) Triple the birth weight C) Gain 6 ounces each week D) Add 2 pounds each month 45. The nurse is caring for a 13 year-old following spinal fusion for scoliosis. Which of the following interventions is appropriate in the immediate post-operative period? A) Raise the head of the bed at least 30 degrees B) Encourage ambulation within 24 hours C) Maintain in a flat position, logrolling as needed D) Encourage leg contraction and relaxation after 48 hours 46. A client asks the nurse about including her 2 and 12 year-old sons in the care of their newborn sister. Which of the following is an appropriate initial statement by the nurse? A) "Focus on your sons' needs during the first days at home." B) "Tell each child what he can do to help with the baby." C) "Suggest that your husband spend more time with the boys." D) "Ask the children what they would like to do for the newborn." 47. A nurse is caring for a 2 year-old child after corrective surgery for Tetralogy of Fallot. The mother reports that the child has suddenly begun seizing. The nurse recognizes this problem is probably due to

A) A cerebral vascular accident B) Postoperative meningitis C) Medication reaction D) Metabolic alkalosis 48. A client with schizophrenia is receiving Haloperidol (Haldol) 5 mg t.i.d.. The client’s family is alarmed and calls the clinic when "his eyes rolled upward." The nurse recognizes this as what type of side effect? A) Oculogyric crisis B) Tardive dyskinesia C) Nystagmus D) Dysphagia 49. A home health nurse is at the home of a client with diabetes and arthritis. The client has difficulty drawing up insulin. It would be most appropriate for the nurse to refer the client to A) A social worker from the local hospital B) An occupational therapist from the community center C) A physical therapist from the rehabilitation agency D) Another client with diabetes mellitus and takes insulin 50. A client was admitted to the psychiatric unit after complaining to her friends and family that neighbors have bugged her home in order to hear all of her business. She remains aloof from other clients, paces the floor and believes that the hospital is a house of torture. Nursing interventions for the client should appropriately focus on efforts to A) Convince the client that the hospital staff is trying to help B) Help the client to enter into group recreational activities C) Provide interactions to help the client learn to trust staff D) Arrange the environment to limit the client’s contact with other clients 51. A client is scheduled for a percutaneous transluminal coronary angioplasty (PTCA). The nurse knows that a PTCA is the A) Surgical repair of a diseased coronary artery B) Placement of an automatic internal cardiac defibrillator C) Procedure that compresses plaque against the wall of the diseased coronary artery to improve blood flow D) Non-invasive radiographic examination of the heart 52. A newborn has been diagnosed with hypothyroidism. In discussing the condition and treatment with the family, the nurse should emphasize A) They can expect the child will be mentally retarded B) Administration of thyroid hormone will prevent problems C) This rare problem is always hereditary D) Physical growth/development will be delayed 53. A priority goal of involuntary hospitalization of the severely mentally ill client is

A) Re-orientation to reality B) Elimination of symptoms C) Protection from harm to self or others D) Return to independent functioning 54. A 19 year-old client is paralyzed in a car accident. Which statement used by the client would indicate to the nurse that the client was using the mechanism of "suppression"? A) "I don't remember anything about what happened to me." B) "I'd rather not talk about it right now." C) "It's the other entire guy's fault! He was going too fast." D) "My mother is heartbroken about this." 55. The nurse is caring for a woman 2 hours after a vaginal delivery. Documentation indicates that the membranes were ruptured for 36 hours prior to delivery. What are the priority nursing diagnoses at this time? A) Altered tissue perfusion B) Risk for fluid volume deficit C) High risk for hemorrhage D) Risk for infection 56. A 3 year-old had a hip spica cast applied 2 hours ago. In order to facilitate drying, the nurse should A) Expose the cast to air and turn the child frequently B) Use a heat lamp to reduce the drying time C) Handle the cast with the abductor bar D) Turn the child as little as possible 57. A client is scheduled for an Intravenous Pyelogram (IVP). In order to prepare the client for this test, the nurse would: A) Instruct the client to maintain a regular diet the day prior to the examination B) Restrict the client's fluid intake 4 hours prior to the examination C) Administer a laxative to the client the evening before the examination D) Inform the client that only 1 x-ray of his abdomen is necessary 58. Following a diagnosis of acute glomerulonephritis (AGN) in their 6 year-old child, the parent’s remark: “We just don’t know how he caught the disease!” The nurse's response is based on an understanding that A) AGN is a streptococcal infection that involves the kidney tubules B) The disease is easily transmissible in schools and camps C) The illness is usually associated with chronic respiratory infections D) It is not "caught" but is a response to a previous B-hemolytic strep infection 59. The nurse is caring for a 20 lbs (9 kg) 6 month-old with a 3 day history of diarrhea, occasional vomiting and fever. Peripheral intravenous therapy has been initiated, with 5% dextrose in 0.33% normal saline with 20 mEq of potassium per liter infusing at 35 ml/hr. Which finding should be reported to the health care provider immediately?

A) 3 episodes of vomiting in 1 hour B) Periodic crying and irritability C) Vigorous sucking on a pacifier D) No measurable voiding in 4 hours 60. While caring for the client during the first hour after delivery, the nurse determines that the uterus is boggy and there is vaginal bleeding. What should be the nurse's first action? A) Check vital signs B) Massage the fundus C) Offer a bedpan D) Check for perineal lacerations 61. The nurse is assessing an infant with developmental dysplasia of the hip. Which finding would the nurse anticipate? A) Unequal leg length B) Limited adduction C) Diminished femoral pulses D) Symmetrical gluteal folds 62. To prevent a valsalva maneuver in a client recovering from an acute myocardial infarction, the nurse would A) Assist the client to use the bedside commode B) Administer stool softeners every day as ordered C) Administer antidysrhythmics prn as ordered D) Maintain the client on strict bed rest 63. On admission to the psychiatric unit, the client is trembling and appears fearful. The nurse’s initial response should be to A) Give the client orientation materials and review the unit rules and regulations B) Introduce him/her and accompany the client to the client’s room C) Take the client to the day room and introduce her to the other clients D) Ask the nursing assistant to get the client’s vital signs and complete the admission search 64. During the admission assessment on a client with chronic bilateral glaucoma, which statement by the client would the nurse anticipate since it is associated with this problem? A) "I have constant blurred vision." B) "I can't see on my left side." C) "I have to turn my head to see my room." D) "I have specks floating in my eyes." 65. A client with asthma has low pitched wheezes present on the final half of exhalation. One hour later the client has high pitched wheezes extending throughout exhalation. This change in assessment indicates to the nurse that the client A) Has increased airway obstruction

B) Has improved airway obstruction C) Needs to be suctioned D) Exhibits hyperventilation 66. Which behavioral characteristic describes the domestic abuser? A) Alcoholic B) Over confident C) High tolerance for frustrations D) Low self-esteem 67. The nurse is caring for a client with a long leg cast. During discharge teaching about appropriate exercises for the affected extremity, the nurse should recommend A) Isometric B) Range of motion C) Aerobic D) Isotonic 68. A client is in her third month of her first pregnancy. During the interview, she tells the nurse that she has several sex partners and is unsure of the identity of the baby's father. Which of the following nursing interventions is a priority? A) Counsel the woman to consent to HIV screening B) Perform tests for sexually transmitted diseases C) Discuss her high risk for cervical cancer D) Refer the client to a family planning clinic 69. A 16 month-old child has just been admitted to the hospital. As the nurse assigned to this child enters the hospital room for the first time, the toddler runs to the mother, clings to her and begins to cry. What would be the initial action by the nurse? A) Arrange to change client care assignments B) Explain that this behavior is expected C) Discuss the appropriate use of "time-out" D) Explain that the child needs extra attention 70. While planning care for a 2 year-old hospitalized child, which situation would the nurse expect to most likely affect the behavior? A) Strange bed and surroundings B) Separation from parents C) Presence of other toddlers D) Unfamiliar toys and games 71. While explaining an illness to a 10 year-old, what should the nurse keep in mind about the cognitive development at this age? A) They are able to make simple association of ideas B) They are able to think logically in organizing facts C) Interpretation of events originate from their own perspective D) Conclusions are based on previous experiences

72. The nurse is has just admitted a client with severe depression. From which focus should the nurse identify a prioriy nursing diagnosis? A) Nutrition B) Elimination C) Activity D) Safety 73. Which playroom activities should the nurse organize for a small group of 7 year-old hospitalized children? A) Sports and games with rules B) Finger paints and water play C) "Dress-up" clothes and props D) Chess and television programs 74. A client is discharged following hospitalization for congestive heart failure. The nurse teaching the family suggests they encourage the client to rest frequently in which of the following positions? A) High Fowler's B) Supine C) Left lateral D) Low Fowler's 75. The nurse is caring for a 10 year-old on admission to the burn unit. One assessment parameter that will indicate that the child has adequate fluid replacement is A) Urinary output of 30 ml per hour B) No complaints of thirst C) Increased hematocrit D) Good skin turgor around burn Answer Key http://pinoybsn.blogspot.com/2006/10/75-items-ms-random-questions-with.html 60 item Exam on CardioVascular Disorders 60 item Exam on CardioVascular Disorders Course Outline a. Basic Anatomy b. Coronary artery disease c. Myocardial infarction d. Heart failure e. Abdominal aortic aneurysm f. Cardiomyopathy g. Pharmacology

h. Laboratory values i. Diagnostic tools Choose the BEST answer 1. Which of the following arteries primarily feeds the anterior wall of the heart? a. Circumflex artery b. Internal mammary artery c. Left anterior descending artery d. Right coronary artery 2. When do coronary arteries primarily receive blood flow? a. During inspiration b. During diastole c. During expiration d. During systole 3. Which of the following illnesses is the leading cause of death in the US? a. Cancer b. Coronary artery disease c. Liver failure d. Renal failure 4. Which of the following conditions most commonly results in CAD? a. Atherosclerosis b. DM c. MI d. Renal failure 5. Atherosclerosis impedes coronary blood flow by which of the following mechanisms? a. Plaques obstruct the vein b. Plaques obstruct the artery c. Blood clots form outside the vessel wall d. Hardened vessels dilate to allow the blood to flow through 6. Which of the following risk factors for coronary artery disease cannot be corrected? a. Cigarette smoking b. DM c. Heredity d. HPN 7. Exceeding which of the following serum cholesterol levels significantly increases the risk of coronary artery disease? a. 100 mg/dl b. 150 mg/dl c. 175 mg/dl

d. 200 mg/dl 8. Which of the following actions is the first priority care for a client exhibiting signs and symptoms of coronary artery disease? a. Decrease anxiety b. Enhance myocardial oxygenation c. Administer sublignual nitroglycerin d. Educate the client about his symptoms 9. Medical treatment of coronary artery disease includes which of the following procedures? a. Cardiac catheterization b. Coronary artery bypass surgery c. Oral medication administration d. Percutaneous transluminal coronary angioplasty 10. Prolonged occlusion of the right coronary artery produces an infarction in which of he following areas of the heart? a. Anterior b. Apical c. Inferior d. Lateral 11. Which of the following is the most common symptom of myocardial infarction? a. Chest pain b. Dyspnea c. Edema d. Palpitations 12. Which of the following landmarks is the corect one for obtaining an apical pulse? a. Left intercostal space, midaxillary line b. Left fifth intercostal space, midclavicular line c. Left second intercostal space, midclavicular line d. Left seventh intercostal space, midclavicular line 13. Which of the following systems is the most likely origin of pain the client describes as knifelike chest pain that increases in intensity with inspiration? a. Cardiac b. Gastrointestinal c. Musculoskeletal d. Pulmonary 14. A murmur is heard at the second left intercostal space along the left sternal border. Which valve area is this? a. Aortic b. Mitral

c. Pulmonic d. Tricuspid 15. Which of the following blood tests is most indicative of cardiac damage? a. Lactate dehydrogenase b. Complete blood count c. Troponin I d. Creatine kinase 16. What is the primary reason for administering morphine to a client with myocardial infarction? a. To sedate the client b. To decrease the client's pain c. To decrease the client's anxiety d. To decrease oxygen demand on the client's heart 17. Which of the followng conditions is most commonly responsible for myocardial infarction? a. Aneurysm b. Heart failure c. Coronary artery thrombosis d. Renal failure 18. What supplemental medication is most frequently ordered in conjuction with furosemide (Lasix)? a. Chloride b. Digoxin c. Potassium d. Sodium 19. After myocardial infarction, serum glucose levels and free fatty acids are both increase. What type of physiologic changes are these? a. Electrophysiologic b. Hematologic c. Mechanical d. Metabolic 20. Which of the following complications is indicated by a third heart sound (S3)? a. Ventricular dilation b. Systemic hypertension c. Aortic valve malfunction d. Increased atrial contractions 21. After an anterior wall myocardial infarction, which of the following problems is indicated by auscultation of crackles in the lungs? a. Left-sided heart failure

b. Pulmonic valve malfunction c. Right-sided heart failure d. Tricuspid valve malfunction 22. Which of the following diagnostic tools is most commonly used to determine the location of myocardial damage? a. Cardiac catheterization b. Cardiac enzymes c. Echocardiogram d. Electrocardiogram 23. What is the first intervention for a client experiencing myocardial infarction? a. Administer morphine b. Administer oxygen c. Administer sublingual nitroglycerin d. Obtain an electrocardiogram 24. What is the most appropriate nursing response to a myocardial infarction client who is fearful of dying? a. "Tell me about your feeling right now." b. "When the doctor arrives, everything will be fine." c. "This is a bad situation, but you'll feel better soon." d. "Please be assured we're doing everything we can to make you feel better." 25. Which of the following classes of medications protects the ischemic myocardium by blocking catecholamines and sympathetic nerve stimulation? a. Beta-adrenergic blockers b. Calcium channel blockers c. Narcotics d. Nitrates 26. What is the most common complication of a myocardial infarction? a. Cardiogenic shock b. Heart failure c. Arrhythmias d. Pericarditis 27. With which of the following disorders is jugular vein distention most prominent? a. Abdominal aortic aneurysm b. Heart failure c. Myocardial infarction d. Pneumothorax 28. What position should the nurse place the head of the bed in to obtain the most accurate reading of jugular vein distention? a. High-fowler's

b. Raised 10 degrees c. Raised 30 degrees d. Supine position 29. Which of the following parameters should be checked before administering digoxin? a. Apical pulse b. Blood pressure c. Radial pulse d. Respiratory rate 30. Toxicity from which of the following medications may cause a client to see a green halo around lights? a. Digoxin b. Furosemide c. Metoprolol d. Enalapril 31. Which ofthe following symptoms is most commonly associated with left-sided heart failure? a. Crackles b. Arrhythmias c. Hepatic engorgement d. Hypotension 32. In which of the following disorders would the nurse expect to assess sacral eddema in bedridden client? a. DM b. Pulmonary emboli c. Renal failure d. Right-sided heart failure 33. Which of the following symptoms might a client with right-sided heart failure exhibit? a. Adequate urine output b. Polyuria c. Oliguria d. Polydipsia 34. Which of the following classes of medications maximizes cardiac performance in clients with heat failure by increasing ventricular contractility? a. Beta-adrenergic blockers b. Calcium channel blockers c. Diuretics d. Inotropic agents

35. Stimulation of the sympathetic nervous system produces which of the following responses? a. Bradycardia b. Tachycardia c. Hypotension d. Decreased myocardial contractility 36. Which of the following conditions is most closely associated with weight gain, nausea, and a decrease in urine output? a. Angina pectoris b. Cardiomyopathy c. Left-sided heart failure d. Right-sided heart failure 37. What is the most common cause of abdominal aortic aneurysm? a. Atherosclerosis b. DM c. HPN d. Syphilis 38. In which of the following areas is an abdominal aortic aneurysm most commonly located? a. Distal to the iliac arteries b. Distal to the renal arteries c. Adjacent to the aortic branch d. Proximal to the renal arteries 39. A pulsating abdominal mass usually indicates which of the following conditions? a. Abdominal aortic aneurysm b. Enlarged spleen c. Gastic distention d. Gastritis 40. What is the most common symptom in a client with abdominal aortic aneurysm? a. Abdominal pain b. Diaphoresis c. Headache d. Upper back pain 41. Which of the following symptoms usually signifies rapid expansion and impending rupture of an abdominal aortic aneurysm? a. Abdominal pain b. Absent pedal pulses c. Angina d. Lower back pain

42. What is the definitive test used to diagnose an abdominal aortic aneurysm? a. Abdominal X-ray b. Arteriogram c. CT scan d. Ultrasound 43. Which of the following complications is of greatest concern when caring for a preoperative abdominal aneurysm client? a. HPN b. Aneurysm rupture c. Cardiac arrythmias d. Diminished pedal pulses 44. Which of the following blood vessel layers may be damaged in a client with an aneurysm? a. Externa b. Interna c. Media d. Interna and Media 45. When assessing a client for an abdominal aortic aneurysm, which area of the abdomen is most commonly palpated? a. Right upper quadrant b. Directly over the umbilicus c. Middle lower abdomen to the left of the midline d. Midline lower abdomen to the right of the midline 46. Which of the following conditions is linked to more than 50% of clients with abdominal aortic aneurysms? a. DM b. HPN c. PVD d. Syphilis 47. Which of the following sounds is distinctly heard on auscultation over the abdominal region of an abdominal aortic aneurysm client? a. Bruit b. Crackles c. Dullness d. Friction rubs 48. Which of the following groups of symptoms indicated a ruptured abdominal aneurysm? a. Lower back pain, increased BP, decreased RBC, increased WBC b. Severe lower back pain, decreased BP, decreased RBC, increased WBC c. Severe lower back pain, decreased BP, decreased RBC, decreased WBC

d. Intermittent lower back pain, decreased BP, decreased RBC, increased WBC 49. Which of the following complications of an abdominal aortic repair is indicated by detection of a hematoma in the perineal area? a. Hernia b. Stage 1 pressure ulcer c. Retroperitoneal rupture at the repair site d. Rapid expansion of the aneurysm 50. Which hereditary disease is most closely linked to aneurysm? a. Cystic fibrosis b. Lupus erythematosus c. Marfan's syndrome d. Myocardial infarction 51. Which of the following treatments is the definitive one for a ruptured aneurysm? a. Antihypertensive medication administration b. Aortogram c. Beta-adrenergic blocker administration d. Surgical intervention 52. Which of the following heart muscle diseases is unrelated to other cardiovascular disease? a. Cardiomyopathy b. Coronary artery disease c. Myocardial infarction d. Pericardial Effusion 53. Which of the following types of cardiomyopathy can be associated with childbirth? a. Dilated b. Hypertrophic c. Myocarditis d. Restrictive 54. Septal involvement occurs in which type of cardiomyopathy? a. Congestive b. Dilated c. Hypertrophic d. Restrictive 55. Which of the following recurring conditions most commonly occurs in clients with cardiomyopathy? a. Heart failure b. DM c. MI d. Pericardial effusion

56. What is the term used to describe an enlargement of the heart muscle? a. Cardiomegaly b. Cardiomyopathy c. Myocarditis d. Pericarditis 57. Dyspnea, cough, expectoration, weakness, and edema are classic signs and symptoms of which of the following conditions? a. Pericarditis b. Hypertension c. Obliterative d. Restricitve 59. Which of the following cardiac conditions does a fourth heart sound (S4) indicate? a. Dilated aorta b. Normally functioning heart c. Decreased myocardial contractility d. Failure of the ventricle to eject all the blood during systole 60. Which of the following classes of drugs is most widely used in the treatment of cardiomyopathy? a. Antihypertensive b. Beta-adrenergic blockers c. Calcium channel blockers d. Nitrates Answer Key http://pinoybsn.blogspot.com/2006/09/60-item-exam-on-cardiovascular_26.html 100 items OB questions by: Jeddah Obstetrics Nursing Questions BY: JEDDAH Answer keys will be posted after one week but rationale will take longer (lol dahil tamad ako!). Please free to post comments and you can answer it. Questions are both NLE and Foreign exam based. Good Luck!! Enjoy answering! Select the best answer 1. Mr. and Mrs. Cremasteric arrive at the clinic for their first pre natal visit. Mr. Crema tells the nurse that the women in his family usually have girl babies first and wonders why some women always have girls. The nurse correct response is: a. “The sex of the baby is determined by the sperm.”

b. “Some women are just more fertile with females.” c. “Nature determines whether the baby will be a girl or a boy.” d. “The sex of the baby is determined by the egg.” 2. The hormone responsible for the development of the ovum during the menstrual cycle is? a. Estrogen b. Progesterone c. Follicle Stimulating hormone (FSH) d. Luteneizing hormone (LH) 3. Which hormone is not responsible for differentiation of male reproductive organs during fetal life? a. Mullerian duct inhibitor (MDI) b. Dyhydrotestosterone c. Dehydroepiandosterone sulfate d. Testosterone 4. Which principal factor causes vaginal pH to be acidic? a. Cervical mucus changes b. Secretion of the Skene’s gland c. The action of the doderlein bacillus d. Secretion of the bartholins gland 5. Family centered nursing care for women and newborn focuses on which of the following? a. Assisting individuals and families achieve their optimal health b. Diagnosing and treating problems promptly c. Preventing further complications from developing d. Conducting nursing research to evaluate clinical skills 6. When reviewing the ethical dilemmas facing maternal and newborn nurses today, which of the following has contributed to their complexity? a. Limitation of available options b. Support for one viable action c. Advancement in technology d. Consistent desirable standards 7. The frenulum and prepuce of the clitoris are formed by the? a. Fossa Navicularis b. Mons veneris c. Labia majora

d. Labia minora 8. The vas deferens is a: a. storage for spermatozoa b. Site of spermatozoa production c. Conduit of spermatozoa d. Passageway of sperm 9. Cremasteric visits the clinic and is told that his sperm count is normal. A normal sperm count ranges from: a. 20 to 100/ml b. 100, 000 to 200, 000/ml c. 100 to 200/ml d. 20 to 100 million/ml 10. During which of the following phase of the menstrual cycle is it ideal for implantation of a fertilized egg to occur? a. Ischemic phase b. Menstrual phase c. Proliferative phase d. Secretory phase 11. Variation on the length of menstrual cycle are due to variations in the number of days in which of the following phase? a. Proliferative phase b. Luteal phase c. Ischemic phase d. Secretory phase Situation: Mrs. Calamares G2P1 1001, comes out of the labor and delivery room and reports ruptured amniotic membranes and contractions that occur every 3 minutes lasting 50-60 seconds. The fetus is in LOA position 12. The nurse’s first action should be to: a. Check the FHR b.Call the physician c. Check the vaginal discharge with nitrazine paper d.Admit Mrs. Calamares to the delivery area 13. When asked to describe the amniotic fluid, Mrs. Calamares states that it is “browntinged”. This indicates that:

a. The fetus had infection b. At some point, the fetus experienced oxygen deprivation c. The fetus is in distress and should be delivered immediately d. The fetus is not experiencing any undue stress 14. The nurse established an IV line, and then connects Calamares to an electronic fetal monitor. The fetal monitoring strip shows FHR deceleration occurring about 30 sec after each contraction begins; the FHR returns to baseline after the contraction is over. This type of deceleration is caused by: a. Fetal head compression b. Umbilical cord compression c. Utero-placental insufficiency d. Cardiac anomalies 15. With this type of deceleration, the nurse’s first action should be to: a. Do nothing, this is a normal occurrence b. Call the physician c. Position the patient on her left side d. Continue monitoring the FHR 16. Which of the following methods would be avoided for a woman who is 38 years old, has 3 children and smokes a pack of cigarette per day? a. Oral contraceptives b. Cervical cap c. Diaphragm d. IUD (Intra-uterine device) 17. A woman using diaphragm for contraception should be instructed to leave it in place for at least how long after intercourse? a. 1 hour b. 6 hours c. 12 hours d. 28 hours 18. When assessing the adequacy of sperm for conception to occur, which of the following is the most helpful criterion? a. sperm count b. sperm motility c. Sperm maturity d. Semen volume

19. A couple with one child had been trying, without success for several years to have another child. Which of the following terms would describe the situation? a. Primary Infertility b. Secondary Infertility c. Irreversible infertility d. Sterility Situation: Melanie a 33y/o G1P0 at 32 weeks AOG is admitted to the Hospital with the diagnosis of PIH. 20. Magnesium Sulfate is ordered per IV. Which of the following should prompt the nurse to refer to the obstetricians prior to administration of the drug? a. BP= 180/100 b. Urine output is 40 ml/hr c. RR=12 bpm d. (+) 2 deep tendon reflex 21. The nurse knows that Melanie is knowledgeable about the occurrence of PIH when she remarks: a. “It usually appears anytime during the pregnancy” b. “Its similar to cardio-vascular disease” c. “PIH occurs during the 1st trimester” d. “PIH occurs after the 20th wks AOG” 22. After several hours of MgSO4 administration to Melanie, she should be observed for clinical manifestations of: a. Hyperkalemia b. Hypoglycemia c. Hypermagnesemia d. Hypercalcemia 23. The nurse instructs Melanie to report prodromal symptoms of seizures associated with PIH. Which of the following will she likely identify? a. Urine output of 15ml/hr b. (-) deep tendon reflex c. sudden increase in BP d. Epigastric pain Situation: The following questions pertain to intrapartum complications:

24. Which of the following may happen if the uterus becomes over stimulated by oxytocin during induction of labor? a. Weak contractions prolonged to more than 70 sec b. Titanic contractions prolonged for more than 90 sec c. Increased pain with bright red vaginal bleeding d. Increased restlessness 25. Which of the following factors is the underlying cause of dystocia? a. Nutritional b. Environmental c. Mechanical d. Medical 26. When Umbilical cord is inserted at the edge of the placenta is termed: a. Central insertion b. Battledore insertion c. Velamentous insertion d. Lateral insertion 27. When fetal surface of the placenta presents a central depression surrounded by a thickened grayish white ring, the condition is known as: a. Placenta succenturiata b. Placenta marginata c. Fenestrated placenta d. Placenta Circumvallata 28. Which of the following is derived form mesoderm? a. lining of the GI tract b. liver c. brain d. skeletal system 29. The average length of the umbilical cord in human is: a. 35 cm b. 55 cm c. 65 cm d. 45 cm 30. Urinary excretion of HCG is maximal between which days of gestation? 50-60

40-50 60-70 30-40 31. Which of the following is not a part of conceptus? a. deciduas b. amniotic fluid c. fetus d. membranes 32. Protection of the fetus against syphilis during the 1st trimester is attributed to: a. amniotic fluid b. langhan’s layer c. syncitiothrophoblast d. placenta Situation: Diane is pregnant with her first baby. She went to the clinic for check up. 33. To determine the clients EDC, which day of the menstrual period will you ask? a. first b. last c. third d. second 34. According to Diane, her LMP is November 15, 2002, using the Naegle’s rule what is her EDC? a. August 22, 2003 b. August 18, 2003 c. July 22, 2003 d. February 22, 2003 35. She complained of leg cramps, which usually occurs at night. To provide relief, the nurse tells Diane to: a. Dorsiflex the foot while extending the knee when the cramps occur b. Dorsiflex the foot while flexing the knee when the cramps occurs c. Plantar flex the foot while flexing the knee when the cramps occur d. Plantar flex the foot while extending the knee when the cramp occur Situation: Marita is a nurse working in a STD clinic (question 36-45) 36. The main symptom of gonorrhea in male is:

a. Maculopapular rash b. Jaundice c. Urinary retention d. Urethral discharge 37. In providing education to your clients, you should take into account the fact that the most effective method known to control the spread of HIV infection is: a. Premarital serological screening b. Prophylactic treatment of exposed person c. On going sex education about preventive behaviors d. Laboratory screening of pregnant woman 38. You counseled one of your clients who developed herpes genitalis concerning follow up care. Women who have developed the disease are at risk of developing: a. Heart and CNS damage b. Cervical cancer c. Infant Pneumonia and eye infection d. Sterility 39. Cremasteric, 19 y/o states that he has Gonorrhea. In performing assessment, the nurse should expect to identify which of the following symptoms? a. Lesion on the palms and soles b. A pinpoint rash on the penis c. Urinary dribbling d. Dysuria 40. The nurse should explain to Rhone, 15 y/o that untreated Gonorrhea in the female frequently leads to: a. Obstruction of the Fallopian tubes b. Ovarian cysts c. Ulceration of the cervix d. Endometrial polyps 41. Diane, a 16 y/o female high school student has syphilis. Treatment is initiated. Before the client leaves the clinic, which of the following actions is essential for the nurse to take? a. Advice the client to avoid sexual contact for 2 months b. Ask the client to identify her sexual contacts c. Arrange for the client to have hearing and vision screening d. Have the client to return to the clinic weekly for blood test

42. Kris complains of fishy smelling, white cheeslike vaginal discharge with pruritus. You suspect that Kris may have: a. Moniliasis b. Trichomoniasis c. Syphilis d. Gonorrhea 43. Demi who has history of repeated Trichomonas infections was advised to have Papsmear by her physician. She asked you what the test is for. Your appropriate response is: a. It’s a screening for cervical cancer b. It’s a screening test for presence of cancer in the female reproductive tract c. It is a diagnostic test for the presence of Trichomonas infection d. It is a test that will show if she has cervical cancer or not. 44. The result of the pap-test is class II. This means that: a. Presence of malignant cells b. Presence of benign or possible malignancy c. Normal finding d. Possible inflammation or infections 45. You should be aware that a major difficulty in preventing spread of gonorrhea is that many women who have the disease: a. Is un aware that they have it b. Have milder form of the disease than most men c. Are more reluctant to seek health care than men d. Acquire the disease without having sexual contact Situation: Mrs. Rhona Mahilum was admitted to the hospital with signs and symptoms of pre-eclampsia 46. Because of the possibility of convulsive seizures, which of the following should the nurse have available at the client’s bed side? a. Oxygen and nasopharyngeal suction b. leather restraints c. cardiac monitor d. venous cutdown set 47. One morning, Rhona tells the nurse that she think she is having contractions. Which of the following approaches should the nurse use to fully assess the presence of uterine co tractions?

a. Place the hand on opposite side of the upper part of the abdomen, and curve them somewhat around the uterine fundus. b. Place the heel of the hand on the abdomen just above the umbilicus firmly c. Place the hand flat on the abdomen over the uterine fundus, with the fingers apart and press lightly d. Place the hand in the middle of the upper abdomen and then move hand several times to different parts of the abdomen 48. Exposure of a woman pregnant of a female offspring to which of the following substance increases the risk of the offspring during reproductive years to cervical and uterine cancer steroids thalidomides diethylstilbestrol tetracyclines 49. In which of the following conditions is vaginal rugae most prominent? a. multiparous women b. before menopause c. after menopause d. nulliparous waman 50. The deepest part o the perineal body surrounding the urethra, vagina and rectum that when damaged can result to cystocele, rectocele and urinary stress incontinence is the? a. Pubococcygeus muscle b. Spinchter of urethra and anus c. Bulbocavernous muscle d. Ischiocavernous muscle Situation: Review of concepts of parturition was made by the clinical instructor to a group of nursing students preliminary to their assignment to Labor and delivery room 51. Which plays an important role in the initiation of labor? a. maternal adrenal cortex b. fetal adrenal cortex c. fetal adrenal medulla d. maternal adrenal medulla 52. Which is not considered an uteroronin? a. Prostaglandin b. Endothelin-1 c. Oxytocin d. Relaxin

53. Which is a primary power of labor? a. uterine contractions b. pushing of the mother c. intrathoracic pressure d. abdominal contraction 54. The lower uterine segment is formed from the: a. cervix b. isthmus and cervix c. body of the uterus d. isthmus 55. Ripening of the cervix occurs during the: a. first stage b. second stage c. third stage d. fourth stage 56. In the second stage of labor, uterine contraction last: a. 20 seconds b. 30 seconds c. 60 seconds d. 120 seconds 57. The time between uterine contractions is: a. intensity b. interval c. duration d. frequency 58. Midpelvic capacity may be precisely determined by: a. imaging studies b. clinical measurement of the sidewall convergence c. clinical measurement of the ischial spine prominence d. sub pubic angel measurement 59. The inanimate bone of the pelvis is not composed of the: a. sacrum

b. ilium c. Pubis d. Ischium 60. Which does not refer to the transverse diameter of the pelvic outlet? a. Bi-ischial diameter b. Bi-spinous diameter c. Bi-tuberous diameter d. Intertuberous diameter 61. The Antero-posterior diameter of the pelvic inlet where the fetus will likely most difficulty during labor is the: a. Diagonal conjugate b. True conjugate c. conjugate Vera d. obstetric conjugate 62. The true conjugate can be measured by subtracting ______ to the diagonal conjugate? a. 2.5 – 3 cm b. 3.5 – 4 cm c. 3 – 4 cm d. 1.5 – 2 cm 63. The most important muscle of the pelvic floor is the: a. levator ani muscle b. ischiocavernous c. bulbocavernous d. pubococcygeous 64. Which pelvic shape has the poorest prognosis fro vaginal delivery? a. platypelloid b. anthropoid c. android d. gynecoid 65. The two pubic bones meet anteriorly at the: a. symphysis pubis b. coccyx c. sacrococcygeal d. sacro-illiac joint

66. In the second stage of labor, expulsion of the fetus from birth canal depends on which important factor? a. Maternal bearing down b. Cervical dilatation c. Uterine contractions d. Adequate pelvic size 67. In what presentation is the head in extreme flexion? a. sinciput b. brow c. vertex d. face Situation: a 26 y/o primigravida admitted to the hospital. Vaginal exam reveals that her cervix is 5cm dilated, 80% effaced and the presenting part in zero station, membranes still intact, occiput is in posterior position 68. Due to fetal position, the nurse caring for her would be correct in telling her that: a. she will not have the urge to bear down when she becomes fully dilated b. she can expect to have more back discomfort than most woman in labor c. the position of baby’s head is optimum for passing through the pelvis d. a caesarian section may be necessary to deliver the baby in thin position 69. Upon IE, you noted that the cervix ix ¼ its original length. This mean that effacement is: a. 25% b. 75% c. 100% d. 50% 70. Because of the position of the fetus, an episiotomy has to be performed to enlarge the birth canal. Which of the following is an advantage of episiotomy over lacerations? a. it is more difficult to repair than laceration b. it is more painful than laceration c. it involve a more blood loss than laceration d. heals more faster than laceration 71. Supporting the perenium at the time of crowning will facilitate: a. flexion of the fetal head b. external rotation

c. expulsion of the fetal head d. expulsion 72. When the bi-parietal diameter of the fetal head passes through the pelvic inlet, this is referred as: a. descent b. flexion c. engagement d. extension 73. Sudden gush of blood or lengthening of the cord after the delivery of infant should warn the nurse of: a. placenta acrreta b. placental separation c. placental retention d. abruption placenta Situation: Nurse Tsunade is a staff nurse in the OB ward of Konoha Medical Hospital 74. When separation begins at the center of the placenta and slides down the birth canal like a folded umbrella this is referred as: a. Duncan mechanism b. Shultz mechanism c. Brandt Andrews mechanism d. Ritgen’s maneuver 75. Which of the following is not true regarding the third stage of labor? a. Care should be taken in the administration of bolus of oxytocin because it can cause hypertension b. Signs of placental separation are lengthening of the cord, sudden gush of blood and sudden change in shape of the uterus c. It ranges from the time of expulsion of the fetus to the delivery of the placenta d. The placenta is delivered approximately 5-15 minutes after delivery of the baby 76. In the immediate postpartum period the action of methylegonovine is to: a. cause sustained uterine contractions b. causes intermittent uterine contractions c. relaxes the uterus d. induces sleep so that the mother can rest after an exhausting labor

77. Rhina is a primipara hospitalized due to preeclampsia. The doctor decided to perform NST. The nurse should apply the fetal transducer over the fetus: a. chest b. back c. head d. buttocks 78. Marisse, a newly delivered multipara complains of heavy and painful breast accompanied by fever. The nurse tells Gina that it is normal breast engorgement as the fever is characterized by a. More than 38 degrees b. Does not last more than 34 hours c. Caused by infection d. Needs to be treated with antibiotic 79. Postpartum depression occurs during which time frame? a. within weeks after delivery b. within 12 weeks c. within 16 weeks d. within 24 hours 80. Demi, a 38 y/o multipara is admitted with a tentative diagnosis of femoral thrombophlebitis. The nurse assesses the patient with: a. burning on urination b. leg pain c. abdominal pain d. increased lochial flow 81. Fever, foul lochial discharge and subinvolution of the uterus are signs of: a. puerperal psychosis b. puerperal sepsis c. postpartum hemorrhage d. hypertensive disorder 82. Which is most important when caring for a high risk postpartum clients? a. discussing hygiene and nutrition b. referring the mother to others for emotional support c. discussing complications and treatment d. promoting mother-newborn contact

83. A direct cause of mis-management of the third stage of labor is: a. inversion of the uterus b. cord prolapse c. prolonged labor d. all of these 84. Rachel, a diabetic woman at 36 weeks gestation is scheduled for biophysical profile in order to: a. ascertain correct gestational age b. determine fetal lung maturity c. determine fetal well being d. determine fetal size and obvious congenital anomaly 85. In a primigravida, the following demotes contracted pelvis except: a. absence of quickening b. absence of lightening c. absence of engagement d. none of these 86. Which of the following changes in Diane’s BP would nurse Tsunade not expect? a. it tends to be highest in sitting b. BP may increase a little in the 2nd tri c. It should be taken at every visit at the clinic d. It is normal for blood pressure to increase as much as 33 in systolic in the 3rd tri when a woman is near delivery 87. Nurse Tsunade referred to Diane to an Obstetrician. At 8 months she was orderd for a contraction stress test and the result is negative. Diane asked when she should be back for her next check up? a. Monthly b. Within 24-hours c. Within a week d. Weekly for 2 weeks then monthly 89. Diane wants to know how many fetal movements per hour is normal. The correct response of Nurse tsunade is: a. twice b. thrice c. four times d. 10-12 times

90. Which of the following statement about L/S ratio in amniotic fluid is correct? a. a slight variation in technique does not significantly affect the accuracy of result b. a L/S ratio of 2:1 is incompatible with life c. a L/S ratio of less than 1:0 is compatible with fetal survival d. when L/S ratio is 2:1 below, majority of infants develop respiratory distress 91. Every visit, you obtain the pregnant woman’s fundic height.. At what age of gestion does the fundic height in cm strongly correlates with gestational age in wks? a. 20-24 wks b. 18-24 wks c. 18-32 wks d. 12-38 wks 92. Which is not an indication of amniocentesis? a. previous pregnancy with chromosomal abnormal fetus b. down syndrome in siblings c. pregnancies in women over 35 y/o d. at 8 wks gestation for chromosomal study Situation: Erica is 24 y/o Filipina married to an American. She is pregnant for the second time and now at 8 weeks AOG. She is RH (-) with blood type B 93. Erica gave birth to a term baby with yellowish skin and sclera. The baby is placed on phototherapy. The treatment is effective when blood test shows: a. Low serum bilirubin b. O2 level of 99% c. Normal RBC and WBC count d. Low platelet count 94. Because of rapidly rising bilirubin level, exchange transfusion was performed on Erica’s NB. The nurse understands that the blood to be transfused to the baby should be: a. Type B, RH + b. Type O, RH – c. ABO compatible, RH – d. Type B, RH – 95. Immediately after delivery of Kikay’s Baby, the nurse should remember to: a. delay clamping of the cord to previde the newborn with more blood b. cut immediately after birth of the baby

c. administer RHoGam to the NB immediately on the 3rd stage of labor d. place the NB in an isolette for phototherapy 96. The doctor ordered Kleihauer-Betke. The nurse know which of the following to be wrong about the test: a. it is used to identify the amount of antibodies in the maternal serum b. it is used to determine presence of fetal blood c. it is used to asses whether the mother is RH – or RH + d. It is used to determine fetal blood type and RH factor 97. Which of the following findings in Erica’s history would identify a need for her to receive RHo (d) immune globulin? a. Rh -, coombs + b. Rh -, Coombs – c. Rh +, Coombs – d. Rh +, Coombs + 98. The portion of the placenta overlying the blastocyst a. decidua capsularis b. decidua vera c. decidua basalis d. decidua parietalis 99. The cardinal function of deciduas is a. Immune resonse b. Production of hormones c. Maintenance of pregnancy d. None of the above 100. O2 and Co2 are exchanged in the placenta through the process of: a. pinocytosis b. diffusion c. facilitated diffusion d. active transport End of test. May God Bless you! lovelots, Jeddah Answer Key

http://pinoybsn.blogspot.com/2006/10/100-items-ob-nursing-questions-answer.html 100 Item Exam On Fundamentals Of Nursing : Oxygenation and Nutrition 100 Item Exam On Fundamentals Of Nursing : Oxygenation and Nutrition FUNDAMENTALS OF NURSING TEST V By : Budek http://www.pinoybsn.tk Content Outline 1. Oxygenation 2. Nutrition 1. Which one of the following is NOT a function of the Upper airway? A. For clearance mechanism such as coughing B. Transport gases to the lower airways C. Warming, Filtration and Humidification of inspired air D. Protect the lower airway from foreign mater 2. It is the hair the lines the vestibule which function as a filtering mechanism for foreign objects A. Cilia B. Nares C. Carina D. Vibrissae 3. This is the paranasal sinus found between the eyes and the nose that extends backward into the skull A. Ehtmoid B. Sphenoid C. Maxillary D. Frontal 4. Which paranasal sinus is found over the eyebrow? A. Ehtmoid B. Sphenoid C. Maxillary D. Frontal

5. Gene De Vonne Katrouchuacheulujiki wants to change her surname to something shorter, The court denied her request which depresses her and find herself binge eating. She accidentally aspirate a large piece of nut and it passes the carina. Probabilty wise, Where will the nut go? A. Right main stem bronchus B. Left main stem bronchus C. Be dislodged in between the carina D. Be blocked by the closed epiglottis 6. Which cell secretes mucus that help protect the lungs by trapping debris in the respiratory tract? A. Type I pneumocytes B. Type II pneumocytes C. Goblet cells D. Adipose cells 7. How many lobes are there in the RIGHT LUNG? A. One B. Two C. Three D. Four 8. The presence of the liver causes which anatomical difference of the Kidneys and the Lungs? A. Left kidney slightly lower, Left lung slightly shorter B. Left kidney slightly higher, Left lung slightly shorter C. Right kidney lower, Right lung shorter D. Right kidney higher, Right lung shorter 9. Surfactant is produced by what cells in the alveoli? A. Type I pneumocytes B. Type II pneumocytes C. Goblet cells D. Adipose cells 10. The normal L:S Ratio to consider the newborn baby viable is A. 1:2 B. 2:1 C. 3:1 D. 1:3

11. Refers to the extra air that can be inhaled beyond the normal tidal volume A. Inspiratory reserve volume B. Expiratory reserve volume C. Functional residual capacity D. Residual volume 12. This is the amount of air remained in the lungs after a forceful expiration A. Inspiratory reserve volume B. Expiratory reserve volume C. Functional residual capacity D. Residual volume 13. Casssandra, A 22 year old grade Agnostic, Asked you, how many spikes of bones are there in my ribs? Your best response is which of the following? A. We have 13 pairs of ribs Cassandra B. We have 12 pairs of ribs Cassandra C. Humans have 16 pairs of ribs, and that was noted by Vesalius in 1543 D. Humans have 8 pairs of ribs. 4 of which are floating 14. Which of the following is considered as the main muscle of respiration? A. Lungs B. Intercostal Muscles C. Diaphragm D. Pectoralis major 15. Cassandra asked you : How many air is there in the oxygen and how many does human requires? Which of the following is the best response : A. God is good, Man requires 21% of oxygen and we have 21% available in our air B. Man requires 16% of oxygen and we have 35% available in our air C. Man requires 10% of oxygen and we have 50% available in our air D. Human requires 21% of oxygen and we have 21% available in our air 16. Which of the following is TRUE about Expiration? A. A passive process B. The length of which is half of the length of Inspiration C. Stridor is commonly heard during expiration D. Requires energy to be carried out 17. Which of the following is TRUE in postural drainage?

A. Patient assumes position for 10 to 15 minutes B. Should last only for 60 minutes C. Done best P.C D. An independent nursing action 18. All but one of the following is a purpose of steam inhalation A. Mucolytic B. Warm and humidify air C. Administer medications D. Promote bronchoconstriction 19. Which of the following is NOT TRUE in steam inhalation? A. It is a dependent nursing action B. Spout is put 12-18 inches away from the nose C. Render steam inhalation for atleast 60 minutes D. Cover the client’s eye with wash cloth to prevent irritation 20. When should a nurse suction a client? A. As desired B. As needed C. Every 1 hour D. Every 4 hours 21. Ernest Arnold Hamilton, a 60 year old American client was mobbed by teen gangsters near New york, Cubao. He was rushed to John John Hopio Medical Center and was Unconscious. You are his nurse and you are to suction his secretions. In which position should you place Mr. Hamilton? A. High fowlers B. Semi fowlers C. Prone D. Side lying 22. You are about to set the suction pressure to be used to Mr. Hamilton. You are using a Wall unit suction machine. How much pressure should you set the valve before suctioning Mr. Hamilton? A. 50-95 mmHg B. 200-350 mmHg C. 100-120 mmHg D. 10-15 mmHg

23. The wall unit is not functioning; You then try to use the portable suction equipment available. How much pressure of suction equipment is needed to prevent trauma to mucus membrane and air ways in case of portable suction units? A. 2-5 mmHg B. 5-10 mmHg C. 10-15 mmHg D. 15-25 mmHg 24. There are four catheter sizes available for use, which one of these should you use for Mr. Hamilton? A. Fr. 18 B. Fr. 12 C. Fr. 10 D. Fr, 5 25. Which of the following, if done by the nurse, indicates incompetence during suctioning an unconscious client? A. Measure the length of the suction catheter to be inserted by measuring from the tip of the nose, to the earlobe, to the xiphoid process B. Use KY Jelly if suctioning nasopharyngeal secretion C. The maximum time of suctioning should not exceed 15 seconds D. Allow 30 seconds interval between suctioning 26. Which of the following is the initial sign of hypoxemia in an adult client? 1. Tachypnea 2. Tachycardia 3. Cyanosis 4. Pallor 5. Irritability 6. Flaring of Nares A. 1,2 B. 2,5 C. 2,6 D. 3,4 27. Which method of oxygenation least likely produces anxiety and apprehension? A. Nasal Cannula B. Simple Face mask C. Non Rebreather mask D. Partial Rebreather mask

28. Which of the following oxygen delivery method can deliver 100% Oxygen at 15 LPM? A. Nasal Cannula B. Simple Face mask C. Non Rebreather mask D. Partial Rebreather mask 29. Which of the following is not true about OXYGEN? A. Oxygen is odorless, tasteless and colorless gas. B. Oxygen can irritate mucus membrane C. Oxygen supports combustion D. Excessive oxygen administration results in respiratory acidosis 30. Roberto San Andres, A new nurse in the hospital is about to administer oxygen on patient with Respiratory distress. As his senior nurse, you should intervene if Roberto will: A. Uses venture mask in oxygen administration B. Put a non rebreather mask in the patient before opening the oxygen source C. Use a partial rebreather mask to deliver oxygen D. Check for the doctor’s order for Oxygen administration 31. Which of the following will alert the nurse as an early sign of hypoxia? A. Client is tired and dyspneic B. The client is coughing out blood C. The client’s heart rate is 50 BPM D. Client is frequently turning from side to side 32. Miguelito de balboa, An OFW presents at the admission with an A:P Diameter ratio of 2:1, Which of the following associated finding should the nurse expect? A. Pancytopenia B. Anemia C. Fingers are Club-like D. Hematocrit of client is decreased 33. The best method of oxygen administration for client with COPD uses: A. Cannula B. Simple Face mask C. Non rebreather mask D. Venturi mask

34. Mang dagul, a 50 year old chronic smoker was brought to the E.R because of difficulty in breathing. Pleural effusion was the diagnosis and CTT was ordered. What does C.T.T Stands for? A. Chest tube thoracotomy B. Chest tube thoracostomy C. Closed tube thoracotomy D. Closed tube thoracostmy 35. Where will the CTT be inserted if we are to drain fluids accumulated in Mang dagul’s pleura? A. 2nd ICS B. 4th ICS C. 5th ICS D. 8th ICS 36. There is a continuous bubbling in the water sealed drainage system with suction. And oscillation is observed. As a nurse, what should you do? A. Consider this as normal findings B. Notify the physician C. Check for tube leak D. Prepare a petrolatum gauze dressing 37. Which of the following is true about nutrition? A. It is the process in which food are broken down, for the body to use in growth and development B. It is a process in which digested proteins, fats, minerals, vitamins and carbohydrates are transported into the circulation C. It is a chemical process that occurs in the cell that allows for energy production, energy use, growth and tissue repair D. It is the study of nutrients and the process in which they are use by the body 38. The majority of the digestion processes take place in the A. Mouth B. Small intestine C. Large intestine D. Stomach 39. All of the following is true about digestion that occurs in the Mouth except A. It is where the digestion process starts

B. Mechanical digestion is brought about by mastication C. The action of ptyalin or the salivary tyrpsin breaks down starches into maltose D. Deglutition occurs after food is broken down into small pieces and well mixed with saliva 40. Which of the following foods lowers the cardiac sphincter pressure? A. Roast beef, Steamed cauliflower and Rice B. Orange juice, Non fat milk, Dry crackers C. Decaffeinated coffee, Sky flakes crackers, Suman D. Coffee with coffee mate, Bacon and Egg 41. Where does the digestion of carbohydrates start? A. Mouth B. Esophagus C. Small intestine D. Stomach 42. Protein and Fat digestion begins where? A. Mouth B. Esophagus C. Small intestine D. Stomach 43. All but one is true about digestion that occurs in the Stomach A. Carbohydrates are the fastest to be digested, in about an hour B. Fat is the slowest to be digested, in about 5 hours C. HCl inhibits absorption of Calcium in the gastric mucosa D. HCl converts pepsinogen to pepsin, which starts the complex process of protein digestion 44. Which of the following is NOT an enzyme secreted by the small intestine? A. Sucrase B. Enterokinase C. Amylase D. Enterokinase 45. The hormone secreted by the Small intestine that stimulates the production of pancreatic juice which primarily aids in buffering the acidic bolus passed by the Stomach A. Enterogastrone B. Cholecystokinin

C. Pancreozymin D. Enterokinase 46. When the duodenal enzyme sucrase acts on SUCROSE, which 2 monosaccharides are formed? A. Galactose + Galactose B. Glucose + Fructose C. Glucose + Galactose D. Fructose + Fructose 47. This is the enzyme secreted by the pancrease that completes the protein digestion A. Trypsin B. Enterokinase C. Enterogastrone D. Amylase 48. The end product of protein digestion or the “Building blocks of Protein” is what we call A. Nucleotides B. Fatty acids C. Glucose D. Amino Acids 49. Enzyme secreted by the small intestine after it detects a bolus of fatty food. This will contract the gallbladder to secrete bile and relax the sphincter of Oddi to aid in the emulsification of fats and its digestion. A. Lipase B. Amylase C. Cholecystokinin D. Pancreozymin 50. Which of the following is not true about the Large Intestine? A. It absorbs around 1 L of water making the feces around 75% water and 25% solid B. The stool formed in the transverse colon is not yet well formed C. It is a sterile body cavity D. It is called large intestine because it is longer than the small intestine 51. This is the amount of heat required to raise the temperature of 1 kg water to 1 degree Celsius A. Calorie

B. Joules C. Metabolism D. Basal metabolic rate 52. Assuming a cup of rice provides 50 grams of carbohydrates. How many calories are there in that cup of rice? A. 150 calories B. 200 calories C. 250 calories D. 400 calories 53. An average adult filipino requires how many calories in a day? A. 1,000 calories B. 1,500 calories C. 2,000 calories D. 2,500 calories 54. Which of the following is true about an individual’s caloric needs? A. All individual have the same caloric needs B. Females in general have higher BMR and therefore, require more calories C. During cold weather, people need more calories due to increase BMR D. Dinner should be the heaviest meal of the day 55. Among the following people, who requires the greatest caloric intake? A. An individual in a long state of gluconeogenesis B. An individual in a long state of glycogenolysis C. A pregnant individual D. An adolescent with a BMI of 25 56. Which nutrient deficiency is associated with the development of Pellagra, Dermatitis and Diarrhea? A. Vitamin B1 B. Vitamin B2 C. Vitamin B3 D. Vitamin B6 57. Which Vitamin is not given in conjunction with the intake of LEVODOPA in cases of Parkinson’s Disease due to the fact that levodopa increases its level in the body? A. Vitamin B1 B. Vitamin B2

C. Vitamin B3 D. Vitamin B6 58. A vitamin taken in conjunction with ISONIAZID to prevent peripheral neuritis A. Vitamin B1 B. Vitamin B2 C. Vitamin B3 D. Vitamin B6 59. The inflammation of the Lips, Palate and Tongue is associated in the deficiency of this vitamin A. Vitamin B1 B. Vitamin B2 C. Vitamin B3 D. Vitamin B6 60. Beri beri is caused by the deficiency of which Vitamin? A. Vitamin B1 B. Vitamin B2 C. Vitamin B3 D. Vitamin C 61. Which of the following is the best source of Vitamin E? A. Green leafy vegetables B. Vegetable oil C. Fortified Milk D. Fish liver oil 62. Among the following foods, which food should you emphasize giving on an Alcoholic client? A. Pork liver and organ meats, Pork B. Red meat, Eggs and Dairy products C. Green leafy vegetables, Yellow vegetables, Cantaloupe and Dairy products D. Chicken, Peanuts, Bananas, Wheat germs and yeasts 63. Which food group should you emphasize giving on a pregnant mother in first trimester to prevent neural tube defects? A. Broccoli, Guava, Citrus fruits, Tomatoes B. Butter, Sardines, Tuna, Salmon, Egg yolk C. Wheat germ, Vegetable Oil, soybeans, corn, peanuts

D. Organ meats, Green leafy vegetables, Liver, Eggs 64. A client taking Coumadin is to be educated on his diet. As a nurse, which of the following food should you instruct the client to avoid? A. Spinach, Green leafy vegetables, Cabbage, Liver B. Salmon, Sardines, Tuna C. Butter, Egg yolk, breakfast cereals D. Banana, Yeast, Wheat germ, Chicken 65. Vitamin E plus this mineral works as one of the best anti oxidant in the body according to the latest research. They are combined with 5 Alpha reductase inhibitor to reduce the risk of acquiring prostate cancer A. Zinc B. Iron C. Selenium D. Vanadium 66. Incident of prostate cancer is found to have been reduced on a population exposed in tolerable amount of sunlight. Which vitamin is associated with this phenomenon? A. Vitamin A B. Vitamin B C. Vitamin C D. Vitamin D 67. Micronutrients are those nutrients needed by the body in a very minute amount. Which of the following vitamin is considered as a MICRONUTRIENT A. Phosphorous B. Iron C. Calcium D. Sodium 68. Deficiency of this mineral results in tetany, osteomalacia, osteoporosis and rickets. A. Vitamin D B. Iron C. Calcium D. Sodium 69. Among the following foods, which has the highest amount of potassium per area of their meat? A. Cantaloupe

B. Avocado C. Raisin D. Banana 70. A client has HEMOSIDEROSIS. Which of the following drug would you expect to be given to the client? A. Acetazolamide B. Deferoxamine C. Calcium EDTA D. Activated charcoal 71. Which of the following provides the richest source of Iron per area of their meat? A. Pork meat B. Lean read meat C. Pork liver D. Green mongo 72. Which of the following is considered the best indicator of nutritional status of an individual? A. Height B. Weight C. Arm muscle circumference D. BMI 73. Jose Miguel, a 50 year old business man is 6’0 Tall and weights 179 lbs. As a nurse, you know that Jose Miguel is : A. Overweight B. Underweight C. Normal D. Obese 74. Jose Miguel is a little bit nauseous. Among the following beverages, Which could help relieve JM’s nausea? A. Coke B. Sprite C. Mirinda D. Orange Juice or Lemon Juice 75. Which of the following is the first sign of dehydration? A. Tachycardia

B. Restlessness C. Thirst D. Poor skin turgor 76. What Specific gravity lab result is compatible with a dehydrated client? A. 1.007 B. 1.020 C. 1.039 D. 1.029 77. Which hematocrit value is expected in a dehydrated male client? A. 67% B. 50% C. 36% D. 45% 78. Which of the following statement by a client with prolonged vomiting indicates the initial onset of hypokalemia? A. My arm feels so weak B. I felt my heart beat just right now C. My face muscle is twitching D. Nurse, help! My legs are cramping 79. Which of the following is not an anti-emetic? A. Marinol B. Dramamine C. Benadryl D. Alevaire 80. Which is not a clear liquid diet? A. Hard candy B. Gelatin C. Coffee with Coffee mate D. Bouillon 81. Which of the following is included in a full liquid diet? A. Popsicles B. Pureed vegetable meat C. Pineapple juice with pulps D. Mashed potato

82. Which food is included in a BLAND DIET? A. Steamed broccoli B. Creamed potato C. Spinach in garlic D. Sweet potato 83. Which of the following if done by the nurse, is correct during NGT Insertion? A. Use an oil based lubricant B. Measure the amount of the tube to be inserted from the Tip of the nose, to the earlobe, to the xiphoid process C. Soak the NGT in a basin of ice water to facilitate easy insertion D. Check the placement of the tube by introducing 10 cc of sterile water and auscultating for bubbling sound 84. Which of the following is the BEST method in assessing for the correct placement of the NGT? A. X-Ray B. Immerse tip of the tube in water to check for bubbles produced C. Aspirating gastric content to check if the content is acidic D. Instilling air in the NGT and listening for a gurgling sound at the epigastric area 85. A terminally ill cancer patient is scheduled for an NGT feeding today. How should you position the patient? A. Semi fowlers in bed B. Bring the client into a chair C. Slightly elevated right side lying position D. Supine in bed 86. A client is scheduled for NGT Feeding. Checking the residual volume, you determined that he has 40 cc residual from the last feeding. You reinstill the 40 cc of residual volume and added the 250 cc of feeding ordered by the doctor. You then instill 60 cc of water to clear the lumen and the tube. How much will you put in the client’s chart as input? A. 250 cc B. 290 cc C. 350 cc D. 310 cc 87. Which of the following if done by a nurse indicates deviation from the standards of NGT feeding?

A. Do not give the feeding and notify the doctor of residual of the last feeding is greater than or equal to 50 ml B. Height of the feeding should be 12 inches about the tube point of insertion to allow slow introduction of feeding C. Ask the client to position in supine position immediately after feeding to prevent dumping syndrome D. Clamp the NGT before all of the water is instilled to prevent air entry in the stomach 88. What is the most common problem in TUBE FEEDING? A. Diarrhea B. Infection C. Hyperglycemia D. Vomiting 89. Which of the following is TRUE in colostomy feeding? A. Hold the syringe 18 inches above the stoma and administer the feeding slowly B. Pour 30 ml of water before and after feeding administration C. Insert the ostomy feeding tube 1 inch towards the stoma D. A Pink stoma means that circulation towards the stoma is all well 90. A client with TPN suddenly develops tremors, dizziness, weakness and diaphoresis. The client said “I feel weak” You saw that his TPN is already empty and another TPN is scheduled to replace the previous one but its provision is already 3 hours late. Which of the following is the probable complication being experienced by the client? A. Hyperglycemia B. Hypoglycemia C. Infection D. Fluid overload 91. To assess the adequacy of food intake, which of the following assessment parameters is best used? A. Food likes and dislikes B. Regularity of meal times C. 3 day diet recall D. Eating style and habits 92. The vomiting center is found in the A. Medulla Oblongata B. Pons C. Hypothalamus

D. Cerebellum 93. The most threatening complication of vomiting in client’s with stroke is A. Aspiration B. Dehydration C. Fluid and electrolyte imbalance D. Malnutrition 94. Which among this food is the richest source of Iron? A. Ampalaya B. Broccoli C. Mongo D. Malunggay leaves 95. Which of the following is a good source of Vitamin A? A. Egg yolk B. Liver C. Fish D. Peanuts 96. The most important nursing action before gastrostomy feeding is A. Check V/S B. Assess for patency of the tube C. Measure residual feeding D. Check the placement of the tube 97. The primary advantage of gastrostomy feeding is A. Ensures adequate nutrition B. It prevents aspiration C. Maintains Gastro esophageal sphincter integrity D. Minimizes fluid-electrolyte imbalance 98. What is the BMI Of Budek, weighing 120 lbs and has a height of 5 feet 7 inches. A. 20 B. 19 C. 15 D. 25 99. Which finding is consistent with PERNICIOUS ANEMIA?

A. Strawberry tongue B. Currant Jelly stool C. Beefy red tongue D. Pale [ HYPOCHROMIC ] RBC 100. The nurse is browsing the chart of the patient and notes a normal serum lipase level. Which of the following is a normal serum lipase value? A. 10 U/L B. 100 U/L C. 200 U/L D. 350 U/L Answer Key http://pinoybsn.blogspot.com/2006/10/100-item-exam-on-fundamentals-of.html 100 Item Exam On Fundamentals Of Nursing : Nursing Process, Physical and Health Assessment and Routine Procedures 100 Item Exam On Fundamentals Of Nursing : Nursing Process, Physical and Health Assessment and Routine Procedures FUNDAMENTALS OF NURSING TEST IV By : Budek http://www.pinoybsn.tk Content Outline 1. The nursing process 2. Physical Assessment 3. Health Assessment 3.a Temperature 3.b Pulse 3.c Respiration 3.d Blood pressure 4. Routine Procedures 4.a Urinalysis specimen collection 4.b Sputum specimen collection 4.c Urine examination 4.d Positioning pre-procedure 4.e Stool specimen collection 1. She is the first one to coin the term “NURSING PROCESS” She introduced 3 steps of nursing process which are Observation, Ministration and Validation.

A. Nightingale B. Johnson C. Rogers D. Hall 2. The American Nurses association formulated an innovation of the Nursing process. Today, how many distinct steps are there in the nursing process? A. APIE – 4 B. ADPIE – 5 C. ADOPIE – 6 D. ADOPIER – 7 3. They are the first one to suggest a 4 step nursing process which are : APIE , or assessment, planning, implementation and evaluation. 1. Yura 2. Walsh 3. Roy 4. Knowles A. 1,2 B. 1,3 C. 3,4 D. 2,3 4. Which characteristic of nursing process is responsible for proper utilization of human resources, time and cost resources? A. Organized and Systematic B. Humanistic C. Efficient D. Effective 5. Which characteristic of nursing process addresses the INDIVIDUALIZED care a client must receive? A. Organized and Systematic B. Humanistic C. Efficient D. Effective 6. A characteristic of the nursing process that is essential to promote client satisfaction and progress. The care should also be relevant with the client’s needs.

A. Organized and Systematic B. Humanistic C. Efficient D. Effective 7. Rhina, who has Menieres disease, said that her environment is moving. Which of the following is a valid assessment? 1. Rhina is giving an objective data 2. Rhina is giving a subjective data 3. The source of the data is primary 4. The source of the data is secondary A. 1,3 B. 2,3 C. 2.4 D. 1,4 8. Nurse Angela, observe Joel who is very apprehensive over the impending operation. The client is experiencing dyspnea, diaphoresis and asks lots of questions. Angela made a diagnosis of ANXIETY R/T INTRUSIVE PROCEDURE. This is what type of Nursing Diagnosis? A. Actual B. Probable C. Possible D. Risk 9. Nurse Angela diagnosed Mrs. Delgado, who have undergone a BKA. Her diagnosis is SELF ESTEEM DISTURBANCE R/T CHANGE IN BODY IMAGE. Although the client has not yet seen her lost leg, Angela already anticipated the diagnosis. This is what type of Diagnosis? A. Actual B. Probable C. Possible D. Risk 10. Nurse Angela is about to make a diagnosis but very unsure because the S/S the client is experiencing is not specific with her diagnosis of POWERLESSNESS R/T DIFFICULTY ACCEPTING LOSS OF LOVED ONE. She then focus on gathering data to refute or prove her diagnosis but her plans and interventions are already ongoing for the diagnosis. Which type of Diagnosis is this? A. Actual B. Probable

C. Possible D. Risk 11. Nurse Angela knew that Stephen Lee Mu Chin, has just undergone an operation with an incision near the diaphragm. She knew that this will contribute to some complications later on. She then should develop what type of Nursing diagnosis? A. Actual B. Probable C. Possible D. Risk 12. Which of the following Nursing diagnosis is INCORRECT? A. Fluid volume deficit R/T Diarrhea B. High risk for injury R/T Absence of side rails C. Possible ineffective coping R/T Loss of loved one D. Self esteem disturbance R/T Effects of surgical removal of the leg 13. Among the following statements, which should be given the HIGHEST priority? A. Client is in extreme pain B. Client’s blood pressure is 60/40 C. Client’s temperature is 40 deg. Centigrade D. Client is cyanotic 14. Which of the following need is given a higher priority among others? A. The client has attempted suicide and safety precaution is needed B. The client has disturbance in his body image because of the recent operation C. The client is depressed because her boyfriend left her all alone D. The client is thirsty and dehydrated 15. Which of the following is TRUE with regards to Client Goals? A. They are specific, measurable, attainable and time bounded B. They are general and broadly stated C. They should answer for WHO, WHAT ACTIONS, WHAT CIRCUMSTANCES, HOW WELL and WHEN. D. Example is : After discharge planning, Client demonstrated the proper psychomotor skills for insulin injection. 16. Which of the following is a NOT a correct statement of an Outcome criteria? A. Ambulates 30 feet with a cane before discharge B. Discusses fears and concerns regarding the surgical procedure C. Demonstrates proper coughing and breathing technique after a teaching session

D. Reestablishes a normal pattern of elimination 17. Which of the following is a OBJECTIVE data? A. Dizziness B. Chest pain C. Anxiety D. Blue nails 18. A patient’s chart is what type of data source? A. Primary B. Secondary C. Tertiary D. Can be A and B 19. All of the following are characteristic of the Nursing process except A. Dynamic B. Cyclical C. Universal D. Intrapersonal 20. Which of the following is true about the NURSING CARE PLAN? A. It is nursing centered B. Rationales are supported by interventions C. Verbal D. Atleast 2 goals are needed for every nursing diagnosis 21. A framework for health assessment that evaluates the effects of stressors to the mind, body and environment in relation with the ability of the client to perform ADL. A. Functional health framework B. Head to toe framework C. Body system framework D. Cephalocaudal framework 22. Client has undergone Upper GI and Lower GI series. Which type of health assessment framework is used in this situation? A. Functional health framework B. Head to toe framework C. Body system framework D. Cephalocaudal framework

23. Which of the following statement is true regarding temperature? A. Oral temperature is more accurate than rectal temperature B. The bulb used in Rectal temperature reading is pear shaped or round C. The older the person, the higher his BMR D. When the client is swimming, BMR Decreases 24. A type of heat loss that occurs when the heat is dissipated by air current A. Convection B. Conduction C. Radiation D. Evaporation 25. Which of the following is TRUE about temperature? A. The highest temperature usually occurs later in a day, around 8 P.M to 12 M.N B. The lowest temperature is usually in the Afternoon, Around 12 P.M C. Thyroxin decreases body temperature D. Elderly people are risk for hyperthermia due to the absence of fats, Decreased thermoregulatory control and sedentary lifestyle. 26. Hyperpyrexia is a condition in which the temperature is greater than A. 40 degree Celsius B. 39 degree Celsius C. 100 degree Fahrenheit D. 105.8 degree Fahrenheit 27. Tympanic temperature is taken from John, A client who was brought recently into the ER due to frequent barking cough. The temperature reads 37.9 Degrees Celsius. As a nurse, you conclude that this temperature is A. High B. Low C. At the low end of the normal range D. At the high end of the normal range 28. John has a fever of 38.5 Deg. Celsius. It surges at around 40 Degrees and go back to 38.5 degrees 6 times today in a typical pattern. What kind of fever is John having? A. Relapsing B. Intermittent C. Remittent D. Constant

29. John has a fever of 39.5 degrees 2 days ago, But yesterday, he has a normal temperature of 36.5 degrees. Today, his temperature surges to 40 degrees. What type of fever is John having? A. Relapsing B. Intermittent C. Remittent D. Constant 30. John’s temperature 10 hours ago is a normal 36.5 degrees. 4 hours ago, He has a fever with a temperature of 38.9 Degrees. Right now, his temperature is back to normal. Which of the following best describe the fever john is having? A. Relapsing B. Intermittent C. Remittent D. Constant 31. The characteristic fever in Dengue Virus is characterized as: A. Tricyclic B. Bicyclic C. Biphasic D. Triphasic 32. When John has been given paracetamol, his fever was brought down dramatically from 40 degrees Celsius to 36.7 degrees in a matter of 10 minutes. The nurse would assess this event as: A. The goal of reducing john’s fever has been met with full satisfaction of the outcome criteria B. The desired goal has been partially met C. The goal is not completely met D. The goal has been met but not with the desired outcome criteria 33. What can you expect from Marianne, who is currently at the ONSET stage of fever? A. Hot, flushed skin B. Increase thirst C. Convulsion D. Pale,cold skin 34. Marianne is now at the Defervescence stage of the fever, which of the following is expected?

A. Delirium B. Goose flesh C. Cyanotic nail beds D. Sweating 35. Considered as the most accessible and convenient method for temperature taking A. Oral B. Rectal C. Tympanic D. Axillary 36. Considered as Safest and most non invasive method of temperature taking A. Oral B. Rectal C. Tympanic D. Axillary 37. Which of the following is NOT a contraindication in taking ORAL temperature? A. Quadriplegic B. Presence of NGT C. Dyspnea D. Nausea and Vomitting 38. Which of the following is a contraindication in taking RECTAL temperature? A. Unconscious B. Neutropenic C. NPO D. Very young children 39. How long should the Rectal Thermometer be inserted to the clients anus? A. 1 to 2 inches B. .5 to 1.5 inches C. 3 to 5 inches D. 2 to 3 inches 40. In cleaning the thermometer after use, The direction of the cleaning to follow Medical Asepsis is : A. From bulb to stem B. From stem to bulb C. From stem to stem

D. From bulb to bulb 41. How long should the thermometer stay in the Client’s Axilla? A. 3 minutes B. 4 minutes C. 7 minutes D. 10 minutes 42. Which of the following statement is TRUE about pulse? A. Young person have higher pulse than older persons B. Males have higher pulse rate than females after puberty C. Digitalis has a positive chronotropic effect D. In lying position, Pulse rate is higher 43. The following are correct actions when taking radial pulse except: A. Put the palms downward B. Use the thumb to palpate the artery C. Use two or three fingers to palpate the pulse at the inner wrist D. Assess the pulse rate, rhythm, volume and bilateral quality 44. The difference between the systolic and diastolic pressure is termed as A. Apical rate B. Cardiac rate C. Pulse deficit D. Pulse pressure 45. Which of the following completely describes PULSUS PARADOXICUS? A. A greater-than-normal increase in systolic blood pressure with inspiration B. A greater-than-normal decrease in systolic blood pressure with inspiration C. Pulse is paradoxically low when client is in standing position and high when supine. D. Pulse is paradoxically high when client is in standing position and low when supine. 46. Which of the following is TRUE about respiration? A. I:E 2:1 B. I:E : 4:3 C I:E 1:1 D. I:E 1:2 47. Contains the pneumotaxic and the apneutic centers

A. Medulla oblongata B. Pons C. Carotid bodies D. Aortic bodies 48. Which of the following is responsible for deep and prolonged inspiration A. Medulla oblongata B. Pons C. Carotid bodies D. Aortic bodies 49. Which of the following is responsible for the rhythm and quality of breathing? A. Medulla oblongata B. Pons C. Carotid bodies D. Aortic bodies 50. The primary respiratory center A. Medulla oblongata B. Pons C. Carotid bodies D. Aortic bodies 51. Which of the following is TRUE about the mechanism of action of the Aortic and Carotid bodies? A. If the BP is elevated, the RR increases B. If the BP is elevated, the RR decreases C. Elevated BP leads to Metabolic alkalosis D. Low BP leads to Metabolic acidosis 52. All of the following factors correctly influence respiration except one. Which of the following is incorrect? A. Hydrocodone decreases RR B. Stress increases RR C. Increase temperature of the environment, Increase RR D. Increase altitude, Increase RR 53. When does the heart receives blood from the coronary artery? A. Systole

B. Diastole C. When the valves opens D. When the valves closes 54. Which of the following is more life threatening? A. BP = 180/100 B. BP = 160/120 C. BP = 90/60 D. BP = 80/50 55. Refers to the pressure when the ventricles are at rest A. Diastole B. Systole C. Preload D. Pulse pressure 56. Which of the following is TRUE about the blood pressure determinants? A. Hypervolemia lowers BP B. Hypervolemia increases GFR C. HCT of 70% might decrease or increase BP D. Epinephrine decreases BP 57. Which of the following do not correctly correlates the increase BP of Ms. Aida, a 70 year old diabetic? A. Females, after the age 65 tends to have lower BP than males B. Disease process like Diabetes increase BP C. BP is highest in the morning, and lowest during the night D. Africans, have a greater risk of hypertension than Caucasian and Asians. 58. How many minutes are allowed to pass if the client had engaged in strenuous activities, smoked or ingested caffeine before taking his/her BP? A. 5 B. 10 C. 15 D. 30 59. Too narrow cuff will cause what change in the Client’s BP? A. True high reading B. True low reading C. False high reading

D. False low reading 60. Which is a preferable arm for BP taking? A. An arm with the most contraptions B. The left arm of the client with a CVA affecting the right brain C. The right arm D. The left arm 61. Which of the following is INCORRECT in assessing client’s BP? A. Read the mercury at the upper meniscus, preferably at the eye level to prevent error of parallax B. Inflate and deflate slowly, 2-3 mmHg at a time C. The sound heard during taking BP is known as KOROTKOFF sound D. If the BP is taken on the left leg using the popliteal artery pressure, a BP of 160/80 is normal. 62. Which of the following is the correct interpretation of the ERROR OF PARALLAX A. If the eye level is higher than the level of the meniscus, it will cause a false high reading B. If the eye level is higher than the level of the meniscus, it will cause a false low reading C. If the eye level is lower than the level of the meniscus, it will cause a false low reading D. If the eye level is equal to that of the level of the upper meniscus, the reading is accurate 63. How many minute/s is/are allowed to pass before making a re-reading after the first one? A. 1 B. 5 C. 15 D. 30 64. Which of the following is TRUE about the auscultation of blood pressure? A. Pulse + 4 is considered as FULL B. The bell of the stethoscope is use in auscultating BP C. Sound produced by BP is considered as HIGH frequency sound D. Pulse +1 is considered as NORMAL 65. In assessing the abdomen, Which of the following is the correct sequence of the physical assessment?

A. Inspection, Auscultation, Percussion, Palpation B. Palpation, Auscultation, Percussion, Inspection C. Inspection, Palpation, Auscultation, Percussion D. Inspection, Auscultation, Palpation, Percussion 66. The sequence in examining the quadrants of the abdomen is: A. RUQ,RLQ,LUQ,LLQ B. RLQ,RUQ,LLQ,LUQ C. RUQ,RLQ,LLQ,LUQ D. RLQ,RUQ,LUQ,LLQ 67. In inspecting the abdomen, which of the following is NOT DONE? A. Ask the client to void first B. Knees and legs are straighten to relax the abdomen C. The best position in assessing the abdomen is Dorsal recumbent D. The knees and legs are externally rotated 68. Dr. Fabian De Las Santas, is about to conduct an ophthalmoscope examination. Which of the following, if done by a nurse, is a Correct preparation before the procedure? A. Provide the necessary draping to ensure privacy B. Open the windows, curtains and light to allow better illumination C. Pour warm water over the ophthalmoscope to ensure comfort D. Darken the room to provide better illumination 69. If the client is female, and the doctor is a male and the patient is about to undergo a vaginal and cervical examination, why is it necessary to have a female nurse in attendance? A. To ensure that the doctor performs the procedure safely B. To assist the doctor C. To assess the client’s response to examination D. To ensure that the procedure is done in an ethical manner 70. In palpating the client’s breast, Which of the following position is necessary for the patient to assume before the start of the procedure? A. Supine B. Dorsal recumbent C. Sitting D. Lithotomy 71. When is the best time to collect urine specimen for routine urinalysis and C/S?

A. Early morning B. Later afternoon C. Midnight D. Before breakfast 72. Which of the following is among an ideal way of collecting a urine specimen for culture and sensitivity? A. Use a clean container B. Discard the first flow of urine to ensure that the urine is not contaminated C. Collect around 30-50 ml of urine D. Add preservatives, refrigerate the specimen or add ice according to the agency’s protocol 73. In a 24 hour urine specimen started Friday, 9:00 A.M, which of the following if done by a Nurse indicate a NEED for further procedural debriefing? A. The nurse ask the client to urinate at 9:00 A.M, Friday and she included the urine in the 24 hour urine specimen B. The nurse discards the Friday 9:00 A M urine of the client C. The nurse included the Saturday 9:00 A.M urine of the client to the specimen collection D. The nurse added preservatives as per protocol and refrigerates the specimen 74. This specimen is required to assess glucose levels and for the presence of albumin the the urine A. Midstream clean catch urine B. 24 hours urine collection C. Postprandial urine collection D. Second voided urine 75. When should the client test his blood sugar levels for greater accuracy? A. During meals B. In between meals C. Before meals D. 2 Hours after meals 76. In collecting a urine from a catheterized patient, Which of the following statement indicates an accurate performance of the procedure? A. Clamp above the port for 30 to 60 minutes before drawing the urine from the port B. Clamp below the port for 30 to 60 minutes before drawing the urine from the port C. Clamp above the port for 5 to 10 minutes before drawing the urine from the port D. Clamp below the port for 5 to 10 minutes before drawing the urine from the port

77. A community health nurse should be resourceful and meet the needs of the client. A villager ask him, Can you test my urine for glucose? Which of the following technique allows the nurse to test a client’s urine for glucose without the need for intricate instruments. A. Acetic Acid test B. Nitrazine paper test C. Benedict’s test D. Litmus paper test 78. A community health nurse is assessing client’s urine using the Acetic Acid solution. Which of the following, if done by a nurse, indicates lack of correct knowledge with the procedure? A. The nurse added the Urine as the 2/3 part of the solution B. The nurse heats the test tube after adding 1/3 part acetic acid C. The nurse heats the test tube after adding 2/3 part of Urine D. The nurse determines abnormal result if she noticed that the test tube becomes cloudy 79. Which of the following is incorrect with regards to proper urine testing using Benedict’s Solution? A. Heat around 5ml of Benedict’s solution together with the urine in a test tube B. Add 8 to 10 drops of urine C. Heat the Benedict’s solution without the urine to check if the solution is contaminated D. If the color remains BLUE, the result is POSITIVE 80. +++ Positive result after Benedicts test is depicted by what color? A. Blue B. Green C. Yellow D. Orange 81. Clinitest is used in testing the urine of a client for glucose. Which of the following, If committed by a nurse indicates error? A. Specimen is collected after meals B. The nurse puts 1 clinitest tablet into a test tube C. She added 5 drops of urine and 10 drops of water D. If the color becomes orange or red, It is considered postitive 82. Which of the following nursing intervention is important for a client scheduled to have a Guaiac Test?

A. Avoid turnips, radish and horseradish 3 days before procedure B. Continue iron preparation to prevent further loss of Iron C. Do not eat read meat 12 hours before procedure D. Encourage caffeine and dark colored foods to produce accurate results 83. In collecting a routine specimen for fecalysis, Which of the following, if done by a nurse, indicates inadequate knowledge and skills about the procedure? A. The nurse scoop the specimen specifically at the site with blood and mucus B. She took around 1 inch of specimen or a teaspoonful C. Ask the client to call her for the specimen after the client wiped off his anus with a tissue D. Ask the client to defecate in a bedpan, Secure a sterile container 84. In a routine sputum analysis, Which of the following indicates proper nursing action before sputum collection? A. Secure a clean container B. Discard the container if the outside becomes contaminated with the sputum C. Rinse the client’s mouth with Listerine after collection D. Tell the client that 4 tablespoon of sputum is needed for each specimen for a routine sputum analysis 85. Who collects Blood specimen? A. The nurse B. Medical technologist C. Physician D. Physical therapist 86. David, 68 year old male client is scheduled for Serum Lipid analysis. Which of the following health teaching is important to ensure accurate reading? A. Tell the patient to eat fatty meals 3 days prior to the procedure B. NPO for 12 hours pre procedure C. Ask the client to drink 1 glass of water 1 hour prior to the procedure D. Tell the client that the normal serum lipase level is 50 to 140 U/L 87. The primary factor responsible for body heat production is the A. Metabolism B. Release of thyroxin C. Muscle activity D. Stress 88. The heat regulating center is found in the

A. Medulla oblongata B. Thalamus C. Hypothalamus D. Pons 89. A process of heat loss which involves the transfer of heat from one surface to another is A. Radiation B. Conduction C. Convection D. Evaporation 90. Which of the following is a primary factor that affects the BP? A. Obesity B. Age C. Stress D. Gender 91. The following are social data about the client except A. Patient’s lifestyle B. Religious practices C. Family home situation D. Usual health status 92. The best position for any procedure that involves vaginal and cervical examination is A. Dorsal recumbent B. Side lying C. Supine D. Lithotomy 93. Measure the leg circumference of a client with bipedal edema is best done in what position? A. Dorsal recumbent B. Sitting C. Standing D. Supine 94. In palpating the client’s abdomen, Which of the following is the best position for the client to assume?

A. Dorsal recumbent B. Side lying C. Supine D. Lithotomy 95. Rectal examination is done with a client in what position? A. Dorsal recumbent B. Sims position C. Supine D. Lithotomy 96. Which of the following is a correct nursing action when collecting urine specimen from a client with an Indwelling catheter? A. Collect urine specimen from the drainage bag B. Detach catheter from the connecting tube and draw the specimen from the port C. Use sterile syringe to aspirate urine specimen from the drainage port D. Insert the syringe straight to the port to allow self sealing of the port 97. Which of the following is inappropriate in collecting mid stream clean catch urine specimen for urine analysis? A. Collect early in the morning, First voided specimen B. Do perineal care before specimen collection C. Collect 5 to 10 ml for urine D. Discard the first flow of the urine 98. When palpating the client’s neck for lymphadenopathy, where should the nurse position himself? A. At the client’s back B. At the client’s right side C. At the client’s left side D. In front of a sitting client 99. Which of the following is the best position for the client to assume if the back is to be examined by the nurse? A. Standing B. Sitting C. Side lying D. Prone 100. In assessing the client’s chest, which position best show chest expansion as well as its movements?

A. Sitting B. Prone C. Sidelying D. Supine Answer Key http://pinoybsn.blogspot.com/2006/09/100-item-exam-on-fundamentals-of_23.html 100 Item Exam on Fundamentals Of Nursing : Stress, Crisis, Crisis Intervention, Communication, Recording, Learning and Documentation 100 Item Exam on Fundamentals Of Nursing : Stress, Crisis, Crisis Intervention, Communication, Recording, Learning and Documentation FUNDAMENTALS OF NURSING TEST III By : Budek http://www.pinoybsn.tk Content Outline 1. Physical response to stress 2. Psychological response to stress 3. Spiritual response to stress 4. Stress management 5. Crisis and Crisis intervention 6. Communication 7. Recording 8. Documentation 9. Learning 1. The coronary vessels, unlike any other blood vessels in the body, respond to sympathetic stimulation by A. Vasoconstriction B. Vasodilatation C. Decreases force of contractility D. Decreases cardiac output 2. What stress response can you expect from a patient with blood sugar of 50 mg / dl? A. Body will try to decrease the glucose level B. There will be a halt in release of sex hormones C. Client will appear restless

D. Blood pressure will increase 3. All of the following are purpose of inflammation except A. Increase heat, thereby produce abatement of phagocytosis B. Localized tissue injury by increasing capillary permeability C. Protect the issue from injury by producing pain D. Prepare for tissue repair 4. The initial response of tissue after injury is A. Immediate Vasodilation B. Transient Vasoconstriction C. Immediate Vasoconstriction D. Transient Vasodilation 5. The last expected process in the stages of inflammation is characterized by A. There will be sudden redness of the affected part B. Heat will increase on the affected part C. The affected part will loss its normal function D. Exudates will flow from the injured site 6. What kind of exudates is expected when there is an antibody-antigen reaction as a result of microorganism infection? A. Serous B. Serosanguinous C. Purulent D. Sanguinous 7. The first manifestation of inflammation is A. Redness on the affected area B. Swelling of the affected area C. Pain, which causes guarding of the area D. Increase heat due to transient vasodilation 8. The client has a chronic tissue injury. Upon examining the client’s antibody for a particular cellular response, Which of the following WBC component is responsible for phagocytosis in chronic tissue injury? A. Neutrophils B. Basophils C. Eosinophils D. Monocytes

9. Which of the following WBC component proliferates in cases of Anaphylaxis? A. Neutrophils B. Basophils C. Eosinophils D. Monocytes 10. Icheanne, ask you, her Nurse, about WBC Components. She got an injury yesterday after she twisted her ankle accidentally at her gymnastic class. She asked you, which WBC Component is responsible for proliferation at the injured site immediately following an injury. You answer: A. Neutrophils B. Basophils C. Eosinophils D. Monocytes 11. Icheanne then asked you, what is the first process that occurs in the inflammatory response after injury, You tell her: A. Phagocytosis B. Emigration C. Pavementation D. Chemotaxis 12. Icheanne asked you again, What is that term that describes the magnetic attraction of injured tissue to bring phagocytes to the site of injury? A. Icheanne, you better sleep now, you asked a lot of questions B. It is Diapedesis C. We call that Emigration D. I don’t know the answer, perhaps I can tell you after I find it out later 13. This type of healing occurs when there is a delayed surgical closure of infected wound A. First intention B. Second intention C. Third intention D. Fourth intention 14. Type of healing when scars are minimal due to careful surgical incision and good healing A. First intention B. Second intention

C. Third intention D. Fourth intention 15. Imelda, was slashed and hacked by an unknown suspects. She suffered massive tissue loss and laceration on her arms and elbow in an attempt to evade the criminal. As a nurse, you know that the type of healing that will most likely occur to Miss Imelda is A. First intention B. Second intention C. Third intention D. Fourth intention 16. Imelda is in the recovery stage after the incident. As a nurse, you know that the diet that will be prescribed to Miss Imelda is A. Low calorie, High protein with Vitamin A and C rich foods B. High protein, High calorie with Vitamin A and C rich foods C. High calorie, Low protein with Vitamin A and C rich foods D. Low calorie, Low protein with Vitamin A and C rich foods 17. Miss Imelda asked you, What is WET TO DRY Dressing method? Your best response is A. It is a type of mechanical debridement using Wet dressing that is applied and left to dry to remove dead tissues B. It is a type of surgical debridement with the use of Wet dressing to remove the necrotic tissues C. It is a type of dressing where in, The wound is covered with Wet or Dry dressing to prevent contamination D. It is a type of dressing where in, A cellophane or plastic is placed on the wound over a wet dressing to stimulate healing of the wound in a wet medium 18. The primary cause of pain in inflammation is A. Release of pain mediators B. Injury to the nerve endings C. Compression of the local nerve endings by the edema fluids D. Circulation is lessen, Supply of oxygen is insufficient 19. The client is in stress because he was told by the physician he needs to undergo surgery for removal of tumor in his bladder. Which of the following are effects of sympatho-adreno-medullary response by the client? 1. Constipation 2. Urinary frequency 3. Hyperglycemia

4. Increased blood pressure A. 3,4 B. 1,3,4 C.1,2,4 D.1,4 20. The client is on NPO post midnight. Which of the following, if done by the client, is sufficient to cancel the operation in the morning? A. Eat a full meal at 10:00 P.M B. Drink fluids at 11:50 P.M C. Brush his teeth the morning before operation D. Smoke cigarette around 3:00 A.M 21. The client place on NPO for preparation of the blood test. Adreno-cortical response is activated and which of the following below is an expected response? A. Low BP B. Decrease Urine output C. Warm, flushed, dry skin D. Low serum sodium levels 22. Which of the following is true about therapeutic relationship? A. Directed towards helping an individual both physically and emotionally B. Bases on friendship and mutual trust C. Goals are set by the solely nurse D. Maintained even after the client doesn’t need anymore of the Nurse’s help 23. According to her, A nurse patient relationship is composed of 4 stages : Orientation, Identification, Exploitation and Resolution A. Roy B. Peplau C. Rogers D. Travelbee 24. In what phase of Nurse patient relationship does a nurse review the client’s medical records thereby learning as much as possible about the client? A. Pre Orientation B. Orientation C. Working D. Termination

25. Nurse Aida has seen her patient, Roger for the first time. She establish a contract about the frequency of meeting and introduce to Roger the expected termination. She started taking baseline assessment and set interventions and outcomes. On what phase of NPR Does Nurse Aida and Roger belong? A. Pre Orientation B. Orientation C. Working D. Termination 26. Roger has been seen agitated, shouting and running. As Nurse Aida approaches, he shouts and swear, calling Aida names. Nurse Aida told Roger “That is an unacceptable behavior Roger, Stop and go to your room now.” The situation is most likely in what phase of NPR? A. Pre Orientation B. Orientation C. Working D. Termination 27. Nurse Aida, in spite of the incident, still consider Roger as worthwhile simply because he is a human being. What major ingredient of a therapeutic communication is Nurse Aida using? A. Empathy B. Positive regard C. Comfortable sense of self D. Self awareness 28. Nurse Irma saw Roger and told Nurse Aida “ Oh look at that psychotic patient “ Nurse Aida should intervene and correct Nurse Irma because her statement shows that she is lacking? A. Empathy B. Positive regard C. Comfortable sense of self D. Self awareness 29. Which of the following statement is not true about stress? A. It is a nervous energy B. It is an essential aspect of existence C. It has been always a part of human experience D. It is something each person has to cope

30. Martina, a Tennis champ was devastated after many new competitors outpaced her in the Wimbledon event. She became depressed and always seen crying. Martina is clearly on what kind of situation? A. Martina is just stressed out B. Martina is Anxious C. Martina is in the exhaustion stage of GAS D. Martina is in Crisis 31. Which of the following statement is not true with regards to anxiety? A. It has physiologic component B. It has psychologic component C. The source of dread or uneasiness is from an unrecognized entity D. The source of dread or uneasiness is from a recognized entity 32. Lorraine, a 27 year old executive was brought to the ER for an unknown reason. She is starting to speak but her speech is disorganized and cannot be understood. On what level of anxiety does this features belongs? A. Mild B. Moderate C. Sever D. Panic 33. Elton, 21 year old nursing student is taking the board examination. She is sweating profusely, has decreased awareness of his environment and is purely focused on the exam questions characterized by his selective attentiveness. What anxiety level is Elton exemplifying? A. Mild B. Moderate C. Sever D. Panic 34. You noticed the patient chart : ANXIETY +3 What will you expect to see in this client? A. An optimal time for learning, Hearing and perception is greatly increased B. Dilated pupils C. Unable to communicate D. Uses palliative coping mechanism 35. When should the nurse starts giving XANAX?

A. When anxiety is +1 B. When the client starts to have a narrow perceptual field and selective inattentiveness C. When problem solving is not possible D. When the client is immobile and disorganized 36. Which of the following behavior is not a sign or a symptom of Anxiety? A. Frequent hand movement B. Somatization C. The client asks a question D. The client is acting out 37. Which of the following intervention is inappropriate for client’s with anxiety? A. Offer choices B. Provide a quiet and calm environment C. Provide detailed explanation on each and every procedures and equipments D. Bring anxiety down to a controllable level 38. Which of the following statement, if made by the nurse, is considered not therapeutic? A. “How did you deal with your anxiety before?” B. “It must be awful to feel anxious.” C. “How does it feel to be anxious?” D. “What makes you feel anxious?” 39. Marissa Salva, Uses Benson’s relaxation. How is it done? A. Systematically tensing muscle groups from top to bottom for 5 seconds, and then releasing them B. Concentrating on breathing without tensing the muscle, Letting go and repeating a word or sound after each exhalation C. Using a strong positive, feeling-rich statement about a desired change D. Exercise combined with meditation to foster relaxation and mental alacrity 40. What type of relaxation technique does Lyza uses if a machine is showing her pulse rate, temperature and muscle tension which she can visualize and assess? A. Biofeedback B. Massage C. Autogenic training D. Visualization and Imagery 41. This is also known as Self-suggestion or Self-hypnosis

A. Biofeedback B. Meditation C. Autogenic training D. Visualization and Imagery 42. Which among these drugs is NOT an anxiolytic? A. Valium B. Ativan C. Milltown D. Luvox 43. Kenneth, 25 year old diagnosed with HIV felt that he had not lived up with God’s expectation. He fears that in the course of his illness, God will be punitive and not be supportive. What kind of spiritual crisis is Kenneth experiencing? 1. Spiritual Pain 2. Spiritual Anxiety 3. Spiritual Guilt 4. Spiritual Despair A. 1,2 B. 2,3 C. 3,4 D. 1,4 44. Grace, believes that her relationship with God is broken. She tried to go to church to ask forgiveness everyday to remedy her feelings. What kind of spiritual distress is Grace experiencing? A. Spiritual Pan B. Spiritual Alienation C. Spiritual Guilt D. Spiritual Despair 45. Remedios felt “EMPTY” She felt that she has already lost God’s favor and love because of her sins. This is a type of what spiritual crisis? A. Spiritual Anger B. Spiritual Loss C. Spiritual Despair D. Spiritual Anxiety 46. Budek is working with a schizophrenic patient. He noticed that the client is agitated, pacing back and forth, restless and experiencing Anxiety +3. Budek said “You appear restless” What therapeutic technique did Budek used?

A. Offering general leads B. Seeking clarification C. Making observation D. Encouraging description of perception 47. Rommel told Budek “ I SEE DEAD PEOPLE “ Budek responded “You see dead people?” This Is an example of therapeutic communication technique? A. Reflecting B. Restating C. Exploring D. Seeking clarification 48. Rommel told Budek, “Do you think Im crazy?” Budek responded, “Do you think your crazy?” Budek uses what example of therapeutic communication? A. Reflecting B. Restating C. Exploring D. Seeking clarification 49. Myra, 21 year old nursing student has difficulty sleeping. She told Nurse Budek “I really think a lot about my x boyfriend recently” Budek told Myra “And that causes you difficulty sleeping?” Which therapeutic technique is used in this situation? A. Reflecting B. Restating C. Exploring D. Seeking clarification 50. Myra told Budek “I cannot sleep, I stay away all night” Budek told her “You have difficulty sleeping” This is what type of therapeutic communication technique? A. Reflecting B. Restating C. Exploring D. Seeking clarification 51. Myra said “I saw my dead grandmother here at my bedside a while ago” Budek responded “Really? That is hard to believe, How do you feel about it?” What technique did Budek used? A. Disproving B. Disagreeing

C. Voicing Doubt D. Presenting Reality 52. Which of the following is a therapeutic communication in response to “I am a GOD, bow before me Or ill summon the dreaded thunder to burn you and purge you to pieces!” A. “You are not a GOD, you are Professor Tadle and you are a PE Teacher, not a Nurse. I am Glen, Your nurse.” B. “Oh hail GOD Tadle, everyone bow or face his wrath!” C. “Hello Mr. Tadle, You are here in the hospital, I am your nurse and you are a patient here” D. “How can you be a GOD Mr. Tadle? Can you tell me more about it?” 53. Erik John Senna, Told Nurse Budek “ I don’t want to that, I don’t want that thing.. that’s too painful!” Which of the following response is NON THERAPEUTIC A. “ This must be difficult for you, But I need to inject you this for your own good” B. “ You sound afraid” C. “Are you telling me you don’t want this injection?” D. “Why are you so anxious? Please tell me more about your feelings Erik” 54. Legrande De Salvaje Y Cobrador La Jueteng, was caught by the bacolod police because of his illegal activities. When he got home after paying for the bail, He shouted at his son. What defense mechanism did Mr. La Jueteng used? A. Restitution B. Projection C. Displacement D. Undoing 55. Later that day, he bought his son ice cream and food. What defense mechanism is Legrande unconsciously doing? A. Restitution B. Conversion C. Redoing D. Reaction formation 56. Crisis is a sudden event in ones life that disturbs a person’s homeostasis. Which of the following is NOT TRUE in crisis? A. The person experiences heightened feeling of stress B. Inability to function in the usual organized manner C. Lasts for 4 months D. Indicates unpleasant emotional feelings

57. Which of the following is a characteristic of crisis? A. Lasts for an unlimited period of time B. There is a triggering event C. Situation is not dangerous to the person D. Person totality is not involved 58. Levito Devin, The Italian prime minister, is due to retire next week. He feels depressed due to the enormous loss of influence, power, fame and fortune. What type of crisis is Devin experiencing? A. Situational B. Maturational C. Social D. Phenomenal 59. Estrada, The Philippine president, has been unexpectedly impeached and was out of office before the end of his term. He is in what type of crisis? A. Situational B. Maturational C. Social D. Phenomenal 60. The tsunami in Thailand and Indonesia took thousands of people and change million lives. The people affected by the Tsunami are saddened and do not know how to start all over again. What type of crisis is this? A. Situational B. Maturational C. Social D. Phenomenal 61. Which of the following is the BEST goal for crisis intervention? A. Bring back the client in the pre crisis state B. Make sure that the client becomes better C. Achieve independence D. Provide alternate coping mechanism 62. What is the best intervention when the client has just experienced the crisis and still at the first phase of the crisis? A. Behavior therapy B. Gestalt therapy C. Cognitive therapy

D. Milieu Therapy 63. Therapeutic nurse client relationship is describes as follows 1. Based on friendship and mutual interest 2. It is a professional relationship 3. It is focused on helping the patient solve problems and achieve health-related goals 4. Maintained only as long as the patient requires professional help A. 1,2,3 B. 1,2,4 C. 2,3,4 D. 1,3,4 64. The client is scheduled to have surgical removal of the tumor on her left breast. Which of the following manifestation indicates that she is experiencing Mild Anxiety? A. She has increased awareness of her environmental details B. She focused on selected aspect of her illness C. She experiences incongruence of action, thoughts and feelings D. She experiences random motor activities 65. Which of the following nursing intervention would least likely be effective when dealing with a client with aggressive behavior? A. Approach him in a calm manner B. Provide opportunities to express feelings C. Maintain eye contact with the client D. Isolate the client from others 66. Whitney, a patient of nurse Budek, verbalizes… “I have nothing, nothing… nothing! Don't make me close one more door, I don't wanna hurt anymore!” Which of the following is the most appropriate response by Budek? A. Why are you singing? B. What makes you say that? C. Ofcourse you are everything! D. What is that you said? 67. Whitney verbalizes that she is anxious that the diagnostic test might reveal laryngeal cancer. Which of the following is the most appropriate nursing intervention? A. Tell the client not to worry until the results are in B. Ask the client to express feelings and concern C. Reassure the client everything will be alright D. Advice the client to divert his attention by watching television and reading newspapers

68. Considered as the most accurate expression of person’s thought and feelings A. Verbal communication B. Non verbal communication C. Written communication D. Oral communication 69. Represents inner feeling that a person do not like talking about. A. Overt communication B. Covert communication C. Verbal communication D. Non verbal communication 70. Which of the following is NOT a characteristic of an effective Nurse-Client relationship? A. Focused on the patient B. Based on mutual trust C. Conveys acceptance D. Discourages emotional bond 71. A type of record wherein , each person or department makes notation in separate records. A nurse will use the nursing notes, The doctor will use the Physician’s order sheet etc. Data is arranged according to information source. A. POMR B. POR C. Traditional D. Resource oriented 72. Type of recording that integrates all data about the problem, gathered by members of the health team. A. POMR B. Traditional C. Resource oriented D. Source oriented 73. These are data that are monitored by using graphic charts or graphs that indicated the progression or fluctuation of client’s Temperature and Blood pressure. A. Progress notes B. Kardex C. Flow chart

D. Flow sheet 74. Provides a concise method of organizing and recording data about the client. It is a series of flip cards kept in portable file used in change of shift reports. A. Kardex B. Progress Notes C. SOAPIE D. Change of shift report 75. You are about to write an information on the Kardex. There are 4 available writing instruments to use. Which of the following should you use? A. Mongol #2 B. Permanent Ink C. A felt or fountain pen D. Pilot Pentel Pen marker 76. The client has an allergy to Iodine based dye. Where should you put this vital information in the client’s chart? A. In the first page of the client’s chart B. At the last page of the client’s chart C. At the front metal plate of the chart D. In the Kardex 77. Which of the following is NOT TRUE about the Kardex A. It provides readily available information B. It is a tool of end of shift reports C. The primary basis of endorsement D. Where Allergies information are written 78. Which of the following, if seen on the Nurses notes, violates characteristic of good recording? A. The client has a blood pressure of 120/80, Temperature of 36.6 C Pulse rate of 120 and Respiratory rate of 22 B. Ate 50% of food served C. Refused administration of betaxolol D. Visited and seen By Dr. Santiago 79. The physician ordered : Mannerix a.c , what does a.c means? A. As desired B. Before meals

C. After meals D. Before bed time 80. The physician ordered, Maalox, 2 hours p.c, what does p.c means? A. As desired B. Before meals C. After meals D. Before bed time 81. The physician ordered, Maxitrol, Od. What does Od means? A. Left eye B. Right eye C. Both eye D. Once a day 82. The physician orderd, Magnesium Hydroxide cc Aluminum Hydroxide. What does cc means? A. without B. with C. one half D. With one half dose 83. Physician ordered, Paracetamol tablet ss. What does ss means? A. without B. with C. one half D. With one half dose 84. Which of the following indicates that learning has been achieved? A. Matuts starts exercising every morning and eating a balance diet after you taught her mag HL tayo program B. Donya Delilah has been able to repeat the steps of insulin administration after you taught it to her C. Marsha said “ I understand “ after you a health teaching about family planning D. John rated 100% on your given quiz about smoking and alcoholism 85. In his theory of learning as a BEHAVIORISM, he stated that transfer of knowledge occurs if a new situation closely resembles an old one. A. Bloom B. Lewin C. Thorndike

D. Skinner 86. Which of the following is TRUE with regards to learning? A. Start from complex to simple B. Goals should be hard to achieve so patient can strive to attain unrealistic goals C. Visual learning is the best for every individual D. Do not teach a client when he is in pain 87. According to Bloom, there are 3 domains in learning. Which of these domains is responsible for the ability of Donya Delilah to inject insulin? A. Cognitive B. Affective C. Psychomotor D. Motivative 88. Which domains of learning is responsible for making John and Marsha understand the different kinds of family planning methods? A. Cognitive B. Affective C. Psychomotor D. Motivative 89. Which of the following statement clearly defines therapeutic communication? A. Therapeutic communication is an interaction process which is primarily directed by the nurse B. It conveys feeling of warmth, acceptance and empathy from the nurse to a patient in relaxed atmosphere C. Therapeutic communication is a reciprocal interaction based on trust and aimed at identifying patient needs and developing mutual goals D. Therapeutic communication is an assessment component of the nursing process 90. Which of the following concept is most important in establishing a therapeutic nurse patient relationship? A. The nurse must fully understand the patient’s feelings, perception and reactions before goals can be established B. The nurse must be a role model for health fostering behavior C. The nurse must recognize that the patient may manifest maladaptive behavior after illness D. The nurse should understand that patients might test her before trust is established

91. Which of the following communication skill is most effective in dealing with covert communication? A. Validation B. Listening C. Evaluation D. Clarification 92. Which of the following are qualities of a good recording? 1. Brevity 2. Completeness and chronology 3. Appropriateness 4. Accuracy A. 1,2 B. 3,4 C. 1,2,3 D. 1,2,3,4 93. All of the following chart entries are correct except A. V/S 36.8 C,80,16,120/80 B. Complained of chest pain C. Seems agitated D. Able to ambulate without assistance 94. Which of the following teaching method is effective in client who needs to be educated about self injection of insulin? A. Detailed explanation B. Demonstration C. Use of pamphlets D. Film showing 95. What is the most important characteristic of a nurse patient relationship? A. It is growth facilitating B. Based on mutual understanding C. Fosters hope and confidence D. Involves primarily emotional bond 96. Which of the following nursing intervention is needed before teaching a client post spleenectomy deep breathing and coughing exercises? A. Tell the patient that deep breathing and coughing exercises is needed to promote good breathing, circulation and prevent complication

B. Tell the client that deep breathing and coughing exercises is needed to prevent Thrombophlebitis, hydrostatic pneumonia and atelectasis C. Medicate client for pain D. Tell client that cooperation is vital to improve recovery 97. The client has an allergy with penicillin. What is the best way to communicate this information? A. Place an allergy alert in the Kardex B. Notify the attending physician C. Write it on the patient’s chart D. Take note when giving medications 98. An adult client is on extreme pain. He is moaning and grimacing. What is the best way to assess the client’s pain? A. Perform physical assessment B. Have the client rate his pain on the smiley pain rating scale C. Active listening on what the patient says D. Observe the client’s behavior 99. Therapeutic communication begins with? A. Knowing your client B. Knowing yourself C. Showing empathy D. Encoding 100. The PCS gave new guidelines including leaflets to educate cancer patients. As a nurse, When using materials like this, what is your responsibility? A. Read it for the patient B. Give it for the patient to read himself C. Let the family member read the material for the patient D. Read it yourself then, Have the client read the material Answer Key http://pinoybsn.blogspot.com/2006/09/100-item-exam-on-fundamentals-of_14.html 100 Item exam on Fundamentals Of Nursing : Infection, Asepsis, Basic concept of stress and Illness Fundamentals Of Nursing : Infection, Asepsis, Basic concept of stress and Illness By : Budek http://www.pinoybsn.tk

FUNDAMENTALS OF NURSING TEST II CONTENT OUTLINE 1. Illness 2. Infection and Asepsis 3. Basic concept of Stress and Adaptation 1. When the General adaptation syndrome is activated, FLIGHT OR FIGHT response sets in. Sympathetic nervous system releases norepinephrine while the adrenal medulla secretes epinephrine. Which of the following is true with regards to that statement? A. Pupils will constrict B. Client will be lethargic C. Lungs will bronchodilate D. Gastric motility will increase 2. Which of the following response is not expected to a person whose GAS is activated and the FIGHT OR FLIGHT response sets in? A. The client will not urinate due to relaxation of the detrusor muscle B. The client will be restless and alert C. Clients BP will increase, there will be vasodilation D. There will be increase glycogenolysis, Pancrease will decrease insulin secretion 3. State in which a person’s physical, emotional, intellectual and social development or spiritual functioning is diminished or impaired compared with a previous experience. A. Illness B. Disease C. Health D. Wellness 4. This is the first stage of illness wherein, the person starts to believe that something is wrong. Also known as the transition phase from wellness to illness. A. Symptom Experience B. Assumption of sick role C. Medical care contact D. Dependent patient role 5. In this stage of illness, the person accepts or rejects a professionals suggestion. The person also becomes passive and may regress to an earlier stage.

A. Symptom Experience B. Assumption of sick role C. Medical care contact D. Dependent patient role 6. In this stage of illness, The person learns to accept the illness. A. Symptom Experience B. Assumption of sick role C. Medical care contact D. Dependent patient role 7. In this stage, the person tries to find answers for his illness. He wants his illness to be validated, his symptoms explained and the outcome reassured or predicted A. Symptom Experience B. Assumption of sick role C. Medical care contact D. Dependent patient role 8. The following are true with regards to aspect of the sick role except A. One should be held responsible for his condition B. One is excused from his societal role C. One is obliged to get well as soon as possible D. One is obliged to seek competent help 9. Refers to conditions that increases vulnerability of individual or group to illness or accident A. Predisposing factor B. Etiology C. Risk factor D. Modifiable Risks 10. Refers to the degree of resistance the potential host has against a certain pathogen A. Susceptibility B. Immunity C. Virulence D. Etiology 11. A group of symptoms that sums up or constitute a disease A. Syndrome B. Symptoms

C. Signs D. Etiology 12. A woman undergoing radiation therapy developed redness and burning of the skin around the best. This is best classified as what type of disease? A. Neoplastic B. Traumatic C. Nosocomial D. Iatrogenic 13. The classification of CANCER according to its etiology Is best described as 1. Nosocomial 2. Idiopathic 3. Neoplastic 4. Traumatic 5. Congenital 6. Degenrative A. 5 and 2 B. 2 and 3 C. 3 and 4 D. 3 and 5 14. Term to describe the reactiviation and recurrence of pronounced symptoms of a disease A. Remission B. Emission C. Exacerbation D. Sub acute 15. A type of illness characterized by periods of remission and exacerbation A. Chronic B. Acute C. Sub acute D. Sub chronic 16. Diseases that results from changes in the normal structure, from recognizable anatomical changes in an organ or body tissue is termed as A. Functional B. Occupational C. Inorganic

D. Organic 17. It is the science of organism as affected by factors in their environment. It deals with the relationship between disease and geographical environment. A. Epidemiology B. Ecology C. Statistics D. Geography 18. This is the study of the patterns of health and disease. Its occurrence and distribution in man, for the purpose of control and prevention of disease. A. Epidemiology B. Ecology C. Statistics D. Geography 19. Refers to diseases that produced no anatomic changes but as a result from abnormal response to a stimuli. A. Functional B. Occupational C. Inorganic D. Organic 20. In what level of prevention according to Leavell and Clark does the nurse support the client in obtaining OPTIMAL HEALTH STATUS after a disease or injury? A. Primary B. Secondary C. Tertiary D. None of the above 21. In what level of prevention does the nurse encourage optimal health and increases person’s susceptibility to illness? A. Primary B. Secondary C. Tertiary D. None of the above 22. Also known as HEALTH MAINTENANCE prevention. A. Primary B. Secondary

C. Tertiary D. None of the above 23. PPD In occupational health nursing is what type of prevention? A. Primary B. Secondary C. Tertiary D. None of the above 24. BCG in community health nursing is what type of prevention? A. Primary B. Secondary C. Tertiary D. None of the above 25. A regular pap smear for woman every 3 years after establishing normal pap smear for 3 consecutive years Is advocated. What level of prevention does this belongs? A. Primary B. Secondary C. Tertiary D. None of the above 26. Self monitoring of blood glucose for diabetic clients is on what level of prevention? A. Primary B. Secondary C. Tertiary D. None of the above 27. Which is the best way to disseminate information to the public? A. Newspaper B. School bulletins C. Community bill boards D. Radio and Television 28. Who conceptualized health as integration of parts and subparts of an individual? A. Newman B. Neuman C. Watson D. Rogers

29. The following are concept of health: 1. Health is a state of complete physical, mental and social wellbeing and not merely an absence of disease or infirmity. 2. Health is the ability to maintain balance 3. Health is the ability to maintain internal milieu 4. Health is integration of all parts and subparts of an individual A. 1,2,3 B. 1,3,4 C. 2,3,4 D. 1,2,3,4 30. The theorist the advocated that health is the ability to maintain dynamic equilibrium is A. Bernard B. Selye C. Cannon D. Rogers 31. Excessive alcohol intake is what type of risk factor? A. Genetics B. Age C. Environment D. Lifestyle 32. Osteoporosis and degenerative diseases like Osteoarthritis belongs to what type of risk factor? A. Genetics B. Age C. Environment D. Lifestyle 33. Also known as STERILE TECHNIQUE A. Surgical Asepsis B. Medical Asepsis C. Sepsis D. Asepsis 34. This is a person or animal, who is without signs of illness but harbors pathogen within his body and can be transferred to another

A. Host B. Agent C. Environment D. Carrier 35. Refers to a person or animal, known or believed to have been exposed to a disease. A. Carrier B. Contact C. Agent D. Host 36. A substance usually intended for use on inanimate objects, that destroys pathogens but not the spores. A. Sterilization B. Disinfectant C. Antiseptic D. Autoclave 37. This is a process of removing pathogens but not their spores A. Sterilization B. Auto claving C. Disinfection D. Medical asepsis 38. The third period of infectious processes characterized by development of specific signs and symptoms A. Incubation period B. Prodromal period C. Illness period D. Convalescent period 39. A child with measles developed fever and general weakness after being exposed to another child with rubella. In what stage of infectious process does this child belongs? A. Incubation period B. Prodromal period C. Illness period D. Convalescent period

40. A 50 year old mailman carried a mail with anthrax powder in it. A minute after exposure, he still hasn’t developed any signs and symptoms of anthrax. In what stage of infectious process does this man belongs? A. Incubation period B. Prodromal period C. Illness period D. Convalescent period 41. Considered as the WEAKEST LINK in the chain of infection that nurses can manipulate to prevent spread of infection and diseases A. Etiologic/Infectious agent B. Portal of Entry C. Susceptible host D. Mode of transmission 42. Which of the following is the exact order of the infection chain? 1. Susceptible host 2. Portal of entry 3. Portal of exit 4. Etiologic agent 5. Reservoir 6. Mode of transmission A. 1,2,3,4,5,6 B. 5,4,2,3,6,1 C. 4,5,3,6,2,1 D. 6,5,4,3,2,1 43. Markee, A 15 year old high school student asked you. What is the mode of transmission of Lyme disease. You correctly answered him that Lyme disease is transmitted via A. Direct contact transmission B. Vehicle borne transmission C. Air borne transmission D. Vector borne transmission 44. The ability of the infectious agent to cause a disease primarily depends on all of the following except A. Pathogenicity B. Virulence C. Invasiveness

D. Non Specificity 45. Contact transmission of infectious organism in the hospital is usually cause by A. Urinary catheterization B. Spread from patient to patient C. Spread by cross contamination via hands of caregiver D. Cause by unclean instruments used by doctors and nurses 46. Transmission occurs when an infected person sneezes, coughs or laugh that is usually projected at a distance of 3 feet. A. Droplet transmission B. Airborne transmission C. Vehicle transmission D. Vector borne transmission 47. Considered as the first line of defense of the body against infection A. Skin B. WBC C. Leukocytes D. Immunization 48. All of the following contributes to host susceptibility except A. Creed B. Immunization C. Current medication being taken D. Color of the skin 49. Graciel has been injected TT5, her last dosed for tetanus toxoid immunization. Graciel asked you, what type of immunity is TT Injections? You correctly answer her by saying Tetanus toxoid immunization is a/an A. Natural active immunity B. Natural passive immunity C. Artificial active immunity D. Artificial passive immunity 50. Agatha, was hacked and slashed by a psychotic man while she was crossing the railway. She suffered multiple injuries and was injected Tetanus toxoid Immunoglobulin. Agatha asked you, What immunity does TTIg provides? You best answered her by saying TTIg provides A. Natural active immunity

B. Natural passive immunity C. Artificial active immunity D. Artificial passive immunity 51. This is the single most important procedure that prevents cross contamination and infection A. Cleaning B. Disinfecting C. Sterilizing D. Handwashing 52. This is considered as the most important aspect of handwashing A. Time B. Friction C. Water D. Soap 53. In handwashing by medical asepsis, Hands are held …. A. Above the elbow, The hands must always be above the waist B. Above the elbow, The hands are cleaner than the elbow C. Below the elbow, Medical asepsis do not require hands to be above the waist D. Below the elbow, Hands are dirtier than the lower arms 54. The suggested time per hand on handwashing using the time method is A. 5 to 10 seconds each hand B. 10 to 15 seconds each hand C. 15 to 30 seconds each hand D. 30 to 60 seconds each hand 55. The minimum time in washing each hand should never be below A. 5 seconds B. 10 seconds C. 15 seconds D. 30 seconds 56. How many ml of liquid soap is recommended for handwashing procedure? A. 1-2 ml B. 2-3 ml C. 2-4 ml D. 5-10 ml

57. Which of the following is not true about sterilization, cleaning and disinfection? A. Equipment with small lumen are easier to clean B. Sterilization is the complete destruction of all viable microorganism including spores C. Some organism are easily destroyed, while other, with coagulated protein requires longer time D. The number of organism is directly proportional to the length of time required for sterilization 58. Karlita asked you, How long should she boil her glass baby bottle in water? You correctly answered her by saying A. The minimum time for boiling articles is 5 minutes B. Boil the glass baby bottler and other articles for atleast 10 minutes C. For boiling to be effective, a minimum of 15 minutes is required D. It doesn’t matter how long you boil the articles, as long as the water reached 100 degree Celsius 59. This type of disinfection is best done in sterilizing drugs, foods and other things that are required to be sterilized before taken in by the human body A. Boiling Water B. Gas sterilization C. Steam under pressure D. Radiation 60. A TB patient was discharged in the hospital. A UV Lamp was placed in the room where he stayed for a week. What type of disinfection is this? A. Concurrent disinfection B. Terminal disinfection C. Regular disinfection D. Routine disinfection 61. Which of the following is not true in implementing medical asepsis A. Wash hand before and after patient contact B. Keep soiled linens from touching the clothings C. Shake the linens to remove dust D. Practice good hygiene 62. Which of the following is true about autoclaving or steam under pressure? A. All kinds of microorganism and their spores are destroyed by autoclave machine

B. The autoclaved instruments can be used for 1 month considering the bags are still intact C. The instruments are put into unlocked position, on their hinge, during the autoclave D. Autoclaving different kinds of metals at one time is advisable 63. Which of the following is true about masks? A. Mask should only cover the nose B. Mask functions better if they are wet with alcohol C. Masks can provide durable protection even when worn for a long time and after each and every patient care D. N95 Mask or particulate masks can filter organism as mall as 1 micromillimeter 64. Where should you put a wet adult diaper? A. Green trashcan B. Black trashcan C. Orange trashcan D. Yellow trashcan 65. Needles, scalpels, broken glass and lancets are considered as injurious wastes. As a nurse, it is correct to put them at disposal via a/an A. Puncture proof container B. Reused PET Bottles C. Black trashcan D. Yellow trashcan with a tag “INJURIOUS WASTES” 66. Miranda Priestly, An executive of RAMP magazine, was diagnosed with cancer of the cervix. You noticed that the radioactive internal implant protrudes to her vagina where supposedly, it should be in her cervix. What should be your initial action? A. Using a long forceps, Push it back towards the cervix then call the physician B. Wear gloves, remove it gently and place it on a lead container C. Using a long forceps, Remove it and place it on a lead container D. Call the physician, You are not allowed to touch, re insert or remove it 67. After leech therapy, Where should you put the leeches? A. In specially marked BIO HAZARD Containers B. Yellow trashcan C. Black trashcan D. Leeches are brought back to the culture room, they are not thrown away for they are reusable

68. Which of the following should the nurse AVOID doing in preventing spread of infection? A. Recapping the needle before disposal to prevent injuries B. Never pointing a needle towards a body part C. Using only Standard precaution to AIDS Patients D. Do not give fresh and uncooked fruits and vegetables to Mr. Gatchie, with Neutropenia 69. Where should you put Mr. Alejar, with Category II TB? A. In a room with positive air pressure and atleast 3 air exchanges an hour B. In a room with positive air pressure and atleast 6 air exchanges an hour C. In a room with negative air pressure and atleast 3 air exchanges an hour D. In a room with negative air pressure and atleast 6 air exchanges an hour 70. A client has been diagnosed with RUBELLA. What precaution is used for this patient? A. Standard precaution B. Airborne precaution C. Droplet precaution D. Contact precaution 71. A client has been diagnosed with MEASLES. What precaution is used for this patient? A. Standard precaution B. Airborne precaution C. Droplet precaution D. Contact precaution 72. A client has been diagnosed with IMPETIGO. What precaution is used for this patient? A. Standard precaution B. Airborne precaution C. Droplet precaution D. Contact precaution 73. The nurse is to insert an NG Tube when suddenly, she accidentally dip the end of the tube in the client’s glass containing distilled drinking water which is definitely not sterile. As a nurse, what should you do? A. Don’t mind the incident, continue to insert the NG Tube B. Obtain a new NG Tube for the client

C. Disinfect the NG Tube before reinserting it again D. Ask your senior nurse what to do 74. All of the following are principle of SURGICAL ASEPSIS except A. Microorganism travels to moist surfaces faster than with dry surfaces B. When in doubt about the sterility of an object, consider it not sterile C. Once the skin has been sterilized, considered it sterile D. If you can reach the object by overreaching, just move around the sterile field to pick it rather than reaching for it 75. Which of the following is true in SURGICAL ASEPSIS? A. Autoclaved linens and gowns are considered sterile for about 4 months as long as the bagging is intact B. Surgical technique is a sole effort of each nurse C. Sterile conscience, is the best method to enhance sterile technique D. If a scrubbed person leaves the area of the sterile field, He/she must do handwashing and gloving again, but the gown need not be changed. 76. In putting sterile gloves, Which should be gloved first? A. The dominant hand B. The non dominant hand C. The left hand D. No specific order, Its up to the nurse for her own convenience 77. As the scrubbed nurse, when should you apply the goggles, shoe cap and mask prior to the operation? A. Immediately after entering the sterile field B. After surgical hand scrub C. Before surgical hand scrub D. Before entering the sterile field 78. Which of the following should the nurse do when applying gloves prior to a surgical procedure? A. Slipping gloved hand with all fingers when picking up the second glove B. Grasping the first glove by inserting four fingers, with thumbs up underneath the cuff C. Putting the gloves into the dominant hand first D. Adjust only the fitting of the gloves after both gloves are on 79. Which gloves should you remove first? A. The glove of the non dominant hand

B. The glove of the dominant hand C. The glove of the left hand D. Order in removing the gloves Is unnecessary 80. Before a surgical procedure, Give the sequence on applying the protective items listed below 1. Eye wear or goggles 2. Cap 3. Mask 4. Gloves 5. Gown A. 3,2,1,5,4 B. 3,2,1,4,5 C. 2,3,1,5,4 D. 2,3,1,4,5 81. In removing protective devices, which should be the exact sequence? 1. Eye wear or goggles 2. Cap 3. Mask 4. Gloves 5. Gown A. 4,3,5,1,2 B. 2,3,1,5,4 C. 5,4,3,2,1 D. 1,2,3,4,5 82. In pouring a plain NSS into a receptacle located in a sterile field, how high should the nurse hold the bottle above the receptacle? A. 1 inch B. 3 inches C. 6 inches D. 10 inches 83. The tip of the sterile forceps is considered sterile. It is used to manipulate the objects in the sterile field using the non sterile hands. How should the nurse hold a sterile forceps? A. The tip should always be lower than the handle B. The tip should always be above the handle C. The handle and the tip should be at the same level

D. The handle should point downward and the tip, always upward 84. The nurse enters the room of the client on airborne precaution due to tuberculosis. Which of the following are appropriate actions by the nurse? 1. She wears mask, covering the nose and mouth 2. She washes her hands before and after removing gloves, after suctioning the client’s secretion 3. She removes gloves and hands before leaving the client’s room 4. She discards contaminated suction catheter tip in trashcan found in the clients room A. 1,2 B. 1,2,3 C. 1,2,3,4 D. 1,3 85. When performing surgical hand scrub, which of the following nursing action is required to prevent contamination? 1. Keep fingernail short, clean and with nail polish 2. Open faucet with knee or foot control 3. Keep hands above the elbow when washing and rinsing 4. Wear cap, mask, shoe cover after you scrubbed A. 1,2 B. 2,3 C. 1,2,3 D. 2,3,4 86. When removing gloves, which of the following is an inappropriate nursing action? A. Wash gloved hand first B. Peel off gloves inside out C. Use glove to glove skin to skin technique D. Remove mask and gown before removing gloves 87. Which of the following is TRUE in the concept of stress? A. Stress is not always present in diseases and illnesses B. Stress are only psychological and manifests psychological symptoms C. All stressors evoke common adaptive response D. Hemostasis refers to the dynamic state of equilibrium 88. According to this theorist, in his modern stress theory, Stress is the non specific response of the body to any demand made upon it.

A. Hans Selye B. Walter Cannon C. Claude Bernard D. Martha Rogers 89. Which of the following is NOT TRUE with regards to the concept of Modern Stress Theory? A. Stress is not a nervous energy B. Man, whenever he encounters stresses, always adapts to it C. Stress is not always something to be avoided D. Stress does not always lead to distress 90. Which of the following is TRUE with regards to the concept of Modern Stress Theory? A. Stress is essential B. Man does not encounter stress if he is asleep C. A single stress can cause a disease D. Stress always leads to distress 91. Which of the following is TRUE in the stage of alarm of general adaptation syndrome? A. Results from the prolonged exposure to stress B. Levels or resistance is increased C. Characterized by adaptation D. Death can ensue 92. The stage of GAS where the adaptation mechanism begins A. Stage of Alarm B. Stage of Resistance C. Stage of Homeostasis D. Stage of Exhaustion 93. Stage of GAS Characterized by adaptation A. Stage of Alarm B. Stage of Resistance C. Stage of Homeostasis D. Stage of Exhaustion 94. Stage of GAS wherein, the Level of resistance are decreased A. Stage of Alarm

B. Stage of Resistance C. Stage of Homeostasis D. Stage of Exhaustion 95. Where in stages of GAS does a person moves back into HOMEOSTASIS? A. Stage of Alarm B. Stage of Resistance C. Stage of Homeostasis D. Stage of Exhaustion 96. Stage of GAS that results from prolonged exposure to stress. Here, death will ensue unless extra adaptive mechanisms are utilized A. Stage of Alarm B. Stage of Resistance C. Stage of Homeostasis D. Stage of Exhaustion 97. All but one is a characteristic of adaptive response A. This is an attempt to maintain homeostasis B. There is a totality of response C. Adaptive response is immediately mobilized, doesn’t require time D. Response varies from person to person 98. Andy, a newly hired nurse, starts to learn the new technology and electronic devices at the hospital. Which of the following mode of adaptation is Andy experiencing? A. Biologic/Physiologic adaptive mode B. Psychologic adaptive mode C. Sociocultural adaptive mode D. Technological adaptive mode 99. Andy is not yet fluent in French, but he works in Quebec where majority speaks French. He is starting to learn the language of the people. What type of adaptation is Andy experiencing? A. Biologic/Physiologic adaptive mode B. Psychologic adaptive mode C. Sociocultural adaptive mode D. Technological adaptive mode 100. Andy made an error and his senior nurse issued a written warning. Andy arrived in his house mad and kicked the door hard to shut it off. What adaptation mode is this?

A. Biologic/Physiologic adaptive mode B. Psychologic adaptive mode C. Sociocultural adaptive mode D. Technological adaptive mode Answer Key http://pinoybsn.blogspot.com/2006/09/fundamentals-of-nursing-infection_12.html 100 Item Exam on Fundamentals of Nursing : History , Concepts and Theories Fundamentals of Nursing : History , Concepts and Theories By : Budek http://www.pinoybsn.tk FUNDAMENTALS OF NURSING TEST I Content Outline 1. Professional Nursing A. Historical perspectives in nursing B. Nursing as a profession C. Theories in nursing D. Health care delivery system 1. The four major concepts in nursing theory are the A. Person, Environment, Nurse, Health B. Nurse, Person, Environment, Cure C. Promotive, Preventive, Curative, Rehabilitative D. Person, Environment, Nursing, Health 2. The act of utilizing the environment of the patient to assist him in his recovery is theorized by A. Nightingale B. Benner C. Swanson D. King 3. For her, Nursing is a theoretical system of knowledge that prescribes a process of analysis and action related to care of the ill person A. King B. Henderson C. Roy

D. Leininger 4. According to her, Nursing is a helping or assistive profession to persons who are wholly or partly dependent or when those who are supposedly caring for them are no longer able to give care. A. Henderson B. Orem C. Swanson D. Neuman 5. Nursing is a unique profession, Concerned with all the variables affecting an individual’s response to stressors, which are intra, inter and extra personal in nature. A. Neuman B. Johnson C. Watson D. Parse 6. The unique function of the nurse is to assist the individual, sick or well, in the performance of those activities contributing to health that he would perform unaided if he has the necessary strength, will and knowledge, and do this in such a way as to help him gain independence as rapidly as possible. A. Henderson B. Abdellah C. Levin D. Peplau 7. Caring is the essence and central unifying, a dominant domain that distinguishes nursing from other health disciplines. Care is an essential human need. A. Benner B. Watson C. Leininger D. Swanson 8. Caring involves 5 processes, KNOWING, BEING WITH, DOING FOR, ENABLING and MAINTAINING BELIEF. A. Benner B. Watson C. Leininger D. Swanson

9. Caring is healing, it is communicated through the consciousness of the nurse to the individual being cared for. It allows access to higher human spirit. A. Benner B. Watson C. Leininger D. Swanson 10. Caring means that person, events, projects and things matter to people. It reveals stress and coping options. Caring creates responsibility. It is an inherent feature of nursing practice. It helps the nurse assist clients to recover in the face of the illness. A. Benner B. Watson C. Leininger D. Swanson 11. Which of the following is NOT TRUE about profession according to Marie Jahoda? A. A profession is an organization of an occupational group based on the application of special knowledge B. It serves specific interest of a group C. It is altruistic D. Quality of work is of greater importance than the rewards 12. Which of the following is NOT an attribute of a professional? A. Concerned with quantity B. Self directed C. Committed to spirit of inquiry D. Independent 13. The most unique characteristic of nursing as a profession is A. Education B. Theory C. Caring D. Autonomy 14. This is the distinctive individual qualities that differentiate a person to another A. Philosophy B. Personality C. Charm D. Character

15. Refers to the moral values and beliefs that are used as guides to personal behavior and actions A. Philosophy B. Personality C. Charm D. Character 16. As a nurse manager, which of the following best describes this function? A. Initiate modification on client’s lifestyle B. Protect client’s right C. Coordinates the activities of other members of the health team in managing patient care D. Provide in service education programs, Use accurate nursing audit, formulate philosophy and vision of the institution. 17. What best describes nurses as a care provider? A. Determine client’s need B. Provide direct nursing care C. Help client recognize and cope with stressful psychological situation D. Works in combined effort with all those involved in patient’s care 18. The nurse questions a doctors order of Morphine sulfate 50 mg, IM for a client with pancreatitis. Which role best fit that statement? A. Change agent B. Client advocate C. Case manager D. Collaborator 19. These are nursing intervention that requires knowledge, skills and expertise of multiple health professionals. A. Dependent B. Independent C. Interdependent D. Intradependent 20. What type of patient care model is the most common for student nurses and private duty nurses? A. Total patient care B. Team nursing C. Primary Nursing D. Case management

21. This is the best patient care model when there are many nurses but few patients. A. Functional nursing B. Team nursing C. Primary nursing D. Total patient care 22. This patient care model works best when there are plenty of patient but few nurses A. Functional nursing B. Team nursing C. Primary nursing D. Total patient care 23. RN assumes 24 hour responsibility for the client to maintain continuity of care across shifts, days or visits. A. Functional nursing B. Team nursing C. Primary nursing D. Total patient care 24. Who developed the first theory of nursing? A. Hammurabi B. Alexander C. Fabiola D. Nightingale 25. She introduces the NATURE OF NURSING MODEL. A. Henderson B. Nightingale C. Parse D. Orlando 26. She described the four conservation principle. A. Levin B. Leininger C. Orlando D. Parse 27. Proposed the HEALTH CARE SYSTEM MODEL.

A. Henderson B. Orem C. Parse D. Neuman 28. Conceptualized the BEHAVIORAL SYSTEM MODEL A. Orem B. Johnson C. Henderson D. Parse 29. Developed the CLINICAL NURSING – A HELPING ART MODEL A. Swanson B. Hall C. Weidenbach D. Zderad 30. Developed the ROLE MODELING and MODELING theory A. Erickson,Tomlin,Swain B. Neuman C. Newman D. Benner and Wrubel 31. Proposed the GRAND THEORY OF NURSING AS CARING A. Erickson, Tomlin, Swain B. Peterson,Zderad C. Bnner,Wrubel D. Boykin,Schoenhofer 32. Postulated the INTERPERSONAL ASPECT OF NURSING A. Travelbee B. Swanson C. Zderad D. Peplau 33. He proposed the theory of morality that is based on MUTUAL TRUST A. Freud B. Erikson C. Kohlberg

D. Peters 34. He proposed the theory of morality based on PRINCIPLES A. Freud B. Erikson C. Kohlberg D. Peters 35. Freud postulated that child adopts parental standards and traits through A. Imitation B. Introjection C. Identification D. Regression 36. According to them, Morality is measured of how people treat human being and that a moral child strives to be kind and just A. Zderad and Peterson B. Benner and Wrubel C. Fowler and Westerhoff D. Schulman and Mekler 37. Postulated that FAITH is the way of behaving. He developed four theories of faith and development based on his experience. A. Giligan B. Westerhoff C. Fowler D. Freud 38. He described the development of faith. He suggested that faith is a spiritual dimension that gives meaning to a persons life. Faith according to him, is a relational phenomenon. A. Giligan B. Westerhoff C. Fowler D. Freud 39. Established in 1906 by the Baptist foreign mission society of America. Miss rose nicolet, was it’s first superintendent. A. St. Paul Hospital School of nursing B. Iloilo Mission Hospital School of nursing C. Philippine General Hospital School of nursing

D. St. Luke’s Hospital School of nursing 40. Anastacia Giron-Tupas was the first Filipino nurse to occupy the position of chief nurse in this hospital. A. St. Paul Hospital B. Iloilo Mission Hospital C. Philippine General Hospital D. St. Luke’s Hospital 41. She was the daughter of Hungarian kings, who feed 300-900 people everyday in their gate, builds hospitals, and care of the poor and sick herself. A. Elizabeth B. Catherine C. Nightingale D. Sairey Gamp 42. She dies of yellow fever in her search for truth to prove that yellow fever is carried by a mosquitoes. A. Clara louise Maas B. Pearl Tucker C. Isabel Hampton Robb D. Caroline Hampton Robb 43. He was called the father of sanitation. A. Abraham B. Hippocrates C. Moses D. Willam Halstead 44. The country where SHUSHURUTU originated A. China B. Egypt C. India D. Babylonia 45. They put girls clothes on male infants to drive evil forces away A. Chinese B. Egyptian C. Indian D. Babylonian

46. In what period of nursing does people believe in TREPHINING to drive evil forces away? A. Dark period B. Intuitive period C. Contemporary period D. Educative period 47. This period ended when Pastor Fliedner, build Kaiserwerth institute for the training of Deaconesses A. Apprentice period B. Dark period C. Contemporary period D. Educative period 48. Period of nursing where religious Christian orders emerged to take care of the sick A. Apprentice period B. Dark period C. Contemporary period D. Educative period 49. Founded the second order of St. Francis of Assisi A. St. Catherine B. St. Anne C. St. Clare D. St. Elizabeth 50. This period marked the religious upheaval of Luther, Who questions the Christian faith. A. Apprentice period B. Dark period C. Contemporary period D. Educative period 51. According to the Biopsychosocial and spiritual theory of Sister Callista Roy, Man, As a SOCIAL being is A. Like all other men B. Like some other men C. Like no other men D. Like men

52. She conceptualized that man, as an Open system is in constant interaction and transaction with a changing environment. A. Roy B. Levin C. Neuman D. Newman 53. In a CLOSED system, which of the following is true? A. Affected by matter B. A sole island in vast ocean C. Allows input D. Constantly affected by matter, energy, information 54. Who postulated the WHOLISTIC concept that the totality is greater than sum of its parts? A. Roy B. Rogers C. Henderson D. Johnson 55. She theorized that man is composed of sub and supra systems. Subsystems are cells, tissues, organs and systems while the suprasystems are family, society and community. A. Roy B. Rogers C. Henderson D. Johnson 56. Which of the following is not true about the human needs? A. Certain needs are common to all people B. Needs should be followed exactly in accordance with their hierarchy C. Needs are stimulated by internal factors D. Needs are stimulated by external factors 57. Which of the following is TRUE about the human needs? A. May not be deferred B. Are not interrelated C. Met in exact and rigid way D. Priorities are alterable

58. According to Maslow, which of the following is NOT TRUE about a self actualized person? A. Understands poetry, music, philosophy, science etc. B. Desires privacy, autonomous C. Follows the decision of the majority, uphold justice and truth D. Problem centered 59. According to Maslow, which of the following is TRUE about a self actualized person? A. Makes decision contrary to public opinion B. Do not predict events C. Self centered D. Maximum degree of self conflict 60. This is the essence of mental health A. Self awareness B. Self actualization C. Self esteem D. Self worth 61. Florence nightingale is born in A. Germany B. Britain C. France D. Italy 62. Which is unlikely of Florence Nightingale? A. Born May 12, 1840 B. Built St. Thomas school of nursing when she was 40 years old C. Notes in nursing D. Notes in hospital 63. What country did Florence Nightingale train in nursing? A. Belgium B. US C. Germany D. England 64. Which of the following is recognized for developing the concept of HIGH LEVEL WELLNESS?

A. Erikson B. Madaw C. Peplau D. Dunn 65. One of the expectations is for nurses to join professional association primarily because of A. Promotes advancement and professional growth among its members B. Works for raising funds for nurse’s benefit C. Facilitate and establishes acquaintances D. Assist them and securing jobs abroad 66. Founder of the PNA A. Julita Sotejo B. Anastacia Giron Tupas C. Eufemia Octaviano D. Anesia Dionisio 67. Which of the following provides that nurses must be a member of a national nurse organization? A. R.A 877 B. 1981 Code of ethics approved by the house of delegates and the PNA C. Board resolution No. 1955 Promulgated by the BON D. RA 7164 68. Which of the following best describes the action of a nurse who documents her nursing diagnosis? A. She documents it and charts it whenever necessary B. She can be accused of malpractice C. She does it regularly as an important responsibility D. She charts it only when the patient is acutely ill 69. Which of the following does not govern nursing practice? A. RA 7164 B. RA 9173 C. BON Res. Code Of Ethics D. BON Res. Scope of Nursing Practice 70. A nurse who is maintaining a private clinic in the community renders service on maternal and child health among the neighborhood for a fee is:

A. Primary care nurse B. Independent nurse practitioner C. Nurse-Midwife D. Nurse specialist 71. When was the PNA founded? A. September 22, 1922 B. September 02, 1920 C. October 21, 1922 D. September 02, 1922 72. Who was the first president of the PNA ? A. Anastacia Giron-Tupas B. Loreto Tupas C. Rosario Montenegro D. Ricarda Mendoza 73. Defines health as the ability to maintain internal milieu. Illness according to him/her/them is the failure to maintain internal environment. A. Cannon B. Bernard C. Leddy and Pepper D. Roy 74. Postulated that health is a state and process of being and becoming an integrated and whole person. A. Cannon B. Bernard C. Dunn D. Roy 75. What regulates HOMEOSTASIS according to the theory of Walter Cannon? A. Positive feedback B. Negative feedback C. Buffer system D. Various mechanisms 76. Stated that health is WELLNESS. A termed define by the culture or an individual. A. Roy

B. Henderson C. Rogers D. King 77. Defined health as a dynamic state in the life cycle, and Illness as interference in the life cycle. A. Roy B. Henderson C. Rogers D. King 78. She defined health as the soundness and wholness of developed human structure and bodily mental functioning. A. Orem B. Henderson C. Neuman D. Clark 79. According to her, Wellness is a condition in which all parts and subparts of an individual are in harmony with the whole system. A. Orem B. Henderson C. Neuman D. Johnson 80. Postulated that health is reflected by the organization, interaction, interdependence and integration of the subsystem of the behavioral system. A. Orem B. Henderson C. Neuman D. Johnson 81. According to them, Well being is a subjective perception of BALANCE, HARMONY and VITALITY A. Leavell and Clark B. Peterson and Zderad C. Benner and Wruber D. Leddy and Pepper 82. He describes the WELLNESS-ILLNESS Continuum as interaction of the environment with well being and illness.

A. Cannon B. Bernard C. Dunn D. Clark 83. An integrated method of functioning that is oriented towards maximizing one’s potential within the limitation of the environment. A. Well being B. Health C. Low level Wellness D. High level Wellness 84. What kind of illness precursor, according to DUNN is cigarette smoking? A. Heredity B. Social C. Behavioral D. Environmental 85. According to DUNN, Overcrowding is what type of illness precursor? A. Heredity B. Social C. Behavioral D. Environmental 86. Health belief model was formulated in 1975 by who? A. Becker B. Smith C. Dunn D. Leavell and Clark 87. In health belief model, Individual perception matters. Which of the following is highly UNLIKELY to influence preventive behavior? A. Perceived susceptibility to an illness B. Perceived seriousness of an illness C. Perceived threat of an illness D. Perceived curability of an illness 88. Which of the following is not a PERCEIVED BARRIER in preventive action? A. Difficulty adhering to the lifestyle

B. Economic factors C. Accessibility of health care facilities D. Increase adherence to medical therapies 89. Conceptualizes that health is a condition of actualization or realization of person’s potential. Avers that the highest aspiration of people is fulfillment and complete development actualization. A. Clinical Model B. Role performance Model C. Adaptive Model D. Eudaemonistic Model 90. Views people as physiologic system and Absence of sign and symptoms equates health. A. Clinical Model B. Role performance Model C. Adaptive Model D. Eudaemonistic Model 91. Knowledge about the disease and prior contact with it is what type of VARIABLE according to the health belief model? A. Demographic B. Sociopsychologic C. Structural D. Cues to action 92. It includes internal and external factors that leads the individual to seek help A. Demographic B. Sociopsychologic C. Structural D. Cues to action 93. Influence from peers and social pressure is included in what variable of HBM? A. Demographic B. Sociopsychologic C. Structural D. Cues to action 94. Age, Sex, Race etc. is included in what variable of HBM? A. Demographic

B. Sociopsychologic C. Structural D. Cues to action 95. According to Leavell and Clark’s ecologic model, All of this are factors that affects health and illness except A. Reservoir B. Agent C. Environment D. Host 96. Is a multi dimensional model developed by PENDER that describes the nature of persons as they interact within the environment to pursue health A. Ecologic Model B. Health Belief Model C. Health Promotion Model D. Health Prevention Model 97. Defined by Pender as all activities directed toward increasing the level of well being and self actualization. A. Health prevention B. Health promotion C. Health teaching D. Self actualization 98. Defined as an alteration in normal function resulting in reduction of capacities and shortening of life span. A. Illness B. Disease C. Health D. Wellness 99. Personal state in which a person feels unhealthy A. Illness B. Disease C. Health D. Wellness 100. According to her, Caring is defined as a nurturant way of responding to a valued client towards whom the nurse feels a sense of commitment and responsibility.

A. Benner B. Watson C. Leininger D. Swanson Answer Key http://pinoybsn.blogspot.com/2006/09/fundamentals-of-nursing-history_06.html 50 item Medical-Surgical Nursing Practice Test Mr. Duffy is admitted to the CCU with a diagnosis of R/O MI. He presented in the ER with a typical description of pain associated with an MI, and is now cold and clammy, pale and dyspneic. He has an IV of D5W running, and is complaining of chest pain. Oxygen therapy has not been started, and he is not on the monitor. He is frightened. 1. The nurse is aware of several important tasks that should all be done immediately in order to give Mr. Duffy the care he needs. Which of the following nursing interventions will relieve his current myocardial ischemia? a. stool softeners, rest b. O2 therapy, analgesia c. Reassurance, cardiac monitoring d. Adequate fluid intake, low-fat diet 2. During the first three days that Mr. Duffy is in the CCU, a number of diagnostic blood tests are obtained. Which of the following patterns of cardiac enzyme elevation are most common following an MI? a. SGOT, CK, and LDH are all elevated immediately. b. SGOT rises 4-6 hours after infarction with CK and LDH rising slowly 24 hours later. c. CK peaks first (12-24 hours), followed by the SGOT (peaks in 24-36 hours) and then the LDH (peaks 3-4 days). d. CK peaks first and remains elevated for 1 to 2 weeks. 3. On his second day in CCU Mr. Duffy suffers a life-threatening cardiac arrhythmia. Considering his diagnosis, which is the most probable arrhythmia? a. atrial tachycardia b. ventricular fibrillation c. atrial fibrillation d. heart block 4. Mr. Duffy is placed on digitalis on discharge from the hospital. The nurse planning with him for his discharge should educate him as to the purpose and actions of his new

medication. What should she or he teach Mr. Duffy to do at home to monitor his reaction to this medication? a. take his blood pressure b. take his radial pulse for one minute c. check his serum potassium (K) level d. weigh himself everyday You are speaking to an elderly group of diabetics in the OPD about eye health and the importance of visits to the ophthalmologist. 5. You decide to discuss glaucoma prevention. Which of the following diagnostic tests should these clients request from their care provider? a. fluorescein stain b. snellen’s test c. tonometry d. slit lamp 6. You also explain common eye changes associated with aging. One of these is presbyopia, which is: a. Refractive error that prevents light rays from coming to a single focus on the retina. b. Poor distant vision c. Poor near vision d. A gradual lessening of the power of accommodation 7. Some of the diabetic clients are interested in understanding what is visualized during funduscopic examination. During your discussion you describe the macular area as: a. Head of the optic nerve, seen on the nasal side of the field, lighter in color than the retina. b. The area of central vision, seen on the temporal side of the optic disc, which is quite avascular. c. Area where the central retinal artery and vein appear on the retina. d. Reddish orange in color, sometimes stippled. 8. One of the clients has noted a raised yellow plaque on the nasal side of the conjunctiva. You explain that this is called: a. a pinguecula, which is normal slightly raised fatty structure under the conjunctiva that may gradually increase with age. b. Icterus, which may be due to liver disease. c. A pterygium, which will interfere with vision. d. Ciliary flush caused by congestion of the ciliary artery.

You are caring for Mr. Kaplan who has chronic renal failure (uremia) 9. You know that all but one of the following may eventually result in uremia. Which option is not implicated? a. glomerular disease b. uncontrolled hypertension c. renal disease secondary to drugs, toxins, infections, or radiations d. all of the above 10. You did the initial assessment on Mr. Kaplan when he came to your unit. What classical signs and symptoms did you note? a. fruity- smelling breath. b. Weakness, anorexia, pruritus c. Polyuria, polydipsia, polyphagia d. Ruddy complexion 11. Numerous drugs have been used on Mr. Kaplan in an attempt to stabilize him. Regarding his diagnosis and management of his drugs, you know that: a. The half-life of many drugs is decreased in uremia; thus dosage may have to be increased to be effective. b. Drug toxicity is a major concern in uremia; individualization of therapy and often a decrease in dose is essential. c. Drug therapy is not usually affected by this diagnosis d. Precautions should be taken with prescription drugs, but most OTC medications are safe for him to use. You are assigned to cardiac clinic to fill in for a colleague for 3 weeks. You begin by reviewing assessment of the cardiovascular system in your mind and asking yourself the following: 12. The point of maximum impulse (PMI) is an important landmark in the cardiac exam. Which statement best describes the location of the PMI in the healthy adult? a. Base of the heart, 5th intercostal space, 7-9 cm to the left of the midsternal line. b. Base of the heart, 7th intercostal space, 7-9 cm to the left of the midsternal line. c. Apex of the heart, intercostal space, 7-9 cm to the left of the midsternal line. d. Apex of the heart, intercostal space, 7-9 cm to the left of the midsternal line. 13. During the physical examination of the well adult client, the health care provider auscultates the heart. When the stethoscope is placed on the 5th intercostal space along the left sternal border, which valve closure is best evaluated? a. Tricuspid

b. Pulmonic c. Aortic d. Mitral 14. The pulmonic component of which heart sound is best heard at the 2nd LICS at the LSB? a. S1 b. S2 c. S3 d. S4 15. The coronary arteries furnish blood supply to the myocardium. Which of the following is a true statement relative to the coronary circulation? a. the right and left coronary arteries are the first of many branches off the ascending aorta b. blood enters the right and left coronary arteries during systole only c. the right coronary artery forms almost a complete circle around the heart, yet supplies only the right ventricle d. the left coronary artery has two main branches, the left anterior descending and left circumflex: both supply the left ventricle Sally Baker, a 40-year-old woman, is admitted to the hospital with an established diagnosis of mitral stenosis. She is scheduled for surgery to repair her mitral valve. 16. Ms. Baker has decided to have surgical correction of her stenosed valve at this time because her subjective complaints of dyspnea, hemoptysis, orthopnea, and paroxysmal nocturnal dyspnea have become unmanageable. These complaints are probably due to: a. thickening of the pericardium b. right heart failure c. pulmonary hypertension d. left ventricular hypertrophy 17. On physical exam of Ms. Baker, several abnormal findings can be observed. Which of the following is not one of the usual objective findings associated with mitral stenosis? a. low-pitched rumbling diastolic murmur, precordial thrill, and parasternal lift b. small crepitant rales at the bases of the lungs c. weak, irregular pulse, and peripheral and facial cyanosis in severe disease d. chest x-ray shows left ventricular hypertrophy 18. You are seeing more clients with diagnoses of mitral valve prolapse. You know those mitral valve prolapse is usually a benign cardiac condition, but may be associated with atypical chest pain. This chest pain is probably caused by:

a. ventricular ischemia b. dysfunction of the left ventricle c. papillary muscle ischemia and dysfunction d. cardiac arrythmias Mr. Oliver, a long term heavy smoker, is admitted to the hospital for a diagnostic workup. His possible diagnosis is cancer of the lung. 19. The most common lethal cancer in males between their fifth and seventh decades is: a. cancer of the prostate b. cancer of the lung c. cancer of the pancreas d. cancer of the bowel 20. Of the four basic cell types of lung cancer listed below, which is always associated with smoking? a. adenocarcinoma b. squamous cell carcinoma (epidermoid) c. undifferenciated carcinoma d. bronchoalveolar carcinoma 21. Chemotherapy may be used in combination with surgery in the treatment of lung cancer. Special nursing considerations with chemotherapy include all but which of the following? a. Helping the client deal with depression secondary to the diagnosis and its treatment b. Explaining that the reactions to chemotherapy are minimal c. Careful observation of the IV site of the administration of the drugs d. Careful attention to blood count results 22. Which of the following operative procedures of the thorax is paired with the correct definition? a. Pneumonectomy: removal of the entire lung b. Wedge resection: removal of one or more lobes of a lung c. Decortication: removal of the reibs or sections of ribs d. Thoracoplasty: removal of fibrous membrane that develops over visceral pleura as a result of emphysema Mr. Liberatore, age 76, is admitted to your unit. He has a past medical history of hypertension, DM, hyperlipidemia. Recently he has had several episodes where he stops talking in midsentence and stares into space. Today the episode lasted for 15 minutes. The admission diagnosis is impending CVA.

23. The episodes Mr. Liberatore has been experiencing are probably: a. small cerebral hemorrhages b. TIA’s or transient ischemic attacks c. Secondary to hypoglycemia d. Secondary to hyperglycemia 24. Mr. Liberatore suffers a left sided CVA. He is right handed. The nurse should expect: a. left-sided paralysis b. visual loss c. no alterations in speech d. no impairment of bladder function 25. Upper motor neuron disease may be manifested in which of the following clinical signs? a. spastic paralysis, hyperreflexia, presence of babinski reflex b. flaccid paralysis, hyporeflexia c. muscle atrophy, fasciculations d. decreased or absent voluntary movement Julie, an 18-year-old girl, is brought into the ER by her mother with the chief complaint of sudden visual disturbance that began half an hour ago and was described as double vision and flashing lights. 26. During your assessment of Julie she tells you all visual symptoms are gone but that she now has a severe pounding headache over her left eye. You suspect Julie may have: a. a tension headache b. the aura and headache of migraine c. a brain tumor d. a conversion reaction 27. You explain to Julie and her mother that migraine headaches are caused by: a. an allergic response triggered by stress b. dilation of cerebral arteries c. persistent contraction of the muscles of the head, neck and face d. increased intracranial pressure 28. A thorough history reveals that hormonal changes associated with menstruation may have triggered Julie’s migraine attack. In investigating Julie’s history what factors would be least significant in migraine?

a. seasonal allergies b. trigger foods such as alcohol, MSG, chocolate c. family history of migraine d. warning sign of onset, or aura 29. A client with muscle contraction headache will exhibit a pattern different for Julie’s. Which of the following is more compatible with tension headache? a. severe aching pain behind both eyes b. headache worse when bending over c. a bandlike burning around the neck d. feeling of tightness bitemporally, occipitally, or in the neck Mr. Snyder is admitted to your unit with a brain tumor. The type of tumor he has is currently unknown. You begin to think about the way brain tumors are classified. 30. Glioma is an intracranial tumor. Which of the following statements about gliomas do you know to be false? a. 50% of all intracranial tumors are gliomas b. gliomas are usually benign c. they grow rapidly and often cannot be totally excised from the surrounding tissue d. most glioma victims die within a year after diagnosis 31. Acoustic neuromas produce symptoms of progressive nerve deafness, tinnitus, and vertigo due to pressure and eventual destruction of: a. CN5 b. CN7 c. CN8 d. The ossicles 32. Whether Mr Snyder’s tumor is benign or malignant, it will eventually cause increased intracranial pressure. Signs and symptoms of increasing intracranial pressure may include all of the following except: a. headache, nausea, and vomiting b. papilledema, dizziness, mental status changes c. obvious motor deficits d. increased pulse rate, drop in blood pressure 33. Mr Snyder is scheduled for surgery in the morning, and you are surprised to find out that there is no order for an enema. You assess the situation and conclude that the reason for this is:

a. Mr. Snyder has had some mental changes due to the tumor and would find an enema terribly traumatic b. Straining to evacuate the enema might increase the intracranial pressure c. Mr. Snyder had been on clear liquids and then was NPO for several days, so an enema is not necessary d. An oversight and you call the physician to obtain the order 34. Postoperatively Mr. Snyder needs vigilant nursing care including all of the following except: a. Keeping his head flat b. Assessments q ½ hour of LOC, VS, papillary responses, and mental status c. Helping him avoid straining at stool, vomiting, or coughing d. Providing a caring, supportive atmosphere for him and his family 35. Potential postintracranial surgery problems include all but which of the following? a. increased ICP b. extracranial hemorrhage c. seizures d. leakage of cerebrospinal fluid Mrs. Hogan, a 43-year-old woman, is admitted to your unit for cholecystectomy. 36. You are responsible for teaching Mrs. Hogan deep breathing and coughing exercises. Why are these exercises especially important for Mrs. Hogan? a. they prevent postoperative atelectasis and pneumonia b. the incision in gallbladder surgery is in the subcostal area, which makes the client reluctant to take a deep breath and cough c. because she is probably overweight and will be less willing to breathe, cough, and move postoperatively 37. On the morning of Mrs. Hogan’s planned cholecystectomy she awakens with a pain in her right scapular area and thinks she slept in poor position. While doing the preop check list you note that on her routine CB report her WBC is 15,000. Your responsibility at this point is: a. to notify the surgeon at once; this is an elevated WBC indicating an inflammatory reaction b. to record this finding in a prominent place on the preop checklist and in your preop notes c. to call the laboratory for a STAT repeat WBC d. none. This is not an unusual finding

38. Mrs. Hogan is scheduled for surgery 2 days later and is to be given atropine 0.3 mg IM and Demerol 50 mg IM one hour preoperatively. Which nursing actions follow the giving of the preop medication? a. have her void soon after receiving the medication b. allow her family to be with her before the medication takes effect c. bring her valuables to the nursing station d. reinforce preop teaching 39. Mrs. Hogan is transported to the recovery room following her cholecystectomy. As you continue to check her vital signs you note a continuing trend in Mrs. Hogan’s status: her BP is gradually dropping and her pulse rate is increasing. Your most appropriate nursing action is to: a. order whole blood for Mrs. Hogan from the lab b. increase IV fluid rate of infusion and place in trendelenburg position c. immediately report signs of shock to the head nurse and/or surgeon and monitor VS closely d. place in lateral sims position to facilitate breathing 40. Mrs. Hogan returns to your clinical unit following discharge from the recovery room. Her vital signs are stable and her family is with her. Postoperative leg exercises should be inititated: a. after the physician writes the order b. after the family leaves c. if Mrs. Hogan will not be ambulated early d. stat 41. An oropharyngeal airway may: a. Not be used in a conscious patient. b. Cause airway obstruction. c. Prevent a patient from biting and occluding an ET tube. d. Be inserted "upside down" into the mouth opening and then rotated into the proper orientation as it is advanced into the mouth. e. All of the above. 42. Endotracheal intubation: a. Can be attempted for up to 2 minutes before you need to stop and ventilate the patient. b. Reduces the risk of aspiration of gastric contents. c. Should be performed with the neck flexed forward making the chin touch the chest. d. Should be performed after a patient is found to be not breathing and two breaths have been given but before checking for a pulse.

43. When giving bag-valve mask ventilations: a. Rapid and forceful ventilations are desirable so that adequate ventilation will be assured b. Effective ventilations can always be given by one person. c. Cricoid pressure may prevent gastric inflation during ventilations. d. Tidal volumes will always be larger than when giving mouth to pocket mask ventilations. 44. If breath sounds are only heard on the right side after intubation: a. Extubate, ventilate for 30 seconds then try again. b. The patient probably only has one lung, the right. c. You have intubated the stomach. d. Pull the tube back and listen again. 45. An esophageal obturator airway (EOA): a. Can be inserted by any person trained in ACLS. b. Requires visualization of the trachea before insertion. c. Never causes regurgitation. d. Should not be used with a conscious person, pediatric patients, or patients who have swallowed caustic substances. 46. During an acute myocardial infarct (MI): a. A patient may have a normal appearing ECG. b. Chest pain will always be present. c. A targeted history is rarely useful in making the diagnosis of MI. d. The chest pain is rarely described as crushing, pressing, or heavy. 47. The most common lethal arrhythmia in the first hour of an MI is: a. Pulseless Ventricular Tachycardia b. Asystole c. Ventricular fibrillation d. First degree heart block. 48. Which of the following is true about verapamil? a. It is used for wide-complex tachycardia. b. It may cause a drop in blood pressure. c. It is a first line drug for Pulseless Electrical Activity. d. It is useful for treatment of severe hypotension. 49. Atropine:

a. Is always given for a heart rate less than 60 bpm. b. Cannot be given via ET tube. c. Has a maximum total dosage of 0.03-0.04 mg/kg IV in the setting of cardiac arrest. d. When given IV, should always be given slowly. 50. Asystole should not be "defibrillated." a. True b. False http://pinoybsn.blogspot.com/2006/10/50-item-ms-practice-test-with-answers.html 109 Questions on Psychotic Disorders 109 Questions on Psychotic Disorders 1. A psychotic client reports to the evening nurse that the day nurse put something suspicious in his water with his medication. The nurse replies, "You're worried about your medication?" The nurse's communication is: A. an example of presenting reality. B. reinforcing the client's delusions. C. focusing on emotional content. D. a nontherapeutic technique called mind reading. 2. A client is admitted to the inpatient unit of the mental health center with a diagnosis of paranoid schizophrenia. He's shouting that the government of France is trying to assassinate him. Which of the following responses is most appropriate? A. "I think you're wrong. France is a friendly country and an ally of the United States. Their government wouldn't try to kill you." B. "I find it hard to believe that a foreign government or anyone else is trying to hurt you. You must feel frightened by this." C. "You're wrong. Nobody is trying to kill you." D. "A foreign government is trying to kill you? Please tell me more about it." 3. Propranolol (Inderal) is used in the mental health setting to manage which of the following conditions? A. Antipsychotic-induced akathisia and anxiety B. The manic phase of bipolar illness as a mood stabilizer C. Delusions for clients suffering from schizophrenia D. Obsessive-compulsive disorder (OCD) to reduce ritualistic behavior

4. A client with borderline personality disorder becomes angry when he is told that today's psychotherapy session with the nurse will be delayed 30 minutes because of an emergency. When the session finally begins, the client expresses anger. Which response by the nurse would be most helpful in dealing with the client's anger? A. "If it had been your emergency, I would have made the other client wait." B. "I know it's frustrating to wait. I'm sorry this happened." C. "You had to wait. Can we talk about how this is making you feel right now?" D. "I really care about you and I'll never let this happen again." 5. How soon after chlorpromazine (Thorazine) administration should the nurse expect to see a client's delusional thoughts and hallucinations eliminated A. Several minutes B. Several hours C. Several days D. Several weeks 6. A client receiving haloperidol (Haldol) complains of a stiff jaw and difficulty swallowing. The nurse's first action is to: A. reassure the client and administer as needed lorazepam (Ativan) I.M. B. administer as needed dose of benztropine (Cogentin) I.M. as ordered. C. administer as needed dose of benztropine (Cogentin) by mouth as ordered. D. administer as needed dose of haloperidol (Haldol) by mouth. 7. A client with a diagnosis of paranoid schizophrenia comments to the nurse, "How do I know what is really in those pills?" Which of the following is the best response? A. Say, "You know it's your medicine." B. Allow him to open the individual wrappers of the medication. C. Say, "Don't worry about what is in the pills. It's what is ordered." D. Ignore the comment because it's probably a joke. 8. The nurse is caring for a client with schizophrenia who experiences auditory hallucinations. The client appears to be listening to someone who isn't visible. He gestures, shouts angrily, and stops shouting in mid-sentence. Which nursing intervention is the most appropriate? A. Approach the client and touch him to get his attention. B. Encourage the client to go to his room where he'll experience fewer distractions. C. Acknowledge that the client is hearing voices but make it clear that the nurse doesn't hear these voices. D. Ask the client to describe what the voices are saying.

9. Yesterday, a client with schizophrenia began treatment with haloperidol (Haldol). Today, the nurse notices that the client is holding his head to one side and complaining of neck and jaw spasms. What should the nurse do? A. Assume that the client is posturing. B. Tell the client to lie down and relax. C. Evaluate the client for adverse reactions to haloperidol. D. Put the client on the list for the physician to see tomorrow 10. A client with paranoid schizophrenia has been experiencing auditory hallucinations for many years. One approach that has proven to be effective for hallucinating clients is to: A. take an as-needed dose of psychotropic medication whenever they hear voices. B. practice saying "Go away" or "Stop" when they hear voices. C. sing loudly to drown out the voices and provide a distraction. D. go to their room until the voices go away. 11. A client with catatonic schizophrenia is mute, can't perform activities of daily living, and stares out the window for hours. What is the nurse's first priority? A. Assist the client with feeding. B. Assist the client with showering. C. Reassure the client about safety. D. Encourage socialization with peers. 12. A client tells the nurse that the television newscaster is sending a secret message to her. The nurse suspects the client is experiencing: A. a delusion. B. flight of ideas. C. ideas of reference. D. a hallucination. 13. The nurse knows that the physician has ordered the liquid form of the drug chlorpromazine (Thorazine) rather than the tablet form because the liquid: A. has a more predictable onset of action. B. produces fewer anticholinergic effects. C. produces fewer drug interactions. D. has a longer duration of action. 14. A client who has been hospitalized with disorganized type schizophrenia for 8 years can't complete activities of daily living (ADLs) without staff direction and assistance. The nurse formulates a nursing diagnosis of Self-care deficient: Dressing/grooming

related to inability to function without assistance. What is an appropriate goal for this client? A. "Client will be able to complete ADLs independently within 1 month." B. "Client will be able to complete ADLs with only verbal encouragement within 1 month." C. "Client will be able to complete ADLs with assistance in organizing grooming items and clothing within 1 month." D. "Client will be able to complete ADLs with complete assistance within 1 month." 15. The nurse is planning care for a client admitted to the psychiatric unit with a diagnosis of paranoid schizophrenia. Which nursing diagnosis should receive the highest priority? A. Risk for violence toward self or others B. Imbalanced nutrition: Less than body requirements C. Ineffective family coping D. Impaired verbal communication 16. The nurse is preparing for the discharge of a client who has been hospitalized for paranoid schizophrenia. The client's husband expresses concern over whether his wife will continue to take her daily prescribed medication. The nurse should inform him that: A. his concern is valid but his wife is an adult and has the right to make her own decisions. B. he can easily mix the medication in his wife's food if she stops taking it. C. his wife can be given a long-acting medication that is administered every 1 to 4 weeks. D. his wife knows she must take her medication as prescribed to avoid future hospitalizations. 17. Benztropine (Cogentin) is used to treat the extrapyramidal effects induced by antipsychotics. This drug exerts its effect by: A. decreasing the anxiety causing muscle rigidity. B. blocking the cholinergic activity in the central nervous system (CNS). C. increasing the level of acetylcholine in the CNS. D. increasing norepinephrine in the CNS. 18. A client is admitted to the inpatient unit of the mental health center with a diagnosis of paranoid schizophrenia. He's shouting that the government of France is trying to assassinate him. Which of the following responses is most appropriate? A. "I think you're wrong. France is a friendly country and an ally of the United States. Their government wouldn't try to kill you." B. "I find it hard to believe that a foreign government or anyone else is trying to hurt you. You must feel frightened by this."

C. "You're wrong. Nobody is trying to kill you." D. "A foreign government is trying to kill you? Please tell me more about it." 19. A dopamine receptor agonist such as bromocriptine (Parlodel) relieves muscle rigidity caused by antipsychotic medication by: A. blocking dopamine receptors in the central nervous system (CNS). B. blocking acetylcholine in the CNS. C. activating norepinephrine in the CNS. D. activating dopamine receptors in the CNS. 20. Most antipsychotic medications exert which of following effects on the central nervous system (CNS)? A. Stimulate the CNS by blocking postsynaptic dopamine, norepinephrine, and serotonin receptors. B. Sedate the CNS by stimulating serotonin at the synaptic cleft. C. Depress the CNS by blocking the postsynaptic transmission of dopamine, serotonin, and norepinephrine. D. Depress the CNS by stimulating the release of acetylcholine. 21. A client is admitted to the psychiatric unit of a local hospital with chronic undifferentiated schizophrenia. During the next several days, the client is seen laughing, yelling, and talking to herself. This behavior is characteristic of: A. delusion. B. looseness of association. C. illusion. D. hallucination. 22. Which of the following medications would the nurse expect the physician to order to reverse a dystonic reaction? A. prochlorperazine (Compazine) B. diphenhydramine (Benadryl) C. haloperidol (Haldol) D. midazolam (Versed) 23. A schizophrenic client states, "I hear the voice of King Tut." Which response by the nurse would be most therapeutic? A. "I don't hear the voice, but I know you hear what sounds like a voice." B. "You shouldn't focus on that voice." C. "Don't worry about the voice as long as it doesn't belong to anyone real." D. "King Tut has been dead for years."

24. A psychotic client reports to the evening nurse that the day nurse put something suspicious in his water with his medication. The nurse replies, "You're worried about your medication?" The nurse's communication is: A. an example of presenting reality. B. reinforcing the client's delusions. C. focusing on emotional content. D. a nontherapeutic technique called mind reading. 25. The nurse is caring for a client with schizophrenia who experiences auditory hallucinations. The client appears to be listening to someone who isn't visible. He gestures, shouts angrily, and stops shouting in mid-sentence. Which nursing intervention is the most appropriate? A. Approach the client and touch him to get his attention. B. Encourage the client to go to his room where he'll experience fewer distractions. C. Acknowledge that the client is hearing voices but make it clear that the nurse doesn't hear these voices. D. Ask the client to describe what the voices are saying 26. A client has been receiving chlorpromazine (Thorazine), an antipsychotic, to treat his psychosis. Which findings should alert the nurse that the client is experiencing pseudoparkinsonism? A. Restlessness, difficulty sitting still, and pacing B. Involuntary rolling of the eyes C. Tremors, shuffling gait, and masklike face D. Extremity and neck spasms, facial grimacing, and jerky movements 27. For several years, a client with chronic schizophrenia has received 10 mg of fluphenazine hydrochloride (Prolixin) by mouth four times per day. Now the client has a temperature of 102° F (38.9° C), a heart rate of 120 beats/minute, a respiratory rate of 20 breaths/minute, and a blood pressure of 210/140 mm Hg. Because the client also is confused and incontinent, the nurse suspects malignant neuroleptic syndrome. What steps should the nurse take? A. Give the next dose of fluphenazine, call the physician, and monitor vital signs. B. Withhold the next dose of fluphenazine, call the physician, and monitor vital signs. C. Give the next dose of fluphenazine and restrict the client to the room to decrease stimulation. D. Withhold the next dose of fluphenazine, administer an antipyretic agent, and increase the client's fluid intake. 28. A schizophrenic client with delusions tells the nurse, "There is a man wearing a red coat who's out to get me." The client exhibits increasing anxiety when focusing on the delusions. Which of the following would be the best response?

A. "This subject seems to be troubling you. Let's walk to the activity room." B. "Describe the man who's out to get you. What does he look like?" C. "There is no reason to be afraid of that man. This hospital is very secure." D. "There is no need to be concerned with a man who isn't even real." 29. Important teaching for women in their childbearing years who are receiving antipsychotic medications includes which of the following? A. Occurrence of increased libido due to medication adverse effects B. Increased incidence of dysmenorrhea while taking the drug C. Continuing previous use of contraception during periods of amenorrhea D. Instruction that amenorrhea is irreversible 30. A client is admitted to a psychiatric facility with a diagnosis of chronic schizophrenia. The history indicates that the client has been taking neuroleptic medication for many years. Assessment reveals unusual movements of the tongue, neck, and arms. Which condition should the nurse suspect? A. Tardive dyskinesia B. Dystonia C. Neuroleptic malignant syndrome D. Akathisia 31. What medication would probably be ordered for the acutely aggressive schizophrenic client? A. chlorpromazine (Thorazine) B. haloperidol (Haldol) C. lithium carbonate (Lithonate) D. amitriptyline (Elavil) 32. A client is admitted with a diagnosis of schizotypal personality disorder. Which signs would this client exhibit during social situations? A. Aggressive behavior B. Paranoid thoughts C. Emotional affect D. Independence needs 33. During the initial interview, a client with schizophrenia suddenly turns to the empty chair beside him and whispers, "Now just leave. I told you to stay home. There isn't enough work here for both of us!" What is the nurse's best initial response? A. "When people are under stress, they may see things or hear things that others don't. Is that what just happened?"

B. "I'm having a difficult time hearing you. Please look at me when you talk." C. "There is no one else in the room. What are you doing?" D. "Who are you talking to? Are you hallucinating?" 34. The definition of nihilistic delusions is: A. a false belief about the functioning of the body. B. belief that the body is deformed or defective in a specific way. C. false ideas about the self, others, or the world. D. the inability to carry out motor activities. 35. A client who's taking antipsychotic medication develops a very high temperature, severe muscle rigidity, tachycardia, and rapid deterioration in mental status. The nurse suspects what complication of antipsychotic therapy? A. Agranulocytosis B. Extrapyramidal effects C. Anticholinergic effects D. Neuroleptic malignant syndrome (NMS) 36. The nurse formulates a nursing diagnosis of Impaired social interaction related to disorganized thinking for a client with schizotypal personality disorder. Based on this nursing diagnosis, which nursing intervention takes highest priority? A. Helping the client to participate in social interactions B. Establishing a one-on-one relationship with the client C. Exploring the effects of the client's behavior on social interactions D. Developing a schedule for the client's participation in social interactions 37. A client with schizophrenia hears a voice telling him he is evil and must die. The nurse understands that the client is experiencing: A. a delusion. B. flight of ideas. C. ideas of reference. D. a hallucination. 38. A client with delusional thinking shows a lack of interest in eating at meal times. She states that she is unworthy of eating and that her children will die if she eats. Which nursing action would be most appropriate for this client? A. Telling the client that she may become sick and die unless she eats B. Paying special attention to the client's rituals and emotions associated with meals C. Restricting the client's access to food except at specified meal and snack times D. Encouraging the client to express her feelings at meal times

39. Which of the following groups of characteristics would the nurse expect to see in the client with schizophrenia? A. Loose associations, grandiose delusions, and auditory hallucinations B. Periods of hyperactivity and irritability alternating with depression C. Delusions of jealousy and persecution, paranoia, and mistrust D. Sadness, apathy, feelings of worthlessness, anorexia, and weight loss 40. The nurse must administer a medication to reverse or prevent Parkinson-type symptoms in a client receiving an antipsychotic. The medication the client will likely receive is: A. benztropine (Cogentin). B. diphenhydramine (Benadryl). C. propranolol (Inderal). D. haloperidol (Haldol). 41. A client is receiving haloperidol (Haldol) to reduce psychotic symptoms. As he watches television with other clients, the nurse notes that he has trouble sitting still. He seems restless, constantly moving his hands and feet and changing position. When the nurse asks what is wrong, he says he feels jittery. How should the nurse manage this situation? A. Ask the client to sit still or leave the room because he is distracting the other clients. B. Ask the client if he is nervous or anxious about something. C. Give an as needed dose of a prescribed anticholinergic agent to control akathisia. D. Administer an as needed dose of haloperidol to decrease agitation. 42. A man is brought to the hospital by his wife, who states that for the past week her husband has refused all meals and accused her of trying to poison him. During the initial interview, the client's speech, only partly comprehensible, reveals that his thoughts are controlled by delusions that he is possessed by the devil. The physician diagnoses paranoid schizophrenia. Schizophrenia is best described as a disorder characterized by: A. disturbed relationships related to an inability to communicate and think clearly. B. severe mood swings and periods of low to high activity. C. multiple personalities, one of which is more destructive than the others. D. auditory and tactile hallucinations. 43. A client has a history of chronic undifferentiated schizophrenia. Because she has a history of noncompliance with antipsychotic therapy, she'll receive fluphenazine decanoate (Prolixin Decanoate) injections every 4 weeks. Before discharge, what should the nurse include in her teaching plan? A. Asking the physician for droperidol (Inapsine) to control any extrapyramidal symptoms that occur

B. Sitting up for a few minutes before standing to minimize orthostatic hypotension C. Notifying the physician if her thoughts don't normalize within 1 week D. Expecting symptoms of tardive dyskinesia to occur and to be transient 44. A client with chronic schizophrenia who takes neuroleptic medication is admitted to the psychiatric unit. Nursing assessment reveals rigidity, fever, hypertension, and diaphoresis. These findings suggest which life-threatening reaction: A. tardive dyskinesia. B. dystonia. C. neuroleptic malignant syndrome. D. akathisia. 45. While looking out the window, a client with schizophrenia remarks, "That school across the street has creatures in it that are waiting for me." Which of the following terms best describes what the creatures represent? A. Anxiety attack B. Projection C. Hallucination D. Delusion 46. A client with schizophrenia tells the nurse, "My intestines are rotted from the worms chewing on them." This statement indicates a: A. delusion of persecution. B. delusion of grandeur. C. somatic delusion. D. jealous delusion. 47. During the assessment stage, a client with schizophrenia leaves his arm in the air after the nurse has taken his blood pressure. His action shows evidence of: A. somatic delusions. B. waxy flexibility. C. neologisms. D. nihilistic delusions. 48. A client with paranoid type schizophrenia becomes angry and tells the nurse to leave him alone. The nurse should A. tell him that she'll leave for now but will return soon. B. ask him if it's okay if she sits quietly with him. C. ask him why he wants to be left alone. D. tell him that she won't let anything happen to him

49. Nursing care for a client with schizophrenia must be based on valid psychiatric and nursing theories. The nurse's interpersonal communication with the client and specific nursing interventions must be: A. clearly identified with boundaries and specifically defined roles. B. warm and nonthreatening. C. centered on clearly defined limits and expression of empathy. D. flexible enough for the nurse to adjust the plan of care as the situation warrants. 50. When discharging a client after treatment for a dystonic reaction, the emergency department nurse must ensure that the client understands which of the following? A. Results of treatment are rapid and dramatic but may not last. B. Although uncomfortable, this reaction isn't serious. C. The client shouldn't buy drugs on the street. D. The client must take benztropine (Cogentin) as prescribed to prevent a return of symptoms. 51. The nurse is caring for a client with schizophrenia. Which of the following outcomes is the least desirable? A. The client spends more time by himself. B. The client doesn't engage in delusional thinking. C. The client doesn't harm himself or others. D. The client demonstrates the ability to meet his own self-care needs. 52. The nurse formulates a nursing diagnosis of Impaired verbal communication for a client with schizotypal personality disorder. Based on this nursing diagnosis, which nursing intervention is most appropriate? A. Helping the client to participate in social interactions B. Establishing a one-on-one relationship with the client C. Establishing alternative forms of communication D. Allowing the client to decide when he wants to participate in verbal communication with the nurse 53. Since admission 4 days ago, a client has refused to take a shower, stating, "There are poison crystals hidden in the showerhead. They'll kill me if I take a shower." Which nursing action is most appropriate? A. Dismantling the showerhead and showing the client that there is nothing in it B. Explaining that other clients are complaining about the client's body odor C. Asking a security officer to assist in giving the client a shower D. Accepting these fears and allowing the client to take a sponge bath

54. Drug therapy with thioridazine (Mellaril) shouldn't exceed a daily dose of 800 mg to prevent which adverse reaction? A. Hypertension B. Respiratory arrest C. Tourette syndrome D. Retinal pigmentation A. "I get upset once in a while, too." B. "I know just how you feel. I'd feel the same way in your situation." C. "I worry, too, when I think people are talking about me." D. "At times, it's normal not to trust anyone." 56. How soon after chlorpromazine (Thorazine) administration should the nurse expect to see a client's delusional thoughts and hallucinations eliminated? A. Several minutes B. Several hours C. Several days D. Several weeks 57. A client is about to be discharged with a prescription for the antipsychotic agent haloperidol (Haldol), 10 mg by mouth twice per day. During a discharge teaching session, the nurse should provide which instruction to the client? A. Take the medication 1 hour before a meal. B. Decrease the dosage if signs of illness decrease. C. Apply a sunscreen before being exposed to the sun. D. Increase the dosage up to 50 mg twice per day if signs of illness don't decrease. 58. A client with paranoid schizophrenia repeatedly uses profanity during an activity therapy session. Which response by the nurse would be most appropriate? A. "Your behavior won't be tolerated. Go to your room immediately." B. "You're just doing this to get back at me for making you come to therapy." C. "Your cursing is interrupting the activity. Take time out in your room for 10 minutes." D. "I'm disappointed in you. You can't control yourself even for a few minutes." 59. Which of the following is one of the advantages of the newer antipsychotic medication risperidone (Risperdal)? A. The absence of anticholinergic effects B. A lower incidence of extrapyramidal effects C. Photosensitivity and sedation D. No incidence of neuroleptic malignant syndrome

60. The etiology of schizophrenia is best described by: A. genetics due to a faulty dopamine receptor. B. environmental factors and poor parenting. C. structural and neurobiological factors. D. a combination of biological, psychological, and environmental factors. 61. A client with schizophrenia who receives fluphenazine (Prolixin) develops pseudoparkinsonism and akinesia. What drug would the nurse administer to minimize extrapyramidal symptoms? A. benztropine (Cogentin) B. dantrolene (Dantrium) C. clonazepam (Klonopin) D. diazepam (Valium) 62. A client with a diagnosis of paranoid schizophrenia comments to the nurse, "How do I know what is really in those pills?" Which of the following is the best response? A. Say, "You know it's your medicine." B. Allow him to open the individual wrappers of the medication. C. Say, "Don't worry about what is in the pills. It's what is ordered." D. Ignore the comment because it's probably a joke. 63. A client tells the nurse that people from Mars are going to invade the earth. Which response by the nurse would be most therapeutic? A. "That must be frightening to you. Can you tell me how you feel about it?" B. "There are no people living on Mars." C. "What do you mean when you say they're going to invade the earth?" D. "I know you believe the earth is going to be invaded, but I don't believe that." 64. A client with schizophrenia tells the nurse he hears the voices of his dead parents. To help the client ignore the voices, the nurse should recommend that he: A. sit in a quiet, dark room and concentrate on the voices. B. listen to a personal stereo through headphones and sing along with the music. C. call a friend and discuss the voices and his feelings about them. D. engage in strenuous exercise. 65. A client with schizophrenia is receiving antipsychotic medication. Which nursing diagnosis may be appropriate for this client? A. Ineffective protection related to blood dyscrasias B. Urinary frequency related to adverse effects of antipsychotic medication C. Risk for injury related to a severely decreased level of consciousness

D. Risk for injury related to electrolyte disturbances 66. A client with persistent, severe schizophrenia has been treated with phenothiazines for the past 17 years. Now the client's speech is garbled as a result of drug-induced rhythmic tongue protrusion. What is another name for this extrapyramidal symptom? A. Dystonia B. Akathisia C. Pseudoparkinsonism D. Tardive dyskinesia 67. The nurse is assigned to a client with catatonic schizophrenia. Which intervention should the nurse include in the client's plan of care? A. Meeting all of the client's physical needs B. Giving the client an opportunity to express concerns C. Administering lithium carbonate (Lithonate) as prescribed D. Providing a quiet environment where the client can be alone 68. A client with a history of medication noncompliance is receiving outpatient treatment for chronic undifferentiated schizophrenia. The physician is most likely to prescribe which medication for this client? A. chlorpromazine (Thorazine) B. imipramine (Tofranil) C. lithium carbonate (Lithane) D. fluphenazine decanoate (Prolixin Decanoate) 69. Propranolol (Inderal) is used in the mental health setting to manage which of the following conditions? A. Antipsychotic-induced akathisia and anxiety B. The manic phase of bipolar illness as a mood stabilizer C. Delusions for clients suffering from schizophrenia D. Obsessive-compulsive disorder (OCD) to reduce ritualistic behavior 70. Every day for the past 2 weeks, a client with schizophrenia stands up during group therapy and screams, "Get out of here right now! The elevator bombs are going to explode in 3 minutes!" The next time this happens, how should the nurse respond? A. "Why do you think there is a bomb in the elevator?" B. "That is the same thing you said in yesterday's session." C. "I know you think there are bombs in the elevator, but there aren't." D. "If you have something to say, you must do it according to our group rules."

71. A 26-year-old client is admitted to the psychiatric unit with acute onset of schizophrenia. His physician prescribes the phenothiazine chlorpromazine (Thorazine), 100 mg by mouth four times per day. Before administering the drug, the nurse reviews the client's medication history. Concomitant use of which drug is likely to increase the risk of extrapyramidal effects? A. guanethidine (Ismelin) B. droperidol (Inapsine) C. lithium carbonate (Lithonate) D. alcohol 72. A client, age 36, with paranoid schizophrenia believes the room is bugged by the Central Intelligence Agency and that his roommate is a foreign spy. The client has never had a romantic relationship, has no contact with family members, and hasn't been employed in the last 14 years. Based on Erikson's theories, the nurse should recognize that this client is in which stage of psychosocial development? A. Autonomy versus shame and doubt B. Generativity versus stagnation C. Integrity versus despair D. Trust versus mistrust 73. During a group therapy session in the psychiatric unit, a client constantly interrupts with impulsive behavior and exaggerated stories that cast her as a hero or princess. She also manipulates the group with attention-seeking behaviors, such as sexual comments and angry outbursts. The nurse realizes that these behaviors are typical of: A. paranoid personality disorder. B. avoidant personality disorder. C. histrionic personality disorder. D. borderline personality disorder. 74. The nurse is teaching a psychiatric client about her prescribed drugs, chlorpromazine and benztropine. Why is benztropine administered? A. To reduce psychotic symptoms B. To reduce extrapyramidal symptoms C. To control nausea and vomiting D. To relieve anxiety 75. A client is admitted to the psychiatric unit with a tentative diagnosis of psychosis. Her physician prescribes the phenothiazine thioridazine (Mellaril) 50 mg by mouth three times per day. Phenothiazines differ from central nervous system (CNS) depressants in their sedative effects by producing: A. deeper sleep than CNS depressants. B. greater sedation than CNS depressants.

C. a calming effect from which the client is easily aroused. D. more prolonged sedative effects, making the client more difficult to arouse. A. Schizophrenia B. Paranoid personality C. Bipolar illness D. Obsessive-compulsive disorder (OCD) 77. A client with paranoid schizophrenia is admitted to the psychiatric unit of a hospital. Nursing assessment should include careful observation of the client's: A. thinking, perceiving, and decision-making skills. B. verbal and nonverbal communication processes. C. affect and behavior. D. psychomotor activity. 78. Which information is most important for the nurse to include in a teaching plan for a schizophrenic client taking clozapine (Clozaril)? A. Monthly blood tests will be necessary. B. Report a sore throat or fever to the physician immediately. C. Blood pressure must be monitored for hypertension. D. Stop the medication when symptoms subside. 79. Important teaching for clients receiving antipsychotic medication such as haloperidol (Haldol) includes which of the following instructions? A. Use sunscreen because of photosensitivity. B. Take the antipsychotic medication with food. C. Have routine blood tests to determine levels of the medication. D. Abstain from eating aged cheese. 80. Positive symptoms of schizophrenia include which of the following? A. Hallucinations, delusions, and disorganized thinking B. Somatic delusions, echolalia, and a flat affect C. Waxy flexibility, alogia, and apathy D. Flat affect, avolition, and anhedonia 81. A client with chronic schizophrenia receives 20 mg of fluphenazine decanoate (Prolixin Decanoate) by I.M. injection. Three days later, the client has muscle contractions that contort the neck. This client is exhibiting which extrapyramidal reaction? A. Dystonia B. Akinesia

C. Akathisia D. Tardive dyskinesia 82. Hormonal effects of the antipsychotic medications include which of the following? A. Retrograde ejaculation and gynecomastia B. Dysmenorrhea and increased vaginal bleeding C. Polydipsia and dysmenorrhea D. Akinesia and dysphasia 83. A client is unable to get out of bed and get dressed unless the nurse prompts every step. This is an example of which behavior? A. Word salad B. Tangential C. Perseveration D. Avolition 84. An agitated and incoherent client, age 29, comes to the emergency department with complaints of visual and auditory hallucinations. The history reveals that the client was hospitalized for paranoid schizophrenia from ages 20 to 21. The physician prescribes haloperidol (Haldol), 5 mg I.M. The nurse understands that this drug is used in this client to treat: A. dyskinesia. B. dementia. C. psychosis. D. tardive dyskinesia. 85. Yesterday, a client with schizophrenia began treatment with haloperidol (Haldol). Today, the nurse notices that the client is holding his head to one side and complaining of neck and jaw spasms. What should the nurse do? A. Assume that the client is posturing. B. Tell the client to lie down and relax. C. Evaluate the client for adverse reactions to haloperidol. D. Put the client on the list for the physician to see tomorrow. 86. A client receiving fluphenazine decanoate (Prolixin Decanoate) therapy develops pseudoparkinsonism. The physician is likely to prescribe which drug to control this extrapyramidal effect? A. phenytoin (Dilantin) B. amantadine (Symmetrel) C. benztropine (Cogentin) D. diphenhydramine (Benadryl)

87. Important teaching for a client receiving risperidone (Risperdal) would include advising the client to: A. double the dose if missed to maintain a therapeutic level. B. be sure to take the drug with a meal because it's very irritating to the stomach. C. discontinue the drug if the client reports weight gain. D. notify the physician if the client notices an increase in bruising. 88. A client is admitted to the psychiatric hospital with a diagnosis of catatonic schizophrenia. During the physical examination, the client's arm remains outstretched after the nurse obtains the pulse and blood pressure, and the nurse must reposition the arm. This client is exhibiting: A. suggestibility. B. negativity. C. waxy flexibility. D. retardation. 89. A client with borderline personality disorder becomes angry when he is told that today's psychotherapy session with the nurse will be delayed 30 minutes because of an emergency. When the session finally begins, the client expresses anger. Which response by the nurse would be most helpful in dealing with the client's anger? A. "If it had been your emergency, I would have made the other client wait." B. "I know it's frustrating to wait. I'm sorry this happened." C. "You had to wait. Can we talk about how this is making you feel right now?" D. "I really care about you and I'll never let this happen again." 90. A client begins clozapine (Clozaril) therapy after several other antipsychotic agents fail to relieve her psychotic symptoms. The nurse instructs her to return for weekly white blood cell (WBC) counts to assess for which adverse reaction? A. Hepatitis B. Infection C. Granulocytopenia D. Systemic dermatitis 91. Which nonantipsychotic medication is used to treat some clients with schizoaffective disorder? A. phenelzine (Nardil) B. chlordiazepoxide (Librium) C. lithium carbonate (Lithane) D. imipramine (Tofranil)

92. A client diagnosed with schizoaffective disorder is suffering from schizophrenia with elements of which of the following disorders? A. Personality disorder B. Mood disorder C. Thought disorder D. Amnestic disorder 93. When teaching the family of a client with schizophrenia, the nurse should provide which information? A. Relapse can be prevented if the client takes the medication. B. Support is available to help family members meet their own needs. C. Improvement should occur if the client has a stimulating environment. D. Stressful family situations can precipitate a relapse in the client. 94. A client is admitted to the psychiatric unit with active psychosis. The physician diagnoses schizophrenia after ruling out several other conditions. Schizophrenia is characterized by: A. loss of identity and self-esteem. B. multiple personalities and decreased self-esteem. C. disturbances in affect, perception, and thought content and form. D. persistent memory impairment and confusion. 95. The nurse is providing care to a client with a catatonic type of schizophrenia who exhibits extreme negativism. To help the client meet his basic needs, the nurse should: A. ask the client which activity he would prefer to do first. B. negotiate a time when the client will perform activities. C. tell the client specifically and concisely what needs to be done. D. prepare the client ahead of time for the activity. 96. The nurse is caring for a client who experiences false sensory perceptions with no basis in reality. These perceptions are known as: A. delusions. B. hallucinations. C. loose associations. D. neologisms. 97. The nurse is aware that antipsychotic medications may cause which of the following adverse effects? A. Increased production of insulin B. Lower seizure threshold

C. Increased coagulation time D. Increased risk of heart failure 98. A client is admitted with a diagnosis of delusions of grandeur. This diagnosis reflects a belief that one is: A. highly important or famous. B. being persecuted. C. connected to events unrelated to oneself. D. responsible for the evil in the world. 99. A man with a 5-year history of multiple psychiatric admissions is brought to the emergency department by the police. He was found wandering the streets disheveled, shoeless, and confused. Based on his previous medical records and current behavior, he is diagnosed with chronic undifferentiated schizophrenia. The nurse should assign highest priority to which nursing diagnosis? A. Anxiety B. Impaired verbal communication C. Disturbed thought processes D. Self-care deficient: Dressing/grooming 100. A client's medication order reads, "Thioridazine (Mellaril) 200 mg P.O. q.i.d. and 100 mg P.O. p.r.n." The nurse should: A. administer the medication as prescribed. B. question the physician about the order. C. administer the order for 200 mg P.O. q.i.d. but not for 100 mg P.O. p.r.n. D. administer the medication as prescribed but observe the client closely for adverse effects. 101. A client is admitted to the psychiatric unit with a diagnosis of borderline personality disorder. The nurse expects the assessment to reveal: A. unpredictable behavior and intense interpersonal relationships. B. inability to function as a responsible parent. C. somatic symptoms. D. coldness, detachment, and lack of tender feelings. 102. A client with disorganized type schizophrenia has been hospitalized for the past 2 years on a unit for chronic mentally ill clients. The client's behavior is labile and fluctuates from childishness and incoherence to loud yelling to slow but appropriate interaction. The client needs assistance with all activities of daily living. Which behavior is characteristic of disorganized type schizophrenia? A. Extreme social impairment

B. Suspicious delusions C. Waxy flexibility D. Elevated affect 103. The nurse is providing care for a female client with a history of schizophrenia who's experiencing hallucinations. The physician orders 200 mg of haloperidol (Haldol) orally or I.M. every 4 hours as needed. What is the nurse's best action? A. Administer the haloperidol orally if the client agrees to take it. B. Call the physician to clarify whether the haloperidol should be given orally or I.M. C. Call the physician to clarify the order because the dosage is too high. D. Withhold haloperidol because it may worsen hallucinations. 104. A client receiving haloperidol (Haldol) complains of a stiff jaw and difficulty swallowing. The nurse's first action is to: A. reassure the client and administer as needed lorazepam (Ativan) I.M. B. administer as needed dose of benztropine (Cogentin) I.M. as ordered. C. administer as needed dose of benztropine (Cogentin) by mouth as ordered. D. administer as needed dose of haloperidol (Haldol) by mouth. 105. A 24-year-old client is experiencing an acute schizophrenic episode. He has vivid hallucinations that are making him agitated. The nurse's best response at this time would be to: A. take the client's vital signs. B. explore the content of the hallucinations. C. tell him his fear is unrealistic. D. engage the client in reality-oriented activities. 106. Which medication can control the extrapyramidal effects associated with antipsychotic agents? A. perphenazine (Trilafon) B. doxepin (Sinequan) C. amantadine (Symmetrel) D. clorazepate (Tranxene) 107. A client with paranoid schizophrenia has been experiencing auditory hallucinations for many years. One approach that has proven to be effective for hallucinating clients is to: A. take an as-needed dose of psychotropic medication whenever they hear voices. B. practice saying "Go away" or "Stop" when they hear voices. C. sing loudly to drown out the voices and provide a distraction. D. go to their room until the voices go away.

108. A dystonic reaction can be caused by which of the following medications? A. diazepam (Valium) B. haloperidol (Haldol) C. amitriptyline (Elavil) D. clonazepam (Klonopin) 109. While pacing in the hall, a client with paranoid schizophrenia runs to the nurse and says, "Why are you poisoning me? I know you work for central thought control! You can keep my thoughts. Give me back my soul!" How should the nurse respond during the early stage of the therapeutic process? A. "I'm a nurse. I'm not poisoning you. It's against the nursing code of ethics." B. "I'm a nurse, and you're a client in the hospital. I'm not going to harm you." C. "I'm not poisoning you. And how could I possibly steal your soul?" D. "I sense anger. Are you feeling angry today?" Answer Key http://pinoybsn.blogspot.com/2006/08/109-questions-and-rationale-on.html 60 Item Pediatric Nursing Exam by Jeddah 60 Items Pediatric Nursing Exam by Jeddah This is a drill exam made in pattern of the Local Nursing Licensure exam. Those questions that are in red color are question made by me, while those in black are collections of questions from the past board exam and reviewers. I’ve even added diseases diagnosed using the Newborn screening act, which I think will come out at the NLE this December. (I will post a full lecture regarding the diseases later). I will post the answers and rationale as soon as I’m finished with them. Please feel free to post comment and answers.. This will improve your test taking strategies. Good luck and enjoy Situation 1: Raphael, a 6 year’s old prep pupil is seen at the school clinic for growth and development monitoring (Questions 1-5) 1. Which of the following is characterized the rate of growth during this period? a. most rapid period of growth b. a decline in growth rate c. growth spurt d. slow uniform growth rate

2. In assessing Raphael’s growth and development, the nurse is guided by principles of growth and development. Which is not included? a. All individuals follow cephalo-caudal and proximo-distal b. Different parts of the body grows at different rate c. All individual follow standard growth rate d. Rate and pattern of growth can be modified 3. What type of play will be ideal for Raphael at this period? a. Make believe b. Hide and seek c. Peek-a-boo d. Building blocks 4. Which of the following information indicate that Raphael is normal for his age? a. Determine own sense self b. Develop sense of whether he can trust the world c. Has the ability to try new things d. Learn basic skills within his culture 5. Based on Kohlberg’s theory, what is the stage of moral development of Raphael? a. Punishment-obedience b. “good boy-Nice girl” c. naïve instrumental orientation d. social contact Situation 2 Baby boy Lacson delivered at 36 weeks gestation weighs 3,400 gm and height of 59 cm (6-10) 6. Baby boy Lacson’s height is a. Long b. Short c. Average d. Too short 7. Growth and development in a child progresses in the following ways EXCEPT a. From cognitive to psychosexual b. From trunk to the tip of the extremities c. From head to toe d. From general to specific 8. As described by Erikson, the major psychosexual conflict of the above situation is a. Autonomy vs. Shame and doubt b. Industry vs. Inferiority c. Trust vs. mistrust d. Initiation vs. guilt

9. Which of the following is true about Mongolian Spots? a. Disappears in about a year b. Are linked to pathologic conditions c. Are managed by tropical steroids d. Are indicative of parental abuse 10. Signs of cold stress that the nurse must be alert when caring for a Newborn is: a. Hypothermia b. Decreased activity level c. Shaking d. Increased RR Situation 3 Nursing care after delivery has an important aspect in every stages of delivery 11. After the baby is delivered, the cord was cut between two clamps using a sterile scissors and blade, then the baby is placed at the: a. Mother’s breast b. Mother’s side c. Give it to the grandmother d. Baby’s own mat or bed 12. The baby’s mother is RH(-). Which of the following laboratory tests will probably be ordered for the newborn? a. Direct Coomb’s b. Indirect Coomb’s c. Blood culture d. Platelet count 13. Hypothermia is common in newborn because of their inability to control heat. The following would be an appropriate nursing intervention to prevent heat loss except a. Place the crib beside the wall b. Doing Kangaroo care c. By using mechanical pressure d. Drying and wrapping the baby 14. The following conditions are caused by cold stress except a. Hypoglycemia b. Increase ICP c. Metabolic acidosis d. Cerebral palsy 15. During the feto-placental circulation, the shunt between two atria is called a. Ductus venosous b. Foramen Magnum c. Ductus arteriosus d. Foramen Ovale

16. What would cause the closure of the Foramen ovale after the baby had been delivered? a. Decreased blood flow b. Shifting of pressures from right side to the left side of the heart c. Increased PO2 d. Increased in oxygen saturation 17. Failure of the Foramen Ovale to close will cause what Congenital Heart Disease? a. Total anomalous Pulmunary Artery b. Atrial Septal defect c. Transposition of great arteries d. Pulmunary Stenosis Situation 4 Children are vulnerable to some minor health problems or injuries hence the nurse should be able to teach mothers to give appropriate home care. 18. A mother brought her child to the clinic with nose bleeding. The nurse showed the mother the most appropriate position for the child which is: a. Sitting up b. With low back rest c. With moderate back rest d. Lying semi flat 19. A common problem in children is the inflammation of the middle ear. This is related to the malfunctioning of the: a. Tympanic membrane b. Eustachian tube c. Adenoid d. Nasopharynx 20. For acute otitis media, the treatment is prompt antibiotic therapy. Delayed treatment may result in complications of: a. Tonsillitis b. Eardrum Problems c. Brain damage d. Diabetes mellitus 21. When assessing gross motor development in a 3 year old, which of the following activities would the nurse expect to finds? a. Riding a tricycle b. Hopping on one foot c. Catching a ball d. Skipping on alternate foot.

22. When assessing the weight of a 5-month old, which of the following indicates healthy growth? a. Doubling of birth weight b. Tripling of birth weight c. Quadrupling of birth weight d. Stabilizing of birth weight 23. An appropriate toy for a 4 year old child is: a. Push-pull toys b. Card games c. Doctor and nurse kits d. Books and Crafts 24. Which of the following statements would the nurse expects a 5-year old boy to say whose pet gerbil just died a. “The boogieman (kamatayan- the man with the scythe) got him” b. “He’s just a bit dead” c. “Ill be good from now own so I wont die like my gerbil” d. “Did you hear the joke about…” 25. When assessing the fluid and electrolyte balance in an infant, which of the following would be important to remember? a. Infant can concentrate urine at an adult level b. The metabolic rate of an infant is slower than in adults c. Infants have more intracellular water that adult do d. Infant have greater body surface area than adults 26. When assessing a child with aspirin overdose, which of the following will be expected? a. Metabolic alkalosis b. Respiratory alkalosis c. Metabolic acidosis d. Respiratory acidosis 27. Which of the following is not a possible systemic clinical manifestation of severe burns? a. Growth retardation b. Hypermetabolism c. Sepsis d. Blisters and edema 28. When assessing a family for potential child abuse risks, the nurse would observe for which of the following? a. Periodic exposure to stress b. Low socio-economic status c. High level of self esteem

d. Problematic pregnancies 29. Which of the following is a possible indicator of Munchausen syndrome by proxy type of child abuse? a. Bruises found at odd locations, with different stages of healing b. STD’s and genital discharges c. Unexplained symptoms of diarrhea, vomiting and apnea with no organic basis d. Constant hunger and poor hygiene 30. Which of the following is an inappropriate interventions when caring for a child with HIV? a. Teaching family about disease transmission b. Offering large amount of fresh fruits and vegetables c. Encouraging child to perform at optimal level d. Teach proper hand washing technique Situation 5 Agata, 2 years old is rushed to the ER due to cyanosis precipitated by crying. Her mother observed that after playing she gets tired. She was diagnosed with Tetralogy of Fallot. 31. The goal of nursing care fro Agata is to: a. Prevent infection b. Promote normal growth and development c. Decrease hypoxic spells d. Hydrate adequately 32. The immediate nursing intervention for cyanosis of Agata is: a. Call up the pediatrician b. Place her in knee chest position c. Administer oxygen inhalation d. Transfer her to the PICU 33. Agata was scheduled for a palliative surgery, which creates anastomosis of the subclavian artery to the pulmonary artery. This procedure is: a. Waterston-Cooley b. Raskkind Procedure c. Coronary artery bypass d. Blalock-Taussig 34. Which of the following is not an indicator that Agata experiences separation anxiety brought about her hospitalization? a. Friendly with the nurse b. Prolonged loud crying, consoled only by mother c. Occasional temper tantrums and always says NO d. Repeatedly verbalizes desire to go home

35. When Agata was brought to the OR, her parents where crying. What would be the most appropriate nursing diagnosis? a. Infective family coping r/t situational crisis b. Anxiety r/t powerlessness c. Fear r/t uncertain prognosis d. Anticipatory grieving r/t gravity of child’s physical status 36. Which of the following respiratory condition is always considered a medical emergency? a. Laryngeotracheobronchitis (LTB) b. Epiglottitis c. Asthma d. Cystic Fibrosis 37. Which of the following statements by the family of a child with asthma indicates a need for additional teaching? a. “We need to identify what things triggers his attacks” b. “He is to use bronchodilator inhaler before steroid inhaler” c. “We’ll make sure he avoids exercise to prevent asthma attacks” d. “he should increase his fluid intake regularly to thin secretions” 38. Which of the following would require careful monitoring in the child with ADHD who is receiving Methylphenidate (Ritalin)? a. Dental health b. Mouth dryness c. Height and weight d. Excessive appetite Situation 6 Laura is assigned as the Team Leader during the immunization day at the RHU 39. What program for the DOH is launched at 1976 in cooperation with WHO and UNICEF to reduce morbidity and mortality among infants caused by immunizable disease? a. Patak day b. Immunization day on Wednesday c. Expanded program on immunization d. Bakuna ng kabtaan 40. One important principle of the immunization program is based on? a. Statistical occurrence b. Epidemiologic situation c. Cold chain management d. Surveillance study 41. The main element of immunization program is one of the following?

a. Information, education and communication b. Assessment and evaluation of the program c. Research studies d. Target setting 42. What does herd immunity means? a. Interruption of transmission b. All to be vaccinated c. Selected group for vaccination d. Shorter incubation 43. Measles vaccine can be given simultaneously. What is the combined vaccine to be given to children starting at 15 months? a. MCG b. MMR c. BCG d. BBR Situation 7: Braguda brought her 5-month old daughter in the nearest RHU because her baby sleeps most of the time, with decreased appetite, has colds and fever for more than a week. The physician diagnosed pneumonia. 44. Based on this data given by Braguda, you can classify Braguda’s daughter to have: a. Pneumonia: cough and colds b. Severe pneumonia c. Very severe pneumonia d. Pneumonia moderate 45. For a 3-month old child to be classified to have Pneumonia (not severe), you would expect to find RR of: a. 60 bpm b. 40 bpm c. 70 bpm d. 50 pbm 46. You asked Braguda if her baby received all vaccines under EPI. What legal basis is used in implementing the UN’s goal on Universal Child Immunization? a. PD no. 996 b. PD no. 6 c. PD no. 46 d. RA 9173 47. Braguda asks you about Vitamin A supplementation. You responded that giving Vitamin A starts when the infant reaches 6 months and the first dose is” a. 200,000 “IU” b. 100,000 “IU”

c. 500,000 “IU” d. 10,000 “IU” 48. As part of CARI program, assessment of the child is your main responsibility. You could ask the following question to the mother except: a. “How old is the child?” b. “IS the child coughing? For how long?” c. “Did the child have chest indrawing?” d. “Did the child have fever? For how long?” 49. A newborn’s failure to pass meconium within 24 hours after birth may indicate which of the following? a. Aganglionic Mega colon b. Celiac disease c. Intussusception d. Abdominal wall defect 50. The nurse understands that a good snack for a 2 year old with a diagnosis of acute asthma would be: a. Grapes b. Apple slices c. A glass of milk d. A glass of cola 51. Which of the following immunizations would the nurse expect to administer to a child who is HIV (+) and severely immunocomromised? a. Varicella b. Rotavirus c. MMR d. IPV 52. When assessing a newborn for developmental dysplasia of the hip, the nurse would expect to assess which of the following? a. Symmetrical gluteal folds b. Trendelemburg sign c. Ortolani’s sign d. Characteristic limp 53. While assessing a male neonate whose mother desires him to be circumcised, the nurse observes that the neonate’s urinary meatus appears to be located on the ventral surface of the penis. The physician is notified because the nurse would suspect which of the following? a. Phimosis b. Hydrocele c. Epispadias d. Hypospadias

54. When teaching a group of parents about seat belt use, when would the nurse state that the child be safely restrained in a regular automobile seatbelt? a. 30 lb and 30 in b. 35 lb and 3 y/o c. 40 lb and 40 in d. 60 lb and 6 y/o 55. When assessing a newborn with cleft lip, the nurse would be alert which of the following will most likely be compromised? a. Sucking ability b. Respiratory status c. Locomotion d. GI function 56. For a child with recurring nephritic syndrome, which of the following areas of potential disturbances should be a prime consideration when planning ongoing nursing care? a. Muscle coordination b. Sexual maturation c. Intellectual development d. Body image 57. An inborn error of metabolism that causes premature destruction of RBC? a. G6PD b. Hemocystinuria c. Phenylketonuria d. Celiac Disease 58. Which of the following would be a diagnostic test for Phenylketonuria which uses fresh urine mixed with ferric chloride? a. Guthrie Test b. Phenestix test c. Beutler’s test d. Coomb’s test 59. Dietary restriction in a child who has Hemocystenuria will include which of the following amino acid? a. Lysine b. Methionine c. Isolensine tryptophase d. Valine 60. A milk formula that you can suggest for a child with Galactosemia: a. Lofenalac b. Lactum

c. Neutramigen d. Sustagen Answer Key http://pinoybsn.blogspot.com/2006/08/60-items-pediatric-nursing-questions.html 89 Item Psychiatric Nursing Exam II : Substance Abuse, Eating disorders and Impulse control disorders 89 Item Psychiatric Nursing Exam II : Substance Abuse, Eating disorders and Impulse control disorders http://www.pinoybsn.tk 1. An unemployed woman, age 24, seeks help because she feels depressed and abandoned and doesn't know what to do with her life. She says she has quit her last five jobs because her coworkers didn't like her and didn't train her adequately. Last week, her boyfriend broke up with her after she drove his car into a tree after an argument. The client's initial diagnosis is borderline personality disorder. Which nursing observations support this diagnosis? A. Flat affect, social withdrawal, and unusual dress B. Suspiciousness, hypervigilance, and emotional coldness C. Lack of self-esteem, strong dependency needs, and impulsive behavior D. Insensitivity to others, sexual acting out, and violence 2.In a toddler, which of the following injuries is most likely the result of child abuse? A. A hematoma on the occipital region of the head B. A 1-inch forehead laceration C. Several small, dime-sized circular burns on the child's back D. A small isolated bruise on the right lower extremity 3. A client is admitted to the emergency department after being found unconscious. Her blood pressure is 82/50 mm Hg. She is 5′ 4" (1.6 m) tall, weighs 79 lb (35.8 kg), and appears dehydrated and emaciated. After regaining consciousness, she reports that she has had trouble eating lately and can't remember what she ate in the last 24 hours. She also states that she has had amenorrhea for the past year. She is convinced she is fat and refuses food. The nurse suspects that she has: A. bulimia nervosa. B. anorexia nervosa. C. depression. D. schizophrenia.

4. A 15-year-old girl with anorexia has been admitted to a mental health unit. She refuses to eat. Which of the following statements is the best response from the nurse? A. "You don't have to eat. It's your choice." B. "I hope you'll eat your food by mouth. Tube feedings and I.V. lines can be uncomfortable." C. "Why do you think you're fat? You're underweight. Here — look in the mirror." D. "You really look terrible at this weight. I hope you'll eat." 5. A client with a history of substance abuse has been attending Alcoholics Anonymous meetings regularly in the psychiatric unit. One afternoon, the client tells the nurse, "I'm not going to those meetings anymore. I'm not like the rest of those people. I'm not a drunk. "What is the most appropriate response? A. "If you aren't an alcoholic, why do you keep drinking and ending up in the hospital?" B. "It's your decision. If you don't want to go, you don't have to." C. "You seem upset about the meetings." D. "You have to go to the meetings. It's part of your treatment plan." 6. A client is admitted to the inpatient adolescent unit after being arrested for attempting to sell cocaine to an undercover police officer. The nurse plans to write a behavioral contract. To best promote compliance, the contract should be written: A. abstractly. B. by the client alone. C. jointly by the client and nurse. D. jointly by the physician and nurse. 7. During which phase of alcoholism is loss of control and physiologic dependence evident? A. Prealcoholic phase B. Early alcoholic phase C. Crucial phase D. Chronic phase 8. Which of the following is important when restraining a violent client? A. Have three staff members present, one for each side of the body and one for the head. B. Always tie restraints to side rails. C. Have an organized, efficient team approach after the decision is made to restrain the client. D. Secure restraints to the gurney with knots to prevent escape. 9. A client who's actively hallucinating is brought to the hospital by friends. They say that the client used either lysergic acid diethylamide (LSD) or angel dust (phencyclidine

[PCP]) at a concert. Which of the following common assessment findings indicates that the client may have ingested PCP? A. Dilated pupils B. Nystagmus C. Paranoia D. Altered mood 10. A severely dehydrated teenager admitted to the hospital with hypotension and tachycardia undergoes evaluation for electrolyte disturbances. Her history includes anorexia nervosa and a 20-lb (9.1-kg) weight loss in the last month. She is 5′ 7" (1.7 m) tall and weighs 80 lb (36.3 kg). Which nursing intervention takes highest priority? A. Initiating caloric and nutritional therapy as ordered B. Instituting behavioral modification therapy as ordered C. Addressing the client's low self-esteem D. Regularly monitoring vital signs and weight 11. A client tells the nurse that he is having suicidal thoughts every day. In conferring with the treatment team, the nurse should make which of the following recommendations? A. A no-suicide contract B. Weekly outpatient therapy C. A second psychiatric opinion D. Intensive inpatient treatment 12. Which of the following etiologic factors predispose a client to Tourette syndrome? A. No known etiology B. Abnormalities in brain neurotransmitters, structural changes in basal ganglia and caudate nucleus, and genetics C. Abnormalities in the structure and function of the ventricles D. Environmental factors and birth-related trauma 13. A client is admitted for detoxification after a cocaine overdose. The client tells the nurse that he frequently uses cocaine but he can control his use if he chooses. Which coping mechanism is he using? A. Withdrawal B. Logical thinking C. Repression D. Denial 14. An 16-year-old boy is admitted to the facility after acting out his aggressions inappropriately at school. Predisposing factors to the expression of aggression include:

A. violence on television. B. passive parents. C. an internal locus of control. D. a single-parent family 15. A client is brought to the emergency department after being beaten by her husband, a prominent attorney. The nurse caring for this client understands that: A. open boundaries are common in violent families. B. violence usually results from a power struggle. C. domestic violence and abuse span all socioeconomic classes. D. violent behavior is a genetic trait passed from one generation to the next. 16. On discharge after treatment for alcoholism, a client plans to take disulfiram (Antabuse) as prescribed. When teaching the client about this drug, the nurse emphasizes the need to: A. avoid all products containing alcohol. B. adhere to concomitant vitamin B therapy. C. return for monthly blood drug level monitoring. D. limit alcohol consumption to a moderate level. 17. During a private conversation, a client with borderline personality disorder asks the nurse to keep his secret and then displays multiple, self-inflicted, superficial lacerations on the forearms. What is the nurse's best response? A. "That's it! You're on suicide precautions." B. "I'm going to tell your physician. Do you want to tell me why you did that?" C. "Tell me what type of instrument you used. I'm concerned about infection." D. "The team needs to know when something important occurs in treatment. I need to tell the others, but let's talk about it first." 18. The nurse is providing care for a client undergoing opiate withdrawal. Opiate withdrawal causes severe physical discomfort and can be life-threatening. To minimize these effects, opiate users are commonly detoxified with: A. barbiturates. B. amphetamines. C. methadone. D. benzodiazepines. 19. The nurse is caring for a client who she believes has been abusing opiates. Assessment findings in a client abusing opiates such as morphine include: A. dilated pupils and slurred speech.

B. rapid speech and agitation. C. dilated pupils and agitation. D. euphoria and constricted pupils. 20. Which of the following signs should the nurse expect in a client with known amphetamine overdose? A. Hypotension B. Tachycardia C. Hot, dry skin D. Constricted pupils 21. A client is admitted to the psychiatric unit with a diagnosis of alcohol intoxication and suspected alcohol dependence. Other assessment findings include an enlarged liver, jaundice, lethargy, and rambling, incoherent speech. No other information about the client is available. After the nurse completes the initial assessment, what is the first priority? A. Instituting seizure precautions, obtaining frequent vital signs, and recording fluid intake and output B. Checking the client's medical records for health history information C. Attempting to contact the client's family to obtain more information about the client D. Restricting fluids and leaving the client alone to "sleep off" the episode 22. Which nursing action is best when trying to diffuse a client's impending violent behavior? A. Helping the client identify and express feelings of anxiety and anger B. Involving the client in a quiet activity to divert attention C. Leaving the client alone until the client can talk about feelings D. Placing the client in seclusion 23. The nurse is working with a client who abuses alcohol. Which of the following facts should the nurse communicate to the client? A. Abstinence is the basis for successful treatment. B. Attendance at Alcoholics Anonymous meetings every day will cure alcoholism. C. For treatment to be successful, family members must participate. D. An occasional social drink is acceptable behavior for the alcoholic 24. Which psychosocial influence has been causally related to the development of aggressive behavior and conduct disorder? A. An overbearing mother B. Rejection by peers C. A history of schizophrenia in the family

D. Low socioeconomic status 25. In group therapy, a client who has used I.V. heroin every day for the past 14 years says, "I don't have a drug problem. I can quit whenever I want. I've done it before." Which defense mechanism is the client using? A. Denial B. Obsession C. Compensation D. Rationalization 26. A client with a history of cocaine addiction is admitted to the coronary care unit for evaluation of substernal chest pain. The electrocardiogram (ECG) shows a 1-mm STsegment elevation the anteroseptal leads and T-wave inversion in leads V3 to V5. Considering the client's history of drug abuse, the nurse expects the physician to prescribe: A. lidocaine (Xylocaine). B. procainamide (Pronestyl). C. nitroglycerin (Nitro-Bid IV). D. epinephrine. 27. A 15-year-old client is brought to the clinic by her mother. Her mother expresses concern about her daughter's weight loss and constant dieting. The nurse conducts a health history interview. Which of the following comments indicates that the client may be suffering from anorexia nervosa? A. "I like the way I look. I just need to keep my weight down because I'm a cheerleader." B. "I don't like the food my mother cooks. I eat plenty of fast food when I'm out with my friends." C. "I just can't seem to get down to the weight I want to be. I'm so fat compared to other girls." D. "I do diet around my periods; otherwise, I just get so bloated." 28. Which is the drug of choice for treating Tourette syndrome? A. fluoxetine (Prozac) B. fluvoxamine (Luvox) C. haloperidol (Haldol) D. paroxetine (Paxil) 29. The client tells the nurse he was involved in a car accident while he was intoxicated. What would be the most therapeutic response from the nurse? A. "Why didn't you get someone else to drive you?" B. "Tell me how you feel about the accident."

C. "You should know better than to drink and drive." D. "I recommend that you attend an Alcoholics Anonymous meeting." a 30. A client voluntarily admits himself to the substance abuse unit. He confesses that he drinks 1 qt or more of vodka each day and uses cocaine occasionally. Later that afternoon, he begins to show signs of alcohol withdrawal. What are some early signs of this condition? A. Vomiting, diarrhea, and bradycardia B. Dehydration, temperature above 101° F (38.3° C), and pruritus C. Hypertension, diaphoresis, and seizures D. Diaphoresis, tremors, and nervousness 31. When monitoring a client recently admitted for treatment of cocaine addiction, the nurse notes sudden increases in the arterial blood pressure and heart rate. To correct these problems, the nurse expects the physician to prescribe: A. norepinephrine (Levophed) and lidocaine (Xylocaine). B. nifedipine (Procardia) and lidocaine. C. nitroglycerin (Nitro-Bid IV) and esmolol (Brevibloc). D. nifedipine and esmolol 32. A client experiencing alcohol withdrawal is upset about going through detoxification. Which of the following goals is a priority? A. The client will commit to a drug-free lifestyle. B. The client will work with the nurse to remain safe. C. The client will drink plenty of fluids daily. D. The client will make a personal inventory of strengths 33. A client is admitted to a psychiatric facility by court order for evaluation for antisocial personality disorder. This client has a long history of initiating fights and abusing animals and recently was arrested for setting a neighbor's dog on fire. When evaluating this client for the potential for violence, the nurse should assess for which behavioral clues? A. A rigid posture, restlessness, and glaring B. Depression and physical withdrawal C. Silence and noncompliance D. Hypervigilance and talk of past violent acts 34. A client is brought to the psychiatric clinic by family members, who tell the admitting nurse that the client repeatedly drives while intoxicated despite their pleas to stop. During an interview with the nurse, which statement by the client most strongly supports a diagnosis of psychoactive substance abuse?

A. "I'm not addicted to alcohol. In fact, I can drink more than I used to without being affected." B. "I only spend half of my paycheck at the bar." C. "I just drink to relax after work." D. "I know I've been arrested three times for drinking and driving, but the police are just trying to hassle me." 35. A client with borderline personality disorder is admitted to the psychiatric unit. Initial nursing assessment reveals that the client's wrists are scratched from a recent suicide attempt. Based on this finding, the nurse should formulate a nursing diagnosis of: A. Ineffective individual coping related to feelings of guilt. B. Situational low self-esteem related to feelings of loss of control. C. Risk for violence: Self-directed related to impulsive mutilating acts. D. Risk for violence: Directed toward others related to verbal threats. 36. A client recently admitted to the hospital with sharp, substernal chest pain suddenly complains of palpitations. The nurse notes a rise in the client's arterial blood pressure and a heart rate of 144 beats/minute. On further questioning, the client admits to having used cocaine recently after previously denying use of the drug. The nurse concludes that the client is at high risk for which complication of cocaine use? A. Coronary artery spasm B. Bradyarrhythmias C. Neurobehavioral deficits D. Panic disorder 37. A client is being admitted to the substance abuse unit for alcohol detoxification. As part of the intake interview, the nurse asks him when he had his last alcoholic drink. He says that he had his last drink 6 hours before admission. Based on this response, the nurse should expect early withdrawal symptoms to: A. begin after 7 days. B. not occur at all because the time period for their occurrence has passed. C. begin anytime within the next 1 to 2 days. D. begin within 2 to 7 days. 38. The nurse is assigned to care for a client with anorexia nervosa. Initially, which nursing intervention is most appropriate for this client? A. Providing one-on-one supervision during meals and for 1 hour afterward B. Letting the client eat with other clients to create a normal mealtime atmosphere C. Trying to persuade the client to eat and thus restore nutritional balance D. Giving the client as much time to eat as desired

39. A client begins to experience alcoholic hallucinosis. What is the best nursing intervention at this time? A. Keeping the client restrained in bed B. Checking the client's blood pressure every 15 minutes and offering juices C. Providing a quiet environment and administering medication as needed and prescribed D. Restraining the client and measuring blood pressure every 30 minutes 40. Which assessment finding is most consistent with early alcohol withdrawal? A. Heart rate of 120 to 140 beats/minute B. Heart rate of 50 to 60 beats/minute C. Blood pressure of 100/70 mm Hg D. Blood pressure of 140/80 mm Hg 41. Which client is at highest risk for suicide? A. One who appears depressed, frequently thinks of dying, and gives away all personal possessions B. One who plans a violent death and has the means readily available C. One who tells others that he or she might do something if life doesn't get better soon D. One who talks about wanting to die 42. Which of the following medical conditions is commonly found in clients with bulimia nervosa? A. Allergies B. Cancer C. Diabetes mellitus D. Hepatitis A 43. A high school student is referred to the school nurse for suspected substance abuse. Following the nurse's assessment and interventions, what would be the most desirable outcome? A. The student discusses conflicts over drug use. B. The student accepts a referral to a substance abuse counselor. C. The student agrees to inform his parents of the problem. D. The student reports increased comfort with making choices. 44. A client who reportedly consumes 1 qt of vodka daily is admitted for alcohol detoxification. To try to prevent alcohol withdrawal symptoms, the physician is most likely to prescribe which drug? A. clozapine (Clozaril) B. thiothixene (Navane)

C. lorazepam (Ativan) D. lithium carbonate (Eskalith) 45. A client is being treated for alcoholism. After a family meeting, the client's spouse asks the nurse about ways to help the family deal with the effects of alcoholism. The nurse should suggest that the family join which organization? A. Al-Anon B. Make Today Count C. Emotions Anonymous D. Alcoholics Anonymous 46. A client is admitted to the psychiatric clinic for treatment of anorexia nervosa. To promote the client's physical health, the nurse should plan to: A. severely restrict the client's physical activities. B. weigh the client daily, after the evening meal. C. monitor vital signs, serum electrolyte levels, and acid-base balance. D. instruct the client to keep an accurate record of food and fluid intake. 47. A young man is remanded by the courts for psychiatric treatment. His police record, which dates to his early teenage years, includes delinquency, running away, auto theft, and vandalism. He dropped out of school at age 16 and has been living on his own since then. His history suggests maladaptive coping, which is associated with: A. antisocial personality disorder. B. borderline personality disorder. C. obsessive-compulsive personality disorder. D. narcissistic personality disorder. 48. A husband and wife seek emergency crisis intervention because he slapped her repeatedly the night before. The husband indicates that his childhood was marred by an abusive relationship with his father. When intervening with this couple, the nurse knows they are at risk for repeated violence because the husband: A. has only moderate impulse control. B. denies feelings of jealousy or possessiveness. C. has learned violence as an acceptable behavior. D. feels secure in his relationship with his wife. 49. A client whose husband just left her has a recurrence of anorexia nervosa. The nurse caring for her realizes that this exacerbation of anorexia nervosa results from the client's effort to: A. manipulate her husband. B. gain control of one part of her life.

C. commit suicide. D. live up to her mother's expectations. 50. A client has approached the nurse asking for advice on how to deal with his alcohol addiction. The nurse should tell the client that the only effective treatment for alcoholism is: A. psychotherapy. B. total abstinence. C. Alcoholics Anonymous (AA). D. aversion therapy. 51. Flumazenil (Romazicon) has been ordered for a client who has overdosed on oxazepam (Serax). Before administering the medication, the nurse should be prepared for which common adverse effect? A. Seizures B. Shivering C. Anxiety D. Chest pain 52. The nurse is caring for a client diagnosed with bulimia. The most appropriate initial goal for a client diagnosed with bulimia is to: A. avoid shopping for large amounts of food. B. control eating impulses. C. identify anxiety-causing situations. D. eat only three meals per day. 53. A client who's at high risk for suicide needs close supervision. To best ensure the client's safety, the nurse should: A. check the client frequently at irregular intervals throughout the night. B. assure the client that the nurse will hold in confidence anything the client says. C. repeatedly discuss previous suicide attempts with the client. D. disregard decreased communication by the client because this is common in suicidal clients. 54. Which of the following drugs should the nurse prepare to administer to a client with a toxic acetaminophen (Tylenol) level? A. deferoxamine mesylate (Desferal) B. succimer (Chemet) C. flumazenil (Romazicon) D. acetylcysteine (Mucomyst)

55. A client is admitted to the substance abuse unit for alcohol detoxification. Which of the following medications is the nurse most likely to administer to reduce the symptoms of alcohol withdrawal? A. naloxone (Narcan) B. haloperidol (Haldol) C. magnesium sulfate D. chlordiazepoxide (Librium) 56. During postprandial monitoring, a client with bulimia nervosa tells the nurse, "You can sit with me, but you're just wasting your time. After you sat with me yesterday, I was still able to purge. Today, my goal is to do it twice." What is the nurse's best response? A. "I trust you not to purge." B. "How are you purging and when do you do it?" C. "Don't worry. I won't allow you to purge today." D. "I know it's important for you to feel in control, but I'll monitor you for 90 minutes after you eat." 57. A client admitted to the psychiatric unit for treatment of substance abuse says to the nurse, "It felt so wonderful to get high." Which of the following is the most appropriate response? A. "If you continue to talk like that, I'm going to stop speaking to you." B. "You told me you got fired from your last job for missing too many days after taking drugs all night." C. "Tell me more about how it felt to get high." D. "Don't you know it's illegal to use drugs?" 58. For a client with anorexia nervosa, which goal takes the highest priority? A. The client will establish adequate daily nutritional intake. B. The client will make a contract with the nurse that sets a target weight. C. The client will identify self-perceptions about body size as unrealistic. D. The client will verbalize the possible physiological consequences of self-starvation. 59. When interviewing the parents of an injured child, which of the following is the strongest indicator that child abuse may be a problem? A. The injury isn't consistent with the history or the child's age. B. The mother and father tell different stories regarding what happened. C. The family is poor. D. The parents are argumentative and demanding with emergency department personnel.

60. For a client with anorexia nervosa, the nurse plans to include the parents in therapy sessions along with the client. What fact should the nurse remember to be typical of parents of clients with anorexia nervosa? A. They tend to overprotect their children. B. They usually have a history of substance abuse. C. They maintain emotional distance from their children. D. They alternate between loving and rejecting their children. 61. In the emergency department, a client with facial lacerations states that her husband beat her with a shoe. After the health care team repairs her lacerations, she waits to be seen by the crisis intake nurse, who will evaluate the continued threat of violence. Suddenly the client's husband arrives, shouting that he wants to "finish the job." What is the first priority of the health care worker who witnesses this scene? A. Remaining with the client and staying calm B. Calling a security guard and another staff member for assistance C. Telling the client's husband that he must leave at once D. Determining why the husband feels so angry 62. The nurse is caring for a client with bulimia. Strict management of dietary intake is necessary. Which intervention is also important? A. Fill out the client's menu and make sure she eats at least half of what is on her tray. B. Let the client eat her meals in private. Then engage her in social activities for at least 2 hours after each meal. C. Let the client choose her own food. If she eats everything she orders, then stay with her for 1 hour after each meal. D. Let the client eat food brought in by the family if she chooses, but she should keep a strict calorie count. 63. The nurse is assigned to care for a suicidal client. Initially, which is the nurse's highest care priority? A. Assessing the client's home environment and relationships outside the hospital B. Exploring the nurse's own feelings about suicide C. Discussing the future with the client D. Referring the client to a clergyperson to discuss the moral implications of suicide 64. A client with anorexia nervosa tells the nurse, "When I look in the mirror, I hate what I see. I look so fat and ugly." Which strategy should the nurse use to deal with the client's distorted perceptions and feelings? A. Avoid discussing the client's perceptions and feelings. B. Focus discussions on food and weight. C. Avoid discussing unrealistic cultural standards regarding weight.

D. Provide objective data and feedback regarding the client's weight and attractiveness. 65. The nurse is caring for a client being treated for alcoholism. Before initiating therapy with disulfiram (Antabuse), the nurse teaches the client that he must read labels carefully on which of the following products? A. Carbonated beverages B. Aftershave lotion C. Toothpaste D. Cheese 66. The nurse is developing a plan of care for a client with anorexia nervosa. Which action should the nurse include in the plan? A. Restrict visits with the family until the client begins to eat. B. Provide privacy during meals. C. Set up a strict eating plan for the client. D. Encourage the client to exercise, which will reduce her anxiety. 67. Victims of domestic violence should be assessed for what important information? A. Reasons they stay in the abusive relationship (for example, lack of financial autonomy and isolation) B. Readiness to leave the perpetrator and knowledge of resources C. Use of drugs or alcohol D. History of previous victimization 68. A client is hospitalized with fractures of the right femur and right humerus sustained in a motorcycle accident. Police suspect the client was intoxicated at the time of the accident. Laboratory tests reveal a blood alcohol level of 0.2% (200 mg/dl). The client later admits to drinking heavily for years. During hospitalization, the client periodically complains of tingling and numbness in the hands and feet. The nurse realizes that these symptoms probably result from: A. acetate accumulation. B. thiamine deficiency. C. triglyceride buildup. D. a below-normal serum potassium level 69. A parent brings a preschooler to the emergency department for treatment of a dislocated shoulder, which allegedly happened when the child fell down the stairs. Which action should make the nurse suspect that the child was abused? A. The child cries uncontrollably throughout the examination. B. The child pulls away from contact with the physician. C. The child doesn't cry when the shoulder is examined.

D. The child doesn't make eye contact with the nurse. 70. When planning care for a client who has ingested phencyclidine (PCP), which of the following is the highest priority? A. Client's physical needs B. Client's safety needs C. Client's psychosocial needs D. Client's medical needs 71. Which outcome criteria would be appropriate for a child diagnosed with oppositional defiant disorder? A. Accept responsibility for own behaviors. B. Be able to verbalize own needs and assert rights. C. Set firm and consistent limits with the client. D. Allow the child to establish his own limits and boundaries. 72. A client is found sitting on the floor of the bathroom in the day treatment clinic with moderate lacerations on both wrists. Surrounded by broken glass, she sits staring blankly at her bleeding wrists while staff members call for an ambulance. How should the nurse approach her initially? A. Enter the room quietly and move beside her to assess her injuries. B. Call for staff back-up before entering the room and restraining her. C. Move as much glass away from her as possible and sit next to her quietly. D. Approach her slowly while speaking in a calm voice, calling her name, and telling her that the nurse is here to help her. 73. A client with anorexia nervosa describes herself as "a whale." However, the nurse's assessment reveals that the client is 5′ 8" (1.7 m) tall and weighs only 90 lb (40.8 kg). Considering the client's unrealistic body image, which intervention should be included in the plan of care? A. Asking the client to compare her figure with magazine photographs of women her age B. Assigning the client to group therapy in which participants provide realistic feedback about her weight C. Confronting the client about her actual appearance during one-on-one sessions, scheduled during each shift D. Telling the client of the nurse's concern for her health and desire to help her make decisions to keep her healthy 74. Eighteen hours after undergoing an emergency appendectomy, a client with a reported history of social drinking displays these vital signs: temperature, 101.6° F (38.7° C); heart rate, 126 beats/minute; respiratory rate, 24 breaths/minute; and blood pressure,

140/96 mm Hg. The client exhibits gross hand tremors and is screaming for someone to kill the bugs in the bed. The nurse should suspect: A. a postoperative infection. B. alcohol withdrawal. C. acute sepsis. D. pneumonia. 75. Clonidine (Catapres) can be used to treat conditions other than hypertension. For which of the following conditions might the drug be administered? A. Phencyclidine (PCP) intoxication B. Alcohol withdrawal C. Opiate withdrawal D. Cocaine withdrawal 76. One of the goals for a client with anorexia nervosa is that the client will demonstrate increased individual coping by responding to stress in constructive ways. Which of the following actions is the best indicator that the client is working toward meeting the goal? A. The client drinks 4 L of fluid per day. B. The client paces around the unit most of the day. C. The client keeps a journal and discusses it with the nurse. D. The client talks almost constantly with friends 76. One of the goals for a client with anorexia nervosa is that the client will demonstrate increased individual coping by responding to stress in constructive ways. Which of the following actions is the best indicator that the client is working toward meeting the goal? A. The client drinks 4 L of fluid per day. B. The client paces around the unit most of the day. C. The client keeps a journal and discusses it with the nurse. D. The client talks almost constantly with friends by telephone. 77. The nurse in the substance abuse unit is trying to encourage a client to attend Alcoholics Anonymous meetings. When the client asks the nurse what he must do to become a member, the nurse should respond: A. "You must first stop drinking." B. "Your physician must refer you to this program." C. "Admit you're powerless over alcohol and that you need help." D. "You must bring along a friend who will support you." 78. An attorney who throws books and furniture around the office after losing a case is referred to the psychiatric nurse in the law firm's employee assistance program. The nurse

knows that the client's behavior most likely represents the use of which defense mechanism? A. Regression B. Projection C. Reaction-formation D. Intellectualization 79. After completing chemical detoxification and a 12-step program to treat crack addiction, a client is being prepared for discharge. Which remark by the client indicates a realistic view of the future? A. "I'm never going to use crack again." B. "I know what I have to do. I have to limit my crack use." C. "I'm going to take 1 day at a time. I'm not making any promises." D. "I will substitue crack for something else" 80. The nurse is assessing a client on admission to the chemical dependency unit for alcohol detoxification. When the nurse asks about alcohol use, this client is most likely to: A. accurately describe the amount consumed. B. underestimate the amount consumed. C. overestimate the amount consumed. D. deny any consumption of alcohol. 81. The nurse is assessing a 15-year-old female who's being admitted for treatment of anorexia nervosa. Which clinical manifestation is the nurse most likely to find? A. Tachycardia B. Warm, flushed extremities C. Parotid gland tenderness D. Coarse hair growth 82. A 38-year-old client is admitted for alcohol withdrawal. The most common early sign or symptom that this client is likely to experience is: A. impending coma. B. manipulating behavior. C. suppression. D. perceptual disorders. 83. The nurse is caring for an adolescent female who reports amenorrhea, weight loss, and depression. Which additional assessment finding would suggest that the woman has an eating disorder?

A. Wearing tight-fitting clothing B. Increased blood pressure C. Oily skin D. Excessive and ritualized exercise 84. A client with a history of polysubstance abuse is admitted to the facility. She complains of nausea and vomiting 24 hours after admission. The nurse assesses the client and notes piloerection, pupillary dilation, and lacrimation. The nurse suspects that the client is going through which of the following withdrawals? A. Alcohol withdrawal B. Cannibis withdrawal C. Cocaine withdrawal D. Opioid withdrawal 85. A client is admitted to the psychiatric unit with a diagnosis of anorexia nervosa. Although she is 5′ 8" (1.7 m) tall and weighs only 103 lb (46.7 kg), she talks incessantly about how fat she is. Which measure should the nurse take first when caring for this client? A. Teach the client about nutrition, calories, and a balanced diet. B. Establish a trusting relationship with the client. C. Discuss cultural stereotypes regarding thinness and attractiveness. D. Explore the reasons why the client doesn't eat. 86. A client is admitted for an overdose of amphetamines. When assessing this client, the nurse should expect to see: A. tension and irritability. B. slow pulse. C. hypotension. D. constipation. 87. Which of the following drugs may be abused because of tolerance and physiologic dependence. A. lithium (Lithobid) and divalproex (Depakote). B. verapamil (Calan) and chlorpromazine (Thorazine) C. alprazolam (Xanax) and phenobarbital (Luminal) D. clozapine (Clozaril) and amitriptyline (Elavil) 88. Which of the following groups are considered to be at highest risk for suicide? A. Adolescents, men over age 45, and persons who have made previous suicide attempts B. Teachers, divorced persons, and substance abusers C. Alcohol abusers, widows, and young married men

D. Depressed persons, physicians, and persons living in rural areas 89. Tourette syndrome is characterized by the presence of multiple motor and vocal tics. A vocal tic that involves repeating one's own sounds or words is known as: A. echolalia. B. palilalia. C. apraxia. D. aphonia. Answer Key http://pinoybsn.blogspot.com/2006/08/89-item-psychiatric-nursing-exam-ii_07.html 50 Item Gastrointestinal Health Problems Test Drill by brewed This is a test drill on gastrointestinal health problems. I created the questions in line with the framework of our own Nurse Licensure Exam. Those in colored ink are my personal contributions while those in black ink are derived from Lippincot's and Springhouse Review Book. These are common board questions. I'll post the answers and rationales soon. You're all free to post a comment. Good luck guys and have fun with the test drill. - brewed Situation 1: Children have a special fascination with the workings of the digestive system. To fully understand the digestive processes, Nurse Lavigña must be knowledgeable of the anatomy and physiology of the gastrointestinal system. 1. The alimentary canal is a continuous, coiled, hollow muscular tube that winds through the ventral cavity and is open at both ends. Its solid organs include all of the following except: a. liver b. gall bladder c. stomach d. pancreas 2. Pharynx is lined with mucous membranes and mucous secreting glands to ease the passage of food. The larygngopharynx serves as passageway for: a. air only b. air and water c. food, fluids and air d. air and food

3. Once food has been placed in the mouth, both mechanical and chemical digestions begin. The six activities of the digestive process are: a. ingestion, mastication, digestion, deglutition, absorption, egestion b. ingestion, mastication, deglutition, digestion, absorption, egestion c. deglutition, ingestion, mastication, egestion, absorption, defecation d. ingestion, digestion, mastication, deglutition, absorption, defecation 4. Most digestive activity occurs in the pyloric region of the stomach. What hormone stimulates the chief cells to produce pepsinogen? a. Gastrin b. Pepsin c. HCl d. Insulin 5. What pancreatic enzyme aids in the digestion of carbohydrates? a. Lipase b. Trypsin c. Amylase d. Chymotrypsin Situation 2: Nurse Dorina is going to perform an abdominal examination to Mr. Lim who was admitted due to on and off pain since yesterday. 6. How will you position Mr. Lim prior to procedure? a. supine with knees flexed b. prone c. lying on back d. sim’s 7. To identify any localized bulging, distention and peristaltic waves, Nurse Dorina must perform which of the following? a. Auscultation b. Inspection c. Palpation d. Percussion 8. In order to identify areas of tenderness and swelling, Nurse Dorina must do: a. deep palpation b. light palpation c. percussion

d. palpation 9. Mr. Lim verbalized pain on the right iliac region. Nurse Dorina knows that the organ affected would be the: a. liver b. sigmoid colon c. appendix d. duodenum 10. Mr. Lim felt pain upon release of Nurse Dorina’s hand. This can be referred as: a. referred pain b. rebound tenderness c. direct tenderness d. indirect tenderness Situation 3: Mrs. Cruz was admitted in the Medical Floor due to pyrosis, dyspepsia and difficulty of swallowing. 11. Based from the symptoms presented, Nurse Yoshi might suspect: a. Esophagitis b. Hiatal hernia c. GERD d. Gastric Ulcer 12. What diagnostic test would confirm the type of problem Mrs. Cruz have? a. barium enema b. barium swallow c. colonoscopy d. lower GI series 13. Mrs. Cruz complained of pain and difficulty in swallowing. This term is referred as: a. Odynophagia b. Dysphagia c. Pyrosis d. Dyspepsia 14. To avoid acid reflux, Nurse Yoshi should advice Mrs. Cruz to avoid which type of diet? a. cola, coffee and tea b. high fat, carbonated and caffeinated beverages

c. beer and green tea d. lechon paksiw and bicol express 15. Mrs. Cruz’ body mass index (BMI) is 25. You can categorized her as: a. normal b. overweight c. underweight d. obese Situation 4: Nurse Gloria is the staff nurse assigned at the Emergency Department. During her shift, a patient was rushed – in the ED complaining of severe heartburn, vomiting and pain that radiates to the flank. The doctor suspects gastric ulcer. 16. What other symptoms will validate the diagnosis of gastric ulcer? a. right epigastric pain b. pain occurs when stomach is empty c. pain occurs immediately after meal d. pain not relieved by vomiting 17. What diagnostic test would yield good visualization of the ulcer crater? a. Endoscopy b. Gastrocopy c. Barium Swallow d. Histology 18. Peptic ulcer disease particularly gastric ulcer is thought to be cause by which of the following microorgamisms? a. E. coli b. H. pylori c. S. aureus d. K. pnuemoniae 19. She is for occult blood test, what specimen will you collect? a. Blood b. Urine c. Stool d. Gastric Juice 20. Preparation of the client for occult blood examination is: a. Fluid intake limited only to 1 liter/day b. NPO for 12 hours prior to obtaining of specimen

c. Increase fluid intake d. Meatless diet for 48 hours prior to obtaining of specimen Situation 5: IBD is a common inflammatory functional bowel disorder also known as spastic bowel, functional colitis and mucous colitis. 21. The client with IBS asks Nurse June what causes the disease. Which of the following responses by Nurse June would be most appropriate? a. “This is an inflammation of the bowel caused by eating too much roughage” b. “IBS is caused by a stressful lifestyle” c. “The cause of this condition is unknown” d. “There is thinning of the intestinal mucosa caused by ingestion of gluten” 22. Which of the following alimentary canal is the most common location for Chron’s disease? a. Descending colon b. Jejunum c. Sigmoid Colon d. Terminal Ileum 23. Which of the following factors is believed to be linked to Chron’s disease? a. Diet b. Constipation c. Heredity d. Lack of exercise 24. How about ulcerative colitis, which of the following factors is believed to cause it? a. Acidic diet b. Altered immunity c. Chronic constipation d. Emotional stress 25. Mr. Jung, had ulcerative colitis for 5 years and was admitted to the hospital. Which of the following factors was most likely of greatest significance in causing an exacerbation of the disease? a. A demanding and stressful job b. Changing to a modified vegetarian diet c. Beginning a weight training program d. Walking 2 miles everyday

Situation 6: A patient was admitted in the Medical Floor at St. Luke’s Hospital. He was asymptomatic. The doctor suspects diverticulosis. 26. Which of the following definitions best describes diverticulosis? a. An inflamed outpouching of the intestine b. A non – inflamed outpouching of the intestine c. The partial impairment of the forward flow of instestinal contents d. An abnormal protrusions of an oxygen trough the structure that usually holds it 27. Which of the following types of diet is implicated in the development of diverticulosis? a. Low – fiber diet b. High – fiber diet c. High – protein diet d. Low – carbohydrate diet 28. Which of the following tests should be administered to client with diverticulosis? a. Proctosocpy b. Barium enema c. Barium swallow d. Gastroscopy 29. To improve Mr. Trinidad’s condition, your best nursing intervention and teaching is: a. Reduce fluid intake b. Increase fiber in the diet c. Administering of antibiotics d. Exercise to increase intraabdominal pressure 30. Upon review of Mr. Trinidad’s chart, Nurse Drew noticed that he weighs 121 lbs and his height is 5 ft, 4 in. After computing for his Body Mass Index (BMI), you can categorize him as: a. obese b. normal c. obese d. underweight Situation 7: Manny, 6 years old was admitted at Cardinal Santos Hospital due to increasing frequency of bowel movements, abdominal cramps and distension. 31. Diarrhea is said to be the leading cause of morbidity in the Philippines. Nurse Harry knows that diarrhea is present if:

a. passage of stool is more than 3 bowel movements per week b. passage of stool is less than 3 bowel movements per day c. passage of stool is more than 3 bowel movements per day d. passage of stool is less than 3 bowel movements per week 32. Diarrhea is believed to be caused by all of the following except a. increase intestinal secretions b. altered immunity c. decrease mucosal absorption d. altered motility 33. What life threatening condition may result in persistent diarrhea? a. hypokalemia b. dehydration c. cardiac dysrhytmias d. leukocytosis 34. Voluminous, watery stools can deplete fluids and electrolytes. The acid base imbalance that can occur is: a. metabolic alkalosis b. metabolic acidosis c. respiratory acidosis d. respiratory alkalosis 35. What is the immediate home care management for diarrhea? a. Milk b. Imodium c. Water d. Oresol Situation 8: Mr. Sean is admitted to the hospital with a bowel obstruction. He complained of colicky pain and inability to pass stool. 36. Which of these findings by Nurse Leonard, would indicate that the obstruction is in the early stages? a. high pitched tinkling or rumbling bowel sounds b. hypoactive bowel sounds c. no bowel sounds auscultated d. normal bowel sounds heard in all four quadrants 37. Nasogastric tube was inserted to Mr. Sean. The NGT’s primary purpose is:

a. nutrition b. decompression of bowel c. passage for medication d. aspiration of gastric contents 38. Mr. Sean has undergone surgery. Post – operatively, which of the following findings is normal? a. absent bowel sounds b. bleeding c. hemorrhage d. bowel movement 39. Client education should be given in order to prevent constipation. Nurse Leonard’s health teaching should include which of the following? a. use of natural laxatives b. fluid intake of 6 glasses per day c. use of OTC laxatives d. complete bed rest 40. Four hours post – operatively, Mr. Sean complains of guarding and rigidity of the abdomen. Nurse Leonard’s initial intervention is: a. assess for signs of peritonitis b. call the physician c. administer pain medication d. ignore the client Situation 9: Mr. Gerald Liu, 19 y/o, is being admitted to a hospital unit complaining of severe pain in the lower abdomen. Admission vital signs reveal an oral temperature of 101.2 0F. 41. Which of the following would confirm a diagnosis of appendicitis? a. The pain is localized at a position halfway between the umbilicus and the right iliac crest. b. Mr. Liu describes the pain as occurring 2 hours after eating c. The pain subsides after eating d. The pain is in the left lower quadrant 42. Which of the following complications is thought to be the most common cause of appendicitis? a. A fecalith

b. Internal bowel occlusion c. Bowel kinking d. Abdominal wall swelling 43. The doctor ordered for a complete blood count. After the test, Nurse Ray received the result from the laboratory. Which laboratory values will confirm the diagnosis of appendicitis? a. RBC 5.5 x 106/mm3 b. Hct 44 % c. WBC 13, 000/mm3 d. Hgb 15 g/dL 44. Signs and symptoms include pain in the RLQ of the abdomen that may be localize at McBurney’s point. To relieve pain, Mr. Liu should assume which position? a. Prone b. Supine, stretched out c. Sitting d. Lying with legs drawn up 45. After a few minutes, the pain suddenly stops without any intervention. Nurse Ray might suspect that: a. the appendix is still distended b. the appendix may have ruptured c. an increased in intrathoracic pressure will occur d. signs and symptoms of peritonitis occur Situation 10: Nurse Nico is caring to a 38-year-old female, G3P3 client who has been diagnosed with hemorrhoids. 46. Which of the following factors would most likely be a primary cause of her hemorrhoids? a. Her age b. Three vaginal delivery pregnancies c. Her job as a school teacher d. Varicosities in the legs 47. Client education should include minimizing client discomfort due to hemorrhoids. Nursing management should include: a. Suggest to eat low roughage diet b. Advise to wear silk undergarments c. Avoid straining during defecation

d. Use of sitz bath for 30 minutes 48. The doctor orders for Witch Hazel 5 %. Nurse Nico knows that the action of this astringent is: a. temporarily relieves pain, burning, and itching by numbing the nerve endings b. causes coagulation (clumping) of proteins in the cells of the perianal skin or the lining of the anal canal c. inhibits the growth of bacteria and other organisms d. causes the outer layers of skin or other tissues to disintegrate 49. Which position would be ideal for the client in the early postoperative period after hemorrhoidectomy? a. High Fowler’s b. Supine c. Side – lying d. Trendelenburg’s 50. Nurse Nico instructs her client who has had a hemorrhoidectomy not to used sitz bath until at least 12 hours postoperatively to avoid which of the following complications? a. Hemorrhage b. Rectal Spasm c. Urinary retention d. Constipation Answer Key http://pinoybsn.blogspot.com/2006/07/50-item-gastrointestinalh_115375823133188151.html 100 Item Obstetrics-Maternal And Child Health Nursing Examination 100 Item Obstetrics-Maternal And Child Health Nursing Examination Budek http://www.pinoybsn.tk Question Pool DISCLAIMER: Care has been taken to verify that all answers and rationale below are accurate. Please comment up if you noticed any errors or contradictions to maintain accuracy and precision of the answers as not to mislead the readers. -Budek

DEGREE OF QUESTION DIFFICULTY 4 – Very hard question, 25% Chance of answering correctly 3 – Hard question, 50% Chance of answering correctly 2 – Moderately hard question, 75% of answering correctly 1 – Easy question, 99% will answer the question correctly Introduction: The questions are coded according to their sources and will only be for my personal reference. RED questions are original questions I created. FATIMA students READ and DIGEST each of these questions carefully. Goodluck. SITUATION : [ND89] Aling Julia, a 32 year old fish vendor from baranggay matahimik came to see you at the prenatal clinic. She brought with her all her three children. Maye, 1 year 6 months; Joy, 3 and Dan, 7 years old. She mentioned that she stopped taking oral contraceptives several months ago and now suspects she is pregnant. She cannot remember her LMP. 1. Which of the following would be useful in calculating Aling Julia's EDC? [3] A. Appearance of linea negra B. First FHT by fetoscope C. Increase pulse rate D. Presence of edema 2. Which hormone is necessary for a positive pregnancy test? [1] A. Progesterone B. HCG C. Estrogen D. Placental Lactogen 3. With this pregnancy, Aling Julia is a [1] A. P3 G3 B. Primigravida C. P3 G4 D. P0 G3 4. In explaining the development of her baby, you identified in chronological order of growth of the fetus as it occurs in pregnancy as [1] A. Ovum, embryo, zygote, fetus, infant B. Zygote, ovum, embryo, fetus, infant C. Ovum, zygote, embryo, fetus, infant D. Zygote, ovum, fetus, embryo, infant

5. Aling Julia states she is happy to be pregnant. Which behavior is elicited by her during your assessment that would lead you to think she is stressed? [3] A. She told you about her drunk husband B. She states she has very meager income from selling C. She laughs at every advise you give even when its not funny D. She has difficulty following instructions 6. When teaching Aling Julia about her pregnancy, you should include personal common discomforts. Which of the following is an indication for prompt professional supervision? [2] A. Constipation and hemorrhoids B. Backache C. Facial edema D. frequent urination 7. Which of the following statements would be appropriate for you to include in Aling Julia's prenatal teaching plan? [1] A. Exercise is very tiresome, it should be avoided B. Limit your food intake C. Smoking has no harmful effect on the growth and development of fetus D. Avoid unnecessary fatigue, rest periods should be included in you schedule 8. The best advise you can give to Aling Julia regarding prevention of varicosities is [3] A. Raise the legs while in upright position and put it against the wall several times a day B. Lay flat for most hours of the day C. Use garters with nylon stocking D. Wear support hose 9. In a 32 day menstrual cycle, ovulation usually occurs on the [2] A. 14th day after menstruation B. 18th day after menstruation C. 20th day after menstruation D. 24th day after menstruation 10. Placenta is the organ that provides exchange of nutrients and waste products between mother and fetus. This develops by [4] A. First month B. Third month C. Fifth month

D. Seventh month 11. In evaluating the weight gain of Aling Julia, you know the minimum weight gain during pregnancy is [3] A. 2 lbs/wk B. 5 lbs/wk C. 7 lbs/wk D. 10 lbs/wk 12. The more accurate method of measuring fundal height is [2] A. Millimeter B. Centimeter C. Inches D. Fingerbreadths 13. To determine fetal position using Leopold's maneuvers, the first maneuver is to [1] A. Determine degree of cephalic flexion and engagement B. Determine part of fetus presenting into pelvis C. Locate the back,arms and legs D. Determine what part of fetus is in the fundus 14. Aling julia has encouraged her husband to attend prenatal classes with her. During the prenatal class, the couple expressed fear of pain during labor and delivery. The use of touch and soothing voice often promotes comfort to the laboring patient. This physical intervention is effective because [2] A. Pain perception is interrupted B. Gate control fibers are open C. It distracts the client away from the pain D. Empathy is communicated by a caring person 15. Which of the following could be considered as a positive sign of pregnancy ? [1] A. Amenorrhea, nausea, vomiting B. Frequency of urination C. Braxton hicks contraction D. Fetal outline by sonography SITUATION : [FFC] Maternal and child health is the program of the department of health created to lessen the death of infants and mother in the philippines. [2] 16. What is the goal of this program?

A. Promote mother and infant health especially during the gravida stage B. Training of local hilots C. Direct supervision of midwives during home delivery D. Health teaching to mother regarding proper newborn care 17. One philosophy of the maternal and child health nursing is [1] A. All pregnancy experiences are the same for all woman B. Culture and religious practices have little effect on pregnancy of a woman C. Pregnancy is a part of the life cycle but provides no meaning D. The father is as important as the mother 18. In maternal care, the PHN responsibility is [2] A. To secure all information that would be needing in birth certificate B. To protect the baby against tetanus neonatorum by immunizing the mother with DPT C. To reach all pregnant woman D. To assess nutritional status of existing children 19. This is use when rendering prenatal care in the rural health unit. It serves as a guide in Identification of risk factors [1] A. Underfive clinic chart B. Home based mother's record C. Client list of mother under prenatal care D. Target list of woman under TT vaccination 20. The schedule of prenatal visit in the RHU unit is [4] A. Once from 1st up to 8th month, weekly on the 9th month B. Twice in 1st and second trimester, weekly on third trimester C. Once in each trimester, more frequent for those at risk D. Frequent as possible to determine the presence of FHT each week SITUATION : Knowledge of the menstrual cycle is important in maternal health nursing. The following questions pertains to the process of menstruation 21. Menarche occurs during the pubertal period, Which of the following occurs first in the development of female sex characteristics? [2] A. Menarche B. Accelerated Linear Growth C. Breast development D. Growth of pubic hair 22. Which gland is responsible for initiating the menstrual cycle? [3]

A. Ovaries B. APG C. PPG D. Hypothalamus 23. The hormone that stimulates the ovaries to produce estrogen is [1] A. GnRH B. LH C. LHRF D. FSH 24. Which hormone stimulates oocyte maturation? [2] A. GnRH B. LH C. LHRF D. FSH 25. When is the serum estrogen level highest in the menstrual cycle? [4] A. 3rd day B. 13th day C. 14th day D. End of menstrual cycle 26. To correctly determine the day of ovulation, the nurse must [2] A. Deduct 14 days at the mid of the cycle B. Subtract two weeks at cycle's end C. Add 7 days from mid of the cycle D. Add 14 days from the end of the cycle 27. The serum progesterone is lowest during what day of the menstrual cycle? [4] A. 3rd day B. 13th day C. 14th day D. End of menstrual cycle 28. How much blood is loss on the average during menstrual period? [4] A. Half cup B. 4 tablespoon C. 3 ounces

D. 1/3 cup 29. Menstruation occurs because of which following mechanism? [2] A. Increase level of estrogen and progesterone level B. Degeneration of the corpus luteum C. Increase vascularity of the endothelium D. Surge of hormone progesterone 30. If the menstrual cycle of a woman is 35 day cycle, she will approximately [2] A. Ovulate on the 21st day with fertile days beginning on the 16th day to the 26th day of her cycle B. Ovulate on the 21st day with fertile days beginning on the 16th day to the 21th day of her cycle C. Ovulate on the 22st day with fertile days beginning on the 16th day to the 26th day of her cycle D. Ovulate on the 22st day with fertile days beginning on the 14th day to the 30th day of her cycle SITUATION : Wide knowledge about different diagnostic tests during pregnancy is an essential arsenal for a nurse to be successful. 31. The Biparietal diameter of a fetus is considered matured if it is atleast [4] A. 9.8 cm B. 8.5 cm C. 7.5 cm D. 6 cm 32. Quickening is experienced first by multigravida clients. At what week of gestation do they start to experience quickening? [4] A. 16th B. 20th C. 24th D. 28th 33. Before the start of a non stress test, The FHR is 120 BPM. The mother ate the snack and the practitioner noticed an increase from 120 BPM to 135 BPM for 15 seconds. How would you read the result? [3] A. Abnormal B. Non reactive C. Reactive D. Inconclusive, needs repeat

34. When should the nurse expect to hear the FHR using a fetoscope? [4] A. 2nd week B. 8th week C. 2nd month D. 4th month 35. When should the nurse expect to hear FHR using doppler Ultrasound? [4] A. 8th week B. 8th month C. 2nd week D. 4th month 36. The mother asks, What does it means if her maternal serum alpha feto protein is 35 ng/ml? The nurse should answer [4] A. It is normal B. It is not normal C. 35 ng/ml indicates chromosomal abberation D. 35 ng/ml indicates neural tube defect 37. Which of the following mothers needs RHOGAM? [1] A. RH + mother who delivered an RH - fetus B. RH - mother who delivered an RH + fetus C. RH + mother who delivered an RH + fetus D. RH - mother who delivered an RH - fetus 38. Which family planning method is recommended by the department of health more than any other means of contraception? [4] A. Fertility Awareness Method B. Condom C. Tubal Ligation D. Abstinence 39. How much booster dose does tetanus toxoid vaccination for pregnant women has? [4] A. 2 B. 5 C. 3 D. 4

40. Baranggay pinoybsn.tk has 70,000 population. How much nurse is needed to service this population? [4] A. 5 B. 7 C. 50 D. 70 SITUATION : [ND2I246] Reproductive health is the exercise of reproductive right with responsibility. A married couple has the responsibility to reproduce and procreate. 41. Which of the following is ONE of the goals of the reproductive health concept? [3] A. To achieve healthy sexual development and maturation B. To prevent specific RH problem through counseling C. Provide care, treatment and rehabilitation D. To practice RH as a way of life of every man and woman 42. Which of the following is NOT an element of the reproductive health? [4] A. Maternal and child health and nutrition B. Family planning C. Prevention and management of abortion complication D. Healthy sexual development and nutrition 43. In the international framework of RH, which one of the following is the ultimate goal? [3] A. Women's health in reproduction B. Attainment of optimum health C. Achievement of women's status D. Quality of life 44. Which one of the following is a determinant of RH affecting woman's ability to participate in social affairs? [3] A. Gender issues B. Socio-Economic condition C. Cultural and psychosocial factors D. Status of women 45. In the philippine RH Framework. which major factor affects RH status? [4] A. Women's lower level of literacy B. Health service delivery mechanism C. Poor living conditions lead to illness

D. Commercial sex workers are exposed to AIDS/STD. 46. Which determinant of reproductive health advocates nutrition for better health promotion and maintain a healthful life? [4] A. Socio-Economic conditions B. Status of women C. Social and gender issues D. Biological, Cultural and Psychosocial factors 47. Which of the following is NOT a strategy of RH? [3] A. Increase and improve contraceptive methods B. Achieve reproductive intentions C. Care provision focused on people with RH problems D. Prevent specific RH problem through information dessemination 48. Which of the following is NOT a goal of RH? [3] A. Achieve healthy sexual development and maturation B. Avoid illness/diseases, injuries, disabilities related to sexuality and reproduction C. Receive appropriate counseling and care of RH problems D. Strengthen outreach activities and the referral system 49. What is the VISION of the RH? [2] A. Attain QUALITY OF LIFE B. Practice RH as a WAY OF LIFE C. Prevent specific RH problem D. Health in the hands of the filipino SITUATION : [SORANGE19] Baby G, a 6 hours old newborn is admitted to the NICU because of low APGAR Score. His mother had a prolonged second stage of labor 50. Which of the following is the most important concept associated with all high risk newborn? [1] A. Support the high-risk newborn's cardiopulmonary adaptation by maintaining adequate airway B. Identify complications with early intervention in the high risk newborn to reduce morbidity and mortality C. Assess the high risk newborn for any physical complications that will assist the parent with bonding D. Support mother and significant others in their request toward adaptation to the high risk newborn

51. Which of the following would the nurse expect to find in a newborn with birth asphyxia? [1] A. Hyperoxemia B. Acidosis C. Hypocapnia D. Ketosis 52. When planning and implementing care for the newborn that has been successfully resuscitated, which of the following would be important to assess? [1] A. Muscle flaccidity B. Hypoglycemia C. Decreased intracranial pressure D. Spontaneous respiration SITUATION : [P-I/46] Nurses should be aware of the different reproductive problems. 53. When is the best time to achieve pregnancy? [2] A. Midway between periods B. Immediately after menses end C. 14 days before the next period is expected D. 14 days after the beginning of the next period 54. A factor in infertility maybe related to the PH of the vaginal canal. A medication that is ordered to alter the vaginal PH is: [2] A. Estrogen therapy B. Sulfur insufflations C. Lactic acid douches D. Na HCO3 Douches 55. A diagnostic test used to evaluate fertility is the postcoital test. It is best timed [2] A. 1 week after ovulation B. Immediately after menses C. Just before the next menstrual period D. Within 1 to 2 days of presumed ovulation 56. A tubal insufflation test is done to determine whether there is a tubal obstruction. Infertility caused by a defect in the tube is most often related to a [3] A. Past infection B. Fibroid Tumor C. Congenital Anomaly

D. Previous injury to a tube 57. Which test is commonly used to determine the number, motility and activity of sperm is the [2] A. Rubin test B. Huhner test C. Friedman test D. Papanicolau test 58. In the female, Evaluation of the pelvic organs of reproduction is accomplished by [2] A. Biopsy B. Cystoscopy C. Culdoscopy D. Hysterosalpingogram 59. When is the fetal weight gain greatest? [3] A. 1st trimester B. 2nd trimester C. 3rd trimester D. from 4th week up to 16th week of pregnancy 60. In fetal blood vessel, where is the oxygen content highest? [3] A. Umbilical artery B. Ductus Venosus C. Ductus areteriosus D. Pulmonary artery 61. The nurse is caring for a woman in labor. The woman is irritable, complains of nausea and vomits and has heavier show. The membranes rupture. The nurse understands that this indicates [1] A. The woman is in transition stage of labor B. The woman is having a complication and the doctor should be notified C. Labor is slowing down and the woman may need oxytocin D. The woman is emotionally distraught and needs assistance in dealing with labor SITUATION : [J2I246] Katherine, a 32 year old primigravida at 39-40 weeks AOG was admitted to the labor room due to hypogastric and lumbo-sacral pains. IE revealed a fully dilated, fully effaced cervix. Station 0. 62. She is immediately transferred to the DR table. Which of the following conditions signify that delivery is near? [2]

I - A desire to defecate II - Begins to bear down with uterine contraction III - Perineum bulges IV - Uterine contraction occur 2-3 minutes intervals at 50 seconds duration A. I,II,III B. I,II,III,IV C. I,III,IV D. II,III,IV 63. Artificial rupture of the membrane is done. Which of the following nursing diagnoses is the priority? [2] A. High risk for infection related to membrane rupture B. Potential for injury related to prolapse cord C. Alteration in comfort related to increasing strength of uterine contraction D. Anxiety related to unfamiliar procedure 64. Katherine complains of severe abdominal pain and back pain during contraction. Which two of the following measures will be MOST effective in reducing pain? [4] I - Rubbing the back with a tennis ball II- Effleurage III-Imagery IV-Breathing techniques A. II,IV B. II,III C. I,IV D. I,II 65. Lumbar epidural anesthesia is administered. Which of the following nursing responsibilities should be done immediately following procedure? [1] A. Reposition from side to side B. Administer oxygen C. Increase IV fluid as indicated D. Assess for maternal hypotension 66. Which is NOT the drug of choice for epidural anesthesia? [4] A. Sensorcaine B. Xylocaine C. Ephedrine D. Marcaine

SITUATION : [SORANGE217] Alpha, a 24 year old G4P3 at full term gestation is brought to the ER after a gush of fluid passes through here vagina while doing her holiday shopping. 67. She is brought to the triage unit. The FHT is noted to be 114 bpm. Which of the following actions should the nurse do first? [2] A. Monitor FHT ever 15 minutes B. Administer oxygen inhalation C. Ask the charge nurse to notify the Obstetrician D. Place her on the left lateral position 68. The nurse checks the perineum of alpha. Which of the following characteristic of the amniotic fluid would cause an alarm to the nurse? [1] A. Greenish B. Scantly C. Colorless D. Blood tinged 69. Alpha asks the nurse. "Why do I have to be on complete bed rest? I am not comfortable in this position." Which of the following response of the nurse is most appropriate? [3] A. Keeping you on bed rest will prevent possible cord prolapse B. Completed bed rest will prevent more amniotic fluid to escape C. You need to save your energy so you will be strong enough to push later D. Let us ask your obstetrician when she returns to check on you 70. Alpha wants to know how many fetal movements per hour is normal, the correct response is [4] A. Twice B. Thrice C. Four times D. 10-12 times 71. Upon examination by the obstetrician, he charted that Alpha is in the early stage of labor. Which of the following is true in this state? [1] A. Self-focused B. Effacement is 100% C. Last for 2 hours D. Cervical dilation 1-3 cm

SITUATION : Maternal and child health nursing a core concept of providing health in the community. Mastery of MCH Nursing is a quality all nurse should possess. 72. When should be the 2nd visit of a pregnant mother to the RHU? [2] A. Before getting pregnant B. As early in pregnancy C. Second trimester D. Third trimester 73. Which of the following is NOT a standard prenatal physical examination? [1] A. Neck examination for goiter B. Examination of the palms of the hands for pallor C. Edema examination of the face hands, and lower extremeties D. Examination of the legs for varicosities 74. Which of the following is NOT a basic prenatal service delivery done in the BHS? [2] A. Oral / Dental check up B. Laboratory examination C. Treatment of diseases D. Iron supplementation 75. How many days and how much dosage will the IRON supplementation be taken? [4] A. 365 days / 300 mg B. 210 days / 200 mg C. 100 days/ 100mg D. 50 days / 50 mg 76. When should the iron supplementation starts and when should it ends? [4] A. 5th month of pregnancy to 2nd month post partum B. 1st month of pregnancy to 5th month post partum C. As early in pregnancy up to 9th month of pregnancy D. From 1st trimester up to 6 weeks post partum 77. In malaria infested area, how is chloroquine given to pregnant women? [4] A. 300 mg / twice a month for 9 months B. 200 mg / once a week for 5 months C. 150 mg / twice a week for the duration of pregnancy D. 100 mg / twice a week for the last trimester of pregnancy 78. Which of the following mothers are qualified for home delivery? [2]

A. Pre term B. 6th pregnancy C. Has a history of hemorrhage last pregnancy D. 2nd pregnancy, Has a history of 20 hours of labor last pregnancy. 79. Which of the following is not included on the 3 Cs of delivery? [2] A. Clean Surface B. Clean Hands C. Clean Equipments D. Clean Cord 80. Which of the following is unnecessary equipment to be included in the home delivery kit? [4] A. Boiled razor blade B. 70% Isopropyl Alcohol C. Flashlight D. Rectal and oral thermometer SITUATION : [NBLUE166] Pillar is admitted to the hospital with the following signs : Contractions coming every 10 minutes, lasting 30 seconds and causing little discomfort. Intact membranes without any bloody shows. Stable vital signs. FHR = 130bpm. Examination reveals cervix is 3 cm dilated with vertex presenting at minus 1 station. 81. On the basis of the data provided above, You can conclude the pillar is in the [1] A. In false labor B. In the active phase of labor C. In the latent phase of labor D. In the transitional phase of labor 82. Pitocin drip is started on Pilar. Possible side effects of pitocin administration include all of the following except [3] A. Diuresis B. Hypertension C. Water intoxication D. Cerebral hemorrhage 83. The normal range of FHR is approximately [3] A. 90 to 140 bpm B. 120 to 160 bpm C. 100 to 140 bpm

D. 140 to 180 bpm 84. A negative 1 [-1] station means that [1] A. Fetus is crowning B. Fetus is floating C. Fetus is engaged D. Fetus is at the ischial spine 85. Which of the following is characteristics of false labor [1] A. Bloody show B. Contraction that are regular and increase in frequency and duration C. Contraction are felt in the back and radiates towards the abdomen D. None of the above 86. Who's Theory of labor pain that states that PAIN in labor is cause by FEAR [4] A. Bradley B. Simpson C. Lamaze D. Dick-Read 87. Which sign would alert the nurse that Pillar is entering the second stage of labor? [1] A. Increase frequency and intensity of contraction B. Perineum bulges and anal orifice dilates C. Effacement of internal OS is 100% D. Vulva encircles the largest diameter of presenting part 88. Nursing care during the second stage of labor should include [1] A. Careful evaluation of prenatal history B. Coach breathing, Bear down with each contraction and encourage patient. C. Shave the perineum D. Administer enema to the patient SITUATION : [NBLUE170] Baby boy perez was delivered spontaneously following a term pregnancy. Apgar scores are 8 and 9 respectively. Routine procedures are carried out. 89. When is the APGAR Score taken? [1] A. Immediately after birth and at 30 minutes after birth B. At 5 minutes after birth and at 30 minutes after birth C. At 1 minute after birth and at 5 minutes after birth

D. Immediately after birth and at 5 minutes after birth 90. The best way to position a newboarn during the first week of life is to lay him [3] A. Prone with head slightly elevated B. On his back, flat C. On his side with his head flat on bed D. On his back with head slightly elevated 91. Baby boy perez has a large sebaceous glands on his nose, chin, and forehead. These are known as [1] A. Milia B. Lanugo C. Hemangiomas D. Mongolian spots 92. Baby boy perez must be carefully observed for the first 24 hours for [2] A. Respiratory distress B. Duration of cry C. Frequency of voiding D. Range in body temperature 93. According to the WHO , when should the mother starts breastfeeding the infant? [4] A. Within 30 minutes after birth B. Within 12 hours after birth C. Within a day after birth D. After infant's condition stabilizes 94. What is the BEST and most accurate method of measuring the medication dosage for infants and children? [3] A. Weight B. Height C. Nomogram D. Weight and Height 95. The first postpartum visit should be done by the mother within [4] A. 24 hours B. 3 days C. a week D. a month

96. The major cause of maternal mortality in the philippines is [3] A. Infection B. Hemorrhage C. Hypertension D. Other complications related to labor,delivery and puerperium 97. According to the WHO, what should be the composition of a commercialized Oral rehydration salt solution? [4] A. Potassium : 1.5 g. ; Sodium Bicarbonate 2.5g ; Sodium Chloride 3.5g; Glucose 20 g. A. Potassium : 1.5 g. ; Sodium Bicarbonate 2.5g ; Sodium Chloride 3.5g; Glucose 10 g. A. Potassium : 2.5 g. ; Sodium Bicarbonate 3.5g ; Sodium Chloride 4.5g; Glucose 20 g. A. Potassium : 2.5 g. ; Sodium Bicarbonate 3.5g ; Sodium Chloride 4.5g; Glucose 10 g. 98. In preparing ORESOL at home, The correct composition recommnded by the DOH is [4] A. 1 glass of water, 1 pinch of salt and 2 tsp of sugar B. 1 glass of water, 2 pinch of salt and 2 tsp of sugar C. 1 glass of water, 3 pinch of salt and 4 tsp of sugar D. 1 glass of water, 1 pinch of salt and 1 tsp of sugar 99. Milk code is a law that prohibits milk commercialization or artificial feeding for up to 2 years. Which law provides its legal basis? [4] A. Senate bill 1044 B. RA 7600 C. Presidential Proclamation 147 D. EO 51 100. A 40 year old mother in her third trimester should avoid [4] A. Traveling B. Climbing C. Smoking D. Exercising Passing : 60/100 CLICK HERE FOR ANSWERS AND RATIONALE Question? Comments? Comment up below or email me : [email protected] Budek http://www.pinoybsn.tk

Posted by Budek on Sunday, July 09, 2006 at 3:59 PM Permalink SITUATION : [ND89] Aling Julia, a 32 year old fish vendor from baranggay matahimik came to see you at the prenatal clinic. She brought with her all her three children. Maye, 1 year 6 months; Joy, 3 and Dan, 7 years old. She mentioned that she stopped taking oral contraceptives several months ago and now suspects she is pregnant. She cannot remember her LMP. 1. Which of the following would be useful in calculating Aling Julia's EDC? [3]B. First FHT by fetoscope 2. Which hormone is necessary for a positive pregnancy test? [1]B. HCG 3. With this pregnancy, Aling Julia is a [1]C. P3 G4 4. In explaining the development of her baby, you identified in chronological order of growth of the fetus as it occurs in pregnancy as [1]C. Ovum, zygote, embryo, fetus, infant 5. Aling Julia states she is happy to be pregnant. Which behavior is elicited by her during your assessment that would lead you to think she is stressed? [3]C. She laughs at every advise you give even when its not funny 6. When teaching Aling Julia about her pregnancy, you should include personal common discomforts. Which of the following is an indication for prompt professional supervision? [2] C. Facial edema 7. Which of the following statements would be appropriate for you to include in Aling Julia's prenatal teaching plan? [1] D. Avoid unnecessary fatigue, rest periods should be included in you schedule 8. The best advise you can give to Aling Julia regarding prevention of varicosities is [3] D. Wear support hose 9. In a 32 day menstrual cycle, ovulation usually occurs on the [2] B. 18th day after menstruation 10. Placenta is the organ that provides exchange of nutrients and waste products between mother and fetus. This develops by [4]

B. Third month 11. In evaluating the weight gain of Aling Julia, you know the minimum weight gain during pregnancy is [3] A. 2 lbs/wk 12. The more accurate method of measuring fundal height is [2] B. Centimeter 13. To determine fetal position using Leopold's maneuvers, the first maneuver is to [1] D. Determine what part of fetus is in the fundus 14. Aling julia has encouraged her husband to attend prenatal classes with her. During the prenatal class, the couple expressed fear of pain during labor and delivery. The use of touch and soothing voice often promotes comfort to the laboring patient. This physical intervention is effective because [2] D. Empathy is communicated by a caring person 15. Which of the following could be considered as a positive sign of pregnancy ? [1] D. Fetal outline by sonography SITUATION : [FFC] Maternal and child health is the program of the department of health created to lessen the death of infants and mother in the philippines. [2] 16. What is the goal of this program? A. Promote mother and infant health especially during the gravida stage 17. One philosophy of the maternal and child health nursing is [1] D. The father is as important as the mother 18. In maternal care, the PHN responsibility is [2] C. To reach all pregnant woman 19. This is use when rendering prenatal care in the rural health unit. It serves as a guide in Identification of risk factors [1] B. Home based mother's record

20. The schedule of prenatal visit in the RHU unit is [4] C. Once in each trimester, more frequent for those at risk SITUATION : Knowledge of the menstrual cycle is important in maternal health nursing. The following questions pertains to the process of menstruation 21. Menarche occurs during the pubertal period, Which of the following occurs first in the development of female sex characteristics? [2] B. Accelerated Linear Growth 22. Which gland is responsible for initiating the menstrual cycle? [3] D. Hypothalamus 23. The hormone that stimulates the ovaries to produce estrogen is [1] A. GnRH 24. Which hormone stimulates oocyte maturation? [2] D. FSH 25. When is the serum estrogen level highest in the menstrual cycle? [4] B. 13th day 26. To correctly determine the day of ovulation, the nurse must [2] D. Add 14 days from the end of the cycle 27. The serum progesterone is lowest during what day of the menstrual cycle? [4] B. 13th day 28. How much blood is loss on the average during menstrual period? [4] B. 4 tablespoon 29. Menstruation occurs because of which following mechanism? [2] B. Degeneration of the corpus luteum 30. If the menstrual cycle of a woman is 35 day cycle, she will approximately [2]

A. Ovulate on the 21st day with fertile days beginning on the 16th day to the 26th day of her cycle SITUATION : Wide knowledge about different diagnostic tests during pregnancy is an essential arsenal for a nurse to be successful. 31. The Biparietal diameter of a fetus is considered matured if it is atleast [4] B. 8.5 cm 32. Quickening is experienced first by multigravida clients. At what week of gestation do they start to experience quickening? [4] A. 16th 33. Before the start of a non stress test, The FHR is 120 BPM. The mother ate the snack and the practitioner noticed an increase from 120 BPM to 135 BPM for 15 seconds. How would you read the result? [3] C. Reactive 34. When should the nurse expect to hear the FHR using a fetoscope? [4] B. 8th week 35. When should the nurse expect to hear FHR using doppler Ultrasound? [4] C. 2nd week 36. The mother asks, What does it means if her maternal serum alpha feto protein is 35 ng/ml? The nurse should answer [4] D. 35 ng/ml indicates neural tube defect 37. Which of the following mothers needs RHOGAM? [1] B. RH - mother who delivered an RH + fetus 38. Which family planning method is recommended by the department of health more than any other means of contraception? [4] D. Abstinence 39. How much booster dose does tetanus toxoid vaccination for pregnant women has? [4] C. 3

40. Baranggay pinoybsn.tk has 70,000 population. How much nurse is needed to service this population? [4] B. 7 SITUATION : [ND2I246] Reproductive health is the exercise of reproductive right with responsibility. A married couple has the responsibility to reproduce and procreate. 41. Which of the following is ONE of the goals of the reproductive health concept? [3] A. To achieve healthy sexual development and maturation 42. Which of the following is NOT an element of the reproductive health? [4] B. Family planning 43. In the international framework of RH, which one of the following is the ultimate goal? [3] D. Quality of life 44. Which one of the following is a determinant of RH affecting woman's ability to participate in social affairs? [3] A. Gender issues 45. In the philippine RH Framework. which major factor affects RH status? [4] C. Poor living conditions lead to illness 46. Which determinant of reproductive health advocates nutrition for better health promotion and maintain a healthful life? [4] A. Socio-Economic conditions 47. Which of the following is NOT a strategy of RH? [3] A. Increase and improve contraceptive methods 48. Which of the following is NOT a goal of RH? [3] D. Strengthen outreach activities and the referral system

49. What is the VISION of the RH? [2] A. Attain QUALITY OF LIFE SITUATION : [SORANGE19] Baby G, a 6 hours old newborn is admitted to the NICU because of low APGAR Score. His mother had a prolonged second stage of labor 50. Which of the following is the most important concept associated with all high risk newborn? [1] B. Identify complications with early intervention in the high risk newborn to reduce morbidity and mortality rn 51. Which of the following would the nurse expect to find in a newborn with birth asphyxia? [1] B. Acidosis 52. When planning and implementing care for the newborn that has been successfully resuscitated, which of the following would be important to assess? [1] D. Spontaneous respiration SITUATION : [P-I/46] Nurses should be aware of the different reproductive problems. 53. When is the best time to achieve pregnancy? [2] C. 14 days before the next period is expected 54. A factor in infertility maybe related to the PH of the vaginal canal. A medication that is ordered to alter the vaginal PH is: [2] D. Na HCO3 Douches 55. A diagnostic test used to evaluate fertility is the postcoital test. It is best timed [2] B. Immediately after menses 56. A tubal insufflation test is done to determine whether there is a tubal obstruction. Infertility caused by a defect in the tube is most often related to a [3] D. Previous injury to a tube 57. Which test is commonly used to determine the number, motility and activity of sperm is the [2]

B. Huhner test 58. In the female, Evaluation of the pelvic organs of reproduction is accomplished by [2] D. Hysterosalpingogram 59. When is the fetal weight gain greatest? [3] C. 3rd trimester 60. In fetal blood vessel, where is the oxygen content highest? [3] B. Ductus Venosus 61. The nurse is caring for a woman in labor. The woman is irritable, complains of nausea and vomits and has heavier show. The membranes rupture. The nurse understands that this indicates [1] A. The woman is in transition stage of labor SITUATION : [J2I246] Katherine, a 32 year old primigravida at 39-40 weeks AOG was admitted to the labor room due to hypogastric and lumbo-sacral pains. IE revealed a fully dilated, fully effaced cervix. Station 0. 62. She is immediately transferred to the DR table. Which of the following conditions signify that delivery is near? [2] I - A desire to defecate II - Begins to bear down with uterine contraction III - Perineum bulges IV - Uterine contraction occur 2-3 minutes intervals at 50 seconds duration B. I,II,III,IV 63. Artificial rupture of the membrane is done. Which of the following nursing diagnoses is the priority? [2] A. High risk for infection related to membrane rupture 64. Katherine complains of severe abdominal pain and back pain during contraction. Which two of the following measures will be MOST effective in reducing pain? [4] I - Rubbing the back with a tennis ball II- Effleurage III-Imagery

IV-Breathing techniques D. I,II 65. Lumbar epidural anesthesia is administered. Which of the following nursing responsibilities should be done immediately following procedure? [1] D. Assess for maternal hypotension 66. Which is NOT the drug of choice for epidural anesthesia? [4] C. Ephedrine SITUATION : [SORANGE217] Alpha, a 24 year old G4P3 at full term gestation is brought to the ER after a gush of fluid passes through here vagina while doing her holiday shopping. 67. She is brought to the triage unit. The FHT is noted to be 114 bpm. Which of the following actions should the nurse do first? [2] A. Monitor FHT ever 15 minutes 68. The nurse checks the perineum of alpha. Which of the following characteristic of the amniotic fluid would cause an alarm to the nurse? [1] A. Greenish 69. Alpha asks the nurse. "Why do I have to be on complete bed rest? I am not comfortable in this position." Which of the following response of the nurse is most appropriate? [3] C. You need to save your energy so you will be strong enough to push later 70. Alpha wants to know how many fetal movements per hour is normal, the correct response is [4] C. Four times 71. Upon examination by the obstetrician, he charted that Alpha is in the early stage of labor. Which of the following is true in this state? [1] D. Cervical dilation 1-3 cm SITUATION : Maternal and child health nursing a core concept of providing health in the community. Mastery of MCH Nursing is a quality all nurse should possess.

72. When should be the 2nd visit of a pregnant mother to the RHU? [2] C. Second trimester 73. Which of the following is NOT a standard prenatal physical examination? [1] D. Examination of the legs for varicosities 74. Which of the following is NOT a basic prenatal service delivery done in the BHS? [2] A. Oral / Dental check up 75. How many days and how much dosage will the IRON supplementation be taken? [4] B. 210 days / 200 mg 76. When should the iron supplementation starts and when should it ends? [4] A. 5th month of pregnancy to 2nd month post partum 77. In malaria infested area, how is chloroquine given to pregnant women? [4] C. 150 mg / twice a week for the duration of pregnancy 78. Which of the following mothers are qualified for home delivery? [2] D. 2nd pregnancy, Has a history of 20 hours of labor last pregnancy. 79. Which of the following is not included on the 3 Cs of delivery? [2] C. Clean Equipments 80. Which of the following is unnecessary equipment to be included in the home delivery kit? [4] C. Flashlight SITUATION : [NBLUE166] Pillar is admitted to the hospital with the following signs : Contractions coming every 10 minutes, lasting 30 seconds and causing little discomfort. Intact membranes without any bloody shows. Stable vital signs. FHR = 130bpm. Examination reveals cervix is 3 cm dilated with vertex presenting at minus 1 station. 81. On the basis of the data provided above, You can conclude the pillar is in the [1] C. In the latent phase of labor

82. Pitocin drip is started on Pilar. Possible side effects of pitocin administration include all of the following except [3] A. Diuresis 83. The normal range of FHR is approximately [3] B. 120 to 160 bpm 84. A negative 1 [-1] station means that [1] B. Fetus is floating 85. Which of the following is characteristics of false labor [1] D. None of the above 86. Who's Theory of labor pain that states that PAIN in labor is cause by FEAR [4] A. Bradley 87. Which sign would alert the nurse that Pillar is entering the second stage of labor? [1] D. Vulva encircles the largest diameter of presenting part 88. Nursing care during the second stage of labor should include [1] B. Coach breathing, Bear down with each contraction and encourage patient. SITUATION : [NBLUE170] Baby boy perez was delivered spontaneously following a term pregnancy. Apgar scores are 8 and 9 respectively. Routine procedures are carried out. 89. When is the APGAR Score taken? [1] C. At 1 minute after birth and at 5 minutes after birth 90. The best way to position a newboarn during the first week of life is to lay him [3] B. On his back, flat 91. Baby boy perez has a large sebaceous glands on his nose, chin, and forehead. These are known as [1] A. Milia

92. Baby boy perez must be carefully observed for the first 24 hours for [2] A. Respiratory distress 93. According to the WHO , when should the mother starts breastfeeding the infant? [4] A. Within 30 minutes after birth 94. What is the BEST and most accurate method of measuring the medication dosage for infants and children? [3] C. Nomogram 95. The first postpartum visit should be done by the mother within [4] A. 24 hours B. 3 days C. a week D. a month 96. The major cause of maternal mortality in the philippines is [3] D. Other complications related to labor,delivery and puerperium 97. According to the WHO, what should be the composition of a commercialized Oral rehydration salt solution? [4] A. Potassium : 1.5 g. ; Sodium Bicarbonate 2.5g ; Sodium Chloride 3.5g; Glucose 20 g. 98. In preparing ORESOL at home, The correct composition recommnded by the DOH is [4] A. 1 glass of water, 1 pinch of salt and 2 tsp of sugar 99. Milk code is a law the prohibits milk commercialization or artificial feeding for up to 2 years. Which law provides its legal basis? [4] D. EO 51 100. A 40 year old mother in her third trimester should avoid [4] C. Smoking 100 Item MEDICAL SURGICAL Nursing Examination

Budek http://www.pinoybsn.tk Question Pool MEDICAL SURGICAL NURSING DEGREE OF QUESTION DIFFICULTY 4 – Very hard question, 25% Chance of answering correctly 3 – Hard question, 50% Chance of answering correctly 2 – Moderately hard question, 75% of answering correctly 1 – Easy question, 99% will answer the question correctly SITUATION : Dervid, A registered nurse, witnessed an old woman hit by a motorcycle while crossing a train railway. The old woman fell at the railway. Dervid Rushed at the scene. 1. As a registered nurse, Dervid knew that the first thing that he will do at the scene is [3] A. Stay with the person, Encourage her to remain still and Immobilize the leg while While waiting for the ambulance. B. Leave the person for a few moments to call for help. C. Reduce the fracture manually. D. Move the person to a safer place. 2. Dervid suspects a hip fracture when he noticed that the old woman’s leg is [4] A. Lengthened, Abducted and Internally Rotated. B. Shortened, Abducted and Externally Rotated. C. Shortened, Adducted and Internally Rotated. D. Shortened, Adducted and Externally Rotated. 3. The old woman complains of pain. John noticed that the knee is reddened, warm to touch and swollen. John interprets that this signs and symptoms are likely related to [2] A. Infection C. Thrombophlebitis B. Inflammation D. Degenerative disease 4. The old woman told John that she has osteoporosis; Dervid knew that all of the following factors would contribute to osteoporosis except [4] A. Hypothyroidism

B. End stage renal disease C. Cushing’s Disease D. Taking Furosemide and Phenytoin. 5. Martha, The old woman was now Immobilized and brought to the emergency room. The X-ray shows a fractured femur and pelvis. The ER Nurse would carefully monitor Martha for which of the following sign and symptoms? [3] A. Tachycardia and Hypotension B. Fever and Bradycardia C. Bradycardia and Hypertension D. Fever and Hypertension SITUATION: Mr. D. Rojas, An obese 35 year old MS Professor of OLFU Lagro is admitted due to pain in his weight bearing joint. The diagnosis was Osteoarthritis. 6. As a nurse, you instructed Mr. Rojas how to use a cane. Mr. Rojas has a weakness on his right leg due to self immobilization and guarding. You plan to teach Mr. Rojas to hold the cane [4] A. On his left hand, because his right side is weak. B. On his left hand, because of reciprocal motion. C. On his right hand, to support the right leg. D. On his right hand, because only his right leg is weak. 7. You also told Mr. Rojas to hold the cane [4] A. 1 Inches in front of the foot. B. 3 Inches at the lateral side of the foot. c. 6 Inches at the lateral side of the foot. D. 12 Inches at the lateral side of the foot. 8. Mr. Rojas was discharged and 6 months later, he came back to the emergency room of the hospital because he suffered a mild stroke. The right side of the brain was affected. At the rehabilitative phase of your nursing care, you observe Mr. Rojas use a cane and you intervene if you see him [4] A. Moves the cane when the right leg is moved. B. Leans on the cane when the right leg swings through. C. keeps the cane 6 Inches out to the side of the right foot. D. Holds the cane on the right side. SITUATION: Alfred, a 40 year old construction worker developed cough, night sweats and fever. He was brought to the nursing unit for diagnostic studies. He told the nurse he did not receive a BCG vaccine during childhood

9. The nurse performs a Mantoux Test. The nurse knows that Mantoux Test is also known as [1] A. PPD B. PDP C. PDD D. DPP 10. The nurse would inject the solution in what route? [1] A. IM B. IV C. ID D. SC 11. The nurse notes that a positive result for Alfred is [2] A. 5 mm wheal B. 5 mm Induration C. 10 mm Wheal D. 10 mm Induration 12. The nurse told Alfred to come back after [2] A. a week B. 48 hours C. 1 day D. 4 days 13. Mang Alfred returns after the Mantoux Test. The test result read POSITIVE. What should be the nurse’s next action? [3] A. Call the Physician B. Notify the radiology dept. for CXR evaluation C. Isolate the patient D. Order for a sputum exam 14. Why is Mantoux test not routinely done in the Philippines? [2] A. It requires a highly skilled nurse to perform a Mantoux test B. The sputum culture is the gold standard of PTB Diagnosis and it will definitively determine the extent of the cavitary lesions C. Chest X Ray Can diagnose the specific microorganism responsible for the lesions D. Almost all Filipinos will test positive for Mantoux Test

15. Mang Alfred is now a new TB patient with an active disease. What is his category according to the DOH? [1] A. I B. II C. III D. IV 16. How long is the duration of the maintenance phase of his treatment? [2] A. 2 months B. 3 months C. 4 months D. 5 months 17. Which of the following drugs is UNLIKELY given to Mang Alfred during the maintenance phase? [3] A. Rifampicin B. Isoniazid C. Ethambutol D. Pyridoxine 18. According to the DOH, the most hazardous period for development of clinical disease is during the first [4] A. 6-12 months after B. 3-6 months after C. 1-2 months after D. 2-4 weeks after 19. This is the name of the program of the DOH to control TB in the country [2] A. DOTS B. National Tuberculosis Control Program C. Short Coursed Chemotherapy D. Expanded Program for Immunization 20. Susceptibility for the disease [ TB ] is increased markedly in those with the following condition except [3] A. 23 Year old athlete with diabetes insipidus B. 23 Year old athlete taking long term Decadron therapy and anabolic steroids C. 23 Year old athlete taking illegal drugs and abusing substances D. Undernourished and Underweight individual who undergone gastrectomy

21. Direct sputum examination and Chest X ray of TB symptomatic is in what level of prevention? [1] A. Primary B. Secondary C. Tertiary D. Quarterly SITUATION: Michiel, A male patient diagnosed with colon cancer was newly put in colostomy. 22. Michiel shows the BEST adaptation with the new colostomy if he shows which of the following? [2] A. Look at the ostomy site B. Participate with the nurse in his daily ostomy care C. Ask for leaflets and contact numbers of ostomy support groups D. Talk about his ostomy openly to the nurse and friends 23. The nurse plans to teach Michiel about colostomy irrigation. As the nurse prepares the materials needed, which of the following item indicates that the nurse needs further instruction? [3] A. Plain NSS / Normal Saline B. K-Y Jelly C. Tap water D. Irrigation sleeve 24. The nurse should insert the colostomy tube for irrigation at approximately [3] A. 1-2 inches B. 3-4 inches C. 6-8 inches D. 12-18 inches 25. The maximum height of irrigation solution for colostomy is [3] A. 5 inches B. 12 inches C. 18 inches D. 24 inches 26. Which of the following behavior of the client indicates the best initial step in learning to care for his colostomy? [1] A. Ask to defer colostomy care to another individual

B. Promises he will begin to listen the next day C. Agrees to look at the colostomy D. States that colostomy care is the function of the nurse while he is in the hospital 27. While irrigating the client’s colostomy, Michiel suddenly complains of severe cramping. Initially, the nurse would [1] A. Stop the irrigation by clamping the tube B. Slow down the irrigation C. Tell the client that cramping will subside and is normal D. Notify the physician 28. The next day, the nurse will assess Michiel’s stoma. The nurse noticed that a prolapsed stoma is evident if she sees which of the following? [1] A. A sunken and hidden stoma B. A dusky and bluish stoma C. A narrow and flattened stoma D. Protruding stoma with swollen appearance 29. Michiel asked the nurse, what foods will help lessen the odor of his colostomy. The nurse best response would be [4] A. Eat eggs B. Eat cucumbers C. Eat beet greens and parsley D. Eat broccoli and spinach 30. The nurse will start to teach Michiel about the techniques for colostomy irrigation. Which of the following should be included in the nurse’s teaching plan? [4] A. Use 500 ml to 1,000 ml NSS B. Suspend the irrigant 45 cm above the stoma C. Insert the cone 4 cm in the stoma D. If cramping occurs, slow the irrigation 31. The nurse knew that the normal color of Michiel’s stoma should be [1] A. Brick Red B. Gray C. Blue D. Pale Pink SITUATION: James, A 27 basketball player sustained inhalation burn that required him to have tracheostomy due to massive upper airway edema.

32. Wilma, His sister and a nurse is suctioning the tracheostomy tube of James. Which of the following, if made by Wilma indicates that she is committing an error? [2] A. Hyperventilating James with 100% oxygen before and after suctioning B. Instilling 3 to 5 ml normal saline to loosen up secretion C. Applying suction during catheter withdrawal D. Suction the client every hour 33. What size of suction catheter would Wilma use for James, who is 6 feet 5 inches in height and weighing approximately 145 lbs? [2] A. Fr. 5 B. Fr. 10 C. Fr. 12 D. Fr. 18 34. Wilma is using a portable suction unit at home, What is the amount of suction required by James using this unit? [4] A. 2-5 mmHg B. 5-10 mmHg C. 10-15 mmHg D. 20-25 mmHg 35. If a Wall unit is used, What should be the suctioning pressure required by James? [4] A. 50-95 mmHg B. 95-110 mmHg C. 100-120 mmHg D. 155-175 mmHg 36. Wilma was shocked to see that the Tracheostomy was dislodged. Both the inner and outer cannulas was removed and left hanging on James’ neck. What are the 2 equipment’s at james’ bedside that could help Wilma deal with this situation? [3] A. New set of tracheostomy tubes and Oxygen tank B. Theophylline and Epinephrine C. Obturator and Kelly clamp D. Sterile saline dressing 37. Which of the following method if used by Wilma will best assure that the tracheostomy ties are not too tightly placed? [2] A. Wilma places 2 fingers between the tie and neck B. The tracheotomy can be pulled slightly away from the neck C. James’ neck veins are not engorged

D. Wilma measures the tie from the nose to the tip of the earlobe and to the xiphoid process. 38. Wilma knew that James have an adequate respiratory condition if she notices that [1] A. James’ respiratory rate is 18 B. James’ Oxygen saturation is 91% C. There are frank blood suction from the tube D. There are moderate amount of tracheobronchial secretions 39. Wilma knew that the maximum time when suctioning James is [1] A. 10 seconds B. 20 seconds C. 30 seconds D. 45 seconds SITUATION : Juan Miguel Lopez Zobel Ayala de Batumbakal was diagnosed with Acute Close Angle Glaucoma. He is being seen by Nurse Jet. 40. What specific manifestation would nurse Jet see in Acute close angle glaucoma that she would not see in an open angle glaucoma? [3] A. Loss of peripheral vision B. Irreversible vision loss C. There is an increase in IOP D. Pain 41. Nurse jet knew that Acute close angle glaucoma is caused by [3] A. Sudden blockage of the anterior angle by the base of the iris B. Obstruction in trabecular meshwork C. Gradual increase of IOP D. An abrupt rise in IOP from 8 to 15 mmHg 42. Nurse jet performed a TONOMETRY test to Mr. Batumbakal. What does this test measures [1] A. It measures the peripheral vision remaining on the client B. Measures the Intra Ocular Pressure C. Measures the Client’s Visual Acuity D. Determines the Tone of the eye in response to the sudden increase in IOP. 43. The Nurse notices that Mr. Batumbakal cannot anymore determine RED from BLUE. The nurse knew that which part of the eye is affected by this change? [3]

A. IRIS B. PUPIL c. RODS [RETINA] D. CONES [RETINA] 44. Nurse Jet knows that Aqueous Humor is produce where? [4] A. In the sub arachnoid space of the meninges B. In the Lateral ventricles C. In the Choroids D. In the Ciliary Body 45. Nurse Jet knows that the normal IOP is [2] A. 8-21 mmHg B. 2-7 mmHg c. 31-35 mmHg D. 15-30 mmHg 46. Nurse Jet wants to measure Mr. Batumbakal’s CN II Function. What test would Nurse Jet implement to measure CN II’s Acuity? [1] A. Slit lamp B. Snellen’s Chart C. Wood’s light D. Gonioscopy 47. The Doctor orders pilocarpine. Nurse jet knows that the action of this drug is to [4] A. Contract the Ciliary muscle B. Relax the Ciliary muscle C. Dilate the pupils D. Decrease production of Aqueous Humor 48. The doctor orders timolol [timoptic]. Nurse jet knows that the action of this drug is [4] A. Reduce production of CSF B. Reduce production of Aquesous Humor C. Constrict the pupil D. Relaxes the Ciliary muscle 49. When caring for Mr. Batumbakal, Jet teaches the client to avoid [1] A. Watching large screen TVs B. Bending at the waist

C. Reading books D. Going out in the sun 50. Mr. Batumbakal has undergone eye angiography using an Intravenous dye and fluoroscopy. What activity is contraindicated immediately after procedure? [4] A. Reading newsprint B. Lying down C. Watching TV D. Listening to the music 51. If Mr. Batumbakal is receiving pilocarpine, what drug should always be available in any case systemic toxicity occurs? [2] A. Atropine Sulfate B. Pindolol [Visken] C. Naloxone Hydrochloride [Narcan] D. Mesoridazine Besylate [Serentil] SITUATION : Wide knowledge about the human ear, it’s parts and it’s functions will help a nurse assess and analyze changes in the adult client’s health. 52. Nurse Budek is doing a caloric testing to his patient, Aida, a 55 year old university professor who recently went into coma after being mauled by her disgruntled 3rd year nursing students whom she gave a failing mark. After instilling a warm water in the ear, Budek noticed a rotary nystagmus towards the irrigated ear. What does this means? [2] A. Indicates a CN VIII Dysfunction B. Abnormal C. Normal D. Inconclusive 53. Ear drops are prescribed to an infant, The most appropriate method to administer the ear drops is [2] A. Pull the pinna up and back and direct the solution towards the eardrum B. Pull the pinna down and back and direct the solution onto the wall of the canal C. Pull the pinna down and back and direct the solution towards the eardrum D. Pull the pinna up and back and direct the solution onto the wall of the canal 54. Nurse Budek is developing a plan of care for a patient with Menieres disease. What is the priority nursing intervention in the plan of care for this particular patient? [1] A. Air, Breathing, Circulation B. Love and Belongingness

C. Food, Diet and Nutrition D. Safety 55. After mastoidectomy, Nurse Budek should be aware that the cranial nerve that is usually damage after this procedure is [3] A. CN I B. CN II C. CN VII D. CN VI 56. The physician orders the following for the client with Menieres disease. Which of the following should the nurse question? [1] A. Dipenhydramine [Benadryl] B. Atropine sulfate C. Out of bed activities and ambulation D. Diazepam [Valium] 57. Nurse Budek is giving dietary instruction to a client with Menieres disease. Which statement if made by the client indicates that the teaching has been successful? [1] A. I will try to eat foods that are low in sodium and limit my fluid intake B. I must drink atleast 3,000 ml of fluids per day C. I will try to follow a 50% carbohydrate, 30% fat and 20% protein diet D. I will not eat turnips, red meat and raddish 58. Peachy was rushed by his father, Steven into the hospital admission. Peachy is complaining of something buzzing into her ears. Nurse Budek assessed peachy and found out It was an insect. What should be the first thing that Nurse Budek should try to remove the insect out from peachy’s ear? [3] A. Use a flashlight to coax the insect out of peachy’s ear B. Instill an antibiotic ear drops C. Irrigate the ear D. Pick out the insect using a sterile clean forceps 59. Following an ear surgery, which statement if heard by Nurse Budek from the patient indicates a correct understanding of the post operative instructions? [2] A. Activities are resumed within 5 days B. I will make sure that I will clean my hair and face to prevent infection C. I will use straw for drinking D. I should avoid air travel for a while

60. Nurse Budek will do a caloric testing to a client who sustained a blunt injury in the head. He instilled a cold water in the client’s right ear and he noticed that nystagmus occurred towards the left ear. What does this finding indicates? [2] A. Indicating a Cranial Nerve VIII Dysfunction B. The test should be repeated again because the result is vague C. This is Grossly abnormal and should be reported to the neurosurgeon D. This indicates an intact and working vestibular branch of CN VIII 61. A client with Cataract is about to undergo surgery. Nurse Budek is preparing plan of care. Which of the following nursing diagnosis is most appropriate to address the long term need of this type of patient? [1] A. Anxiety R/T to the operation and its outcome B. Sensory perceptual alteration R/T Lens extraction and replacement C. Knowledge deficit R/T the pre operative and post operative self care D. Body Image disturbance R/T the eye packing after surgery 62. Nurse Budek is performing a WEBERS TEST. He placed the tuning fork in the patients forehead after tapping it onto his knee. The client states that the fork is louder in the LEFT EAR. Which of the following is a correct conclusion for nurse Budek to make? [4] A. He might have a sensory hearing loss in the left ear B. Conductive hearing loss is possible in the right ear C. He might have a sensory hearing loss in the right hear, and/or a conductive hearing loss in the left ear. D. He might have a conductive hearing loss in the right ear, and/or a sensory hearing loss in the left ear. 63. Aling myrna has Menieres disease. What typical dietary prescription would nurse Budek expect the doctor to prescribe? [2] A. A low sodium , high fluid intake B. A high calorie, high protein dietary intake C. low fat, low sodium and high calorie intake D. low sodium and restricted fluid intake SITUATION : [ From DEC 1991 NLE ] A 45 year old male construction worker was admitted to a tertiary hospital for incessant vomiting. Assessment disclosed: weak rapid pulse, acute weight loss of .5kg, furrows in his tongue, slow flattening of the skin was noted when the nurse released her pinch. Temperature: 35.8 C , BUN Creatinine ratio : 10 : 1, He also complains for postural hypotension. There was no infection.

64. Which of the following is the appropriate nursing diagnosis? [1] A. Fluid volume deficit R/T furrow tongue B. Fluid volume deficit R/T uncontrolled vomiting C. Dehydration R/T subnormal body temperature D. Dehydration R/T incessant vomiting 65. Approximately how much fluid is lost in acute weight loss of .5kg? [1] A. 50 ml B. 750 ml C. 500 ml D. 75 ml 66. Postural Hypotension is [1] A. A drop in systolic pressure less than 10 mmHg when patient changes position from lying to sitting. B. A drop in systolic pressure greater than 10 mmHg when patient changes position from lying to sitting C. A drop in diastolic pressure less than 10 mmHg when patient changes position from lying to sitting D. A drop in diastolic pressure greater than 10 mmHg when patient changes position from lying to sitting 67. Which of the following measures will not help correct the patient’s condition [1] A. Offer large amount of oral fluid intake to replace fluid lost B. Give enteral or parenteral fluid C. Frequent oral care D. Give small volumes of fluid at frequent interval 68. After nursing intervention, you will expect the patient to have [1] 1. Maintain body temperature at 36.5 C 2. Exhibit return of BP and Pulse to normal 3. Manifest normal skin turgor of skin and tongue 4. Drinks fluids as prescribed A. 1,3 B. 2,4 C. 1,3,4 D. 2,3,4

SITUATION: [ From JUN 2005 NLE ] A 65 year old woman was admitted for Parkinson’s Disease. The charge nurse is going to make an initial assessment. 69. Which of the following is a characteristic of a patient with advanced Parkinson’s disease? [1] A. Disturbed vision B. Forgetfulness C. Mask like facial expression D. Muscle atrophy 70. The onset of Parkinson’s disease is between 50-60 years old. This disorder is caused by [1] A. Injurious chemical substances B. Hereditary factors C. Death of brain cells due to old age D. Impairment of dopamine producing cells in the brain 71. The patient was prescribed with levodopa. What is the action of this drug? [1] A. Increase dopamine availability B. Activates dopaminergic receptors in the basal ganglia C. Decrease acetylcholine availability D. Release dopamine and other catecholamine from neurological storage sites 72. You are discussing with the dietician what food to avoid with patients taking levodopa? [3] A. Vitamin C rich food B. Vitamin E rich food C. Thiamine rich food D. Vitamin B6 rich food 73. One day, the patient complained of difficulty in walking. Your response would be [2] A. You will need a cane for support B. Walk erect with eyes on horizon C. I’ll get you a wheelchair D. Don’t force yourself to walk SITUATION: [ From JUN 2005 NLE ] Mr. Dela Isla, a client with early Dementia exhibits thought process disturbances. 74. The nurse will assess a loss of ability in which of the following areas? [2]

A. Balance B. Judgment C. Speech D. Endurance 75. Mr. Dela Isla said he cannot comprehend what the nurse was saying. He suffers from: [1] A. Insomnia B. Aphraxia C. Agnosia D. Aphasia 76. The nurse is aware that in communicating with an elderly client, the nurse will [1] A. Lean and shout at the ear of the client B. Open mouth wide while talking to the client C. Use a low-pitched voice D. Use a medium-pitched voice 77. As the nurse talks to the daughter of Mr. Dela Isla, which of the following statement of the daughter will require the nurse to give further teaching? [1] A. I know the hallucinations are parts of the disease B. I told her she is wrong and I explained to her what is right C. I help her do some tasks he cannot do for himself D. Ill turn off the TV when we go to another room 78. Which of the following is most important discharge teaching for Mr. Dela Isla [2] A. Emergency Numbers B. Drug Compliance C. Relaxation technique D. Dietary prescription SITUATION : Knowledge of the drug PROPANTHELINE BROMIDE [Probanthine] Is necessary in treatment of various disorders. 79. What is the action of this drug? [4] A. Increases glandular secretion for clients affected with cystic fibrosis B. Dissolve blockage of the urinary tract due to obstruction of cystine stones C. Reduces secretion of the glandular organ of the body D. Stimulate peristalsis for treatment of constipation and obstruction 80. What should the nurse caution the client when using this medication [4]

A. Avoid hazardous activities like driving, operating machineries etc. B. Take the drug on empty stomach C. Take with a full glass of water in treatment of Ulcerative colitis D. I must take double dose if I missed the previous dose 81. Which of the following drugs are not compatible when taking Probanthine? [4] A. Caffeine B. NSAID C. Acetaminophen D. Alcohol 82. What should the nurse tell clients when taking Probanthine? [4] A. Avoid hot weathers to prevent heat strokes B. Never swim on a chlorinated pool C. Make sure you limit your fluid intake to 1L a day D. Avoid cold weathers to prevent hypothermia 83. Which of the following disease would Probanthine exert the much needed action for control or treatment of the disorder? [4] A. Urinary retention B. Peptic Ulcer Disease C. Ulcerative Colitis D. Glaucoma SITUATION : [ From DEC 2000 NLE ] Mr. Franco, 70 years old, suddenly could not lift his spoons nor speak at breakfast. He was rushed to the hospital unconscious. His diagnosis was CVA. 84. Which of the following is the most important assessment during the acute stage of an unconscious patient like Mr. Franco? [1] A. Level of awareness and response to pain B. Papillary reflexes and response to sensory stimuli C. Coherence and sense of hearing D. Patency of airway and adequacy of respiration 85. Considering Mr. Franco’s conditions, which of the following is most important to include in preparing Franco’s bedside equipment? [1] A. Hand bell and extra bed linen B. Sandbag and trochanter rolls C. Footboard and splint

D. Suction machine and gloves 86. What is the rationale for giving Mr. Franco frequent mouth care? [1] A. He will be thirsty considering that he is doesn’t drink enough fluids B. To remove dried blood when tongue is bitten during a seizure C. The tactile stimulation during mouth care will hasten return to consciousness D. Mouth breathing is used by comatose patient and it’ll cause oral mucosa dying and cracking. 87. One of the complications of prolonged bed rest is decubitus ulcer. Which of the following can best prevent its occurrence? [1] A. Massage reddened areas with lotion or oils B. Turn frequently every 2 hours C. Use special water mattress D. Keep skin clean and dry 88. If Mr. Franco’s Right side is weak, What should be the most accurate analysis by the nurse? [4] A. Expressive aphasia is prominent on clients with right sided weakness B. The affected lobe in the patient is the Right lobe C. The client will have problems in judging distance and proprioception D. Clients orientation to time and space will be much affected SITUATION : [ From JUN 1988 NLE ] a 20 year old college student was rushed to the ER of PGH after he fainted during their ROTC drill. Complained of severe right iliac pain. Upon palpation of his abdomen, Ernie jerks even on slight pressure. Blood test was ordered. Diagnosis is acute appendicitis. 89. Which result of the lab test will be significant to the diagnosis? [1] A. RBC : 4.5 TO 5 Million / cu. mm. B. Hgb : 13 to 14 gm/dl. C. Platelets : 250,000 to 500,000 cu.mm. D. WBC : 12,000 to 13,000/cu.mm 90. Stat appendectomy was indicated. Pre op care would include all of the following except? [1] A. Consent signed by the father B. Enema STAT C. Skin prep of the area including the pubis D. Remove the jewelries

91. Pre-anesthetic med of Demerol and atrophine sulfate were ordered to : [3] A. Allay anxiety and apprehension B. Reduce pain C. Prevent vomiting D. Relax abdominal muscle 92. Common anesthesia for appendectomy is [3] A. Spinal B. General C. Caudal D. Hypnosis 93. Post op care for appendectomy include the following except [1] A. Early ambulation B. Diet as tolerated after fully conscious C. Nasogastric tube connect to suction D. Deep breathing and leg exercise 94. Peritonitis may occur in ruptured appendix and may cause serious problems which are [2] 1. Hypovolemia, electrolyte imbalance 2. Elevated temperature, weakness and diaphoresis 3. Nausea and vomiting, rigidity of the abdominal wall 4. Pallor and eventually shock A. 1 and 2 B. 2 and 3 C. 1,2,3 D. All of the above 95. If after surgery the patient’s abdomen becomes distended and no bowel sounds appreciated, what would be the most suspected complication? [1] A. Intussusception B. Paralytic Ileus C. Hemorrhage D. Ruptured colon 96. NGT was connected to suction. In caring for the patient with NGT, the nurse must [2] A. Irrigate the tube with saline as ordered B. Use sterile technique in irrigating the tube

C. advance the tube every hour to avoid kinks D. Offer some ice chips to wet lips 97. When do you think the NGT tube be removed? [1] A. When patient requests for it B. Abdomen is soft and patient asks for water C. Abdomen is soft and flatus has been expelled D. B and C only Situation: Amanda is suffering from chronic arteriosclerosis Brain syndrome she fell while getting out of the bed one morning and was brought to the hospital, and she was diagnosed to have cerebrovascular thrombosis thus transferred to a nursing home. 98. What do you call a STROKE that manifests a bizarre behavior? [4] A. Inorganic Stroke B. Inorganic Psychoses C. Organic Stroke D. Organic Psychoses 99. The main difference between chronic and organic brain syndrome is that the former [2] A. Occurs suddenly and reversible B. Is progressive and reversible C. tends to be progressive and irreversible D. Occurs suddenly and irreversible 100. Which behavior results from organic psychoses? [4] A. Memory deficit B. Disorientation C. Impaired Judgement D. Inappropriate affect Answer Key http://pinoybsn.blogspot.com/2006/07/100-item-medical-surgical_115218420022881261.html 50 Item Psychiatric Nursing Exam by Budek PSYCHIATRIC NURSING By Budek

1. 60 year old post CVA patient is taking TPA for his disease, the nurse understands that this is an example of what level of prevention? a. primary b. secondary c. tertiary d. nota 2. A female client undergoes yearly mammography. This is a type of what level of prevention? a. primary b. secondary c. tertiary d. nota 3. A Diabetic patient was amputated following an unexpected necrosis on the right leg, he sustained and undergone BKA. He then underwent therapy on how to use his new prosthetic leg. this is a type of what level of prevention? a. primary b. secondary c. tertiary d. nota 4. As a care provider, The nurse should do first: a. Provide direct nursing care. b. Participate with the team in performing nursing intervention. c. Therapeutic use of self. d. Early recognition of the client’s needs. 5. As a manager, the nurse should: a. Initiates nursing action with co workers. b. Plans nursing care with the patient. c. Speaks in behalf of the patient. d. Works together with the team. 6. the nurse shows a patient advocate role when a. defend the patients right b. refer patient for other services she needs c. work with significant others d. intercedes in behalf of the patient.

7. which is the following is the most appropriate during the orientation phase ? a. patients perception on the reason of her hospitalization b. identification of more effective ways of coping c. exploration of inadequate coping skills d. establishment of regular meeting of schedules 8. preparing the client for the termination phase begins : a. pre orientation b. orientation c. working d. termination 9. a helping relationship is a process characterized by : a. recovery promoting b. mutual interaction c. growth facilitating d. health enhancing 10. During the nurse patient interaction, the nurse assess the ff: to determine the patients coping strategy : a. how are you feeling right now? b. do you have anyone to take you home? c. what do you think will help you right now? d. How does your problem affect your life? 11. As a counsellor, the nurse performs which of the ff: task? a. encourage client to express feelings and concerns b. helps client to learn a dance or song to enable her to participate in activities c. help the client prepare in group activities d. assist the client in setting limits on her behaviour 12. Freud stresses out that the EGO a. Distinguishes between things in the mind and things in the reality. b. Moral arm of the personality that strives for perfection than pleasure. c. Reservoir of instincts and drives d. Control the physical needs instincts. 13. A 16 year old child is hospitalized, according to Erik Erikson, what is an appropriate intervention?

a. tell the friends to visit the child b. encourage patient to help child learn lessons missed c. call the priest to intervene d. tell the child’s girlfriend to visit the child. 14. NMS is characterized by : a. hypertension, hyperthermia, flushed and dry skin. b. Hypotension, hypothermia, flushed and dry skin. c. Hypertension, hyperthermia, diaphoresis d. Hypertension, hypothermia, diaphoresis 15. Which of the following drugs needs a WBC level checked regularly? a. Lithane b. Clozaril c. Tofranil d. Diazepam Angelo, an 18 year old out of school youth was caught shoplifting in a department store. He has history of being quarrelsome and involving physical fight with his friends. He has been out of jail for the past two years 16. Initially, The nurse identifies which of the ff: Nursing diagnosis: a. self centred disturbance b. impaired social interaction c. sensory perceptual alteration d. altered thought process 17. which of the ff: is not a characteristic of PD? a. disregard rights of others b. loss of cognitive functioning c. fails to conform to social norms d. not capable of experiencing guild or remorse for their behaviour 18. the most effective treatment modality for persons if anti social PD is a. hypnotherapy b. gestalt therapy c. behaviour therapy d. crisis intervention 19. Which of the following is not an example of alteration of perception?

a. ideas of reference b. flight of ideas c. illusion d. hallucination 20. The type of anxiety that leads to personality disorganization is : a. Mild b. moderate c. severe d. panic 21. A client is admitted to the hospital. Twelve hours later the nurse observes hand tremors, hyperexicitability, tachycardia, diaphoresis and hypertension. The nurse suspects alcohol withdrawal. The nurse should ask the client: a. at what time was your last drink taken? b. Why didn’t you tell us you’re a drinker? c. Do you drink beer or hard liquor? d. How long have you been drinking? 22. client with a history of schizophrenia has been admitted for suicidal ideation. The client states "God is telling me to kill myself right now." The nurse's best response is: a. I understand that god’s voice are real to you, But I don’t hear anything. I will stay with you. b. The voices are part of your illness, it will stop if you take medication c. The voices are all in your imagination, think of something else and itll go away d. Don’t think of anything right now, just go and relax. 23. In assessing a client's suicide potential, which statement by the client would give the nurse the HIGHEST cause for concern? a. my thoughts of hurting my self are scary to me b. I’d like to go to sleep and not wake up c. I’ve thought about taking pills and alcohol till I pass out d. Id like to be free from all these worries 24. A client with paranoid schizophrenia has persecutory delusions and auditory hallucinations and is extremely agitated. He has been given a PRN dose of Thorazine IM. Which of the following would indicate to the nurse that the medication is having the desired effect? a. Complains of dry mouth b. State he feels restless in his body c. Stops pacing and sits with the nurse d. Exhibits increase activity and speech

25. A client who was wandering aimlessly around the streets acting inappropriately and appeared disheveled and unkempt was admitted to a psychiatric unit and is experiencing auditory and visual hallucinations. The nurse would develop a plan of care based on: a. borderline personality disorder b. anxiety disorder c. schizophrenia d. depression 26. A decision is made to not hospitalize a client with obsessive-compulsive disorder. Of the following abilities the client has demonstrated, the one that probably most influenced the decision not to hospitalize him is his ability to: a. Hold a job. b. Relate to his peers. c. Perform activities of daily living. d. Behave in an outwardly normal 27. A client is admitted to the inpatient psychiatric unit. He is unshaven, has body odor, and has spots on his shirt and pants. He moves slowly, gazes at the floor, and has a flat affect. The nurse's highest priority in assessing the client on admission would be to ask him: a. How he sleeps at night. b. If he is thinking about hurting himself. c. About recent stresses. d. How he feels about himself. 28. The nurse should know that the normal therapeutic level of lithium is : a. .6 to 1.2 meq/L b. 6 to 12 meq/L c. .6 to .12 cc/ml d. .6 to .12 cc3/L 29. The patient complaint of vomiting, diarrhea and restlessness after taking lithane. The nurse’s initial intervention is : a. Recognize that this is a sign of toxicity and withhold the next medication. b. Notify the physician. c. Check V/S to validate patient’s concerns. d. Recognize that this is a normal side effects of lithium and still continue the drug. 30. The client is taking TOFRANIL. The nurse should closely monitor the patient for : a. Hypertension

b. Hypothermia c. Increase Intra Ocular Pressure d. Increase Intra Cranial Pressure 31. A client was hospitalized with major depression with suicidal ideation for 1 week. He is taking venlafaxine (Effexor), 75 mg three times a day, and is planning to return to work. The nurse asks the client if he is experiencing thoughts of self-harm. The client responds, "I hardly think about it anymore and wouldn't do anything to hurt myself." The nurse judges: a. The client to be decompensating and in need of being readmitted to the hospital. b. The client to need an adjustment or increase in his dose of antidepressant. c. The depression to be improving and the suicidal ideation to be lessening. d. The presence of suicidal ideation to warrant a telephone call to the client's physician 32. The client is taking sertraline (Zoloft), 50 mg q AM. The nurse includes which of the following in the teaching plan about Zoloft? a. Zoloft causes erectile dysfunction in men. b. Zoloft causes postural hypotension c. Zoloft increases appetite and weight gain d. It may take 3-4 weeks before client will start feeling better. 33. After 3 days of taking haloperidol, the client shows an inability to sit still, is restless and fidgety, and paces around the unit. Of the following extrapyramidal adverse reactions, the client is showing signs of: a. Dystonia. b. Akathisia. c. Parkinsonism. d. Tardive dyskinesia. 34. After 10 days of lithium therapy, the client's lithium level is 1.0 mEq/L. The nurse knows that this value indicates which of the following? a. A laboratory error. b. An anticipated therapeutic blood level of the drug. c. An atypical client response to the drug. d. A toxic level. 35. When caring for a client receiving haloperidol (Haldol), the nurse would assess for which of the following? a. Hypertensive episodes. b. Extrapyramidal symptoms. c. Hypersalivation.

d. Oversedation. 36. A client is brought to the hospital’s emergency room by a friend, who states, "I guess he had some bad junk (heroin) today." In assessing the client, the nurse would likely find which of the following symptoms? a. Increased heart rate, dilated pupils, and fever. b. Tremulousness, impaired coordination, increased blood pressure, and ruddy complexion. c. Decreased respirations, constricted pupils, and pallor. d. Eye irritation, tinnitus, and irritation of nasal and oral mucosa. 37. The client has been taking the monoamine oxidase inhibitor (MAOI) phenelzine (Nardil), 10 mg bid. The physician orders a selective serotonin reuptake inhibitor (SSRI), paroxetine (Paxil), 20 mg given every morning. The nurse: a. Gives the medication as ordered. b. Questions the physician about the order. c. Questions the dosage ordered. d. Asks the physician to order benztropine (Cogentin) for the side effects. 38. Which of the following client statements about clozapine (Clozaril) indicates that the client needs additional teaching? a. "I need to have my blood checked once every several months while I’m taking this drug." b. "I need to sit on the side of the bed for a while when I wake up in the morning." c. "The sleepiness I feel will decrease as my body adjusts to clozapine." d. "I need to call my doctor whenever I notice that I have a fever or sore throat." 39. A client has been taking lithium carbonate (Lithane) for hyperactivity, as prescribed by his physician. While the client is taking this drug, the nurse should ensure that he has an adequate intake of: a. Sodium. b. Iron. c. Iodine. d. Calcium. 40. The client has been taking clomipramine (Anafranil) for his obsessive-compulsive disorder. He tells the nurse, "I'm not really better, and I've been taking the medication faithfully for the past 3 days just like it says on this prescription bottle." Which of the following actions would the nurse do first? a. Tell the client to continue taking the medication as prescribed because it takes 5 to 10 weeks for a full therapeutic effect.

b. Tell the client to stop taking the medication and to call the physician. c. Encourage the client to double the dose of his medication. d. Ask the client if he has resumed smoking cigarettes. 41. The nurse judges correctly that a client is experiencing an adverse effect from amitriptyline hydrochloride (Elavil) when the client demonstrates: a. An elevated blood glucose level. b. Insomnia. c. Hypertension. d. Urinary retention. 42. Which of the following health status assessments must be completed before the client starts taking imipramine (Tofranil)? a. Electrocardiogram (ECG). b. Urine sample for protein. c. Thyroid scan. d. Creatinine clearance test. 43. A client comes to the outpatient mental health clinic 2 days after being discharged from the hospital. The client was given a 1-week supply of clozapine (Clozaril). The nurse reviews information about clozapine with the client. Which client statement indicates an accurate understanding of the nurse's teaching about this medication? a."I need to call my doctor in 2 weeks for a checkup." b."I need to keep my appointment here at the hospital this week for a blood test." c. "I can drink alcohol with this medication." d. "I can take over-the-counter sleeping medication if I have trouble sleeping." 44. The client is taking risperidone (Risperdal) to treat the positive and negative symptoms of schizophrenia. Which of the following negative symptoms will improve?. a. Abnormal thought form. b. Hallucinations and delusions. c. Bizarre behaviour. d. Asocial behaviour and anergia. 45. The nurse would teach the client taking tranylcypromine sulfate (Parnate) to avoid which food because of its high tyramine content? a. Nuts. b. Aged cheeses. c. Grain cereals. d. Reconstituted milk.

46. Which of the following clinical manifestations would alert the nurse to lithium toxicity? a. Increasingly agitated behaviour. b. Markedly increased food intake. c. Sudden increase in blood pressure. d.Anorexia with nausea and vomiting. 47. The client with depression has been hospitalized for 3 days on the psychiatric unit. This is the second hospitalization during the past year. The physician orders a different drug, tranylcypromine sulfate (Parnate), when the client does not respond positively to a tricyclic antidepressant. Which of the following reactions should the client be cautioned about if her diet includes foods containing tryaminetyramine? a. Heart block. b. Grand mal seizure. c. Respiratory arrest. d. Hypertensive crisis. 48. After the nurse has taught the client who is being discharged on lithium (Eskalith) about the drug, which of the following client statements would indicate that the teaching has been successful? a. "I need to restrict eating any foods that contain salt." b. "If I forget a dose, I can double the dose the next time I take it." c. "I'll call my doctor right away for any vomiting, severe hand tremors, or muscle weakness." d. "I should increase my fluid” 49. A nurse is caring for a client with Parkinson's disease who has been taking carbidopa/levodopa (Sinemet) for a year. Which of the following adverse reactions will the nurse monitor the client for? a. dykinesia b. glaucoma c. hypotension d. respiratory depression 50. A client is taking fluoxetine hydrochloride (Prozac) for treatment of depression. The client asks the nurse when the maximum therapeutic response occurs. The nurse's best response is that the maximum therapeutic response for fluoxetine hydrochloride may occur in the: a. 10-14 days b. First week

c. Third week d. Fourth week Answer Key http://pinoybsn.blogspot.com/2006/07/50-item-psychiatric-exam-answers-and.html

View more...

Comments

Copyright ©2017 KUPDF Inc.
SUPPORT KUPDF